VTNE study set 2

Réussis tes devoirs et examens dès maintenant avec Quizwiz!

The term myiasis indicates a problem with which of the following? Flies Mites Worms Lice Fleas

Flies Myiasis refers to infestation with larvae of flies (diptera). Pediculosis is the term for infestation with lice.

What is the ideal age for tail docking in puppies? At birth 3 to 5 days 4 to 6 months or at the time of sterilization 2 to 3 weeks

3 to 5 days

Abdominal radiographs should be taken at which of the following times? Systole Full expiration Full inspiration Diastole

Full expiration Abdominal radiographs are best taken at full expiration because this is when the diaphragm is pushed cranially, which increases the amount of space in the peritoneum and minimizes the overlap of abdominal organs on the radiograph. It is usually not possible to time a radiographic exposure based on the cardiac cycle.

While assisting with a field necropsy on a 9-year old Thoroughbred mare, you notice the pictured parasite within the stomach. What is this parasite? Strongyloides westeri Onchocerca Gasterophilus intestinalis Parascaris equorum

Gasterophilus intestinalis Gasterophilus are not worms but rather bot fly larvae and are also known as the stomach bot. There are several species; G. nasalis, G. haemorrhoidalis, and G. intestinalis. While they may appear disgusting, they typically do not cause any clinical signs. It has been reported that bot fly infestation can lead to stomach rupture, but this is extremely rare.

Of the following, which drug is considered an aminoglycoside? Enrofloxacin Doxycycline Gentamicin Itraconazole Cephalexin

Gentamicin

A 5-month old female cat presents for weight loss, chronic diarrhea and steatorrhea. You are looking at a stained fecal smear from the cat as shown in the image below (this is a magnified 40X image; the organism is approximately 15 x 8 um). What is the organism here? (has a face) Giardia Tritrichomonas Toxascaris Toxacara

Giardia

The pet in this photo is being tested for what condition? (eye) Pannus Glaucoma Kerratoconjunctivitis sicca Hypertension

Glaucoma

Dorzolamide ophthalmic drops may be used in treating what eye condition? Keratoconjunctivitis sicca Glaucoma Corneal ulcer Pannus

Glaucoma Dorzolamide is a carbonic anhydrase inhibitor that is a common eye drop used in the treatment of glaucoma (increased eye pressures). This drug helps to decrease aqueous production in the eye, thus lowering intraocular pressure.

You are performing treatments on a horse and you need to apply dimethyl sulfoxide (DMSO) topically over a region of tenosynovitis in a horse. Which of the following personal protective equipment should be worn? Surgical gown Protective goggles Filtered respirator Gloves

Gloves DMSO is an anti-inflammatory drug. It may be administered topically, orally, or diluted and administered IV. Gloves should be worn when applying or handling DMSO because it can be absorbed through the skin. Nitrile gloves are preferable to latex because DMSO can be absorbed through latex gloves.

Which blood chemistry value may be artificially lowered if analysis of a blood sample is delayed? Creatinine Sodium Glucose Alanine aminotransferase

Glucose

Which of the following areas should be avoided when giving intramuscular injections in cattle? Neck Gluteal muscles Pectoral muscles Shoulders

Gluteal muscles The gluteal region should be avoided because of increased chance of injection-induced abscesses at this site.

Bacteria that stain red by safranin or basic fuchsin on a Gram stain are known as which of the following? Gram-negative Gram-neutral Gram-red Gram-positive

Gram-negative

Which of these dog breeds is most predisposed to development of gastric dilatation and volvulus? Great Dane Dalmatian Chihuahua Cocker Spaniel

Great Dane

Nervous tissue without myelin is known as which of the following? Grey matter Dura mater Pia mater White matter

Grey matter Grey matter is nervous tissue that is not myelinated. White matter is myelinated. Myelin is a dielectric material that insulates and provides a sheath around the axon of a neuron. Dura mater and pia mater (along with arachnoid) are part of the meninges of the brain.

An elevated PCV is considered normal for which of the following breeds? Greyhound Akita Cavalier King Charles Spaniel Chihuahua

Greyhound Due to their sporting nature, Greyhounds tend to have packed cell volumes that are normally higher than other breeds.

What species offspring is labeled "precocious"? Guinea Pig Cat Hamster Rat Rabbit Ferret

Guinea Pig Due to the long gestation period for guinea pigs, the development of the fetus occurs in utero. After 58 days, the young are born with their eyes open, fully furred and able to walk. This is know as "precocious". The opposite is "altricial". This means to be born with eyes closed, hairless and helpless.

A 13.2-lb cat is being sent home with an oral solution of Tramadol for post-operative pain control. At a dose of 3 mg/kg, how many milliliters should be given if the solution comes in a 5 mg/ml concentration? 5 ml 3.6 ml 8 ml 18 ml

3.6 ml

How much 2% lidocaine should be drawn up if performing an intra-articular injection at a dose of 2mg/kg in a 38-kg dog? 12 ml 38 ml 7.6 ml 3.8 ml

3.8 ml 2% lidocaine is equivalent to 20 mg/ml. 38 kg x (2 mg/kg) = 76 mg 76 mg / (20 mg/ml) = 3.8 ml

Your clinic has decided to start harvesting blood from volunteer donors to administer to patients. How long can whole blood units be stored in the refrigerator if properly collected? 35 days 3 months 21 days 1 year 45 days

35 days Whole blood units may be stored for 35 days if the unit is collected aseptically, stored appropriately and mixed daily.

How long does estrus in the dog typically last? 14-28 days 6 months 1-3 days 4-13 days

4-13 days Estrus typically lasts 4 to 13 days, with an average of 9 days. The entire estrous cycle in the dog is approximately 6 months

Where would you find an arrector pili muscle? Penis Ear Ovary Heart Hair follicle

Hair follicle

What is the number one cause of spreading nosocomial infections in a hospital setting? Hands of hospital staff Dirty floors Contaminated food and water dishes Animal feces

Hands of hospital staff

Rats are known to produce a substance called "red tears" or chromodacyorrhea. Where are red tears produced? Harderian gland Nasal tissue gland Retro-orbital gland Conjunctival gland

Harderian gland

A 2-year old Dalmatian presents after being hit by a car. Anisocoria is noted during the initial physical examination. What type of trauma would be suspected? Orthopedic injury Splenic torsion Ruptured bladder Pneumothorax Head trauma

Head trauma

Stopping a blood vessel from bleeding is known as what? Hemolysis Coagulopathy Incision Hemostasis

Hemostasis

A 10 kg Miniature Schnauzer is to receive 22mg/kg cefazolin intravenously following a laceration repair. If the concentration of the cefazolin is 50mg/ml, how many milliliters should be administered? 6.0 2.2 1.5 3.2 4.4

4.4

A client presents with her 8-week old puppy. She wants to know when she can be spayed. What is the ideal recommended age for spays and neuters? 5-6 months 10-12 weeks 8-10 months The younger the better, as young as 2 weeks

5-6 months

You are instructed to administer 12 liters of sterile isotonic IV fluids via a fluid pump over the next 24 hours. What is the appropriate fluid rate to administer this volume of IV fluids? 1 liter/hour 500 mL/hour 12 liters/hour 250 mL/hour

500 mL/hour 12 liters (12000 mLs) divided by 24 hours would equal 500 mLs/hour.

In general, at what age are puppy teeth (deciduous) considered to be retained? 18 months 1 year 3 months 6 months

6 months

A 2-ounce bottle of Dermacool spray contains how many milliliters? 40 mL 15 mL 30 mL 60 mL

60 mL There are 30 mL in one ounce. 2-ounce bottle X 30 mL/ounce = 60 mL

A 5-year old Labrador has developed hemorrhagic gastroenteritis. The clinician has instructed you to start him on Metronidazole at a dose of 10 mg/kg intravenously twice daily. Metronidazole is available in a 5 mg/ml solution. How many milliliters are necessary per dose? The dog's body weight is 80 lbs. 40 ml 18.2 ml 73 ml 160 ml

73 ml

What percentage of mammary gland tumors in the cat are malignant? 90% 50% 20% 100%

90%

While caring for a week old orphan puppy, it is important to monitor body temperature. Rectal temperature should be between: 92-94F 94-99F 101-103F 99-101F

94-99F If the temperature falls below 94F, the heart rate will drop and intestinal motion ceases. Death will occur if not corrected.

A patient who has stopped breathing after administration of a short-acting anesthetic can be described as being which of the following? Apneic Dyspneic Eupneic Atelectic

Apneic This patient would be apneic, which means the patient is not breathing. Dyspnea implies difficulty breathing. Eupnea is normal respiration. Atelectasis is the description given to collapsed lungs.

Impulses are carried away from the neuronal cell body by which of the following structures? Spinous process Axon Dendrite Afferent nerve

Axon

Which lab test is used to assess kidney function and its breakdown product of protein? Albumin BUN Total bilirubin Creatinine Glucose

BUN

In which species would you find the bursa of Fabricius? Llamas Birds Rabbits Rats

Birds

A dog is going to be euthanized due to aggressive behavior, and the owner wants the dog tested for rabies. The dog has not bitten anyone. What should be submitted? Heart Serum Salivary gland Brain Urine

Brain

A male rabbit is also referred to as a ______. Gilt Brock Boar Buck

Buck

Which of the following is a local anesthetic of choice for use in veterinary dentistry because it can provide 3-5 hours of pain relief? Preservative-free morphine Lidocaine Proparacaine Bupivacaine

Bupivacaine

Which of the following is considered a partial mu agonist/antagonist? Hydromorphone Naloxone Morphine Butorphanol

Butorphanol

Release of what molecule from the sarcoplasmic reticulum ultimately helps to achieve muscle contraction? Calcium Adenosine triphosphate Chloride Sodium

Calcium

What is the best construction form for a tortoise outdoor enclosure? Cement blocks Pressure treated wood Chicken wire Chain link fence Stacked rocks or pavers

Cement blocks

What is a round or spherical shaped bacteria called? Spirochete Pleomorphic Coccus Nematode Bacillus

Coccus

What is the term given to the initial milk produced by a female after giving birth? Colostrum Allantois Amnion Chorion

Colostrum

A purple top (or lavender top) is used to run which blood test? Biochemistry panel Complete blood count Potassium bromide level Coagulation panel

Complete blood count

What is the shape of a bacillus such as the bacteria that causes anthrax, Bacillus anthracis? Cylindrical Spiral Circular Round and organized in chains

Cylindrical

Which breed is over-represented and prone to intervertebral disk disease? Labradors Dachshunds Chihuahuas St. Bernards Schnauzers

Dachshunds

Which laboratory test is used to differentiate white blood cell types? Methylene blue wet mount Giemsa's stain Diff-Quick stain Gram stain

Diff-Quick stain

The Greyhound is an example of which of the following breeds? Brachycephalic Brachygnathic Mesaticephalic Dolichocephalic

Dolichocephalic

What is the approximate minute volume of a 4 kg cat? 40 ml 4 L 250 ml 100 ml 25 ml 1 L

1 L

A 620-kg horse weighs how many pounds? 281 pounds 1,364 pounds 1,625 pounds 1,240 pounds

1,364 pounds

A dog presents for a broken adult canine tooth. The tooth is broken at the tip and a small area of pulp is exposed. Exposure of pulp in a tooth should be treated in which way? A sealant Antibiotics and soft food until it heals It should be left alone since treatment is not required Endodontic treatment

Endodontic treatment

A laryngoscope might aid in which procedure? Endotracheal tube placement Jugular catheter placement Anal gland expression Ear drum examination

Endotracheal tube placement

A 5-week old cat received emergency surgery for a laceration. What antibiotic should NOT be used in this patient? Amoxicillin Cefazolin Clindamycin Enrofloxacin Ticarcillin

Enrofloxacin Young animals should not receive enrofloxacin (Baytril) due to risk of cartilage defects (mostly in young large breed dogs). This drug must be used with caution and at lower doses in cats because it may cause blindness.

You are performing a cytology from a lesion on the chin of a cat (see image). The cytology shows many cells with pink-staining granules. These are most likely which cell type? Lymphocytes Eosinophils Neutrophils Basophils

Eosinophils

You should not give a phenothiazine tranquilizer to an animal with which disease? Glaucoma Diabetes Hypothyroidism Epilepsy Cushings

Epilepsy Phenothiazine tranquilizers such as acepromazine or chlorpromazine are traditionally thought to be contraindicated in patients with seizure disorders. More recent research has shown that they are not nearly as dangerous as originally perceived and some don't consider these medications as being contraindicated.

Which neurotransmitter is associated with the sympathetic nervous system? Histamine Epinephrine Acetylcholine GABA Glutamate

Epinephrine Neurotransmitters transmit signals from a neuron to a cell across a synapse. Epinephrine and norepinephrine are the neurotransmitters associated with the sympathetic nervous system. Acetylcholine is associated with the parasympathetic nervous system.

Ruminants frequently belch or burp to release gas from the forestomach. What is the term for this process known as? Apnea Fermentation Eructation Casting

Eructation

Which of these cell types are all typically found in circulation? Rubriblasts, thrombocytes, erythrocytes Lymphoblasts, erythrocytes, leukocytes Leukocytes, thrombocytes, metamyelocytes Erythrocytes, leukocytes, thrombocytes

Erythrocytes, leukocytes, thrombocytes The cellular makeup of the blood includes erythrocytes (red blood cells), leukocytes (white blood cells), and thrombocytes (platelets)

When a calf is born, milk goes immediately to the omasum and bypasses the rumen and reticulum. Which anatomic structure makes this possible? Reticular folds Esophageal groove Gastric rugae Epiploic foramen

Esophageal groove

Which hormone causes the vaginal cells of the female dog to proliferate and form cornified epithelium? Testosterone Cortisol Progesterone Prolactin Estrogen

Estrogen The correct answer is estrogen. As estrogen rises during proestrus, the vaginal epithelium proliferates, cornifies, and keratinizes, presumably in preparation for mating.

Where is a meibomian gland located? Foot pads Testicles Eyelid Ear canal

Eyelid

Canine Distemper Virus can be transmitted to which species? Pig Cat Horse Ferret

Ferret

During which of the following periods is a human embryo or fetus most sensitive to adverse effects from radiation? Third trimester The is no difference in the risk based on the stage of pregnancy First trimester Second trimester

First trimester The first 3 months of pregnancy are the most important in terms of radiation safety because the embryo/fetus is most sensitive at this time. This heightened sensitivity is due to the organogenesis that occurs during the first trimester.

Smooth muscle is found in what regions? Hollow organs Within the muscles of the eye Within the muscles of mastication Extremities

Hollow organs

Which of these animals does not have a gall bladder? Cat Horse Rabbit Cow Bird

Horse

The coffin bone is a layman's term for part of the anatomy of which species? Pig Dog Horse Sheep

Horse The coffin bone is the third phalanx (or distal phalanx/P3) in horses. It is also known as the "toe bone".

Which of the following species have hypsodont teeth? Horses Monkeys Dogs Pigs

Horses

Which of the following species is unable to vomit? Dogs Birds Cows Cats Horses

Horses

Which species is not susceptible to "hoof-and-mouth" disease? Horses Cattle Pigs Sheep

Horses

You are asked to prepare a patient for a bone marrow aspirate. Which of the following sites are commonly used in dogs to obtain a bone marrow sample? Femoral head or sacrum Femoral condyle and greater tubercle of the humerus Humeral head and iliac crest Sternum or iliac crest

Humeral head and iliac crest

An 8-month old Siamese kitten presents for ataxia, and the veterinarian suspects the kitten has "water on the brain" due to her enlarged head size. What is the term for this? Paracervical edema Brachycephalus Hydrocephalus Craniostosis

Hydrocephalus Hydrocephalus is a build-up of cerebrospinal fluid inside the skull, leading to brain swelling. Hydrocephalus means "water on the brain".

Phenylpropanolamine is used to treat urinary incontinence in dogs. It can commonly cause which side effect that should be discussed with the owner? Hypocalcemia Hypertension Profound hyperglycemia Renal failure

Hypertension

Hyperthyroidism in cats can be associated with which condition? Dilated cardiomyopathy (DCM) Bradycardia Hypertrophic cardiomyopathy (HCM) Patent ductus arteriosus

Hypertrophic cardiomyopathy (HCM) Hypertrophic cardiomyopathy is a thickening of the heart muscle (myocardium) and is often associated with hyperthyroidism or hypertension.

Which of the following is known as Addison's disease? Hypoadrenocorticism Hyperthyroidism Hyperadrenocorticism Hypoparathyroidism

Hypoadrenocorticism Hypoadrenocorticism is a decrease in circulating cortisol. Hyperadrenocorticism is an increase in circulating cortisol and is known as Cushing's disease.

When looking under the microscope at a blood sample of a patient with iron deficiency, which of the following terms decribes what you would most likely see? Hyperchromasia Hypovolemia Hypochromasia Hyponatremia

Hypochromasia

A farmer brings the one-half inch grub shown in the image to you one spring day, telling you it emerged from a hole in the back of one of his prize show cattle. What is the name of the parasite in the image? Habronema Hypoderma bovis Musca autumnalis Cuterebra

Hypoderma bovis

An owner called and said that her dog ate a pack of sugarless gum containing xylitol. You tell her to bring her dog in right away. Which of the following could this cause? Hypercalcemia Hypoglycemia Hyperglycemia Hypophosphatemia

Hypoglycemia Xylitol is a sugar alcohol found in many sugarless candies, sweets, and gum. Xylitol toxicity can be life-threatening. Dogs that ingest doses of >0.1 g/kg of xylitol are at risk for developing hypoglycemia, while dogs that ingest >0.5 g/kg may develop acute liver failure. Supportive care and dextrose for treatment of hypoglycemia are indicated.

Which of the following is not a common cause of dystocia? Hypokalemia Decreased pelvic diameter Uterine inertia Oversize fetus

Hypokalemia Hypokalemia is not considered a cause of dystocia. Hypocalcemia, on the other hand, can result in decreased ability to contract and will be a cause for dystocia. Oxytocin release will lead to contraction and depletion of calcium stores; inadequate release of oxytocin may result in uterine inertia. An oversized fetus, as is common in small breed dogs, may necessitate a planned cesarean delivery. Animals that have a decreased pelvic diameter will also be at risk of developing dystocia. A common cause of decreased pelvic diameter is previous trauma.

A urine sample with a specific gravity of less than 1.008 is considered to be which of the following? Hyposthenuric Isosthenuric Hypersthenuric Normosthenuric

Hyposthenuric

What is the average cloacal temperature of a chicken? 104F 98.8F 107F 102F

107F

Which of the following is characteristic of an animal in shock? Decreased heart rate Increased heart rate Normal temperature Increased blood pressure

Increased heart rate

If the kidney has a drop in blood flow and a part of the kidney dies, what would this area be termed? Diuretic Dialyzed Azotemic Infarcted

Infarcted

Which of the following correctly defines the term stomatitis? Inflammation of the periodontal ligament and alveolar bone Inflammation of the tongue Inflammation of the oral mucosal surfaces Inflammation of the region housing the tonsils

Inflammation of the oral mucosal surfaces

A 2-year old dachshund presents with acute urticaria and facial swelling. Which of the following would be a most likely cause for this presentation (puffy face)? Flea allergy dermatitis Contact dermatitis Drug eruption Insect sting

Insect sting

Which route of drug administration usually allows for the most rapid onset of action? Subcutaneous Oral Intravenous Intramuscular

Intravenous

Your dog's histopathology results have arrived and show an adenoma. What is true about an adenoma? Is always easy to excise and small in nature Does not cause morbidity Is not malignant Is frequently life-threatening

Is not malignant

The surgeon accidentally drops this clamp on the floor. You pick it up and he asks for another one. What is the name of this instrument? (slit going down half way) Mayo clamp Backhaus clamp Kelly clamp Carmalt clamp

Kelly clamp

When placing an epidural in a dog or cat, the spinal needle is inserted into the space located between which two vertebrae? L6, L7 L7, S1 L6, S1 L8, S1

L7, S1 There are 7 lumbar vertebrae, when placing an epidural it is done between the seventh lumbar vertebra and the first sacral vertebra.

Where is the most recommended area to administer a subcutaneous injection in a horse? Over the withers Caudal aspect of the forearm Dorsal aspect of the flank Lateral aspect of the neck

Lateral aspect of the neck

The term pediculosis indicates a problem with which of the following? Fleas Flies Worms Lice Mites

Lice Pediculosis refers to infestation with lice. It does not differentiate between biting and sucking lice. Myiasis is the term for infestation with fly larvae.

"Grass tetany" may been seen in sheep or goats due to a deficiency in which mineral? Copper Magnesium Potassium Sulfur

Magnesium

Struvite crystals are one of the most common observed in urine. What are the chemical constituents of struvite? Magnesium, bilirubin, and phosphate Magnesium, ammonium, and phosphate Ammonium, silica, and carbon Calcium and phosphate

Magnesium, ammonium, and phosphate

Which of the following is not a potential job responsibility of the technician during colic surgery? Opening surgical packs Administering anesthesia Making the initial incision into the abdomen Placing a jugular catheter

Making the initial incision into the abdomen

Which of the following should be clean, but does not need to be sterile for surgical procedures? Gloves Gown Drape Mask

Mask

The 4th upper premolars are connected with which structure in dogs? Maxillary Sinus Nasal cavity Pharynx Frontal Sinus

Maxillary Sinus

After performing a blood smear in a patient it is determined that macrocytosis is present. Which value would be expected to be elevated based on this observation on a complete blood count? Red cell distribution width Hemoglobin Red blood cell number Mean corpuscular volume

Mean corpuscular volume Macrocytosis is expected to result in an increase in mean corpuscular volume. Mean corpuscular volume is a measurement of the mean volume in a group of red blood cells. If there is a higher volume, then one can deduce that the cell is larger (macrocytosis). Red blood cell number as well as hemoglobin content should remain unaffected. Red cell distribution is a value that helps the interpreter determine if there is anisocytosis present.

When placing a bandage on the limb, what is the main purpose for leaving the tips of the toes exposed? Monitor for swelling Prevent odor Prevent swelling Prevent soiling Allow for toe nail trims

Monitor for swelling

Ivermectin is effective against which organisms? Cestodes Nematodes Protozoa Trematodes

Nematodes Nematodes or roundworms are susceptible to ivermectin. Ivermectin has activity against ticks, mites, nematodes, and even lice.

What is the basic structural and functional unit of the kidney? Nephron Glomerulus Hepatocyte Neuron

Nephron

What is the preferred stain to identify reticulocytes and Heinz bodies in erythrocytes? Romanowsky stain Wright's stain Wright's-Giemsa stain New methylene blue

New methylene blue

You are doing a CBC on an anemic patient. Which of the following cells, if you see them while doing the differential, will change the final leukocyte count? Lymphocytes Platelets Reticulocytes Nucleated Red Blood Cells

Nucleated Red Blood Cells

A dog presents for stumbling, has a head tilt, and you notice his eyes are moving back and forth rapidly in a horizontal motion. What is the term for this type of eye movement? Mydriasis Miosis Ataxia Hypermetria Nystagmus

Nystagmus This type of eye movement is termed nystagmus. It is most commonly seen with brain lesions or with vestibular disease. Mydriasis is the term for dilated pupils; miosis is the term for constricted pupils. Ataxia is the term for an unsteady gait. Hypermetria is an exaggerated gait caused most often by cerebellar disease.

You are getting the weight of a patient in the exam room and are going to write it in the record. What part of a SOAP record would you put the weight in? P S O A

O

A cat with type A blood is in need of a transfusion, what type of blood is it able to receive? Only AB blood Both AB and A blood Only A blood Only B blood

Only A blood A cat with type A blood can only receive type A blood. Type B cats can only receive type B blood, while AB cats are able to receive type A or type B blood.

You are instructed to give a patient fluids per os. How are these administered? Orally Into the bone Rectally Intra-arterial Intravenous Subcutaneous

Orally Per os, often abbreviated P.O., means orally or by mouth.

Bone is made by which type of cell? Osteoclasts Chondrocytes Osteocytes Osteoblasts

Osteoblasts Osteoblasts are mononucleate cells that are responsible for bone formation and come from the bone marrow. Osteocytes are actually cells inside of the bone. Osteoclasts are large cells that dissolve the bone. Chondrocytes are cartilage cells.

What hormone stimulates uterine contraction? Oxytocin Luteinizing hormone Antidiuretic hormone Prolactin

Oxytocin Oxytocin is the hormone responsible for uterine contraction and is sometimes administered to animals with dystocia to help expel the fetus. Prolactin stimulates lactation, and oxytocin causes the milk letdown. Luteinizing hormone (LH) stimulates ovulation. Antidiuretic hormone results in water re-absorption when released.

Which of the following tests amplifies DNA? Cytology Histopathology ELISA (Enzyme-linked immunosorbent assay) PCR (Polymerase chain reaction) Flow cytometry

PCR (Polymerase chain reaction) PCR is a method of amplifying a segment of nucleic acid (DNA or RNA) segments. ELISA is a test that detects the presence of antibodies or antigens. Flow cytometry is a technique used to sort cells. Cytology and histopathology are qualitative tests to look at cellular morphology under the microscope.

What is another common name for herpes infection in psittacine birds? Pacheco's disease Chlamydiosis Newcastle disease West Nile

Pacheco's disease

What is the proper terminology to describe a laceration under the right forelimb paw? Palmar aspect Posterior aspect Anterior aspect Plantar aspect

Palmar aspect

A dog is very weak in the hindend but is able to stand with support. The dog is being supported under the abdomen, and the paws are individually turned over so that the dorsal paw is touching the ground. The dog is not able to correct this posture. This is an evaluation of: Malalignment of the vertebral column Spinal pain Proprioception Mentation

Proprioception

A new client presents her puppy and is inquiring about crate training. You tell her that crate training can be used for all of the following except: Potty-training Sleeping Traveling Punishment

Punishment The puppy crate should be a place of comfort and a place the pet seeks to go. Punishing a puppy with the crate may result in a negative association with the crate.

When running a PCV, why are the red cells at the bottom and white cells at the top of the hematocrit tube? Red cells are heavier Hemolysis makes red cells sink White cells are smaller Red cells have a low specific gravity White cells are heavier

Red cells are heavier

You are instructed to give a medication which you are told is an antipyretic. What will this medication do? Provide sedation Reduce gastric acid Prevent vomiting Reduce fever

Reduce fever

Which of the following would be elevated in a pet with regenerative anemia? Reticulocyte count Lymphocyte count Hemoglobin concentration Hematocrit

Reticulocyte count

What is the term commonly used to describe diarrhea in calves? Trots Creeps Scours Runs

Scours Scours is the common term for diarrhea in calves. It can be a serious health problem in calves and is best prevented by good husbandry.

This canine patient presents following his neuter surgery. He was neutered at the local animal shelter and the owner did not follow instructions to use an E-collar. He now has an open wound and a swollen scrotum. The scrotum is filled with blood due to self trauma. This could be called which of the following? Scrotal ablation Scrotal atrophy Scrotal torsion Scrotal hematoma

Scrotal hematoma

What does the term "euthanasia" literally mean? The good death Stopping of the heartbeat To stop the breath of life Cardiac arrest

The good death

Which bone is not associated with the forelimb? Tibia Metacarpal Ulna Humerus Radius

Tibia The tibia is a bone of the hindlimb also known as the "shinbone". The tibia is located distal to the femur and proximal to the tarsus.

Which of the following is a chelonian? Rabbit Goat Tortoise Bird

Tortoise Turtles and tortoises are from the order Chelonia, and are often referred to as chelonians.

Rostral indicates which direction? Towards the back Towards the tail Towards the nose Towards the head

Towards the nose

Keratoconjunctivitis sicca (dry eye) can result from the use of which medication? Enalapril Prednisone Enrofloxacin Cyclosporine Trimethoprim-sulfa (TMS)

Trimethoprim-sulfa (TMS)

When applying a bone plate to fix a fracture, which of the following instruments would be least useful? Trocar Tap Point-to-point forceps Drill guide Screw driver

Trocar

The eardrum is also known as which of the following? Ortolani Otic diaphragm Percussionum Tympanum

Tympanum

Gout is an accumulation of which of the following? Uric acid Bilirubin Cholesterol Albumin Urea

Uric acid Gout occurs when uric acid accumulates, usually due to poor nitrogen metabolism and excretion.

The broad ligament attaches which structure to the body? Spleen Uterus Bladder Intestines

Uterus

Which drug can be given as a tablet into the conjunctival sac in the eye to induce vomiting? Apomorphine Morphine Hydrogen peroxide Xylazine

Apomorphine Apomorphine can be given as an injectable or in tablet form that can be dissolved into the eye to induce emesis. Giving it orally is not as effective. It acts by dopamine stimulation.

If a dog has atrophy of its muscles, what appearance will the muscles take on? Appear sunken with loss of tone Have calcification and be hard to the touch Appear larger than normal Be irregular and lumpy on palpation

Appear sunken with loss of tone

What is the average gestation length of a cow? Approximately 285 days Approximately 350 days Approximately 200 days Approximately 120 days

Approximately 285 days Cows have an average gestation length of 285 days (about 9 months). The average gestation length of dogs and cats is 63 days. Horses have a gestation length of approximately 340 days. Although the average for horses is approximately 11 months, it can vary by as much as 30 days.

If an owner is instructed to give a medication "prn", how often should it be given? Every 4 hours Hourly As needed Once daily Continuous via infusion

As needed

A 10-year old Golden Retriever presents with a distended abdomen, and when you touch his belly you notice a fluid wave. What is the medical term for free fluid in the abdomen? Ascites Pericardial effusion Pleural effusion Hemoabdomen

Ascites Ascites is free fluid in the abdomen. Pleural effusion is fluid around the lungs in the pleural space. Pericardial effusion is fluid surrounding the heart in the pericardial sac. Hemoabdomen is one type of ascites, specifically blood in the abdomen.

While performing an epidural on a dog, your needle begins to fill with a clear pale straw colored fluid, what is the appropriate action? This is normal and it is ok to aspirate, if there is no blood give the full calculated dose of anesthetic. You must withdraw the needle completely and start over. Aspirate and if there is no blood it is ok to administer half of the calculated dose of anesthetic. You must withdraw the needle completely and no further attempts should be made.

Aspirate and if there is no blood it is ok to administer half of the calculated dose of anesthetic. The fluid is cerebrospinal fluid (CSF); this means you have punctured the ligamentum flavum and the needle has entered the subarachnoid space. Only half of the dose should be administered.

Barium for an upper GI radiograph series should not be given to a patient with which of the following? Aspiration pneumonia Partial small intestinal obstruction Dehydration Diarrhea Renal insufficiency

Aspiration pneumonia Patients with aspiration pneumonia or who are at risk for aspiration pneumonia should not be given oral barium for an upper GI series due to the risk of aspiration of the barium. Dehydration and renal failure are contraindications for other types of contrast such as iodide contrast agents, which are hyperosmolar and can cause osmotic diuresis and diarrhea. Barium is commonly given to animals with partial obstructions and/or diarrhea as part of their diagnostic work-up and is not contraindicated.

Which heart rhythm is a flatline, or a state of no cardiac electrical activity? Asystole Ventricular fibrillation Atrioventricular block Arrhythmia

Asystole Asystole is defined as the complete absence of electrical activity in the myocardium.

Which joint allows rotational movement of the head from side to side (from right to left or left to right)? Coxofemoral Atlantooccipital Atlantoaxial Scapulohumeral

Atlantoaxial The atlantoaxial joint is a pivot joint which allows rotation of the head from side to side. It is located between the atlas and axis. The atlantooccipital joint provides the up and down motion of the head and is located between the occipital condyles of the skull and C1 (atlas). The coxofemoral joint is the hip joint. The scapulohumeral joint is where the scapula joins with the proximal humerus.

There is an enzyme in about 40% of rabbits that neutralizes a common drug. What drug is not used in rabbits because of this? Butorphanol Enrofloxicin Atropine Metoclopramide

Atropine Atropine esterase is produced by rabbits and acts to neutralize atropine.

As teeth make contact and grind together they gradually become worn. What is this process known as? Dentition Attrition Hypertrophy Thinning

Attrition

When anesthetizing a rabbit for anesthesia, which vein is most often used? Auricular Lateral saphenous Medial saphenous Cephalic Jugular

Auricular The auricular (ear vein) is most often used in rabbits when IV access is necessary. For short procedures in private practice, subcutaneous or intramuscular injectable anesthetics are are often given, or a gas induction is used.

Agglutination of blood and spherocytes seen on a blood smear are indicative of which type of disease? Rickettsial Auto-immune Bacterial Viral

Auto-immune Agglutination occurs in immune-mediated disorders due to antibodies coating the red blood cells. This results in clumping of the erythrocytes. Spherocytes are often seen with Immune Mediated Hemolytic Anemia (IMHA).

What is the name for a steam sterilizer? Radiator Autoclave Filtration chamber Steamvac

Autoclave There are physical and chemical ways to sterilize. Physical means include filtration, radiation, and heat. Filtration separates particles from gases or liquids and is used for sterilizing many pharmaceuticals. Gloves and suture materials are sterilized via radiation, destroying microorganisms without causing high temperatures that could damage the materials. Heat is the most common means of sterilization. This includes autoclaving which (is steam under pressure). This is used to sterilize surgical packs/instruments.

Which of the following laboratory values are most associated with the kidneys? Amylase and lipase BUN and creatinine ALT and ALP Albumin and globulin

BUN and creatinine The main renal values are BUN and creatinine. ALT and ALP are liver enzymes, amylase and lipase are pancreatic enzymes, and albumin and globulin are proteins.

During surgery, which is NOT considered sterile? Surgical instruments Surgical gloves Surgical drape Back of surgical gown

Back of surgical gown The back of the surgical gown is not considered sterile. Surgeons should pass each other face to face or back to back if needed to maintain sterility of the front of the gown.

What instrument is being used to temporarily appose the skin edges of this tail wound in a cat? Allis tissue forceps Halsted mosquito forceps Backhaus towel clamp Brown-Adson forceps

Backhaus towel clamp

A 3-month old kitten presents for mucoid diarrhea and anemia. The clinic where you work has recently seen several other puppies and kittens with similar clinical signs that were infected with Strongyloides stercoralis. You look up and find that this parasite is passed in the feces in the L1 larval form. What is the best way to recover and identify this parasite? Fecal sedimentation Baermann fecal technique Direct fecal smear Fecal flotation

Baermann fecal technique

You are examining a blood agar plate from a submitted bacterial culture and see an area of complete hemolysis, leaving a clear zone around a bacterial colony. What type of hemolysis is this? Alpha hemolysis Gamma hemolysis Delta hemolysis Beta hemolysis

Beta hemolysis

What is the primary defense method for a pig? Biting Striking Kicking Running

Biting

Which of the following is the best method for killing Parvovirus? Direct sunlight Bleach Alcohol Hydrogen peroxide

Bleach

What health problem might you see in a dog who has an elevated prothrombin time (PT) on his lab report? Seizures Bleeding Hives Alopecia

Bleeding Prothrombin time (PT) is a blood test that measures how long it takes for blood to clot and is a test commonly used to check for bleeding problems. Prothrombin, or factor II, is one of the clotting factors made by the liver. If the PT is increased, this means the blood takes longer to clot and could lead to bleeding or bruising.

You are asked to run an Azostix strip on a dog prior to anesthesia. What does this test for? Creatinine Blood urea nitrogen Amylase Alanine aminotransferase Blood glucose

Blood urea nitrogen The Azostix tests for BUN. It is a small strip to which a drop of blood is applied and gives a result in one minute.

What is a good way to assess daily changes in hydration status of a patient that is in the hospital for several days or longer? Respiratory rate Scleral color Body weight Heart rate

Body weight Body weight is a good way to assess hydration of a patient. It is especially helpful when monitoring hydration status of hospitalized patients that are weighed daily. Tacky or dry mucus membranes and increased skin tenting are clinical signs of dehydration. Heart rate is often increased when patients are dehydrated, but increased heart rate can also be from other causes, such as stress.

Which of following typically appear hyperechoic (bright white) on ultrasound imaging? Bone and gas Renal medulla and gas Colon and liver Bone and fluid Spleen and lung Liver and spleen

Bone and gas

What tissue is the MOST sensitive to radiation therapy? Soft Tissue Cartilage Muscles Bone marrow

Bone marrow

Estrogen toxicity most commonly causes which side effect? Hypoglycemia Seizures Bone marrow suppression Hypertension

Bone marrow suppression

Which vaccination is most commonly given the intranasal route? Periodontal Distemper Parvo Bordetella Rabies

Bordetella Bordetella bronchiseptica is a common cause of kennel cough. An intranasal vaccine against Bordetella is widely used. There is also an injectable vaccine available for Bordetella.

A 2-year old bulldog presents with respiratory distress, specifically on inhalation, and stertor. What, of the following, would be the most common cause of respiratory distress in this patient? Tracheal mass Brachycephalic syndrome Laryngeal paralysis Tracheal foreign body Asthma

Brachycephalic syndrome

What part of the brain controls heartbeat, respiration, and blood pressure? Cerebellum Brain stem Hypothalamus Cerebral cortex

Brain stem The brain stem is made of the midbrain, pons, and medulla. It controls basic vital functions such as breathing, heartbeat, and blood pressure.

This instrument is commonly used in surgery for grasping tissues. What is the name of this instrument? (look like tweezers) Brown-Adson forceps Kelly forceps Debakey forceps Allis tissue forceps

Brown-Adson forceps

When dogs are mating, the male dog "ties" with the female so that the penis cannot be removed from the vagina for a period of time. This is due to the swelling of which structure? Os penis Bulbis glandis Prostate Scrotum Epididymis

Bulbis glandis The bulbis glandis is erectile tissue on the penis that swells during mating and locks or ties the penis in the vagina.

Which cranial nerve is known for three major branches? CN XII CN I CN V CN XI

CN V Cranial nerve V (five) is the Trigeminal nerve and has three major branches: First branch: Ophthalmic nerve- sensory to the eye socket and its contents, the anterior nasal cavity, and the skin of the nose and forehead. Second branch: Maxillary nerve- sensory to maxilla, nasal cavity/sinuses, palate, and part of the face. Third branch: Mandibular nerve (the largest branch)- sensory fibers to the lower jaw, the floor of the mouth, the anterior two-thirds of the tongue, the lower teeth, and supplies motor fibers to the muscles of mastication (note: that this is the only branch that supplies motor function).

When evaluating urine sediment, a clear-colored square crystal with an X across the surface is noted throughout the sample. What crystal is this likely to be? Ammonium biurate Calcium oxalate Struvite Silica

Calcium oxalate

For which very contagious and rapidly fatal disease are ferrets vaccinated against? Feline panleukopenia Canine distemper Canine adenovirus Feline rhinotracheitis Canine parvovirus Canine bordetella

Canine distemper

When performing CPR in the neonatal calf which are the two most critical procedures to begin? Establishing venous access and administration of drugs Establishing an airway and venous access Cardiac compressions and administration of drugs Cardiac compressions and establishing an airway Establishing venous access and cardiac compressions

Cardiac compressions and establishing an airway Establishing an airway and adequate cardiac compression rate first contribute to positive outcomes when performing CPR in the neonatal calf.

Which species commonly has laryngospasm when you are trying to place an endotracheal tube during anesthetic induction? Horse Cow Dog Cat

Cat

Which of the following species is an induced ovulator? Cat Dog Horse Human

Cat Cats, rabbits, and ferrets are induced ovulators. This means they will ovulate after being bred. Dogs have a diestrous cycle, meaning they cycle in the spring and fall. Animals that cycle continuously throughout the year have polyestrous cycles. Horses are seasonally polyestrous and cycle continuously during specific seasons of the year. Humans are spontaneous ovulators and will ovulate regardless of mating.

A 2-year old male neutered Great Dane presents on emergency with a distended abdomen and a history of unproductive retching. A catheter must be placed to provide immediate fluid therapy. What is the best site for catheterization? Lateral saphenous vein Femoral artery Cephalic vein Dorsal pedal artery Jugular vein

Cephalic vein

Which of the following drugs should not be given to any animal intended for human consumption? Amprolium Prostaglandin F2-alpha Chloramphenicol Oxytetracycline

Chloramphenicol

Many factors can contribute to insulin resistance in pets. Which of the following can lead to insulin resistance and type II diabetes? Loss of acinar cells of the pancreas Increased drinking and urinating Chronic use of corticosteroids High-protein diet

Chronic use of corticosteroids Type II diabetes occurs mostly due to insulin resistance and can occur from things such as chronic use of corticosteroids, obesity, or pancreatitis. Acinar cells secrete digestive enzymes. Beta cells of the pancreas actually make insulin, and degeneration of these cells can lead to type I diabetes. Increased drinking and urinating is a clinical symptom of diabetes but doesn't cause insulin resistance. A high-protein, low-carbohydrate diet actually helps with glycemic control.

Which chemotherapy drug is contraindicated for use in cats? Vincristine Cisplatin Prednisone Doxorubicin L-Aspariginase

Cisplatin Cisplatin cannot be used in cats as it causes pulmonary toxicity. Remember the phrase "Cisplatin splats cats"

What is the single most important thing to do when removing a stomach tube from a patient? Twist the tube as you are pulling it out Flush the tube with 5-10 ml of water Clamp or pinch off stomach tube prior to removing Advance the tube 3cm infuse 10ml of air and then remove

Clamp or pinch off stomach tube prior to removing Clamping or pinching off a stomach tube just prior to removing the tube is essential to prevent spillage of the contents within the tube as it is being removed. Any inadvertent spillage could result in contents entering the airway, subsequently causing aspiration pneumonia. None of the other answer choices effectively prevent this potential complication.

Some bacteria produce beta-lacatamases that destroy or inactivate penicillins. What beta-lactamase inhibitor is added to amoxicillin to prevent this from occurring? Chloride Sulfamethoxazole Clavulanic acid Procaine

Clavulanic acid Clavulanic acid is a beta-lactamase inhibitor added to amoxicillin to prevent some types of antibiotic resistance. Sulfamethoxazole is a sulfonamide bacteriostatic antibiotic typically given with trimethoprim. Sulfa medications can cause side effects such as dry eye, acute hepatitis, and bone-marrow toxicity. Procaine is a local anesthetic, historically added to penicillin to reduce the pain associated with the injection. Clavulanic acid contains a potassium salt, but not chloride.

A very young puppy presents with a history of milk coming from his nose and seems to cough and gurgle when he eats. What deformity commonly results in this presentation? Cleft palate Esophageal stricture Stenotic nares Occluded sinus Persistent right aortic arch

Cleft palate

Which intestinal parasite is NOT considered zoonotic to humans? Toxocara Ancylostoma Coccidia Giardia

Coccidia Coccidia are host-specific. Cystoisospora (the coccida found in dogs and cats) is not infective to humans. The other parasites listed are zoonotic.

Which of the following are intended to shield the user from the primary X-ray beam? Lead gloves Collimator Dosimeter badge Thyroid shield Apron

Collimator Only the collimator contains enough lead to shield the primary radiation beam. The other equipment is only intended to absorb scatter radiation. Even when wearing appropriate radiation-protective equipment, no part of the technician should ever be in the primary field of a radiograph.

A dog has ruptured a chordae tendinae. What condition can this lead to? Congestive heart failure Paralyzed diaphragm muscle Chronic osteoarthritis and lameness Incontinence

Congestive heart failure The chordae tendinae are cord-like tendons that connect the papillary muscles to the tricuspid valve and the mitral valve in the heart. If this structure ruptures, the valves can no longer function properly, and this can lead to congestive heart failure.

A subtotal colectomy in which a small amount of colonic contents spills into the abdomen in a cat is an example of which of the following? Infected (dirty) surgery Clean surgery Contaminated surgery Clean-contaminated surgery

Contaminated surgery

Through which structures does light pass as it moves from the outside to the inside of the eye? Cornea, pupil, lens, retina Lens, retina, cornea, vitreous Retina, lens, pupil, cornea Cornea, iris, retina, lens

Cornea, pupil, lens, retina

You are performing a urinalysis and identify this six-sided crystal (Hexagon). What type of crystal is this? Cystine Calcium oxalate Struvite Urate

Cystine

These parasites are potential zoonotic pathogens EXCEPT for which of the following? Giardia spp. Cryptosporidium parvum Toxacara canis Demodex canis Toxoplasma gondii

Demodex canis Toxoplasma can cause birth defects in humans. Cryptosporidium and Giardia can cause severe diarrhea. Toxocara canis can cause visceral larva migrans. Demodex is a mite that commonly lives in the hair follicles. It is not contagious to humans.

Occasionally, animals will present with dystocia. What does the term dystocia mean? Pus-filled uterus Difficult birth Distended abdomen Fluid around the brain

Difficult birth

Because of its potential side effects that can occur due to toxic serum levels, a blood level of which medication should be monitored closely? Spironolactone Pimobendan Furosemide Digoxin

Digoxin

Which of the following medications is commonly found over-the-counter (OTC)? Doxycycline Hydrocodone Pimobendan Diphenhydramine

Diphenhydramine

Colic is a common condition in horses. Which of the following is not a common cause of colic? Intestinal incarceration Displaced abomasum Enterolithiasis Intestinal volvulus

Displaced abomasum Horses are not ruminants and therefore do not have an abomasum. The abomasum is the fourth stomach in ruminants and functions in a similar fashion to a monogastric stomach. Enteroliths are stones that can form in the gastrointestinal tract that result in an obstruction. Intestinal incarceration is the trapping of intestines through a small opening. Finally, an intestinal volvulus occurs when a section of intestine spins about its axis and occludes its own blood supply.

To obtain an accurate blood pressure in this patient, the doppler crystal should be placed: Proximal to the blood pressure cuff but distal to the metatarsal pad Distal to the blood pressure cuff but proximal to the metatarsal pad Beneath the blood pressure cuff In the webbing between digits 3 and 4

Distal to the blood pressure cuff but proximal to the metatarsal pad To obtain an accurate blood pressure measurement, the crystal must always be placed distal to the cuff so that blood flow is properly occluded.

Which of these are used in the major cross-match for blood products to be transfused to a dog? Donor red blood cells, donor plasma Donor red blood cells, recipient plasma Recipient red blood cells, donor plasma Recipient red blood cells, recipient plasma

Donor red blood cells, recipient plasma

The pancreas connects to which part of the intestine? Duodenum Ileum Jejunum Cecum

Duodenum The pancreas connects to the duodenum (the first segment of the small intestine). It secretes digestive enzymes into the duodenum via the pancreatic duct.

What are the three layers of the meninges? Cortex, brainstem, cerebellum Limbic layer, cortex, cerebrospinal fluid Dura mater, arachnoid mater, pia mater Dura mater, cerebrospinal fluid, cortex

Dura mater, arachnoid mater, pia mater The meninges are layers that cover the brain and spinal cord. The meninges consist of the dura mater, the arachnoid mater, and the pia mater. The meninges, along with the cerebrospinal fluid, function to protect the central nervous system (CNS).

A patient presents for difficulty swallowing and is found to have a bone lodged at the back of its throat during physical exam. Which of the following is the term for this presenting complaint? Ptyalism Dysphagia Stomatitis Anuria

Dysphagia The term for difficulty swallowing is dysphagia. It may be caused by an oral or esophageal foreign body, esophagitis, neurologic disease (myasthenia gravis), etc. Anuria is the term for absence of urination. Ptyalism is drooling and is sometimes observed in patients with dysphagia. Stomatitis is inflammation of the soft structures in the oral cavity (cheeks, gums, tongue, etc.).

You are preparing to collect abdominal fluid from a horse presented for signs of colic. Collection of abdominal fluid for cytologic examination should be collected in what type of sample tube? Clot tube (red top) EDTA (purple top) Sodium heparin (green top) Sodium citrate (blue top)

EDTA (purple top) Cells from fluid samples are best preserved and prevented from clotting in an EDTA tube.

What is the most sensitive and preferred test for detecting Giardia? ELISA Direct smear PCR Fecal flotation with zinc centrifugation

ELISA The most sensitive test for detecting Giardia is the ELISA. This will detect Giardia in greater than 95% of cases. The Fecal flotation with zinc centrifugation will only detect Giardia less than 70% of the time. A direct smear can be used to look for trophozoites but is not highly sensitive, and multiple smears may be needed to find the organism.

Coprophagy is the term used to describe an animal that is doing what? Eating feces Regurgitating and then eating what was regurgitated Digging holes Straining to defecate

Eating feces An animal that is eating its own feces or feces of other animals is said to be coprophagic. Coporophagy is a normal behavior of rabbits and many rodents.

A dog has dislocated his hip joint and the doctor has reduced the dislocation. What can be used to hold the hip in place? Ehmer sling External fixator Strict cage rest Robert Jones bandage

Ehmer sling

Which of the following should be done with a dog after a myelogram to decrease the risk of procedure-related complications? Stabilize the neck Elevate the head Place a chest tube Administer a bolus of crystalloid IV fluids

Elevate the head After a myelogram, dogs have a risk of having a seizure due to the contrast administration. The risk can be decreased by keeping the head elevated after the procedure.

What instrument is used to perform a castration in horses? Freedom forceps Rongeurs Kelly carmalt Emasculator

Emasculator The correct answer is an emasculator. This device crushes the spermatic cord as well as cuts it. The crushed end stays with the patient to prevent hemorrhage. Care must be taken when applying the device to ensure that the crushed end stays with the patient, otherwise major bleeding can occur.

What is phagocytosis when discussing physiology of cells? Engulfing large particles such as bacteria into the cell The passive process of diffusion of small particles across the cell membrane Uptake of liquids into the cell Pushing waste out through the cell membrane

Engulfing large particles such as bacteria into the cell "Phago" means eating, and phagocytosis is the process of a cell engulfing SOLID particles into the cell such as bacteria. Endocytosis is the uptake of liquids into the cell.

You help take an impression smear from the lip of a cat and see the following cells on cytology. What type of cells are these? (pink speckled with purple blobs) Basophils Neutrophils Mast cells Eosinophils

Eosinophils

Your neighbor just bought some pet mice and asks you about breeding them. He wants to know how long their estrus and gestation are. Which of the following is most accurate? Mice are induced ovulators, gestation 12-14 days Estrus 26-28 days, gestation 23-25 days Estrus 14-18 days, gestation 18-20 days Estrus 4-6 days, gestation 19-21 days

Estrus 4-6 days, gestation 19-21 days Mice come into estrus every 4-6 days and gestation average is around 20 days.

What does the abbreviation "q" stand for on a prescription? Give Discontinue Every Orally

Every "q" means every. It may say something such as: Give 1 tablet q 12 hours for 14 days. This would mean give 1 tablet every 12 hours for 14 days. The abbreviation for orally is "P.O." (per os).

Why should atropine, a competitive antagonist of acetylcholine receptors, be used cautiously in horses (i.e. what is a major side effect of the drug)? Excessive use can result in intestinal stasis (colic) Excessive use can result in diarrhea Excessive use can result in bradycardia Excessive use can result in intense constriction of the pupil

Excessive use can result in intestinal stasis (colic) Atropine is commonly used topically to dilate the pupil in horses with uveitis; with excessive administration, atropine can cause intestinal motility to decrease, possibly resulting in abdominal pain (colic) in the horse. Atropine is also used to increase the heart rate during general anesthesia.

Aside from diabetes, what is another cause for glucosuria in a cat? High carbohydrate diet Estrus Dehydration Excitement

Excitement

Which of the following is a common cause of epistaxis in horses? Exercise-induced pulmonary hemorrhage Tooth abscess Allergic rhinitis Sinus infection

Exercise-induced pulmonary hemorrhage Exercise-induced pulmonary hemorrhage is a common cause of epistaxis (bloody nose) in the horse. The other choices do not commonly cause epistaxis.

A patient who has been diagnosed with pancreatitis should avoid foods that are high in which of the following? Water Fat Protein Carbohydrates Vitamins

Fat Pancreatitis means inflammation of the pancreas, the glandular organ that secretes enzymes needed to digest food. When something causes these enzymes to be activated prematurely, they can actually begin to digest the pancreas itself, resulting in pain and inflammation. There is evidence that feeding a low protein, high fat diet can induce pancreatitis.

When performing a median sternotomy as seen in the image below, the surgeon will use the retractor seen in the image (thick metal with knobs sticking out on top). What is its name? Finochietto retractor Balfour retractor Gelpis retractor Army-Navy retractor

Finochietto retractor A Finochietto retractor is specifically designed for rib retraction. Balfour retractors are designed for retraction of the abdominal wall. Gelpis retractors are used to increase visibility in small areas such as the knee joint. Army-Navy retractors are very versatile and can be used to retract in many situations.

Flea allergy dermatitis is a very common cause of pruritus in dogs in endemic areas. This is usually an allergy to what? Flea feces Flea saliva Flea cytoskeleton (chitin) Flea eggs

Flea saliva Many pets are allergic to flea saliva and it causes a dermatologic hypersensitivity. It may manifest with symptoms of pruritus, erythematous skin, secondary pyodermas, and hot spots.

You are working in the ICU; a dog on a high rate of fluids has a very sudden drop in urine production in his urine collection bag. What do you do first? Immediately notify the veterinarian on call immediately Perform a urine sediment examination Flush the urinary catheter with sterile saline Run a BUN and creatinine to check kidney function

Flush the urinary catheter with sterile saline First, flush the catheter and make sure the line didn't get kinked, etc. If the urine production had been good and suddenly urine production slowed dramatically, it is likely a result of some mechanical issue. If the catheter flushes normally and all appears OK, call the veterinarian immediately.

What is Mycology the study of? Fungi Parasites Bacteria Muscles

Fungi Mycology is the branch of biology that is the study of fungi. The study of muscles is myology (with no c). The study of bacteria is bacteriology. The study of parasites is parasitology.

You are working with an aggressive chihuahua, and the standard muzzles available in the practice will not fit this dog. Which alternative could you try? Cover the dog's head with a towel No muzzle, just grasp firmly around the neck region so the dog cannot turn its head to bite Use your hand to firmly close the dog's mouth during examination Gauze muzzle

Gauze muzzle

What term describes an enlarged thyroid gland? Goiter Thyroidosis Thyroiditis Hypothyroidism

Goiter Goiter is the abnormal gross enlargement of the thyroid gland. Thyroiditis is inflammation of the thyroid tissue. Thyroidosis is not a medical term. Hypothyroidism is inadequate production of thyroid hormone.

Which of the following is accurate about a cryptorchid dog? Has one testicle larger than the other Has both male and female sexual organs Has an additional testicle Is prone to prostate infections Has one or both testicles retained and not in the scrotum

Has one or both testicles retained and not in the scrotum

The surgeon clamps off a bleeding blood vessel. It could be said that the surgeon achieved which of the following? Hemolysis Fibrinolysis Coagulation Hemostasis

Hemostasis

You are checking the fax machine and get a report from the local emergency clinic that a patient from the hospital where you work presented overnight for severe urticaria. What did this patient have? Tremoring Hives Diarrhea Limping Nasal discharge

Hives

The tibiotarsal joint may also be referred to as which of the following? Stifle Elbow Hock Shoulder

Hock The tibiotarsal joint (the joint between the distal tibia and the tarsus) is also commonly called the hock.

Which of the following types of radiographs is helpful in confirming the presence of free air or fluid, if it is suspected based on survey films (DV, VD and lateral projections)? Bisecting angle technique Vertical beam Skyline view Horizontal beam

Horizontal beam Standing a patient upright and taking a horizontal beam radiograph can be helpful in confirming the presence of air or fluid in a body cavity (thorax, abdomen) because this position allows fluid to settle ventrally and air to rise dorsally. The bisecting angle technique is a radiographic technique used to prevent distortion when it is not possible to place the film parallel to the structure of interest and perpendicular to the x-ray beam (common in dental radiography). A skyline view is a technique used to decrease superimposition of structures, usually in the equine carpus.

The viscosity of synovial fluid is directly related to which component in the fluid? Acetic acid Hyaluronic acid Cartilage Erythrocytes

Hyaluronic acid Hyaluronic acid is found in all connective tissues. It is highest in the joints where it acts as a cushion. In inflammatory conditions, the fluid gets thinner, and thus the overall quality of the joint fluid decreases. To test the viscosity or "stickiness" of the fluid, take a drop of synovial fluid between two fingers, press and pull your fingers apart. The stringy component (viscosity) of the fluid is related to the hyaluronic acid component.

What immunoglobulin is the only one to pass through the human placenta, thereby providing protection to the fetus in utero? IgE IgG IgA IgM IgD

IgG Antibodies come in different varieties and differ in their biological properties, functional locations, and ability to deal with different antigens. IgG is the only class of antibody to cross the placenta from mother to fetus; it is also the most abundant of all antibodies.

Colostrum provides passive immunity to the neonate because it is rich in which of the following? Folate Electrolytes Immunoglobulins Antigens

Immunoglobulins

What causes Metabolic Bone Disease (MBD) in tortoises? UVB light Warm temperature A diet mostly of fresh greens Improper calcium/phosphorus ratio Sun exposure

Improper calcium/phosphorus ratio If the dietary calcium is too low, it will cause malformation and softening of bones with deformed shells (called pyramiding). Typical tortoise diets are low in calcium and high in phosphorus, hence the need for most tortoises to receive a calcium supplement. Too much calcium or over-supplementation may cause zinc, copper, and iodine deficiency, and could lead to bladder stones. If the temperature is too low, it will hinder the ability to digest food properly. UVB light or natural sunlight is needed to absorb and metabolize calcium. There are other factors that contribute to MBD, but improper care and diet are the main factors.

Where is the thymus located? In the brain Ventral neck region In the cranial mediastinum in the chest In the abdomen just above the kidneys

In the cranial mediastinum in the chest The thymus is an immune organ located in the mediastinum. It extends from the thoracic inlet to the pericardium upon which it is molded. The thyroid gland is located in the ventral neck region. The adrenal glands are located just above the kidneys. The hypothalamus is located in the brain.

What does the word anhidrosis mean? Lack of drinking Hidden disease Without tears Inability to sweat

Inability to sweat The cause is unknown, but some horses lack the ability to sweat. This condition in known as anhidrosis. Keep horses with anhidrosis as cool as possible during hot or humid weather. Use of mist, sponging, fans, shade or stall rest can be helpful during exposure to heat.

You pull a 6 year old film out of a folder to compare to radiographs that you took today. The old film appears to have yellow-brown staining. What is the likely cause of this? Incomplete drying of the film Film was left in developer too long Storing the film at a hot temperature Incomplete washing of the film

Incomplete washing of the film Yellow-brown staining over time is the result of incomplete removal of chemicals (usually from the fixer) during the final washing step in film development. A film that is not dried completely commonly has artifacts around the edges of the film.

What is meant by a "left shift" when describing a complete blood count? Decreased numbers of neutrophils in the blood Increased numbers of lymphoblasts in the blood Increased numbers of immature neutrophils in the blood Increased total number of white blood cells Decreased numbers of reticulocytes in the blood

Increased numbers of immature neutrophils in the blood A left shift refers to an increase in the number of immature neutrophils, usually bands, in the blood. This occurs when the bone marrow is increasing neutrophil production in response to inflammatory signals.

Which of the following nerve blocks can be used to decrease anesthetic requirements and provide analgesia for a dental extraction of a maxillary tooth? Infraorbital block Inferior alveolar block Mental nerve block Auriculopalpebral block

Infraorbital block The infraorbital block or maxillary block are local anesthetic techniques used to decrease anesthetic requirements and provide analgesia for procedures involving the maxilla. The inferior alveolar block and mental nerve block are local anesthetic techniques for the mandible.

Where are the anal sacs located? The dorsal external anus region In the large intestine just caudal to the cecum Inside the anus at the 12 and 6 o'clock positions Inside the anus at the 4 and 8 o'clock positions

Inside the anus at the 4 and 8 o'clock positions

An example of a clean-contaminated surgery would be which of the following? Ovariohysterectomy Nephrectomy Splenectomy Intestinal foreign body removal

Intestinal foreign body removal

Which of the following routes of administration would give a drug the highest bioavailability? Oral Intravenous Subcutaneous Intramuscular

Intravenous

Which is NOT true of cytology? It is read under the microscope One method of sample collection is a fine needle aspirate It evaluates tissue architecture It can be useful to help differentiate a tumor from inflammatory tissue

It evaluates tissue architecture Tissue architecture is studied via histopathology. Cytology is only the study of a smaller number of cells, not intact tissue.

Water comes from the ultrasonic scaler for which purpose? It prevents dulling of the instrument It smoothes the surface of the tooth It helps cool the instrument tip It prevents introduction of bacteria into the gumline

It helps cool the instrument tip The ultrasonic scaler uses water to cool the instrument tip and to flush debris from the tooth as it is scaled away. By cooling the instrument tip, the water helps to prevent thermal damage to the tooth. The polisher is the instrument used to help smooth the tooth surface.

Which of the following is NOT true concerning a penrose drain? It is also referred to as a closed suction drain Fluid drains around the penrose tube, not through it It is a passive drain It is a soft, thin-walled, collapsible, latex rubber tube

It is also referred to as a closed suction drain

Which of the following is TRUE regarding the presence of crystals in a dog or cat urine sample? It is never normal Crystals are not present in urine It is always normal It is usually normal

It is usually normal

A six month old Collie comes into your clinic for a technician exam and deworming. What antiparasitic drug should be used with caution in this breed? Sulfadimethoxine Pyrantel pamoate Propofol Fenbendazole Ivermectin

Ivermectin Collies, Australian Shepherds,and Shelties have a greater sensitivity to Ivermectin thought to be caused by a greater permeability in the blood-to-brain barrier. These breeds may show symptoms of toxicity even at therapeutic doses.

An arthrogram involves contrast administration into what region? Bladder Esophagus Subarachnoid space Epidural space Subdural space Joint

Joint An arthrogram involves contrast administration into a joint. A myelogram involves contrast administration into the subarachnoid space. Contrast is injected into the bladder for a cystogram and ingested into the esophagus for an esophagram.

An 8-year old cat suffering from intestinal lymphoma has been prescribed TPN. Which of the following is necessary for the administration of TPN? Esophagostomy tube Subcutaneous catheter Cephalic catheter A-line Jugular catheter

Jugular catheter TPN stands for total parenteral nutrition. TPN solutions contain 10% (or greater) dextrose and must be given in a central catheter to avoid phlebitis. TPN solutions cannot be administered through peripheral catheters. Medications should never be injected into A-lines.

Which of the following is an appropriate thing to do for a horse that is diagnosed with tetanus? Have exercise increased daily to prevent muscle atrophy Kept in a dark stable with extra padding Turned out to pasture Placed in a UV-lighted barn

Kept in a dark stable with extra padding Horses with tetanus are light sensitive and are sensitive to loud noises and handling. It is best to keep affected horses in a dark, quiet, padded stall.

What structure in the eye is responsible for focus? Cornea Retina Iris Sclera Lens

Lens

Which of the following is NOT part of the axial skeleton? Spine Ribs Skull Limbs

Limbs The axial skeleton forms the core body structure. The appendicular skeleton includes the appendages, or limbs.

Which type of cell is rich in histamine and heparin? Neutrophil Erythrocyte Mast cell Platelet

Mast cells

Which of the following scissors are the most heavy duty and should be used for cutting through dense or thick tissues and fascia? Olsen-Hegar Iris Metzenbaum Mayo

Mayo Mayo scissors are heavy duty scissors compared to the other options. Metzenbaum and iris scissors have thinner blades and should be used for thin, delicate tissues only. Olsen-Hegar is a type of needle driver which includes a scissor for cutting suture material and should not be used for tissue due to potential to damage the tissue with the needle holder tip.

A dog presents for his first day of heartworm treatment. He tested positive for heartworms last week, and the infection was confirmed by chest radiographs and finding microfilariae in his blood. Which drug will this dog receive for killing the heartworms? Milbemycin Selamectin Ivermectin Melarsomine

Melarsomine

A 10-year-old domestic short hair cat presents for facial excoriations. The cat is extremely itchy on its face. Which of the following medications may cause this side effect? Famotidine Metronidazole Methimazole Metoclopramide

Methimazole Methimazole (Tapazole) may cause intense facial pruritus as a side effect. This medication is used to treat hyperthyroidism.

Which of the following is a diagnostic stage of Dirofilaria immitis, the parasite that causes heartworm, in dogs? Larval stage 1 (L1) Larval stage 3 (L3) Nymphal stage Microfilarial stage

Microfilarial stage The diagnostic stage is the stage of the parasite that is used to detect it with a diagnostic test. For heartworm, microfilaria can be identified on a concentrated blood sample.

How is West Nile Virus transmitted? Bot fly bite Mosquito bite Through respiratory secretions Ingested in the environment

Mosquito bite

Which of the following is the most common factor that prevents a diagnostic radiograph in a horse? Bandages Oversedation Low line voltage Motion

Motion Motion is a problem with any radiograph on any species. However, in horses, due to their fight or flight nature, getting them to stand still can be tricky.

A lab report has a note from the pathologist that says the "segs" were elevated. What blood cell type is a "seg"? Platelet Neutrophil Lymphocyte Red blood cell

Neutrophil

What is the proper name for the "third eyelid"? Frenulum Ranula Nictitating membrane Tympanic membrane

Nictitating membrane The nictitating membrane is the "third eyelid". The tympanic membrane is the "ear drum". A frenulum is a fold of tissue that prevents movement (such as with a persistent frenulum in the bull regarding the penis). Palpebra is the proper term in general for eyelid. A ranula is a mucocele usually found under the tongue region.

If a break in sterile technique occurs during surgery, what should you do? Send the patient home on at least 10 days of antibiotics Notify the surgeon immediately Ask the surgeon for permission to administer additional intraoperative antibiotics Abort the surgery to reduce the likelihood of infection

Notify the surgeon immediately

Which organ is responsible for producing most of the digestive enzymes used to breakdown food? Spleen Stomach Pancreas Liver

Pancreas

Which of the following is a general term for giving birth that can be applied to all species? Queening Pyometra Whelping Parturition

Parturition

The non steroidal anti-inflammatory drug, ketoprofen, should not be used in which patients? Patients with hypertension Horses Cats Patients with renal impairment

Patients with renal impairment The correct answer is patients with renal impairment. Ketoprofen is an NSAID that is eliminated by the kidneys. It is one of the few NSAIDs that are tolerated well by cats. It can also be used in horses, and there is no contraindication of its use in patients with hypertension.

Which of the following drugs is commonly given by intravenous injection to humanely euthanize small animals? Ketamine Digoxin Phenobarbital Pentobarbital Cyanide

Pentobarbital

Which medication given for diarrhea can cause the stools to appear dark in color, like melena? Tylosin Forti-flora Pepto Bismol Sucralfate Metronidazole

Pepto Bismol Pepto Bismol and some formulations of KaoPectate contain bismuth subsalicylate, which causes darkening of the stool and acts by anti-secretory effects on the intestine. Sucralfate is given to help sooth esophageal and gastric ulcers. Metronidazole and Tylosin are antibiotics. Forti-flora is given to help restore the normal gastrointestinal flora.

Which vessel transports blood from the intestines to the liver? Renal artery Hepatic duct Vena cava Portal vein

Portal vein

A large canine patient that has been in the hospital developed a decubital ulcer. What is likely causing this? Pressure from laying in one position for too long MRSA infection Kidney function is declining Improper nutrition

Pressure from laying in one position for too long A decubital ulcer is the proper term for a pressure sore. This is more common in patients that are not ambulatory and lay in the same position for too long; this is why patients must be turned and moved regularly.

Potassium citrate is most often given to dogs for which of the following reasons? Immune function Arthritis pain Boost energy level Prevent bladder stones

Prevent bladder stones Potassium citrate is often supplemented in the diet to help prevent formation of calcium oxalate bladder stones.

A bitch with a swollen vulva and bloody vulvar discharge that attracts but will not accept a male is likely in which stage of the estrous cycle? Estrus Proestrus Anestrus Diestrus

Proestrus In proestrus, vulvar edema is present with a bloody discharge. The bitch is attractive to males but will not stand for mating. A bitch that is in estrus will typically accept a male for mating. During estrus, the vulva is not quite as swollen and the discharge becomes more clear.

Which hormone maintains pregnancy? Luteinizing hormone Estrogen Progesterone Prostaglandin

Progesterone Estrogen is made when the follicle stimulates GnRH from the hypothalamus and causes LH release from the anterior pituitary. LH causes the follicle to ovulate. The follicle then becomes the corpus luteum, and progesterone maintains the pregnancy. Prostaglandin actually causes luteal death.

Which anesthetic is short-acting, white-opaque in color, and can cause transient apnea on induction? Etomidate Thiopental Propofol Ketamine

Propofol

What is the proper term for when the globe (eye) comes out of the orbit due to trauma? Miosis Buphthalmos Mydriasis Proptosis

Proptosis A proptosed globe is an eye that has actually popped out of the orbit. Buphthalmos describes an enlarged and bulging globe which is usually caused by increased intraocular pressure (such as with glaucoma). Miosis is the term for constricted pupils and mydriasis is the term for dilated pupils.

Giardia is what type of parasite? Cestode Ascarid Coccidial Protozoan Trematode

Protozoan Protozoa are unicellular organisms. Giardia exists in two forms: the motile trophozoite and the cyst form. Trematodes are flukes. Cestodes are tapeworms. Coccidia are a different type of protozoan parasite consisting of several species of Cystoisospora, Cryptosporidium, Toxoplasma, and others.

Where do adult heartworms live? Aorta Pulmonary arteries Lungs Left ventricle

Pulmonary arteries Most adult heartworms reside in the pulmonary arteries.

Which of the following animals develops dental caries? Goats and camelids Rabbits and chinchillas Horses and cats Cattle and sheep Pigs and reptiles

Rabbits and chinchillas Rabbits, chinchillas, and dogs develop dental caries. Caries are very rare in the other species listed. Caries are cavities.

When administering a blood transfusion to a rat which species is able to donate? Rat only Rat or mouse Rat, mouse, or ferret Rat or ferret

Rat only The rat is the only species that can donate blood to be given to another rat.

What is an erythrocyte? Red blood cell Platelet Cell with lack of central pallor White blood cell

Red blood cell

What is the most common reason for placing a urinary catheter in a male cat? Relieve urethral blockage Therapeutic infusion into the bladder Contrast radiography Collect urine sample

Relieve urethral blockage

Drugs that have no accepted medical use, are not considered safe, and have high potential for abuse are the most controlled by the DEA (Drug Enforcement Administration). Drugs under this category such as heroin are listed as what type of category? Schedule III controlled substance Schedule I controlled substance Schedule IV controlled substance Schedule II controlled substance Schedule V controlled substance

Schedule I controlled substance Schedule I substances are the most controlled and are unlikely to be available at a veterinary clinic. Schedule II substances such as morphine and fentanyl are frequently the most tightly controlled substances in a veterinary hospital.

Of the following list of retractors which one is not self-retaining? Weitlaner retractor Senn Rake retractor Balfour retractor Gelpis retractor

Senn Rake retractor

A plain red top tube or a tiger top tube is used for collecting which of the following? Hematocrit Plasma Urine Serum CBC

Serum

Which of the following tests would be most helpful in the diagnosis of feline pancreatitis? Serum pancreatic lipase immunoreactivity (PLI) Serum lipase Serum amylase Serum glucose

Serum pancreatic lipase immunoreactivity (PLI) Amylase and lipase are not sufficiently sensitive or specific in the cat to diagnose pancreatitis, but PLI has been shown to correlate very well with the presence and severity of pancreatitis in cats.

Which of the following tests is best for evaluating a dog with possible exocrine pancreatic insufficiency? Serum amylase Serum lipase Serum pancreatic lipase immunoreactivity (PLI) Serum trypsin-like immunoreactivity (TLI)

Serum trypsin-like immunoreactivity (TLI) TLI is a sensitive and specific test for exocrine pancreatic insufficiency in dogs. The other tests listed are used more commonly to diagnose pancreatitis.

What is the most likely period of time for a scrotal hernia to occur in a stallion? Shortly after a trailer ride Shortly after heavy jumping Shortly after a race Shortly after breeding

Shortly after breeding

The vomeronasal organ is closely associated with which sense? Balance Sight Smell Taste

Smell

Antivenin is typically given in cases of which of the following? Congestive heart failure Snake bites Toxin ingestion Prolonged seizure activity

Snake bites Antivenin contains venom-neutralizing antibodies and provides some passive immunity against viperine snakes (mostly Crotalidae species, or rattlesnakes).

A rabbit presents with a history of sneezing, nasal discharge, and conjunctivitis. The veterinarian suspects the rabbit has a Pasteurella infection, most commonly known as which of the following? Snuffles Rabbit fever Kennel cough Q fever

Snuffles

For which procedure is a celiotomy performed? Lung lobectomy Hemilaminectomy Pericardial window Spay

Spay

Which of the following is an important safety feature that one must follow when using biomedical lasers for surgery? Special safety glasses must be worn The operator must hold his breath while the laser is being activated The patient's incision site may only be scrubbed with 0.05% dilute chlorhexadine solution The patient must be maintained on oxygen only

Special safety glasses must be worn

Which organ can be completely removed and is not required to sustain life? Liver Intestines Heart Spleen

Spleen The spleen can be surgically removed. The other organs listed are essential to life. Only a portion of the liver could be resected as the liver is needed to sustain life. The liver also has the ability to regenerate.

What does the term "metastasis" used in veterinary medicine most often describe? Degeneration of a tumor without treatment Spread of cancer from the primary site to another site A malignant tumor that is locally aggressive A non-cancerous lesion

Spread of cancer from the primary site to another site A malignant tumor is synonymous with cancer. Malignant tumors may metastasize, or spread to other sites. Very common areas to which tumors metastasize are the lymph nodes and lungs. This is why lymph node aspirations and chest radiographs are often done as part of cancer staging.

Ketones in the urine may be caused by unregulated diabetes (diabetic ketoacidosis). What is another potential cause of ketonuria? Renal failure Muscle tremoring Increased exercise Starvation

Starvation Ketones are intermediate products of fat metabolism. Diabetes is the most common condition in which ketones are produced. Starvation can also cause ketonuria, except without the hyperglycemia.

How do oral flea control products such as Program or Sentinel work? Stopping egg hatching Sterilizing the male flea to prevent reproduction Killing adult fleas directly Paralyzing the nervous system of the flea

Stopping egg hatching These products interfere with chitin synthesis. The flea's "egg tooth", which is made of chitin, is no longer able to help the flea hatch from the egg. Another possible correct choice, but not provided, would be that females produce sterile eggs. Female fleas may produce 150-300 eggs per week (or up to 50 eggs a day). These products do not kill adult fleas.

Which bacterium is gram-positive? Campylobacter Salmonella E. coli Streptococcus

Streptococcus

What is a ranula? Sludge accumulation in the gallbladder Anal gland infection Sublingual mucocele Cyst on the eyelid

Sublingual mucocele

Which of the following suture/needle combination is most commonly used in veterinary medicine? Eyed, straight needle Eyed, curved needle Swaged, curved needle Swaged, straight needle

Swaged, curved needle A swaged needle is one that has the suture connected to the needle. It is essentially "eye-less". Curved needles are preferentially used to suture all anatomic structures such as fascia, subcutaneous tissue, dermis, and skin. Eyed needles have a hole at the end through which suture is fed. These types of needles are more traumatic to the tissues and not as commonly used.

What does bog spavin describe? Rotation of the coffin joint Abscessation of the hoof wall Inflammation of the superficial digital flexor tendon Tarsocrural effusion

Tarsocrural effusion Bog spavin is a term used in the equine field. It describes the accumulation of synovial fluid in the tarsocrural joint. Rotation of the coffin joint occurs with laminitis. "Bowed tendon" and tendonitis are the descriptive terms for inflammation of a tendon. If in the correct location the abscessed hoof wall may be termed a subsolar abscess. These result in extreme pain and lameness and must be distinguished from laminitis.

NAVTA stands for which of the following? The National Association of Veterinary Technicians in America The North American Veterinary Technicians Association The National American Veterinary Technicians Association The Northern Association of Veterinary Technicians in America

The National Association of Veterinary Technicians in America This is the correct name of the organization that represents veterinary technicians and assistants in the United States.

What is a sarcolemma? The cell membrane which encloses a muscle fiber A contractile cell in the muscle A calcium storage unit in the muscle Connective tissue putting muscle fibers into groups

The cell membrane which encloses a muscle fiber A sarcolemma is a thin membrane enclosing a striated muscle fiber, or simply the cell wall of a muscle cell. A sarcomere is the contractile unit of a myofibril, or muscle cell. Sarcomeres are repeating units and are located between Z lines of the muscle cells. Perimysium is connective tissue that groups individual muscle fibers into bunches or groups.

When placing lateral radiographs of the stifle joint on a viewer for a veterinarian to evaluate, how should the film be oriented? The cranial or dorsal side of the limb should be to the left and the distal aspect should be toward the top of the viewbox The cranial or dorsal side of the limb should be to the right and the proximal aspect should be toward the top of the viewbox The cranial or dorsal side of the limb should be to the left and the proximal aspect should be toward the top of the viewbox The cranial or dorsal side of the limb should be to the right and the distal aspect should be toward the top of the viewbox

The cranial or dorsal side of the limb should be to the left and the proximal aspect should be toward the top of the viewbox

Which of the following is true regarding dentition in dogs? The dog has a total of 6 incisors The lower first molars in the dog are triple-rooted The incisors on the maxilla should rest caudal to the incisors on the mandible The incisors on the mandible should be palatal to the incisors on the maxilla

The incisors on the mandible should be palatal to the incisors on the maxilla

What is the primary toxic principle in chocolate? Arginine Cholinesterase inhibitors Serotonin Theobromine Sucrose

Theobromine Methylxanthines are the primary toxic principles in chocolate. Specifically, these are theobromine and caffeine.

A dog with which condition should not have blood drawn from the jugular vein? Thrombocytopenia Hyperadrenocorticism Congestive heart failure Renal failure Hypertension

Thrombocytopenia Thrombocytopenia is the term for decreased platelets. Dogs that have low platelets have problems clotting and could potentially have a serious bleed from the site of venipuncture.

When performing anesthesia in the horse, which of the following should always be done prior to induction and intubation? Throughly rinse the oral cavity Float the molars Apply a small amount of lidocaine to the larynx Insert a tube into the esophagus so that you are able to place the endotracheal tube properly

Throughly rinse the oral cavity The oral cavity should always be rinsed to remove food particles to prevent introducing foreign material into the trachea.

What is the best way to restrain a unwilling rat for an exam? Thumb and forefinger under front legs, other hand at base of tail Sitting in the palm of your hand Scruffed just like a cat Grab at tip of tail and compress body to table

Thumb and forefinger under front legs, other hand at base of tail

What is the purpose of the rheostat of the light source on a microscope? To adjust light intensity To focus light on the object To raise and lower the light source relative to the stage To adjust the wavelength of light

To adjust light intensity

What is the main job of the nucleus of a cell? To contain DNA To enclose organelles To make protein To produce ATP (energy)

To contain DNA The nucleus holds the genetic material (DNA) of the cell. Ribosomes synthesize protein. Cytoplasm holds the organelles. Mitochondria produce most of the ATP (powerhouse of the cell).

Why do you use a heparin flush to flush an IV catheter? To prevent phlebitis To reduce pain associated with flushing the catheter To prevent infection associated with the catheter To prevent clot formation in the catheter

To prevent clot formation in the catheter

What is the purpose of polishing the teeth after a dental cleaning? To strengthen the enamel To smooth the microscopic defects on the tooth surface To prevent cavity formation To remove any remaining calculus on the teeth

To smooth the microscopic defects on the tooth surface

Torsion of an organ means that what has happened to it? Become necrotic Filled with air Been pushed to a different location by another organ Twisted on itself

Twisted on itself

What is the most common blood type in cats in the United States? Type B Type AB Type A Type O

Type A Cats most commonly have type A blood. Some cats have type B blood, many of which are exotic-type species. Type B cats have strong anti-A alloantibodies, so type A blood given to a type B cat results in life-threatening acute hemolytic transfusion reactions. A cat should always have a blood type test before a transfusion.

In a non-rebreathing anesthesia circuit, which of the following is NOT used? Oxygen Unidirectional valve Reservoir bag Scavenger

Unidirectional valve

The word "idiopathic" is commonly used in veterinary medicine. What does this word imply? Self-inflicted Unknown cause Inflicted by another Acquired in the hospital

Unknown cause Idiopathic means that the cause or mechanism for a disease is unknown. Nosocomial means acquired in the hospital

Which of the following are triple rooted teeth in dogs? Lower 1st molars Lower 2nd premolars Upper 1st premolars Upper 4th premolars

Upper 4th premolars

What is the "antidote" or treatment for Flagyl toxicity in animals? Phenobarbital Sucralfate Valium Metoclopramide

Valium Flagyl (metronidazole) can cause neurologic side effects. The treatment of these side effects is Valium (diazepam) and supportive care. The treatment using Valium is related to modulating GABA receptors in the cerebellar and vestibular systems.

Which of the following vitamins are fat-soluble? Vitamins A, D, E, and K Vitamin B12, Vitamin K Vitamin C, Vitamin K Vitamin C, folate

Vitamins A, D, E, and K

Which of the following is not a complication associated with the placement of an epidural in a dog that is under anesthesia? Vomiting Hypotension Respiratory depression Urinary retention

Vomiting

Which of these is NOT part of the regular signalment for a pet? Breed Age Spay/neuter status Weight

Weight

You are sterilizing this retractor for a wound repair surgery. Which retractor is this? (mini rakes at the end) Balfour Malleable Gelpi Weitlaner

Weitlaner

Hamsters are susceptible to proliferative ileitis. What is the common name for this debilitating disease? Tyzzer's disease Wet tail Snuffles Tularemia

Wet tail Wet tail is the common name given to the disease that plagues hamsters in stressful situations. It is called "wet tail" because of the diarrhea that soils the rear of the hamster. Caused by a mulitude of issues (stress being one of the most common), this condition is contagious and debilitating. An overgrowth of bacteria causes profound diarrhea.

You are asked to run an electrocardiogram (ECG) strip on a pet. There are three leads on your machine. One is white, one is black, and one is red. Where should these be placed? White: right hindlimb, black: left forelimb, red: right forelimb White: left forelimb, black: right forelimb, red: right hindlimb White: left hindlimb, black: right forelimb, red: left forelimb White: right forelimb, black: left forelimb, red: left hindlimb

White: right forelimb, black: left forelimb, red: left hindlimb

Which of these drugs can be used to reverse anesthesia induced by xylazine in a cat? Yohimbine Flumazenil Naloxone Medetomidine

Yohimbine

What is the test of choice for diagnosing Addison's disease in dogs? Urine cortisol:creatinine ratio Insulin:glucose ratio ACTH stimulation test Estrogen level Pre- and post-prandial bile acids test

ACTH stimulation test

Trichomoniasis causes which of the following conditions in cattle? Abortions Severe watery diarrhea Mucopurulent nasal discharge Dry, scaly skin lesions

Abortions

What type of folding method is preferred for cloth drapes that would allow for easy unfolding and placement of the drape? Accordion folding Square folding Diagonal folding Triangle folding

Accordion folding

A butterfly catheter is most commonly used to: Administer an IV injection Give intraosseous fluids Give tube feedings Collect a urine sample

Administer an IV injection

Which material is used to record impressions or imprints of the teeth for orthodontic evaluation? Alginate Casting gel Plaster of Paris Gel foam

Alginate This material is mixed, placed into an impression tray, and placed over the teeth. After it sets, the tray is removed in one motion in the direction of the long axis of the teeth. Plaster of Paris is the classic material that used to be used to make casts; however, nowadays most doctors use fiberglass to make casts. Gel foam is a collagen sponge that surgeons use to help create clots during surgical procedures.

You are responsible for anesthetizing an adult cat. What size endotracheal tube would you have ready for this patient? #2 #6 #8 #4

#4 The average adult cat is properly intubated with a #4 endotracheal tube. A #2 would be too small for an adult. It could be used for a young kitten. The #6 and #8 would be too big for a cat and would be applicable for small to medium dogs.

When doing a tie-over bandage, which suture would be the best suited for the "loop sutures" located around the wound periphery? 2-0 catgut suture 5-0 Polysorb suture 0 or 1-0 monofilament non-absorbable suture 2-0 multifilament absorbable suture 5-0 non-absorbable suture

0 or 1-0 monofilament non-absorbable suture For the loop sutures, you want a strong and non-absorbable suture. A 0 or 1-0 size suture is the largest and strongest of these choices.

You are working in the critical care unit and learning to perform central venous pressure readings. What is a normal CVP reading? 15-20 cm H2O 90-120 cm H2O 10-15 cm H2O Less than 0 cm H2O 0-10 cm H2O

0-10 cm H2O

What percentage of hemoglobin should be saturated with oxygen in a healthy animal? >85% >80% >90% >95% >65%

>95% In a healthy animal, 95 to 100% of the hemoglobin should be saturated with oxygen. Below 90%, suggests fairly significant hypoxemia.

A 4-year-old domestic short hair cat presents with her pupils being noticeably of two different sizes (see image). What is the proper term for this condition? Anisocoria Buphthalmos Miosis Mydriasis

Anisocoria

What is the most common swine gastrointestinal parasite? Stephanurus dentatus Trichuris suis Ascaris suum Erysipelothrix rhusiopathiae

Ascaris suum

Which of the following is NOT part of the small intestine? Ascending colon Jejunum Ileum Duodenum

Ascending colon

Which drug inhibits platelet aggregation? Metronidazole Amoxicillin Aspirin Enalapril

Aspirin

Pyrethrin (which is used to kill fleas) is derived or extracted from which of the following plants? Bougainvillea Stargazer lily Hibiscus Chrysanthemum

Chrysanthemum Pyrethrin comes from the Chrysanthemum plant and acts on the nervous system to cause flea death.

What is most often seen in a cat with heartworm disease? Seizuring Unkempt haircoat Petechial hemorrhages Coughing Syncope

Coughing

The retina is innervated by which cranial nerve? Cranial Nerve IV Cranial Nerve III Cranial Nerve VI Cranial Nerve II Cranial Nerve I

Cranial Nerve II The optic nerve (II) provides the sensory input to the brain to produce visual images. CN I is the olfactory nerve, CN III is the oculomotor nerve, CN IV is the trochlear nerve, and CN VI is the abducens nerve.

The tooth crown is covered with which of the following? Dentin Enamel Cementum Pulp

Enamel

What test can be performed to confirm diabetes if a stressed cat has hyperglycemia? Fructosamine Insulin level Blood glucose curve Urine culture

Fructosamine

Where would you find pia mater? Inside the ear canals In the brain In the lining of the urinary bladder Under the tongue

In the brain

Which of the following is a measure of red blood cell size on a complete blood count? Mean Corpuscular Hemoglobin (MCH) Hematocrit Mean Corpuscular Volume (MCV) Mean Corpuscular Hemoglobin Concentration (MCHC)

Mean Corpuscular Volume (MCV) Red blood cell size (in femtoliters) is described by the MCV. This value can be determined electronically by automated analyzers or manually by dividing the PCV by the RBC concentration and multiplying by 10.

Sodium fluoride is the preferred anticoagulant for which of the following purposes? Assessment of most enzymatic activities Assessment of white blood cell morphology Assessment of the coagulation cascade Measurement of blood glucose

Measurement of blood glucose

Which of the following drugs increases gastrointestinal motility? Metoclopramide Barium Famotidine Diphenhydramine

Metoclopramide

Accumulations of desquamated cells, food particles, and bacteria along the teeth are known as which of the following? Caries Plaque Calculus Tartar

Plaque

The bottom of a tortoise shell is called what? Terrapin Bridge Carapace Plastron Scute

Plastron The carapace is the upper shell and the plastron is the bottom shell. Scutes are a layer of keratin covering the shell. A bony bridge is an area that joins the carapace and the plastron together. A terrapin is a type of turtle which lives in fresh or brackish water.

Which blood cells play an important role in clotting? Platelets Metamyelocytes Erythrocytes Leukocytes

Platelets When a wound occurs, platelets act with fibrin to form a clot and stop bleeding. A metamyelocyte is a very immature white blood cell (that will eventually become a band) and is typically only seen in the bone marrow.

Name this dental instrument (measure with it). Probe Luxator Polisher Curette

Probe

Periodontal pockets are measured by which dental instrument? Scaler Curet Elevator Probe

Probe

What is the causative agent of visceral larval migrans in humans? Whipworm Hookworm Roundworm Tapeworm

Roundworm The correct answer is roundworm. The roundworm, Toxocara canis, is the most common cause of visceral larval migrans in humans. Transmission is usually fecal-oral. Puppies may acquire the disease transplacentally.

What is a frightened sheep most likely to do? Kick Run Head butt Bite

Run

Parrots have the unique ability to mimic sound. What anatomical structure in birds is responsible for producing sound? Syrinx Cloaca Larynx Choana

Syrinx

Which essential amino acid is critical for cats? Tyrosine Lysine Arginine Taurine

Taurine Taurine deficiency in cats is associated with dilated cardiomyopathy (DCM) and retinal degeneration (think taurine Deficiency = DCM).

Where would you find a Sertoli cell tumor? Adrenal gland Pancreas Kidney Brain Testicle

Testicle

A 9-year old Greyhound presents to your clinic with an ulcerating mast cell tumor, located subcutaneously over the left scapula. You immediately schedule the dog for a wide surgical excision after no evidence of metastatic disease is identified on a complete blood count, serum chemistry, thoracic radiographs, and abdominal ultrasound. Which drug is contraindicated in this patient? Acepromazine Thiopental Ivermectin Propofol Atropine

Thiopental

The elbow is extended by which muscle? Biceps brachii Latissimus dorsi Triceps brachium Supraspinatus

Triceps brachium

How many upper incisors does a goat have? Two Four Six One Zero

Zero

Periodontal disease can refer to processes affecting which of the following? Gingiva Enamel Dentin Tongue Hard palate

Gingiva

Which of the following is the correct dental formula for guinea pigs and chinchillas? 2(I2/2 C0/0 PM 0/0 M 3/3) =20 2(I1/1 C0/0 PM1/1 M3/3) = 20 2(I1/1 C1/1 PM1/1 M2/2) =20 2(I3/3 C1/1 PM1/1 M2/2) = 28

2(I1/1 C0/0 PM1/1 M3/3) = 20

What is the permanent dental formula for a dog? 2(I3/3 C1/1 P2/4 M3/3) 2(I3/3 C1/1 P3/2 M1/1) 2(I3/3 C1/1 P4/4 M2/3) 2(I3/3 C1/1 P3/4 M1/3)

2(I3/3 C1/1 P4/4 M2/3)

A 285-gram guinea pig weighs: 2.8 pounds 0.285 kg 2.85 kg 1.2 pounds

0.285 kg 285 / 1000 =

Which of the following components or settings influence x-ray beam quality? kVp and collimation kVp and filtration Collimation and filtration kVp and mAs mAs and filtration mAs and collimation

kVp and filtration The "quality" of an x-ray beam refers to the energy distribution of the photons that are emitted. mAs and collimation have no effect on the energy of the emitted photons. mAs specifies the quantity of photons emitted; collimation constrains the area where the photons are emitted. The kVp is related to the peak energy of the photons emitted. Applied filtration preferentially absorbs lower-energy photons to affect beam quality.

Electrolytes (potassium, sodium, chloride) are measured in what units? mEq/L mg/dL mmol g/dL

mEq/L

A canine patient just received open hip reduction surgery. What type of bandage do you use on this limb? The Spica splint The Ehmer sling The Velpeau sling The Schroeder-Thomas splint

The Ehmer sling

Horses have unique structures that are paired diverticuli of the Eustachian tube located near the pharyngeal region of the horse. What is the common name of these structures? The dorsal pharyngeal recesses The hyoid apparatus The oropharyngeal openings The nasopharyngeal pouches The guttural pouches

The guttural pouches

What is hematuria? The presence of blood in the urine The presence of hemoglobin in the urine The presence of urea in the blood The presence of ammonia in the urine

The presence of blood in the urine Hematuria is the presence of blood in urine. Hemoglobinuria is the presence of hemoglobin in the urine. Myoglobinuria is the presence of myoglobin in the urine.

You are assisting the emergency veterinarian with a dog who has a GDV. The doctor is ready to place an orogastric tube and the patient is awake on the exam table. Which of the following is true regarding this procedure? The tube should be held above the patient to help prevent aspiration The tube should be held lower than the patient to aid in removal of gastric contents The tube should only be placed after the stomach is trocharized The tube should only be placed while the patient is anesthetized

The tube should be held lower than the patient to aid in removal of gastric contents

You see that the veterinarian you are working with has a patient scheduled to come in today for PU/PD. Which of the following complaints do you expect the owner to have? Their animal is vomiting and having diarrhea Their animal is sneezing and has nasal discharge Their animal is scratching and licking excessively Their animal is coughing and having difficulty breathing Their animal is drinking and urinating excessively

Their animal is drinking and urinating excessively

What is true regarding chelonians? (turtle) They don't have eyelids They don't have a spleen They don't have a urinary bladder They don't have a diaphragm

They don't have a diaphragm

The surgeon asks for the Brown-Adsons. Which type of instrument is this? Small retractor Tissue scissors Thumb forceps Small curved clamp

Thumb forceps

For which of the following procedures would an epidural be of most benefit? Pericardial window Forelimb amputation Splenectomy Tibial Plateau Leveling Osteotomy (TPLO)

Tibial Plateau Leveling Osteotomy (TPLO) Epidural anesthesia provides analgesia and is of great benefit in orthopedic surgeries involving the hindlimbs in dogs. These are routinely performed in TPLO procedures, hip replacement surgeries, etc. and are also often used in cesarean sections.

When giving an intramuscular injection, why do you pull back suction on the syringe before injecting? To prime the needle before injecting To make sure the needle is securely on the syringe and no air leaks through To make sure you are not inadvertently in a blood vessel To make sure there is no air bubble in the needle tip

To make sure you are not inadvertently in a blood vessel When giving an intramuscular injection, you should pull back suction on the syringe to make sure that you are not accidentally in a blood vessel. Certain medications, if inadvertently given IV instead of IM, could cause serious concerns.

Why should pigs be fed from a bin or trough and not on the ground? It causes gastroesophageal reflux It causes them to swallow excessive amounts of air To prevent development of trichobezoars They cannot swallow well unless their food is elevated from the floor

To prevent development of trichobezoars A trichobezoar is a ball of swallowed hair that collects in the stomach and can fail to pass through the intestines. Pigs are constantly shedding hair, so it accumulates on the ground; when eating food off that ground, pigs will ingest enough over time to put them at risk for formation of a trichobezoar.

An elastrator band is used for which purpose? To aid in ambulation for pets that have had back or orthopedic surgery To reduce stomach size in obese pets Castration To hold pressure on the abdomen in cases of hemoabdomen Valve replacement in the heart

To reduce stomach size in obese pets Castration An elastrator band is used to castrate young calves. It is placed on the scrotum. In 2-3 weeks after placement, the scrotum and testes are sloughed.

What device or instrument is typically used to help diagnose glaucoma? Tonometer Ophthalmoscope Slit lamp Schirmer Tear Test

Tonometer The tonometer (Tonopen) is used to measure intraocular pressures. Glaucoma is the condition of raised intraocular pressures. Normal eye pressures for dogs is typically between 10-20 and for cats 10-25 but these must be interpreted in light of clinical symptoms. Eye pressures lower than 10 can be caused by uveitis (inflammation in the eye).

A direct life cycle is seen in which of the following parasites? Toxocara canis Dirofilaria immitis Fasciola hepatica Dipylidium caninum

Toxocara canis Toxocara canis (roundworm) has a direct life cycle. In a direct life cycle, the immature parasite can infect the same host from which it came (i.e., this worm lives in the intestine, eggs are passed in feces, and the eggs are passed to the next host directly). The other parasites all require an intermediate host. Intermediate hosts are: Dirofilaria (mosquito), Dipylidium (flea), and Fasciola (snail)

If a disease is zoonotic, what does this imply? Not contagious Host-specific Transmissible between animals but not to people Transmissible from animals to people

Transmissible from animals to people

Which disease does an intradermal caudal skin fold test check for in cattle? Moraxella bovis Anthrax Tuberculosis Dermatophytosis

Tuberculosis

The dog pictured enters the clinic having an allergic reaction to an unknown allergen. The correct term for the skin condition shown in the photograph is: Erythema Urticaria Dermatophytosis Pemphigus

Urticaria Urticaria is also known as "hives" or "wheals". It is frequently a symptom of an allergic reaction to vaccines or other allergens. Erythema is redness of the skin, which can be seen in allergic reactions, but is not demonstrated in the photo. Pemphigus is the shortened name for a number of different auto-immune skin diseases which usually show up as crusting or ulceration of the skin, particularly on the nose and paw pads. Dermatophytosis is another term for ringworm, which is a fungal infection of the skin that causes alopecia and red, circular, itchy plaques.

What is the most accurate method to estimate a horse's weight out on a farm call? Approximate the weight by height Approximate the weight based on body condition score Use a weight measuring tape Pick up a foot and use that weight to approximate the weight

Use a weight measuring tape Specialized measuring tapes are commonly used to approximate the weight of a horse. There are markings that show weights and the tape is wrapped around the barrel of the horse just caudal to the withers.

Which of the following is NOT a good recommendation to make to an owner after having had a declaw performed on their cat? Use clay based litter in the litter box Restrict his activity as much as possible Discourage licking or chewing of the toes Monitor the bandages for slippage

Use clay based litter in the litter box Typical "gravel-type" clay litter will stick to the incisions and may perpetuate complications with healing and subsequent osteomyelitis. Therefore, it is important to confirm that owners are using approved litter such as torn paper towels or newspaper to help prevent small particles from embedding into the healing incisions. The other answer choices are good recommendations to make.

Mulberry heart disease in swine is caused by which of the following? Strict confinement Vitamin E deficiency Toxicity of eating the fruit of mulberry plants Erysipelothrix rhusiopathiae

Vitamin E deficiency

Which of the following drugs is a sedative, analgesic, and also provides muscle relaxation? Tilitamine Xylazine Ketamine Acepromazine

Xylazine Xylazine is a clonidine analogue and is an alpha-2 adrenergic agonist. It is used for sedation, anesthesia, muscle relaxation, and analgesia in animals (mainly horses). It can be reversed with yohimbine. It may also be used to induce emesis in cats. Ketamine is a dissociative anesthetic and provides analgesia, but causes muscle rigidity, not relaxation. Tilitamine is in the same class as ketamine, and also causes muscle rigidity (because of this is often given with the muscle relaxant zolazepam in the form of Telazol). Acepromazine is a phenothiazine tranquilizer that provides sedation and some muscle relaxation, but does not provide analgesia.

Which breed is most likely to have a portosystemic shunt? Bedlington terrier Labrador Yorkshire terrier Rottweiler

Yorkshire terrier A portosystemic shunt is an abnormal vessel that allows blood to bypass the liver. The Yorkshire terrier is overrepresented for this condition, but the condition can be seen in any breed. It is most common in toy breeds and dogs that are the "runt" of the litter.

You are performing an ACTH stimulation test. You are instructed to give Cortrosyn at a dose of 5 micrograms/kg IV. The Cortrosyn has been reconstituted to a concentration of 0.25 mg/mL. Your patient weighs 27 pounds. How much Cortrosyn should you administer? 0.08 mL 0.85 mL 2.1 mL 2.4 mL 0.24 mL

0.24 mL

A dog having a rough recovery from anesthesia has been ordered to receive 0.02 mg/kg of acepromazine. The dog's body weight is 40 lbs, and the acepromazine is diluted to a 1 mg/ml solution. How much acepromazine should this patient be administered? 0.36 ml 0.8 ml 1.2 ml 0.036 ml

0.36 ml First, convert the body weight to kilograms. 40 lbs / (2.2 lbs/kg) = 18.2 kg Now determine the amount of milligrams needed. 0.02 mg/kg x (18.2 kg) = 0.36 mg (note the kilograms cancel out) Finally determine the amount of milliliters needed. 0.36 mg / (1 mg/ml) = 0.36 ml (note the milligrams cancel out)

A 47 pound Standard Poodle has come to your hospital for a hind limb lameness and the veterinarian would like you to give an injection of Carprofen SQ for pain. The dosage is 2.2 mg/kg. Using the image, how many milliliters will you give to this patient? 2.0 ml 4.6 ml 4.1 ml 0.9 ml

0.9 ml

Which is safe to put in the ears if the tympanum may be ruptured? Chlorhexidine Neomycin 0.9% NaCl Gentamicin

0.9% NaCl Saline (0.9% sodium chloride) is considered safe. The others listed (especially aminoglycoside antibiotics) could potentially be ototoxic if the eardrum is not intact.

What is the normal respiratory rate of dogs? 8-12 breaths per minute 6-12 breaths per minute 10-30 breaths per minute 25-50 breaths per minute

10-30 breaths per minute

A dog has snail bait poisoning (metaldehyde) and presents with seizures. His temperature is 109F, and you begin cooling techniques while the veterinarian treats the seizures. At what temperature should you stop trying to cool him? 102F 107F He should not be cooled 100F 103F

103F When the temperature reaches 103F, it is time to cease the cooling techniques, as the temperature can continue to drop and get dangerously low.

How many thoracic vertebrae do horses have? 18 12 13 9

18

What is the minimum volume of water required by mammals to sustain life? 20 - 25 mls/kg/hr 2 - 4 mls/kg/hr 30 - 35 mls/kg/hr 10 - 12 mls/kg/hr

2 - 4 mls/kg/hr This is the minimum requirement for sustaining life.

What minimum quantity of feces is required to run a reliable fecal test for ova and parasites? 5 grams 2 grams 10 grams 0.5 grams

2 grams Two grams of feces, about the size of a garbanzo bean, is the smallest quantity of feces that would be acceptable to run a reliable fecal flotation with zinc centrifugation. Five grams is ideal.

A 12 pound DSH is presented to your hospital for a dental cleaning. The attending veterinarian has asked you to induce the cat with Propofol at a dosage of 4 mg/kg IV. The concentration of Propofol is 10 mg/ml. How many milliliters will you draw up in the syringe? 0.22 mls 2.18 mls 21.8 mls 4.8 mls

2.18 mls You must first convert pounds to kilograms dividing 12 by 2.2 = 5.45 kg. 5.45 kg multiplied by 4 mg/kg = 21.8 mg. 21.8 mg divided by 10 mg/ml = 2.18 mls. In a real clinical setting, you would likely round the number down and draw up 2 mL.

How many feline blood types exist? 3 2 4 1 13

3 There are three feline blood types: A, B, and AB. Group A is the most common, as 98% of cats are type A. Type B is more rare and seen most often in some exotic breeds. Only approximately 1.7% of cats have type B blood. AB is rare in all breeds. Always crossmatch cats unless you know their blood types. Dogs have 13 blood types.

A patient has come into the hospital after being stung by a bee. The patient's muzzle is severely swollen, and the doctor has ordered a dose of dexamethasone sodium phosphate to be given intravenously. The patient weight is 30 kg, and the dose is 0.5 mg/kg. The concentration of the dexamethasone is 4 mg/ml. How many milliliters is this patient to receive? 3.75 ml 12 ml 1.7 ml 8 ml

3.75 ml

If you have 25% mannitol and you wish to give 500 mg per kg to a 15 kg dog, how much should you give? 30 mls 60 mls 300 mls 600 mls 3 mls

30 mls The correct answer is 30 mls. 25% corresponds to 250 mg/ml strength. At a dose of 500 mg/kg, the total dose for a 15 kg dog should be 7,500 mg, which divided by 250 mg/ml equals 30 mls.

Which of the following is the best approximation of the typical tidal volume of a 20 kg dog? 800 ml 8 L per minute 100 ml 300 ml 4 L

300 ml The tidal volume is volume of air that is normally displaced during inspiration and expiration. Typical tidal volume for a dog is about 15 ml/kg (slightly more for smaller dogs and slightly less for large dogs). Tidal volume can be used to help estimate minute volume and the necessary fresh gas flow rates with various gas anesthesia machines.

For a veterinary X-ray technician, what is the maximum permissible dose of radiation they can receive in one year? 100 mrem 1 rem 5 Gy 5 Sv 10 rad 50 mSv

50 mSv

How many cervical vertebrae do cows have? 9 6 7 13

7

Which organization establishes standards for label information and the description of ingredients on pet food sold in the United States? AKC FDA AAFCO USDA

AAFCO AAFCO is the Association of American Feed Control Officials and is the organization that sets these standards. They ensure that adequate information is communicated to the consumer about the food product.

What is the test of choice for diagnosing Addison's disease in dogs? Urine cortisol:creatinine ratio Insulin:glucose ratio ACTH stimulation test Estrogen level Pre- and post-prandial bile acids test

ACTH stimulation test

What is the most common cause of flexural and angular deformities in horses? Incorrect shoeing Excessive riding Laminitis Abnormal development

Abnormal development Flexure and angular deformities occur as a result of musculotendinous and musculoskeletal growth deformities. These abnormalities may be present at birth or develop over the first few months to years of life. Treatment may be as simple as corrective shoeing or bandaging, but more advanced cases may require surgical intervention. The three main types of flexural deformities are carpal, distal interphalangeal (coffin joint), and metacarpophalangeal.

What is the function of soda lime in an anesthetic machine? Absorb carbon dioxide Absorb oxygen Absorb carbon monoxide Absorb anesthetic gases Absorb nitrogen

Absorb carbon dioxide

Which of the following drugs are toxic to cats? Acetaminophen Famotidine Ketoprofen Prednisolone Xylazine

Acetaminophen

An "FHO" would most likely be performed on a patient with which of the following problems? Arthritic hip Ovarian cysts Incontinence Blindness from glaucoma

Arthritic hip "FHO" stands for Femoral Head Osteotomy. This procedure is removal of the femoral head from the hip joint. It is often performed when there is severe arthritis of a hip or due to fracture or disease from trauma to that hip joint.

What is the biggest complication in patients with megaesophagus? Anemia Aspiration pneumonia Spinal cord compression Glaucoma

Aspiration pneumonia

Which of the following correctly indicates where electrons are generated in an x-ray machine? At the filament of the cathode At the filament of the anode At the target of the anode At the target of the cathode

At the filament of the cathode The cathode flament of the x-ray machine is where electrons are generated. The kVp applied directs those electrons to the anode, where they hit a tungsten target to produce x-rays.

What is the major concern when a patient has a long recovery post-op and is laterally recumbent for an extended period of time? Pericardial effusion Anemia Hypovolemia Atelectasis

Atelectasis Atelectasis is collapse of all or part of a lung. In a patient who is recumbent for a long period of time, there is increased risk that the lung could collapse (usually the lung that is down). This is why it is so important to turn the patient every 15 to 20 minutes after recovery. Turning the patient will help them to recover more quickly and will help to prevent atelectasis.

Which of the following components of an anesthetic machine needs to be replaced regularly? Vaporizer Barium lime One-way valves Breathing tubes

Barium lime Barium lime is a carbon dioxide absorber that can become exhausted (lose its ability to absorb carbon dioxide) and must be replaced. The other components listed can be re-used.

A patient needs to be fasted for which of the following blood test? Fructosamine ACTH stimulation test Bile acids Phenobarbital level T4 (thyroid level)

Bile acids

Assessing which of the following on serum chemistry is most appropriate in the assessment of liver function? Glucose Bilirubin Creatinine Blood Urea Nitrogen (BUN)

Bilirubin Bilirubin is an indicator of liver function. However, bilirubin levels must be interpreted cautiously because factors unrelated to liver function can also cause elevated bilirubin; this is true for all of the choices listed. BUN and creatinine are primarily indicators of kidney function.

What is a comedone? Hyperkeratinized skin Blackhead Sebaceous wart Fungus

Blackhead A comedone is another term for blackhead. Comedones are fairly common in dogs with seborrhea (excess production of sebum), endocrinopathies, and demodex.

The menace test will elicit which response? Which two nerves is this testing? Gag reflex, Cranial nerves XII and VIII Blink, Cranial nerves II and VII Smell, Cranial nerves I and IV Ear twitch, Cranial nerves V and III

Blink, Cranial nerves II and VII The menace is when you wave your hand to the eye to try and elicit a blinking response. This is testing vision via the Optic nerve (Cranial nerve II). I-Olfactory nerve (smell) II-Optic nerve III-Oculomotor nerve IV-Trochlear nerve V-Trigeminal nerve (sensory to the face) VI-Abducens nerve VII-Facial nerve VIII-Vestibulocochlear nerve IX-Glossopharyngeal nerve (sensory for gag reflex) X-Vagus nerve (motor for gag reflex) XI-Accessory nerve XII-Hypoglossal nerve

What does a beta-2 adrenergic agonist cause? Salivation Bronchodilation Constricted pupils Slowed heart rate

Bronchodilation The beta-2 adrenergic agonists are often used in treating cases of "asthma" or other conditions that require bronchodilation as a treatment. Examples of short acting beta-2 agonists are terbutaline and albuterol.

A 3-year old female Doberman presents for a routine spay. The owner states that she has not recently been in heat and has been healthy her entire life. The veterinarian is concerned that Doberman Pinchers are predisposed to having von Willebrand's Disease (vWD). Which of the following diagnostic tests would give the best indication of whether the dog has an increased risk of bleeding due to this disorder? Chest radiographs Total protein level Buccal mucosal bleeding test Abdominal ultrasound

Buccal mucosal bleeding test

In cattle, black leg is caused by which of the following clostridium species? C. chauvoei C. novyi C. septicum C. botulinum

C. chauvoei C. chauvoei causes black leg in cattle. C. novyi causes black disease, C. botulinum causes botulism, and C. septicum causes malignant edema.

A 1-year old pregnant female bulldog presents with a history of unproductive labor. The dog has not had contractions in several hours, and the veterinarian has requested a chemistry panel. What value would be most important on this panel? Sodium Albumin Calcium Potassium Phosphorous

Calcium Low ionized calcium levels are an indication of eclampsia. Changes in sodium, potassium and albumin, while important in the overall assessment of the patient, do not give information as to the ability of the patient to produce uterine contractions.

Which test measures somatic cells to identify mastitis in cows? Connecticut Mastitis Test Somatic Udder Test California Mastitis Test Bacterial Somatic Count Test

California Mastitis Test The California Mastitis Test (also called the CMT) counts somatic cells, mostly neutrophils, in cow's milk to identify mastitis (bacterial udder infection).

Which of the following terms best describes the region at the corner of the eye where the upper and lower eyelids meet. Canthus Sclera Lid Iris

Canthus The "canthus" is the palpebral commissure (where the eyelids come together at the corners). There is a medial and a lateral canthus to an eye. Medial is towards the nose and lateral is towards the ear.

Use of an automated hematology analyzer makes which of the following unnecessary when performing a CBC? Platelet estimate Cell count on a hemocytometer Assessment of red blood cell morphology on a blood smear Differential white blood cell count

Cell count on a hemocytometer There is no need to count cells by hemocytometer if you are using an automated hematology analyzer since the analyzer is likely more accurate than a manual cell count. However, a hand differential count is still more accurate than a machine count and provides more detailed information. A platelet estimate is still necessary because platelet clumps may affect the accuracy of a machine platelet count. Automated hematology analyzers also do not report red blood cell morphologic characteristics such as spherocytes, Heinz bodies, or hemoparasites, all of which may be important to the clinician.

A veterinarian asks you to give a 1 ml/kg dose of lidocaine to a dog that is having ventricular premature complexes. You realize after it is too late that you miscalculated the dose and gave 10 times what was asked for. What is the most common early sign of lidocaine toxicity in dogs? Apnea Profound bradycardia Central nervous system depression Anaphylaxis Peripheral neuropathy

Central nervous system depression In dogs, toxicity of lidocaine is manifested primarily as CNS signs. Drowsiness or agitation may progress to muscle twitching and convulsions at higher doses. This occurs before respiratory or cardiac depression. Hypotension may develop if an IV bolus is given too rapidly. Cats are more sensitive to lidocaine toxicity and may show cardiac suppression and CNS excitation.

In what anatomical location can the atlas and axis be found? Pelvis Cervical region Abdominal cavity Mandible

Cervical region The cervcial (neck) vertebrae C1-2 are known as the atlas and axis and articulate with each other.

Which type of gloving method would result in the least chance for contamination? Assisted gloving Double open gloving Closed gloving Open gloving

Closed gloving

What is the most rostral structure of the eye? Pupil Lens Retina Cornea

Cornea

What is the term used to describe the junction between the skin and hoof of the horse's distal limb? Laminae Periople Coronet Transitional epithelium Frog

Coronet The transition is called the coronet or coronary band; the hoof is formed by epithelial keratinization over a greatly modified dermis.

You are examining a blind dog and want to know if the dog has a sense of smell. You wave a treat in front of the dog's nose, and he smells it right away. Which nerve were you testing? Cranial nerve I (one) Cranial nerve X (ten) Cranial nerve II (two) Cranial nerve V (five)

Cranial nerve I (one) The Olfactory nerve deals with the sense of smell and is Cranial nerve I. Cranial nerves: I-Olfactory nerve II-Optic nerve III-Oculomotor nerve IV-Trochlear nerve V-Trigeminal nerve VI-Abducens nerve VII-Facial nerve VIII-Vestibulocochlear nerve IX-Glossopharyngeal nerve X-Vagus nerve XI-Accessory nerve XII-Hypoglossal nerve

Which of the following is an ectoparasite of animals? Ctenocephalides felis Ancylostoma caninum Paragonimus kellicotti Feline infectious peritonitis Filaroides osleri

Ctenocephalides felis By definition, an ectoparasite is a eukaryotic organism that lives on the outside of the host. Examples include lice, flies, fleas, and mites. Ctenocephalides is a flea. Ancylostoma, Filaroides, and Paragonimus are all endoparasites. Feline infectious peritonitis is not a parasite, it has a viral cause.

Which of the following terms describes an incision into the bladder? Cystotomy Cystocentesis Dystocia Cystectomy

Cystotomy When a word ends with "-otomy" it means an incision or to cut into organ. For example, an enterotomy is an incision into the intestine just as a cystotomy is an incision into the bladder. Words that end in "-ectomy" mean that they are being removed. So a cystectomy would be the removal of the bladder. Words that end in "-centesis" imply the removal of fluid. A cystocentesis is what would be performed to obtain a urine sample via a needle and syringe inserted into the bladder. Dystocia is a general term used to describe an abnormal or difficult labor. There can be many different causes of dystocia.

Organelles reside in which of the following compartments of the cell? Ribosome Cytoplasm Nucleus Cell membrane

Cytoplasm Organelles reside in the cytoplasm. Some examples include ribosomes, endoplasmic reticulum, lysosomes, mitochondria, and the Golgi complex. The nucleus contains the cell's DNA for replication. The cell membrane (cell wall) is what separates the cell from the external environment. The cell wall is semi-permeable and will allow certain compounds and electrolytes in and out of the cell.

The modified Knott's technique can be used to isolate the diagnostic stage of which of the following parasites? Strongyloides stercoralis Dirofilaria immitis Cystoisospora canis Giardia lamblia Oxyuris equi

Dirofilaria immitis The modified Knott's technique allows differentiation of microfilariae circulating in the bloodstream. It involves mixing blood with formalin, centrifuging, and analyzing the sediment under a microscope. Dirofilaria immitis is the only blood-borne parasite of the answer choices. The other parasite that can be recovered in this technique is Dipetalonema reconditum.

The acronym DIC stands for which of the following? Disseminated inflammatory cardiomyopathy Dissipated infectious coagulopathy Disseminated intravascular coagulation Dilated immune-mediated cardiomyopathy

Disseminated intravascular coagulation DIC, or Disseminated intravascular coagulation is the condition in which coagulation factors are excessively consumed. This prevents additional clots from being formed and also impairs the breakdown of existing clots. Patients in DIC are both pro-thrombic (producing micro-clots that get trapped in the capillaries) and anti-thrombic (unable to create new clots). These patients have severe bleeding tendencies. DIC most often follows severe trauma, inflammatory conditions, or sepsis.

A patient in the hospital has the orders "NPO" on its cage. What does this order mean? Only give the patient a limited amount of food and water. It is ok to give the patient water, but no food. The patient should always have food and water available. Do not give the patient any food or water.

Do not give the patient any food or water.

Which breed most commonly has von Willebrand's disease? Labrador Retriever English Bulldog Shar pei Doberman Pinscher

Doberman Pinscher

When drawing blood for a fructosamine level in a canine, how many hours do you have to wait after insulin is given to draw the blood? 1 hour 8 hours 6 hours Doesn't matter 12 hours

Doesn't matter

Which chemotherapeutic agent has been associated with cardiotoxicity? L-asparaginase Cyclophosphamide Doxorubicin Lomustine

Doxorubicin

Which of these antibiotic choices is bacteriostatic? Penicillin Enrofloxacin Cephalexin Doxycycline

Doxycycline Tetracycline antibiotics (such as doxycycline) are bacteriostatic, meaning they do not kill bacteria, but instead inhibit their growth. Doxycycline inhibits bacterial protein synthesis by binding to the 30S ribosomal subunit. The other antibiotics listed are bactericidal (kill bacteria).

Which of the following is required for chinchilla health? Dust baths Cedar shavings Vitamin C Monthly dental adjustments A warm (90 degree) basking spot

Dust baths Chinchillas require dust baths every 1-2 days in order to maintain coat health. The dust bath should not be left in the enclosure at all times since frequent bathing may cause conjunctivitis. Chinchillas are heat-intolerant, and the enclosure should not be kept at temperatures above 80 degrees Fahrenheit. Cedar shavings should be avoided as bedding, since they can cause respiratory irritation. Chinchillas do not require supplemental Vitamin C. Healthy chinchillas do not require dental adjustments, although dental disease is a common problem.

On average, cows have their estrous cycle how often? Every 21 days Every 6 months Every 30 days Every 60 days

Every 21 days

Syncope is a condition that one should be ready to report if observed. What is this? Vomiting Fainting Seizuring Head bobbing

Fainting

Sheep mostly breed during which season? Winter Spring Fall Summer

Fall

Where is the easiest place to take a dog's pulse? Dorsal pedal artery Carotid artery Brachial artery Femoral artery

Femoral artery The easiest place to take the pulse is the femoral artery which is located in the medial thigh area just caudal to the inguinal area.

Which of these drugs is considered a Class II controlled substance by the Drug Enforcement Administration? Ketamine Diazepam Fentanyl Buprenorphine

Fentanyl

Which of the following species is an induced ovulator? Pig Mouse Ferret Horse

Ferret Induced ovulator means that the animal does not ovulate until the act of breeding occurs. Cats and Ferrets are the most common induced ovulators. Camels and Llamas are also induced ovulators.

You are scrubbing in to assist in a surgery. The scrub should occur in which order? Forearms, wrists, hands, fingers Fingertips, hands, wrists, forearms Between fingers, wrists, forearms, fingertips Elbows, forearms, hands, fingers

Fingertips, hands, wrists, forearms The scrub should start at the fingertips and work up towards the forearms. The scrub should not go back in the reverse direction. The elbows are not typically included in the scrub at all.

Developed radiographs that appear yellow are usually due to a problem with which of the following? Film storage Intensifying screen Developer solution Fixer solution

Fixer solution Yellow radiographs are usually seen when the fixer solution is low or fixation time is too short.

What is the name of the dye commonly applied to the eye to identify a corneal ulcer? Fluorescein dye Schirmer dye Rose bengel dye Lugol dye

Fluorescein dye

Which of the following drugs would be the most likely to cause a nephrotoxicity? Amoxicillin Gentamicin Cephalexin Enrofloxacin

Gentamicin Aminoglycosides, such as gentamicin, are known for their potential toxic effects to the kidneys (nephrotoxicity) and ear (ototoxicity). In some cases, these drugs may cause tubular necrosis of the proximal renal tubular cells, so caution and monitoring are important.

In the name Clostridium perfringens, Clostridium is the: Order Genus Class Family Species

Genus Clostridium is the genus and perfringens is the species. The genus name is always capitalized.

All of the following can be expected to be found on a complete blood count EXCEPT for which of the following? Glucose Hemoglobin Hematocrit Neutrophils

Glucose A glucose level is not part of a complete blood count. Glucose would be reported on a chemistry panel.

In the United States, what is the appropriate color of an oxygen gas tank? Yellow Blue Gray Green Black

Green

What equine gastrointestinal parasite, when deposited at an abnormal site on the skin or mucous membranes, is responsible for causing the eosinophilic granulomas known as summer sores? Onchocerca cervicalis Strongylus vulgaris Oxyuris equi Strongyloides westeri Habronema spp.

Habronema spp.

Which of the following fluid types is a colloid? Hypertonic saline Lactated Ringers Sodium chloride 0.9% Hetastarch 5% dextrose in saline

Hetastarch A colloid contains larger molecules than a crystalloid. This size difference allows the fluid to stay in the intravascular space for longer and thus helps to increase oncotic pressure. Examples of colloids are Hetastarch, blood products (whole blood, plasma, etc.), and Dextrans.

You see that a Shar Pei is scheduled to come in for a second opinion about his entropion. What is wrong with the pet? His eyelids roll inward He is unable to retract his penis into the prepuce He has severe itching He has a hernia

His eyelids roll inward Entropion is when the eyelid edges roll inward towards the globe of the eye so that the lashes are rubbing against the cornea. It is a painful condition that often causes tearing and secondary infections, ulcers, etc. It is treated surgically. The proper term for itching is pruritus. Paraphimosis is the term for an extruded penis that cannot be retracted. A hernia is a hole or tear in the body wall from which abdominal contents can protrude subcutaneously.

A male ferret is also referred to as which of the following? Jill Boar Jack Hob

Hob

Which species is unable to vomit? Pigs Horses Cows Birds

Horses

Which species has wolf teeth? Pigs Cows Cats Dogs Horses

Horses Wolf teeth are small teeth that are the first premolars in the horse. It is thought they may be deciduous teeth. They are located immediately rostral to the first cheek teeth in horses.

You have a small canine patient in cardiac arrest that needs emergency drugs. The animal does not have venous access, and placing a venous catheter would be too difficult and take too long. What is the best way to get the drugs to this animal quickly? Intraosseous catheter Intracardiac injection Intramuscular injection Subcutaneous injection

Intraosseous catheter

The dog penis differs from that of other animals in what way? It is composed of erectile tissue It has spiny epithelial projections It is composed of tiny suction cups at the tip It has a non-articulating bone known as the os penis

It has a non-articulating bone known as the os penis

A cataract is an opaque protein deposit on which structure? Sclera Fundus Lens Cornea

Lens A cataract is an opacification or clouding of the lens.

When discussing teeth, what is the tooth surface closest to the tongue termed? Buccal Lingual Gingival Frontal

Lingual

Heart murmurs are often referred to in terms of "grade". What does this refer to? Interval Rhythm Location Loudness

Loudness

You are performing an ear cytology and see some purple bowling pin-appearing organisms. What are these most likely? Malassezia Pseudomonas Gram-negative rods Pollen

Malassezia

An animal with inflammation of the uterine lining is said to have which of the following conditions? Metritis Pyometra Uterinitis Cystic ovaries

Metritis Metritis is inflammation of the lining of the uterus (the endometrium). Pyometra is pus in the uterus.

What is a Knott's test is useful for detecting? Leptospira Microfiliariae Ancylostoma Aelurostrongylus

Microfiliariae

What is the most common white blood cell found in circulation in a normal dog? Monocyte Neutrophil Lymphocyte Eosinophil

Neutrophil

In a cow that is having difficulty calving and seems to have her legs splaying out from underneath her, you are most concerned about damage to which nerve? Peroneal Tibial Femoral Obturator

Obturator

What is the definitive host for the causative agent of equine protozoal myelitis/myeloencephalitis? Opossum Horse Raccoon Wildebeest Donkey

Opossum The opossum is the definitive host for Sarcocystis, the causative agent of equine protozoal myelitis.

The movement of fluid from an area of lower solute concentration to an area of higher solute concentration across a semi-permeable membrane is called: Osmosis Mitosis Meiosis Diuresis

Osmosis In chemistry, osmosis is the movement of solute molecules through a semi-permeable membrane to an area of higher solute concentration. Net movement of solvent occurs from the less concentrated (hypotonic) to the more concentrated (hypertonic) solution to help equalize the concentration on either side of the membrane.

You are cleaning up the laboratory one morning and notice a slide that was left on the microscope overnight. You peek in to see what is on the slide (purple/red shapes). What are these oval shaped structures? Ancylostoma eggs Pollen Immature Demodex mites Otodectes eggs

Otodectes eggs

The egg is fertilized in which of the following structures? Vagina Oviduct Uterus Peritoneum

Oviduct Fertilization of the ovum occurs in the oviduct.

What is feline "parvo" virus is known as? Calicivirus Panleukopenia Rhinotracheitis Feline Leukemia

Panleukopenia Feline panleukopenia is sometimes referred to as "feline parvo". It causes feline infectious enteritis. It can be associated with cerebellar disease in young kittens.

What is the most common drug in most euthanasia solutions? Phenobarbital Propofol Pentobarbital Thiopental

Pentobarbital

Which species other than cattle is considered cloven hoofed? Rabbit Camel Pigs Horse

Pigs

Which species has ovaries that look like grape clusters? Pigs Sheep Cows Horses

Pigs In the pig the follicles and corpora lutea sit almost on the surface of the ovary giving the appearance of a cluster of grapes. Pigs are also litter-bearing and thus usually have more follicles.

Lactose-fermenting bacteria such as Escherichia coli and Klebsiella appear what color on MacConkey agar? Clear or white Pink or red Blue or green Tan or grey

Pink or red MacConkey agar contains lactose; when lactose-fermenting organisms are grown, they produce acid from lactose that causes them to turn red. Bacteria that do not ferment lactose are typically clear colonies.

Which of the following should never be given in a bolus to a patient? Hetastarch Lidocaine 0.9% Sodium chloride Potassium chloride Mannitol

Potassium chloride Rapid infusion of potassium can cause cardiac arrest. There is never a good reason to bolus potassium chloride; however sometimes it is inadvertently bolused when administering fluids which had potassium chloride added to them. Administration of greater than 0.5 meq/kg of potassium chloride can be deadly. All other choices as long as appropriately dosed may be bolused.

A female dog has some vaginal bleeding, swollen vulva, and attracts males but will not allow them to mount. What stage of the estrus cycle is she in? Estrus Proestrus Anestrus Diestrus

Proestrus The correct answer is proestrus. During this phase, there is vulvar swelling, vaginal bleeding, and attraction of males but no mounting is allowed. When the female will stand to be mounted, she is in estrus. This is a vaginal cytology from a dog in proestrus.

Culture or draining lesions of which bacteria gives a characteristic sweet smell? E. coli Nocardia Staphylococcus Pseudomonas Campylobacter

Pseudomonas Pseudomonas aeruginosa is a gram-negative bacterium that often has a sweet or fruity smell.

Which of the following is a zoonotic disease? Psittacosis Coccidioides White muscle disease Pacheco's disease

Psittacosis

Which of the following would most commonly be enlarged on chest films of a heartworm positive dog? Left atrium Pulmonary arteries Aorta Lymph nodes

Pulmonary arteries

Upon gram staining, gram-positive bacteria will appear what color when visualized under the microscope? Orange Blue Pink Purple

Purple

Which of the following species has a double row of incisors? Guinea Pig Hamster Chinchilla Ferret Rabbit Rat

Rabbit

Which vaccine can a pregnant dog receive? Parvo Bordetella Rabies Distemper

Rabies Pregnant dogs may receive killed antigens such as the Rabies vaccine. The DHPP vaccine is modified live. The Bordetella intranasal is modified live, and the Bordetella injectable is cellular antigen extract and should not be given to pregnant animals.

A dog sustains a humeral fracture. Which nerve would most likely be damaged? Suprascapular nerve Ulnar nerve Radial nerve Sciatic nerve

Radial nerve

Which bone is adjacent to the ulna? Scapula Radius Femur Tibia

Radius The radius is adjacent to the ulna. The femur and tibia are bones of the hind limb.

After taking a radiograph exposure of a patient abdomen at a 40 cm focal-film distance with settings of kVp=60, mAs=2, you have to repeat the exposure using a horizontal beam technique at 80 cm focal-film distance. Assuming there is no change in the tissue penetrated and no other differences aside from the distance in radiographic technique or equipment, what settings should you use to obtain the same radiographic density? kVp=240, mAs=2 kVp=120, mAs=2 kVp=60, mAs=4 kVp=60, mAs=8

kVp=60, mAs=8 To obtain the same radiographic density, you would not change the energy (kVp) but you would need to change the exposure to account for the difference in distance, which is based on the inverse square law. This law states that the exposure at twice the distance will be reduced to 1/4. Therefore, the mAs must be increased 4X. If there had been other equipment such as a grid on one of the views, there might also be other correction factors to take into account.

Which of these are used in the minor cross-match for blood products when looking for a compatible blood donor for a dog? Recipient red blood cells, recipient plasma Donor red blood cells, donor plasma Donor red blood cells, recipient plasma Recipient red blood cells, donor plasma

Recipient red blood cells, donor plasma The correct answer is recipient red blood cells, donor plasma. In a minor crossmatch, you are looking to see if the factors in the plasma of the donor are going to react to the recipient's red blood cells.

A patient with DIC (Disseminated Intravascular Coagulation) may show abnormal red blood cells called schistocytes. What are schistocytes? Nucleated red blood cells Red blood cells with nuclear fragments Red blood cell fragments Large blue tinged red blood cells

Red blood cell fragments Patients with DIC frequently have schistocytes and decreased platelets on their blood smears. Schistocytes are red blood cell fragments.

Which of the following is a technician not allowed to do? Give injections intramuscularly Refill medications without veterinarian approval Perform a dental prophylaxis Dispense controlled substances even with veterinarian approval

Refill medications without veterinarian approval

What is a common presenting complaint in a patient with megaesophagus? Regurgitation Hematochezia Stiff, stilted gait Syncope Muscle tremors

Regurgitation

A dog tries to swallow his ball and it gets stuck in the back of his throat. He is breathing very slowly and is not ventilating properly. He is alert but the obstructive breathing is a concern. Which of the following abnormalities would you expect? Hypothermia Respiratory acidosis Pulmonary hypertension Respiratory alkalosis

Respiratory acidosis Respiratory obstruction leads to accumulation of carbon dioxide. Increased PCO2 results in lower blood pH. This is respiratory acidosis.

You are going to assist with a patella surgery and are asked to use this instrument (tiny rake, 3). What instrument is this? Gelpi retractor Senn Rake retractor Balfour retractor Army Navy retractor

Senn Rake retractor

A suture pack with a cutting needle would be used on which of the following? Intestine Gall bladder Urinary bladder Skin

Skin

Fasciola hepatica requires which intermediate host? Mosquito Flea Rat Snail

Snail Fasciola hepatica is the liver fluke. It is typically found in wet environments where snails proliferate. Snails are the intermediate host for this parasite, and the most common definitive hosts are sheep and cattle. It is a flat worm, resembling a leaf.

Activated charcoal would be used for which of the following? Foreign body ingestion Snail bait ingestion Fluoroscopy Thermal skin burns

Snail bait ingestion

An animal that has been diagnosed with underlying cardiac insufficiency may need to be restricted on intake of: Potassium Thiamine Sodium Magnesium

Sodium Water follows salt, so an animal consuming more sodium is likely to retain more water. This may be detrimental to an animal with underlying heart disease.

Which of the following would not typically be caused by tetanus in the horse? Inability to chew or swallow Muscular rigidity Convulsions Spontaneous abortion Respiratory arrest

Spontaneous abortion The toxin that causes tetanus in horses results in convulsions and muscle rigidity that starts in the hindquarters and moves towards the forequarters. In some cases, the tongue is paralyzed and the animal can not chew or swallow. Paralysis of the respiratory muscles can lead to respiratory arrest. Spontaneous abortions are not a symptom of tetnus.

What is the main effect of Dopram (doxopram hydrochloride)? Stimulates respiration Slows heart rate Stops seizures Induces emesis

Stimulates respiration Dopram (doxopram hydrochloride) is a potent respiratory stimulant. In veterinary medicine, it is used to aid in diagnosis of laryngeal paralysis.

Electrocautery is often used for which of the following reasons? Make an incision Provide a shock to the heart Stop bleeding vessels Sterilize the skin prior to surgery

Stop bleeding vessels Electrocautery is used to achieve hemostasis by cauterizing bleeding vessels.

Streptococcus equi is a bacterium that causes swollen glands in horses. The common term for this condition is: Strangles Choke Sweeney Founder

Strangles

What is the term used to describe difficulty in urination? Pollakiuria Anuria Oliguria Stranguria

Stranguria Stranguria describes straining or difficulty to urinate. Pollakiuria refers to an increase in frequency in urination. Oliguria is a decrease in urination. Anuria is the complete absence of urine formation.

A guinea pig presents with swollen glands in the neck (see image). What organism most often causes this problem? Escherica coli Malassezia pachydermatis Streptococcus zooepidemicus Staphylococcus aureus

Streptococcus zooepidemicus Cervical lymphadenitis is a swelling or abscess of the cervical lymph nodes in a guinea pig's neck. It is most often caused by Streptococcus zooepidemicus. It is a bacteria most commonly transmitted through broken oral mucosa. It is treated by surgical draining and flushing of the abscess. Enrofloxacin (Baytril) is the antibiotic of choice in adult guinea pigs. It is thought that this bacterium could be zoonotic in immune-compromised humans, so you the technician should wear gloves and take extra precautions when treating a guinea pig with a swelling such as this.

A dog refuses a rectal temperature. You take his temperature under the axillary region and get a reading of 99.5F. What is likely his core body temperature? 99.5F 102.1F 101.5F 98.7F

101.5F

You draw up a canine distemper-parvo combination vaccine. How long is the vaccine considered viable at room temperature? 20 minutes 8 hours 24 hours 1 hour

1 hour Vaccines should be discarded if they have been left out at room temperature for longer than one hour. Most references state that if they are refrigerated they can be used within 24 hours, but this is controversial. The best option is to use the vaccine shortly after it is drawn up to avoid any concerns.

Your practice recently purchased a portable ultrasound machine. A client calls and asks how early her mare can have an ultrasound to determine if she is pregnant. What is the very earliest that a pregnancy can be confirmed with an ultrasound in a horse? 45 days 63 days 30 days 11 days

11 days

The gestation period for hamsters makes them a valuable research model. What is their gestation period? 21 days 15 days 30 days 56 days

15 days

What is the average gestation for goats and sheep? 100 days 150 days 83 days 240 days

150 days This list of gestations should be committed to memory: Llama 1 year (350 days), Horse 11 months (330 days), Cow 9 months (280 days), Sheep/Goat 5 months (150 days), Pig 4 months (114 days), Dog/Cat 2 months (63 days), Ferret 1.5 months (42 days)

Surgeons performing orthopedic surgery will need cerclage wire from time to time to reconstruct bone fractures. Which of the following gauges represents the largest diameter wire? 20 gauge 26 gauge 22 gauge 18 gauge

18 gauge The most common sizes used in small animal medicine range from 18-26 gauge; the lower the gauge number, the larger the diameter of the wire. In this question, the 26 gauge wire would be the smallest diameter wire.

How long after birth can foals absorb protective maternal antibodies? 1-3 hours 18-24 hours Up to 48 hours 3-6 hours

18-24 hours Foals and calves can absorb maternal antibodies from colostrum directly into the bloodstream for 18-24 hours after birth. After this time the gut closes for this direct transfer. Calves should get at least 2 liters and foals 1 liter of colostrum before this occurs. Adequate colostrum helps prevent against failure of passive transfer which can lead to increased risk of infections.

You have just completed a manual differential and obtained the values below. The total white blood cell count is 21,800. Segmented Neutrophils 88% Lymphocytes 3% Monocytes 8% Eosinophils 1% What is the absolute value of the Segmented Neutrophils? 88 247.72 19,184 1,918,400

19,184 The percentage of each white blood cell type counted is multiplied by the total white blood cell count to determine the absolute value of each of the cell types present. 21,800 x 0.88= 19,184 Lymphocyte 654 Monocyte 1744 Eosinophil 218 The total of all absolute values should equal the total white blood cell count.

Which of the following suture size is the largest? 2-0 2 6-0 1-0

2

Thiopental is a barbiturate used to induce anesthesia. Once reconstituted it is a 2% solution. The attending veterinarian asks you to induce a 27 kg dog with a dosage of 10 mg/kg. How many milligrams do you need, and how many milliliters will you draw up into the syringe? 13.5 mg / 270 mls 270 mg / 13.5 mls 135 mg / 27 mls 27 mg / 2.7 mls

270 mg / 13.5 mls 27 kg multiplied by 10 mg/kg = 270 mg. A 2% solution is equal to 20 mg/ml. 270 mg divided by 20 mg/ml = 13.5 mls

You are helping admit a patient to the hospital. Rudy is a 5-year old female spayed Yorkie with pancreatitis. The hospital is very busy and all of the fluid pumps are currently being used by other patients. You are asked to start fluids at 24mL/hr for Rudy. Calculate the drip rate for this patient. Her body weight is 8 pounds. The drop size for your administration set is 10 drops/mL. 2 drops/minute 12 drops/minute 21 drops/minute 4 drops/minute

4 drops/minute 24 mL per hour divided by 60 minutes in an hour: 0.4 mL per minute You administration set will give 10 drops per 1 mL. So, multiply 0.4mL/min X 10 drops/1mL (the mLs cancel out) = 4 drops/minute

Regarding fluid types, which cannot be given subcutaneously? Isolyte Lactated Ringers 0.9% NaCl 5% Dextrose

5% Dextrose Only use isotonic fluids for subcutaneous administration. This means that the osmotic properties are equal to that of the extracellular fluid. Dextrose given subcutaneously can cause sloughing of the skin and abscess formation.

The heart rate of an adult dog should range between which of the following? 70-160 bpm 180-240 bpm 24-50 bpm 120-200 bpm

70-160 bpm A dog heart rate ranges between 70-160 bpm. As with the cat, there may be variation. An adult cat typically has a resting heart rate between 150-220 bpm. Keep in mind that if the cat is very excited, the heart rate might be faster. Remember that puppies and kittens will have faster heart rates.

What is the normal rectal temperature for a horse? 98.5-99.5 102.5-103.5F 99-101.5F 101.5-102.5F

99-101.5F Average rectal temperature for a horse is around 100 degrees Fahrenheit. Averages for other species are: Cattle 101F, Cat/Dog 101-102F, Goats/Sheep 102.3F, Pig 102.5F, and Rabbit 103.1F

Which of the following is the best description of an epulis? A gingival lesion caused by inflammation A gingival lesion caused by trauma A non-malignant oral tumor A developmental tooth disorder

A non-malignant oral tumor An epulis is a non-malignant tumor of the periodontal ligament. These tumors do not metastasize but can be locally invasive. The most aggressive form of this condition, an acanthomatous epulis, can invade and destroy bone.

In cattle, a balling gun is sometimes used for which of the following. Euthanasia Castration Administering medications Floating teeth

Administering medications

What is it called when a limb is moved outward or away from the median plane of the body? Abduction Adduction Flexion Extension

Abduction Abduction involves movement away from the midline. Adduction is movement toward midline.

The doctor is trying to retrieve a foreign body out of a dog's ear and is using this tool (bent and long). What instrument is this? Alligator forceps Mosquito hemostat Allis tissue forceps Crile clamp

Alligator forceps The alligator forceps is a two-bladed instrument with a handle for compressing or grasping tissues. It works really well for grasping foreign bodies from the ear canal.

You are scrubbed in and assisting with a digit amputation. The surgeon asks for this instrument (clamps are long and flat). What instrument is this? Alligator forceps Allis tissue forceps Towel clamp Debakey forceps

Allis tissue forceps Allis tissue forceps have inward-curving toothed blades and a ratcheted handle. Although they are called tissue forceps, the are relatively traumatic and should not be used on delicate tissues.

MRI-compatible equipment might be made of which of the following metals? Aluminum Nickel Iron Cobalt

Aluminum For safety purposes, whenever there is any question, no metal should be brought in or near an MRI suite. However, magnets in MRI machines attract ferrous metals which are iron, nickel, and cobalt. There are many non-ferromagnetic metals including aluminum, titanium, brass, and others. Many MRI-compatible devices are made with these. Alloys such as stainless steel may or may not contain ferromagnetic metals.

A Dalmation presents with difficulty urinating. Bladder stones are confirmed with the ultrasound and a cystotomy is performed. Which crystal is commonly observed in the Dalmatian and not expected to be seen in other dog breeds? Calcium oxalate Silicate Ammonium biurate Struvite

Ammonium biurate Ammonium biurate crystals are abnormal in all other breeds except for Dalmatians. Some Dalmatians have a genetic defect that results in the production of ammonium biurate crystals. If the defect is severe, they will be predisposed to developing ammonium biurate uroliths such as the dog in the photograph.

What causes cutaneous larval migration in humans? Toxocara Dipylidium Echinococcus Ancylostoma

Ancylostoma The correct answer is Ancylostoma. These are hookworms; when they come in contact with unprotected skin, the infective larvae penetrate the epidermis but generally cannot penetrate the basement membrane. They therefore migrate aimlessly, and the disease is usually self-limiting in humans.

The natural ovulatory season for mares in the northern hemisphere occurs during approximately which months? December - February September - November April - August March - May

April - August

A 6-year old collie presents after falling out of a moving vehicle. The dog has an open tibial fracture and pulmonary contusions. What would be the best way to immediately stabilize the fracture? Splint Robert-Jones bandage External stabilization Tourniquet Internal stabilization

Robert-Jones bandage

Snakes have the same organs as mammals except they lack which of the following? Liver Lungs Bladder Spleen

Bladder

An extremely fractious cat has been admitted to the facility. Due to its disposition, you are unable to remove it from its carrier. What opioid normally given by injection can be given orally for sedation or analgesia? Hydromorphone Buprenorphine Morphine Dexmedetomedine Acepromazine

Buprenorphine Buprenorphine, a commonly-used pain medication in small animals, can be absorbed through the oral mucosa. None of the other medications can be absorbed in this manner.

Which part of the brain coordinates motor activity? Disease of this part of the brain can cause a hypermetric gait. Frontal lobe Cerebral cortex Cerebellum Brainstem

Cerebellum The cerebellum has a major role in motor control. It helps to coordinate voluntary movement and is located just above the brainstem. Hypermetria is an exaggerated gait in which the movements extend beyond what is intended.

What is the vertebral formula for dogs and cats? Cervical 6, Thoracic 12, Lumbar 5, Sacral 13 Cervical 7, Thoracic 13, Lumbar 7, Sacral 3 Cervical 6, Thoracic 10, Lumbar 7, Sacral 7 Cervical 3, Thoracic 12, Lumbar 6, Sacral 7

Cervical 7, Thoracic 13, Lumbar 7, Sacral 3

Microscopic evaluation of peritoneal fluid would be useful in a horse with which of the following? Colic Coughing Founder Obesity

Colic Collecting abdominal fluid for microscopic evaluation can be useful if a horse is showing symptoms of colic (abdominal pain). The fluid is evaluated to look for inflammatory cells, presence of bacteria, or neoplasia. There are many different causes of colic in a horse. Symptoms of colic can include biting or kicking at the abdomen or sides, or pawing or rolling on the ground.

Most distemper-parvo vaccinations are of which type? Canarypox Killed virus Cellular antigen extract Modified live virus

Modified live virus Most distemper and parvo vaccinations are modified live virus vaccinations. Rabies is a killed virus vaccination in general. There is a feline-only rabies vaccination that is a canarypox vaccine. The injectable form of Bordetella vaccine is a cellular antigen extract vaccine.

What is cerumen? Ear wax A mucoid substance secreted by the stomach The oily part of the tear film Oil produced by sebaceous glands

Ear wax Cerumen is the proper term for ear wax. It is the yellowish waxy hydrophobic protective substance that is secreted in the ear canal. Sebum is an oily substance that is produced by the sebaceous glands.

This instrument is most often used to examine which part of the body? (light source that forms skinny to stick in certain areas) Eyes Ears Mouth Nares

Ears

A horse with extended forelimbs and arched and tense back that is painful when turning most likely has which of the following conditions? Fractured shoulder Laminitis Colic Spinal cord compression

Laminitis Laminitis, also known as founder, is a very painful condition in which the lamina in the hoof becomes inflamed. The horse tries to compensate for this pain and postures with an arched back and extended forelimbs in an attempt to take weight off the toes. It is especially painful for the horse when walking or turning on hard surfaces.

What personal protective equipment would be required for a dental cleaning? Eye protection, cap, booties, gloves Mask, eye protection, cap Mask, eye protection, gloves Booties, cap, mask, gloves

Mask, eye protection, gloves For a dental, a face shield or eye protection is needed due to the splatter from dental cleaning. A mask is needed to prevent inhalation of this splatter or mist. Gloves are required to prevent the spread of infection. Dental cleanings are not sterile procedures, so a cap and booties would not be required.

You are helping the neurologist with a Tensilon test in a dog. Tensilon is the trade name for edrophonium chloride. By administering this drug, you are testing for which disease? Hydrocephalus Myasthenia gravis Diskospondylitis Old dog vestibular disease Meningitis

Myasthenia gravis

Opioids are commonly used in veterinary medicine. What is one of the most common side effects of opioids? Diarrhea Increased heart rate Respiratory depression Elevated temperature

Respiratory depression

Choose in order the corresponding acronyms for: once daily, twice daily, three times daily, and four times daily BID, SID, QID, TID SID, BID, TID, QID TID, BID, SID QID SID, BID, QID, TID QID, SID, TID, BID

SID, BID, TID, QID

A dog is having his teeth cleaned and has a discolored upper right third premolar. The doctor would like you to take an x-ray of this tooth. Which patient position would make the radiograph easiest to take? Dorsal recumbency Left lateral recumbency Sternal recumbency Right lateral recumbency

Sternal recumbency

What is the causative agent of Equine strangles? Staphylococcus aureus Equine herpesvirus 1 (EHV1) Moraxella bovis Streptococcus equi

Streptococcus equi

When handling a raptor, what is the anatomic structure that can cause the most damage to a handler? Beaks Tails Talons Wings

Talons

The fetus does most of its growing during which time period? First trimester Second trimester Third trimester It grows at the same rate throughout gestation

Third trimester

Which parasite is commonly transmitted transplacentally to puppies? Cystoisospora Toxocara Trichuris Dipylidium

Toxocara

The lining of the urinary bladder is composed of which cell type? Transitional cell Columnar cell Squamous cell Parietal cell

Transitional cell

An incision made from xyphoid to pubis would be found where? Over the dorsal hip region Ventral chest over the sternum Ventral midline Ventral cervical region

Ventral midline

Which area of the horse is generally referred to as the withers? Where the dorsal neck joins the thorax The area ventral to the neck and extending between the forelimbs The region between the tuber coxae The male genitalia

Where the dorsal neck joins the thorax

You are doing a fecal flotation and observe these eggs under the microscope. They have bipolar plugs; one plug on each end (footballs). What are these? Whipworm eggs Hookworm eggs Roundworm eggs Tapeworm eggs

Whipworm eggs The bipolar plugs on the eggs are typical of Trichuris vulpis (whipworm).

Which of the following would you use to induce emesis in cats? Apomorphine Naltrexone Activated charcoal Xylazine

Xylazine The correct answer is xylazine. This is an alpha-2 agonist and produces emesis reliably in the cat.

What is the normal sulcus depth of the teeth in dogs when examining with a periodontal probe? 0-1 mm 0-3 mm 3-5 mm 3-8 mm

0-3 mm

A 60-kg dog needs to be sedated with dexmedetomidine and butorphanol. The doctor would like a dose of 10 ug/kg for dexmedetomidine and 0.1 mg/kg for butorphanol. Dexmedetomidine comes in a 1 mg/ml solution, and butorphanol is in a 10 mg/ml solution. How many milliliters of each medication will need to be administered for the patient's sedation? 60 ml of dexmedetomidine and 0.6 ml of butorphanol 0.6 ml of dexmedetomidine and 0.6 ml of butorphanol 0.6 ml of dexmedetomidine and 6 ml of butorphanol 0.06 ml of dexmedetomidine and 0.3 ml of butorphanol

0.6 ml of dexmedetomidine and 0.6 ml of butorphanol For dexmedetomidine, do the following math: 60 kg x (10 ug/kg) = 600 ug Note the units and convert to work with milligrams as the solution is in milligrams per milliliter. 600 ug / (1000 mg/ug) = 0.6 mg 0.6 mg / (1 mg/ml) = 0.6 ml For butorphanol, do the following math: 60 kg x 0.1 mg/kg = 6 mg 6 mg / (10 mg/ml) = 0.6 ml

When performing a direct fecal smear to look for "swimmers" or parasitic trophozoites, the feces cannot be older than which of the following? 5 minutes 12 hours 1 hour 24 hours if not refrigerated

1 hour

A patient is having a seizure in middle of the treatment room. The doctor quickly asks for some diazepam at a dose of 0.5 mg/kg. The patient's body weight is approximately 24.2 lbs. How many milliliters do you need to draw up? Valium is available as a 5 mg/ml solution. 1.1 ml 8 ml 0.5 ml 2.4 ml

1.1 ml

The veterinarian would like you to give famotidine to a 23 kg Doberman Pinscher that has been hospitalized for vomiting. The dosage is 0.5 mg/kg. Using the image, how many milliliters you will be administering? 1.2 ml 0.12 ml 1.4 ml 11.5 ml

1.2 ml 23 kg multiplied by 0.5 mg/kg= 11.5 mg divided by 10 mg/ml= 1.2 ml

A 6.7 kg Chihuahua has been prescribed a 2 mg/kg/day constant rate infusion of metoclopramide. The metoclopramide is to be added to the intravenous fluids. If the metoclopramide is 5 mg/ml and the Chihuahua's fluid rate is 17 ml/hr, how many milliliters of metoclopramide should be added to a 250 ml bag of intravenous fluids? 1.6 mL 4.6 mL 13.4 mL 0.56 mL

1.6 mL Even though this may not seem like an entry level question, there may be a constant rate infusion question on the VTNE. Let's walk through the math in this question: 6.7 kg x 2 mg/kg/day = 13.4 mg/day 13.4 mg/day / 24 hours/day = 0.56 mg/hour 0.56 mg/hr / 5 mg/ml (conc. of metoclopramide) = 0.11 ml metoclopramide/hour 250 ml/bag / 17 ml/hr = 14.7 hours of intravenous fluids/bag 14.7 hours/bag x 0.11 ml metoclopramide/hour = 1.6 ml of metoclopramide/250 ml bag

A 1200-gram bird needs to receive an injection of an antibiotic. The antibiotic dose to be administered is 15 mg/kg. It comes in a 10 mg/ml solution. How many milliliters does this bird need? 1800 ml 9 ml 1.8 ml 18 ml

1.8 ml

Removal of sutures or staples before an incision has healed can result in unwanted complications. What is the typically recommended time window for suture/staple removal? 20-28 days after surgery 5-7 days after surgery 14-19 days after surgery 10-14 days after surgery

10-14 days after surgery Incisions will usually be sufficiently strong by 10-14 days after surgery and will no longer require skin sutures/staples. Leaving the sutures/staples in longer than 14 days can result in unwanted suture or staple reaction and is of no benefit to the incision.

What is the normal temperature for a cat or dog? 100-102.6F 95.7-103.5F 103.5-104.5F 97.8-98.5F

100-102.6F

A patient's fluid rate is currently set at 150 ml/hr. The doctor asks you to decrease the fluid rate by 25%. What is the new rate? 37.5 ml 112.5 ml 90 ml 87.5 ml

112.5 ml First, we need to determine how much 25% of 150ml is. 150 ml x 0.25 = 37.5 ml Now, subtract the answer above from the original fluid rate to determine the new rate. 150 ml - 37.5 ml = 112.5 ml

How many pairs of cranial nerves are there? 15 12 10 8

12 There are twelve pairs of cranial nerves. They are designated by Roman numerals as follows: I - Olfactory nerve II - Optic nerve III - Oculomotor nerve IV - Trochlear nerve V - Trigeminal nerve VI - Abducens nerve VII - Facial nerve VIII - Vestibulocochlear nerve IX - Glossopharyngeal nerve X - Vagus nerve XI - Accessory nerve XII - Hypoglossal nerve

Typically, when should a dog or cat be offered food following gastrointestinal surgery? 6-12 hours after surgery 3-5 days after surgery 12-24 hours after surgery 48-72 hours after surgery

12-24 hours after surgery Most animals undergoing intestinal surgery should be offered food and water within 12-24 hours after surgery. Further delaying the delivery of nutrition may result in slower healing. Trying to feed a patient within 6-12 hours after major surgery is usually unrewarding because the patient is still recovering from anesthesia and is frequently under the effects of strong pain medications which contribute to inappetence and nausea.

You are monitoring an adult cat under anesthesia. Since induction, the heart has been slowing down. What is the approximate lowest acceptable heart rate for this cat? 140 120 180 160

120 If the heart rate drops below 100-120 beats per minute in an anesthetized cat, the heart rate needs to be increased and the anesthetic depth should be evaluated. If appropriate, atropine may be administered in this situation, and anesthetic gas turned down if possible.

A 20-kg patient is being sent home on famotidine to help with potential esophagitis. How many 10 mg tablets should be sent home with the owner, if the patient requires 0.5 mg/kg PO BID x 7 days? 7 21 14 40

14 20 kg x (0.5 mg/kg) = 10 mg (This is the amount the patient needs per dose, which is equivalent to 1 tablet.) 1 x 2= 2 (This is the number of pills needed per day, since the dosing is BID.) 2 x 7= 14 (This is the number of pills needed for the 7 day treatment course.)

Pregnancy check in the mare can be performed via ultrasound as early as which of the following? 60 days 14 days 42 days 30 days

14 days

What is the maximum volume that can be injected intramuscularly in the larger muscle groups of an average adult horse? 25 ml 5 ml 3 ml 15 ml

15 ml

The heart rate of an adult cat should range between which of the following? 80-120 bpm 30-60 bpm 120-150 bpm 150-220 bpm

150-220 bpm An adult cat typically has a resting heart rate between 150-220 bpm. Keep in mind that if the cat is very excited the heart rate might be faster. A dog's heart rate ranges between 70-160 bpm. As with the cat, there may be variation. Remember that puppies and kittens will have faster heart rates.

A 65-pound Golden Retriever "Marley" presents with a history of gastroenteritis over the past 48 hours. He is approximately 5% dehydrated and the attending veterinarian would like to hospitalize him on IV fluids for the day. Calculate the volume deficit in this patient. 3 L 480 mL 325 mL 1500 mL

1500 mL 65 pounds/ 2.2 pound/kg (pounds cancel out) = 30 kg 30 kg X 0.05 (which is 5%) = 1.5 L volume deficit Since 1.5 L is not an answer choice, continue converting into mL: There are 1000 mL in 1 liter: 1.5 L X 1000mL/liter (liters cancel out)= 1500 mL total volume deficit

How tall would a horse that measures 66 inches be, in hands? 14 - 3 hands 15 hands 16 - 2 hands 16 hands

16 - 2 hands A hand is equivelent to 4 inches. Thus 66 divided by 4 = 16 hands + 2 inches or 16 - 2 hands.

A client calls and would like her dog's phenobarbital refilled. He is currently getting 1/4 grain every 12 hours. How many milligrams is in 1/4 grain? 32.4 mg 58.0 mg 64.8 mg 16.2 mg

16.2 mg 1 grain is equal to 64.8 mg. Therefore, 1/4 of 64.8 mg is equal to 16.2 mg. Phenobarbital is commonly dispensed in "grains".

Which of the following catheters would allow the largest volume of fluids to be given over the shortest amount of time? 18g x 2" 22g x 3/4" 16g x 1" 24g x 1"

16g x 1" Catheters that are shorter in length and larger in diameter can deliever the largest volume of fluids in the shortest amount of time. This is known as Poiseuille's Law.

Which of the following people is not allowed to restrain a patient for a radiograph? 17-year old male veterinary assistant that is working at the clinic for the summer 63-year old veterinarian who owns the practice 22-year old female pre-veterinary student that is volunteering to get experience for her vet school application 26-year old female veterinary technician who is 5-months pregnant

17-year old male veterinary assistant that is working at the clinic for the summer

Determination of adequate passive transfer of maternal antibodies (IgG) to a neonatal foal is typically performed at what age? 6-8 hours of age 10-12 hours of age 2-4 hours of age 18-24 hours of age

18-24 hours of age Newborn foals are immunocompetent but have few antibodies in their body when they are born; thus, they rely on acquiring antibodies from their dam's colostrum. In the foal, the IgG should be checked around 18-24 hours of age to allow for the foal to absorb the maternal antibodies that are consumed. The foal's gut "closes" or loses its ability to absorb colostral antibodies around 18 hours of age.

You are making serial dilutions of a sample in the laboratory. You start by making a 1:5 dilution of a serum sample. You then take this new solution and make a serial 1:5 dilution of it. What is the dilution ratio of this new solution? 1:25 1:5 1:50 1:20 1:1

1:25 When calculating the final dilution ratio of a serial dilution, you must multiply the dilution factors. Here 1:5 is multiplied by 1:5 and the final dilution is (1 x 1):(5 x 5) = 1:25

How many mammary glands does the goat have? 1 4 6 2

2

Which is the dental formula for a cat? 2 (I 3/3 C 1/1 P 3/2 M 1/1)= 30 2 (I 3/2 C 2/2 P 3/3 M 0/1)= 32 2 (I 3/3 C 1/1 P 4/4 M 2/3)= 42 2 (I 2/2 C 1/1 P 3/3 M 2/1)= 30

2 (I 3/3 C 1/1 P 3/2 M 1/1)= 30

Approximately what percentage of body weight should a horse consume in forage (i.e. hay) per day? 5% 2% 12% 8%

2% The average horse should consume approximately 2% of its body weight in forage. In a 1000 lb horse, this would equal 20 lbs of hay/day.

A 12-pound cat has presented with diabetic ketoacidosis and is currently in shock. The doctor orders that a 40 ml/kg bolus of 0.9% saline be administered. How many milliliters will this patient receive? 220 ml 36 ml 480 ml 9 ml

220 ml The body weight needs to be converted to kilograms. 12 lb / (2.2 lb/kg) = 5.5 kg 5.5 kg x (40 ml/kg) = 220 ml

A neonatal foal should have his immunoglobulin G (IgG) measured at what time after birth? 7 days 1 hour 24 hours 30 days

24 hours Neonatal foals have a functioning immune system but have minimal antibodies present in the blood when born; therefore, they rely on passive transfer of maternal antibodies by consumption of colostrum from the mare. The IgG should be evaluated in foals between 16-24 hours of age. This is testing for failure of passive transfer.

What is the normal respiratory rate of a cat? 24-42 breaths per minute 12-18 breaths per minute 40-76 breaths per minute 6-14 breaths per minute

24-42 breaths per minute Cats have a resting respiratory rate of approximately 24-42 breaths per minute. Dogs have a resting respiratory rate of approximately 10-30 breaths per minute. Horses have a respiratory rate of approximately 8-16 breaths per minute.

What is the ideal time period to withhold food in large ruminants prior to general anesthesia for an elective procedure? 24-48 hours Ruminants should not be fasted prior to anesthesia 8-12 hours 4-6 hours

24-48 hours Bloat results from continuous fermentation in the rumen when the ability to eructate is absent. In general there is less risk of bloat if cattle are fasted for 48 hours prior to anesthesia. This may not always be possible in emergency situations, but ideally for an elective procedure should fast for at least 24-48 hours.

A patient is slowly being weaned off fluid therapy. Her body weight is 10 kg, and she is currently at a rate of 30 ml/hr. The doctor wishes to decrease her fluid rate by 15%. What is her new rate? 25.5 ml 27 ml 22 ml 4.5 ml

25.5 ml 30 ml x 0.15 = 4.5 ml Now we subtract the answer above from the original rate. 30 ml - 4.5 ml = 25.5 ml is the new fluid rate. Notice that the body weight is not necessary to calculate this rate change.

What is the standard temperature and duration accepted for autoclaving? 250 degrees Fahrenheit for 13 minutes 1200 degrees Fahrenheit for 60 minutes 400 degrees Fahrenheit for 20 minutes 150 degrees Fahrenheit for 55 minutes

250 degrees Fahrenheit for 13 minutes Autoclaving standard is 250 degrees Fahrenheit (121 degrees Celsius) for 13 minutes.

Your radiograph technique chart indicates that you should use a kVp setting of 50 and mAs setting of 12.5. Which of the following would be appropriate? 125 mA, 1/30 sec 125 mA, 1/20 sec 250 mA, 1/5 sec 250 mA, 1/20 sec 250 mA, 1 /10 sec

250 mA, 1/20 sec mAs is a measure of the total exposure. It is the product of mA and time in seconds. Of the choices here, only 250 mA and 1/20 sec yields a product of 12.5 mAs. Generally, it is also best to maximize the mA and minimize exposure time to decrease motion artifacts.

What is the maximum time a fecal can be used for fecal flotation with zinc centrifugation to look for ova and parasites if kept in the refrigerator? 12 hours 3 days 1 week 2 hours

3 days Ideally, a fecal should be collected and performed within 24 hours. However, the maximum time that the feces can still be used if refrigerated is 3 days. Fresher is always better!

How many milliliters of dextrose should be added to a 60 ml syringe of 0.9% NaCl to make a 2.5% dextrose solution? The patient weighs 20 kg, and the dextrose in the hospital comes in a 50% concentration. 3 ml 0.6 ml 120 ml 6 ml 1.5 ml

3 ml 0.025 x 60= 1.5 / 0.50 = x x = 3 ml Note that the body weight is completely unnecessary to calculate the answer to this problem. 200 ml of 25% dextrose should be added to one liter in order to make a 5% solution.

How many milliliters are in 3 Liters? 30,000 mL 3.3 mL 300 mL 3,000 mL

3,000 mL

How many milligrams are in 100 ml of a 3% solution? 0.3 mg 3,000 mg 3 mg 30 mg

3,000 mg A 3% solution is equal to 30 mg/mL. Now take 100 mL X 30 mg/mL = 3000 mg= 3 grams (since there are 1000 mg in 1 gram)

Cats do not have first and second premolars on the mandible. Therefore, what is the number in the Triadan system for the premolar that is closest to the canine on the left mandible? 405 307 404 305 407

307 In the Triadan system, each tooth has a predictable number, even with teeth missing. Therefore, even though cats do not have PM 1 and 2 (numbered 305 and 306 on the left mandible), the third premolar which is the tooth closest to the canine is 307. In all species, the canine tooth ends in 04 and the first molar ends with 09.

"Zoe", a 40 kg Labrador Retriever is recovering after a liver mass resection. The doctor asks you to start "Zoe" on a fentanyl constant rate infusion at 4 micrograms per kilogram per hour (ug/kg/hr). The concentration of fentanyl is 0.05mg/mL. The fluid rate is currently set at 100 mL/hour. How much fentanyl will you add to a liter of Normosol-R? 32 mL 14 mL 8 mL 61 mL

32 mL 1 liter = 1000 1,000 mL / 100mL/hr = 10 hours 4 ug x 40kg x 10 hours= 1600 ug Now convert the ug into mg by dividing by 1,000: 1600 / 1,000 = 1.6 mg 1.6mg / 0.05mg/mL = 32 mL To be truly accurate when adding drugs to a bag like this, you should discard 32 mL of fluids from the bag prior to adding the 32 mL of fentanyl.

You remove an IV catheter from a peripheral vein and place a pressure bandage. What is the maximum amount of time this bandage should stay on? 4 hours 30 minutes 12 hours 1 hour

4 hours Venous return can become compromised if a compression bandage or pressure wrap is left on for more than 4 hours. It is best to remove the pressure bandage once the bleeding has stopped so that the bandage is not forgotten and venous return is not compromised.

How many milliliters of 50% dextrose should be added to a 100 ml solution of Lactated Ringer's Solution to make it 2.5%? 50 ml 5 ml 1 ml 0.5 ml 10 ml

5 ml To answer this question, the following formula should be implemented: C(1)V(1) = C(2)V(2) C is the concentration V is the volume C(1) =0.5 V(1) = x C(2) = 0.025 V(2) = 100 ml 0.5x = 0.025(100) x = 5 ml 5 ml of 50% dextrose should be added to 100 ml in order to make a 2.5% solution.

Tramadol is to be sent home with a 27 pound Beagle following a routine ovariohysterectomy. The veterinarian wants you to fill the prescription giving 2 mg/kg every 8 hours PO for 3 days. Tramadol comes in 50 mg tablets. How many tablets will you dispense? 18 tablets 5 tablets 9 tablets Dispense as many tablets as the owner would like in case the patient is painful for more than 3 days.

5 tablets First you must convert pounds to kilograms dividing 27 by 2.2 = 12.2 kg. 12.2 kg multiplied by 2 mg/kg = 24.5 mg. This patient will get 1/2 tablet by mouth (25 mg) every 8 hours or 3 times daily. 0.5 tablet multiplied by 3 times daily is 1.5 tablets per day multiplied by 3 days is 4.5 tablets. The appropriate number of tablets to dispense is 5.

A ferret presents to your hospital in lateral recumbency. A blood glucose test confirms hypoglycemia, and dehydration is making venous access very difficult. The veterinarian directs you to administer subcutaneous fluids. Which of the following fluids can not be given via this method? 5% dextrose in 0.9% Sodium Chloride Lactated Ringer's Solution 0.9% Sodium Chloride Plasma-Lyte

5% dextrose in 0.9% Sodium Chloride

5 grams is equal to how many milligrams? 50 500 0.05 5,000

5,000 1 gram is equal to 1,000 mg. To calculate this, you multiply 5 by 1,000. 5 grams = 5,000 mg.

Which of the following is not an anticipated finding when evaluating a urine sample after performing a cystocentesis? Struvite crystals Erythrocytes Calcium oxalate crystals 5-10 white blood cells per high-powered field

5-10 white blood cells per high-powered field There should be less than 0-5 white blood cells per high-powered field in a urine sample. Erythrocytes are not uncommon when performing a cystocentesis. This can occur secondary to trauma to the bladder wall when performing urine collection. Keep in mind that an excessive presence of blood can be abnormal. Struvite and calcium oxalate crystaluria is not necessarily abnormal.

Without a heartworm test, what is the latest age that heartworm preventative could safely be started in a dog? 2 months 4 months 6 months 1 year

6 months

A 60 lb Bulldog is to receive intravenous fluids at a maintenance rate. Assuming the maintenance rate can be calculated as 60 ml/kg/day, what would the fluid rate be per hour? Round answer to the closest milliliter/hour. 30 108 150 68 163

68 The answer is 68. 60 lb/2.2 kg/lb = 27.27 kgs in weight. 27.27 x 60 ml/kg = 1636 ml/day. 1636 ml/day / 24hrs/day = 68 ml/hour

How many lumbar vertebrae do dogs and cats have? 13 18 5 7

7

It is estimated that blood urea nitrogen and creatinine levels become elevated when what percentage of kidney function is lost? 50% 75% 90% 25%

75% BUN and creatinine are not considered very sensitive tests of renal function because they often do not become elevated until at least 75% of kidney function is lost. Urine becomes isosthenuric when approximately 66% of kidney function is lost.

Socialization plays an important role in companion animal behavior. At what age is it vital that painful and/or traumatic experiences be avoided? 3-8 weeks 8-10 weeks 6 months 16-20 weeks

8-10 weeks Between the ages of 8 and 10 weeks puppies experience a fear period. Because of this, painful and traumatic situations should be avoided. Between the ages of 3 and 8 weeks puppies socialize best with other dogs. At 16-20 weeks puppies go through a period when they are curious and exploring new environments.

The respiratory rate for an adult horse should generally be: 5-8 breaths per minute 30-40 breaths per minute 40-50 breaths per minute 8-20 breaths per minute

8-20 breaths per minute

Counting 20 heartbeats in a 15 second time period means the patient's heart rate is how many beats per minute? 40 60 80 120

80

A 4-kg puppy is in need of subcutaneous fluids due to dehydration. The doctor prescribed a dose of 20 ml/kg. How many milliliters should this patient receive? 200 ml 24 ml 80 ml 5 ml

80 ml To obtain the answer the following math should have been performed: 4 kg x (20 ml/kg) = 80 ml Notice the kilograms were canceled out.

An adult canine patient is in cardiac arrest and it is your job to administer thoracic compressions. How fast should you give compressions? 180-200 beats per minute 40-60 beats per minute As fast as you can 80-120 beats per minute

80-120 beats per minute

Some orphaned kittens are left on the doorstep of the clinic where you work. You volunteer to take care of these kittens. What volume can the stomach of a kitten or puppy hold on average? 9 mL/kg 30 mL/kg 2 mL/kg 15 mL/kg

9 mL/kg The stomach volume of a kitten is around 9 mL/kg. Giving more than this at a time can lead to regurgitation and causes increased risk of aspiration.

When taking a radiograph of an equine limb, you take a radiograph where the portable X-ray unit source is 90 cm away from the cassette. You then move closer so that the X-ray source is now 30 cm away from the cassette. If no changes were made to the settings of the machine, what will the beam intensity be at the second radiograph relative to the first one? 6 times greater at 30 cm compared to 90 cm 3 times greater at 30 cm compared to 90 cm 9 times greater intensity at 30 cm compared to 90 cm 1/6 intensity at 30 cm compared to 90 cm 1/3 intensity at 30 cm compared to 90 cm

9 times greater intensity at 30 cm compared to 90 cm The inverse square law states the intensity of the x-ray beam is inversely proportional to the square of the distance from the X-ray source. Therefore, the intensity is greater as the cassette is moved closer to the source. To calculate the answer, the intensity of beam 1 is (x/90)^2 and the intensity of beam 2 is (x/30)^2 where x is the number of photons produced by the x-ray machine. The math here works out as follows: (x/30)^2 / (x/90)^2 = [(x/30)/(x/90)]^2 = (90/30)^2 = (3)^2 = 9

"Bernie," a 4-year-old Maltese mix presents to your clinic after having ingested a full milk chocolate bar about 20 minutes prior to his arrival; this means he consumed about 1.55 ounces, according to the package the owner brought. How many milligrams of theobromine would this be? (Hint: Milk chocolate contains about 60mg per ounce of theobromine, the toxin we are concerned about.) Bernie weighs 15 pounds. 240 mg 74 mg 900 mg 93 mg

93 mg This question is asking about how many mg he ingested, so his body weight is irrelevant for this particular problem. He consumed 1.55 oz X 60 mg/oz (oz cancel out)= 93 mg ingested The toxic dose of chocolate can vary per patient. At 20mg/kg may be hyperactive, agitated, and have GI symptoms. Around 50mg/kg may start to have cardiac effects such as tachycardia or arrhythmias. Doses above 60 mg/kg may start to cause neurologic symptoms such as tremoring, seizures, etc.; in severe cases death may occur. To see what effects Bernie might have, let's take it one step further: He weighs 15 pounds/2.2 pounds/kg = 6.8 kg 93 mg ingested/6.8kg = 13.6 mg/kg dose ingested (this is well below the 20mg/kg that may cause hyperactivity) Going off the information about side effects above, Bernie is not likely to have any severe effects other than possibly some GI upset. If he just ate the chocolate, the vet may still wish to induce vomiting, or just monitor.

You are responsible for autoclaving surgical packs in the hospital. You are using a steam sterilizer and heat to a temperature of 250 degrees Fahrenheit (121 degrees Celsius) for 30 minutes. What minimum humidity level is needed in order to provide adequate sterilization? 80-85% humidity 30-35% humidity 97-99% humidity 10-15% humidity

97-99% humidity The relative humidity in the steam sterilizer must reach 97-99% humidity in order to provide proper sterilization.

Which of the following is considered a normal pulse oximetry reading? 98% 90% 50% 1%

98% Pulse oximetry measures oxygen saturation. Normal oxygenation is greater than 96% on average. It can be difficult to always get an accurate reading on an awake animal due to pigmented skin, fur, and/or the cooperation of the animal. Typical places for readings on an awake animal are the ears or lips. During anesthesia, the reading is on the tongue. If the pulse oximetry drops during anesthesia, wet and massage the tongue and take another reading.

A reticulocyte count should be included on a complete blood cell count in which of the following instances? A 13-year old cat with a packed cell volume of 19% A 3-year old dog with a total white blood cell count of 32,000/ul An 8-year old cat with a platelet count of 30,000/ul A 9-year old horse with a total white blood cell count of 2,000/ul

A 13-year old cat with a packed cell volume of 19% A reticulocyte count is performed to assess whether a patient is generating a bone marrow response to anemia. Reticulocytes are immature red blood cells that are released from the bone marrow when the marrow is stimulated to increase red blood cell production. Therefore, anemia (or a low packed cell volume) is the primary reason to perform a reticulocyte count.

Which anesthetic breathing circuit should be used for rabbit anesthesia? As long as you are using a nonprecision vaporizer the breathing circuit does not matter A breathing circuit with plenty of dead space A breathing circuit with minimal dead space As long as you are using a precision vaporizer the breathing circuit does not matter

A breathing circuit with minimal dead space As with small animal anesthesia, as a general rule, any patient under 7 kilograms should be placed on a nonrebreathing circuit. These offer minimal dead space, less resistance to breathing, and provide faster changes in anesthetic depth. Precision vaporizers should always be used in small mammal anesthesia, but this does not change the need for minimal dead space.

What is a pneumocystogram? A radiograph of the bladder taken after intravenous injection of positive contrast that is excreted via the kidneys Fluoroscopy of the kidney and bladder taken after intravenous injection of positive contrast that is excreted via the kidneys A radiograph of the bladder taken after direct injection of positive contrast A radiograph of the bladder taken after having had air injected into it

A radiograph of the bladder taken after having had air injected into it A pneumocystogram is a radiograph of the bladder taken after insertion of a urinary catheter, draining of the urine, and injection of air (negative contrast) into the bladder via the catheter. This procedure allows the bladder wall to be visible and makes certain types of bladder stones more easily viewable. A radiograph of the bladder taken after direct injection of positive contrast is called a cystogram or a positive-contrast cystogram. A radiograph of the bladder taken after intravenous injection of positive contrast that is excreted via the kidneys is called an intravenous pyelogram (IVP) or excretory urogram.

Which organ is responsible for producing cortisol? Pancreas Liver Kidney Posterior pituitary Adrenal gland

Adrenal gland The adrenal gland is responsible for producing specific hormones in the body, including cortisol, epinephrine, and aldosterone.

When is the appropriate time to immerse surgical instruments in surgical milk? Just prior to placing them in the ultrasonic cleaner After being cleaned in the ultrasonic cleaner After a preliminary rinsing of the instruments Immediately after surgery

After being cleaned in the ultrasonic cleaner

Which of these is an effective drug to induce emesis in the dog? Apomorphine Xylazine Azathioprine Magnesium hydroxide

Apomorphine The correct answer is apomorphine. Apomorphine is an opioid dopaminergic agonist that acts on the chemoreceptor trigger zone to induce vomiting in dogs. It can be administered IM, SC, IV, or in the conjunctival sac. Xylazine is a fairly effective emetic in the cat but is not used in the dog for this purpose. Azathioprine is an immunosuppressive drug not used to induce vomiting. Magnesium hydroxide or Milk of Magnesia is a cathartic but is not used for vomiting.

What is the average gestation length of a cat? Approximately 60 days Approximately 90 days Approximately 290 days Approximately 340 days

Approximately 60 days

When performing abdominal surgery, which of the following instruments is frequently used retract the abdominal wall? Meyerding retractor Malleable retractor Finochietto retractor Balfour retractor

Balfour retractor

Circling is usually a result of which type of disease? Renal Brain Cardiac Spinal cord

Brain

Chyme is a part of which bodily function? Coagulation Motor movement Digestion Protection of the ear canal

Digestion

Which method is used to determine the sex of young mice? Palpation of testicles in males Distance between the anus and genitalia Penile extrusion Tail length

Distance between the anus and genitalia Anogenital distance is the best way to determine whether juvenile mice are male or female.

A dog in respiratory distress should not be placed in which position for radiographs? Left lateral Right lateral Dorsal Sternal

Dorsal It is contraindicated to place an animal in respiratory distress on its back. Doing so makes it more difficult for the patient to fully expand their lungs.

What is the most common location for placement of an arterial catheter (a-line) in a dog? Radial artery Pulmonary artery Femoral artery Dorsal metatarsal artery Brachial artery

Dorsal metatarsal artery

The most common method of identification of pigs is what? Ear notching Lip tattoo Freeze branding Ear tags

Ear notching

The radiograph cassette is usually placed on the tabletop to radiograph which of the following areas in a medium-sized dog? Spine Abdomen Thorax Elbow Pelvis

Elbow Tabletop technique is usually used to image structures that are not too thick (usually <10cm thick) because there is decreased scatter, and a grid and Bucky are not needed. In this case, the elbow is the only structure on this list that is not thick enough to warrant the use of a grid and Bucky.

Which breed almost always needs a Cesarean section to give birth? Bedlington Terrier English Bulldog Yorkshire Terrier Chihuahua

English Bulldog

In which species will you never perform a reticulocyte count? Canine Equine Bovine Feline

Equine Horses do not release reticulocytes (immature erythrocytes) into circulation.

In which situation would perioperative antibiotics NOT be indicated? Surgery on a patient with hyperadrenocorticism Surgery on a patient with diabetes mellitus Excisional biopsy of a 2cm mass An orthopedic procedure lasting 1 hour Surgery on a patient treated with glucocorticoids

Excisional biopsy of a 2cm mass Diabetes mellitus, hyperadrenocorticism, and chronic steroid therapy can all result in immunosuppression; these patients should be treated with perioperative antibiotics unless the surgeon specifically requested that an antibiotic not be used in the event that an intraoperative sample is needed for bacterial culture. Any patient having an orthopedic procedure involving the placement of implants should have perioperative antibiotics. A clean procedure performed under 90 minutes in an otherwise healthy and immunocompetent patient does not require perioperative antibiotics. An excisional biopsy of a 2cm mass would fall under this category.

A dog has been brought into your clinic showing stage two signs of rabies. What are the animal's symptoms? Paralytic, excessive salivation, and change in bark Animal will be asymptomatic Excitability, viciousness, biting, and snapping Sudden change in behavior, unusually shy or nervous, seeks solitude

Excitability, viciousness, biting, and snapping Stage two of rabies is the furious stage. Animals will bite, snap, and attack for no reason, spreading the disease in their saliva though bite wounds. Stage one is the prodromal stage which is characterized by changes in attitude. The animal may become shy, nervous, and apprehensive. Stage three of the disease is the paralytic stage. Muscles will become uncoordinated and paralysis sets in. The animal will lose the ability to swallow and excessively salivate.

A client's dog has been diagnosed with a splenic mass that requires surgical removal. The dog has also been previously diagnosed with liver disease. The owner has pet insurance and is submitting a claim for the laparotomy. She is afraid the insurance company will deny the claim because of the liver disease and asks you to omit that from the medical record until the claim is approved. Of the following, which would be the most appropriate action to take? Explain to the owner that the medical record can not be altered but you can omit the liver disease from the claim form. Explain to the owner that by law you can not change the medical record. Let the owner fill out the claim form by herself and you will be absolved of any responsibility of committing insurance fraud. By law you must report attempted fraud to the insurance company.

Explain to the owner that by law you can not change the medical record.

This instrument would be most useful in an operation involving which organ?(mini scissors) Bladder Intestine Liver Eye

Eye The image shows iris scissors. Iris scissors, hence their name, are often used during ocular surgery. They may be used on other tissues and are a small sharp scissor.

Where on a primate's body are they tested for tuberculosis? Eyelid Leg Arm Foot

Eyelid Primates are tested for tuburculosis with an intradermal skin test on the eyelid. This allows the reading to be done without having to restrain the animal. It is much safer for personnel and less stress for patient.

The term pseudocyesis means which of the following? Resembling a cystic structure Infertility Ovarian cysts False pregnancy

False pregnancy

What are Omega 3 and Omega 6? Fatty acids Amino acids Antifungals Fat soluble vitamins

Fatty acids Omega 3 and Omega 6 are types of essential fatty acids (meaning they are needed in the diet because the body does not produce them). These acids are needed in immune and inflammatory responses, certain hormone production, blood clotting, and cell growth. They are used in veterinary medicine most often for skin disease and as an arthritis supplement.

What procedure performed in horses includes filing and smoothing their teeth? Quidding Floating Cribbing Raking

Floating

A client who feeds his dog "ad libitum" follows which schedule? Once daily Free choice Multiple small meals throughout the day Twice daily

Free choice Ad libitum feeding means free choice feeding, or leaving the food out at all times for the animal to eat when he wants. SID = Once daily BID = Twice daily

A thoracic radiograph should be taken at which time? Full inspiration Diastole Full exhalation Systole

Full inspiration

What is the number one way to prevent transmission of disease in the large-animal hospital setting? Hand hygiene Gowning Foot baths Separation of species

Hand hygiene

Which bone is between the scapula and radius? Central tarsal bone Tibia Ulna Humerus

Humerus The humerus lies distal to the scapula and proximal to the ulna. The tibia and central tarsal bone are bones of the hind limb.

Which of the following can cause severe gastritis and should be used with extreme caution when inducing vomiting? Hydrogen peroxide Xylazine Apomorphine Hydromorphone

Hydrogen peroxide

What is the causative agent of what is commonly known as "Ich" in fish? Ichthyophthirius multocida Ichthyophthirius multifiliis Ichthyophthirius magnum Ichthyophthirius multifilementum

Ichthyophthirius multifiliis

Which medication should never be administered to a turtle? Enrofloxacin Ivermectin Ketoconazole Ampicillin

Ivermectin Ivermectin can cause depression, paralysis, coma, or death in chelonians (turtles, tortoises, etc.). It is believed that they have a more permeable blood-brain barrier. Ivermectin should never be given to this species.

What is the most common location of an intravenous catheter in the horse? Cephalic vein Saphenous vein Auricular vein Femoral vein Jugular vein

Jugular vein The jugular vein is very accessible and carries a low risk of injury to the person placing the catheter when compared to trying to place a catheter on a limb. Jugular veins are large and easy to find along the neck of the horse as well as easy to maintain.

Which of the following is an abnormal finding in a free catch of urine in a dog? Bacteria Bilirubin Ketones Calcium oxalate crystals

Ketones The observance of ketones is always abnormal. This suggests that the patient is diabetic and is unable to take glucose out of the blood stream and into cells for metabolism. In rare cases, starvation can cause mild ketonuria in a patient that does not have diabetes. The presence of bacteria in a free catch is always a possibility, as a free catch of urine is never sterile. Bilirubin and calcium oxalate crystals are normal findings in a sample of urine.

What clinical syndrome can result from bedding a horse on shavings made from the Black Walnut (Juglans nigra) tree? Nothing/no Effect Laminitis Foot abscessation Contact alopecia Contact dermatitis

Laminitis Contact with Black Walnut shavings can cause laminitis in the horse within 10-12 hours of being placed on the bedding. Severity of laminitis can be mild to severe and may also be accompanied by limb edema.

If the serum at the top of a microhematocrit tube appears cloudy, how is it described? Normal Icteric Hemolyzed Lipemic

Lipemic

Which bacteria are known for causing a head tilt and neurologic symptoms in ruminants? Listeria E. coli Salmonella Campylobacter

Listeria

In dogs, which of the following are all permanent teeth with 2 roots? Mandibular premolars 2, 3, and 4, and molar 1 Maxillary premolars 2, 3, and 4, and molar 1 Maxillary molars 1 and 2 Mandibular premolar 1 and molars 2 and 3

Mandibular premolars 2, 3, and 4, and molar 1 While exceptions do occur, the following are generally accepted and true. In dogs, all of the permanent incisors and canine teeth have 1 root. In the mandible, premolar (PM) 1 has one root. PM 2, 3, and 4 all have two roots. M 1 and 2 have two roots. M3 can have one or two roots. In the maxilla, PM 1 has one root. PM 2 and 3 have two roots, although PM 3 can sometimes have 3 roots. PM 4 and M1 and 2 all have 3 roots.

Which of the following drugs is considered an anti-emetic? Metoclopramide Methimazole Methocarbamol Methoxyflurane

Metoclopramide Metoclopramide (or Reglan) is an anti-emetic that acts on the dopamine receptors that communicate with the vomiting (emetic) center of the brain.

A dog or cat with a hypomotile intestinal tract might most benefit from which of the following medications? Metoclopramide Metronidazole Omeprazole Famotidine

Metoclopramide Metoclopramide (Reglan) is used in cases of hypomotility due to its prokinetic properties. Cisapride is another drug that is used to treat hypomotility.

Which cell structure is responsible for producing ATP? Lysosome Mitochondria Golgi complex Endoplasmic reticulum

Mitochondria The mitochondria are responsible for producing the ATP necessary for all cellular functions. The endoplasmic reticulum acts as a transportation system for proteins to be moved as well as for protein synthesis. The Golgi complex modifies and packages compounds that came from the endoplasmic reticulum. They are then released into the cytoplasm for use within the cell. Lysosomes contain enzymes which digest intracellular bacteria and non-functional organelles.

A patient in ventricular fibrillation needs to be defibrillated, and you are assisting. Someone yells "clear" before the defibrillation. What do you do? Move away from the patient and table Hold the ECG clamps with your fingers to prevent interference from the shock Hold the patient firmly on the table Touch the table so you are grounded

Move away from the patient and table

What type of organism is typically detected with an acid-fast stain? Endospores Fungi Mycobacteria Mycoplasma Gram-negative

Mycobacteria Mycobacteria and Nocardia species are usually detected with an acid-fast stain.

Where might you find the parasite Oestrus ovis? Under the skin of a rabbit Under the skin of a cow Nasal cavity of the sheep In the ear of a goat

Nasal cavity of the sheep Oestrus ovis is the sheep nasal bot. Flies deposit their larvae around the nares. They enter the nasal cavity and migrate to the sinuses where they mature. They then leave the nose and pupate in the soil. The word "ovis" should have been a hint that the parasite is likely found in the sheep (ovine). Cuterebra is a bot that can be found under the skin, notably under a hole found on the skin in several different species. Hypoderma bovis is the warble fly whose larvae cause "warbles" or bumps under the skin in cattle.

What is OSHA? Occupational Staff and Health Agency Occupational Safety and Health Association Obligatory Safety and Health Association Occupational Safety and Health Administration

Occupational Safety and Health Administration OSHA (Occupational Safety and Health Administration) is the main federal agency charged with the enforcement of safety and health legislation. They set standards for safety and practice, and veterinary services falls under their regulation.

What is the common term for Psittacosis (Ornithosis)? Psittacine beak and feather Parrot fever Avian pox Aspergillosis Pacheco's

Parrot fever Psittacosis is also known as Parrot fever or Chlamydiosis. The treatment of choice is doxycycline. It is a reportable disease, and any bird suspected of having Psittacosis should be immediately isolated. In birds, C. psittaci may manifest itself as an upper respiratory infection with nasal, and/or ocular discharge, diarrhea, or a combination of all three. In some cases, birds may be infected but show no signs. It is caused by the obligate intracellular bacterium Chlamydia psittaci. It is especially found in cockatiels and parakeets. After exposure, the incubation period can vary from 3 days to several weeks and can survive in the environment for several months. It is zoonotic, especially in immunocompromised people, and protection should be worn when handling a suspected case.

Which anatomic structure is the passageway for both the respiratory and digestive systems? Esophagus Pharynx Bronchi Tongue Larynx

Pharynx The pharynx leads from the oral and nasal cavities to the larynx and esophagus. It helps to get food to the esophagus and air to the lungs.

A Gram stain causes Gram-negative bacteria to turn which color? Pink Green Blue Orange Purple

Pink

What is cholestasis? Inability of the pancreas to produce enzymes Rupture of the gall bladder Enlarged liver lobes Bladder stone stuck in the urethra Poor flow of bile from the liver to the GI tract

Poor flow of bile from the liver to the GI tract Cholestasis is a condition in which the bile has difficulty flowing from the liver into the duodenum. The prefix chole- means bile and the term stasis means to stand still. The enzymes associated with cholestasis are alkaline phosphatase (ALP) and gamma glutamyltransferase (GGT).

Which injectable medication comes in an emulsion containing egg lecithin and soybean oil? Thiopental Metoclopramide Propofol Ketamine

Propofol

What instrument is used to measure the urine specific gravity? Manometer Barometer Glucometer Refractometer

Refractometer

Casts in the urine generally indicate damage in what part of the urinary system? Urethra Bladder Renal tubules Glomerulus Ureters

Renal tubules

Which of the following animals do not have canine teeth? Pigs Rodents Ferrets Horses

Rodents Rodents do not have canine teeth. Horses, pigs and ferrets all have canine teeth.

Which type of fracture occurs at the growth plate and is seen in young animals? Oblique Transverse Comminuted Salter-Harris Greenstick

Salter-Harris

Cardiac Output (CO) is determined by what variables? Heart rate X Systemic vascular resistance Systolic pressure X Stroke volume Systemic vascular resistance X Systolic pressure Stroke volume x Heart rate

Stroke volume x Heart rate

You find coccidia (Cystoisospora) on a fecal float for a puppy that presents with mucoid diarrhea with streaks of blood. What medication is used to treat coccidia? Sulfadimethoxine Pyrantel Praziquantel Metronidazole Fenbendazole

Sulfadimethoxine

You are taking indirect blood pressure readings on a cat with hyperthyroidism. Which is true when choosing a cuff for the patient? The cuff diameter should be about 40% the circumference of the limb The cuff diameter should be about 75% the circumference of the limb Indirect blood pressure measurements do not involve using a cuff The size of the cuff is not significant if placed tightly enough

The cuff diameter should be about 40% the circumference of the limb Oscillometric and Doppler readings are indirect blood pressure measurements and are most commonly used in the clinical setting. A blood pressure cuff is used, and the general rule is that the cuff diameter should be about 40% of the limb circumference. It is important that the cuff be proportional to patient size. If the cuff is too large, the reading will be inaccurately low. If the cuff is too small, the reading will be inaccurately too high. It is best to take an average of several readings for the most accurate measurement.

Which of the following is considered part of the plantar surface? The top surface of the forepaw The top surface of the hindpaw The underside of the hindpaw The underside of the forepaw

The underside of the hindpaw

Where is the thyroid gland located? Cranial to the bladder Above the right kidney Near the hypothalamus in the brain Ventral neck

Ventral neck The thyroid gland is located in the ventral neck region.

Under which circumstances are pigs most likely to bite each other's tails? When a pig has an open wound When they are fed a grain-only diet When there is more than one male in a group When the facility is over-crowded

When the facility is over-crowded

Which of the following pieces of information should the technician be most aware of prior to inducing a patient for anesthesia? When the patient defecated last When the patient urinated last When the patient ate last When the patient last had an anesthetic procedure

When the patient ate last Patients that are anesthetized with a full stomach have a higher incidence of vomiting or regurgitating at the time of anesthesia. Therefore, it is very important to have patients fasted prior to anesthesia. Depending on the species, some patients may be fasted anywhere between just a few hours to over 12 hours.

Pyuria indicates which of the following? Protein in the urine Bilirubin in the urine Bacteria in the urine White blood cells in the urine Red blood cells in the urine

White blood cells in the urine Pyuria indicates white blood cells in the urine (Py=Pus, uria=urine). Pus is an accumulation of white blood cells. Hematuria is red blood cells in the urine. Proteinuria is protein in the urine. Bilirubinuria is bilirubin in the urine. Bacteriuria is bacteria in the urine.

You want to give a 22 pound cat a 5 mg/kg dose of a drug. The drug comes as a 2.5% solution. How many milliliters should you give the cat? 20 .2 8.8 4.4 .4 2

2

You are opening the boxes from today's shipments and find soda lime. What is this used for? Suction canister Autoclave Anesthesia machine Vaccine refrigerator

Anesthesia machine Soda lime or ("Sodasorb") is used in the anesthesia machine. In the machine, these granules absorb carbon dioxide exhaled by the patient in a rebreathing anesthesia circuit.

Which structure is NOT a part of the equine large intestine? Pelvic Flexure Cecum Left Dorsal Colon Ileum Right Dorsal Colon

Ileum The ileum is part of the small intestine. The large intestine of the horse starts with the cecum and continues with the right ventral colon, sternal flexure, left ventral colon, pelvic flexure, left dorsal colon, diaphragmatic flexure, right dorsal colon, transverse colon, and small colon.

Which is true regarding epinephrine? It makes the blood pressure drop It causes bradycardia It is a sympathomimetic It may cause a low blood sugar

It is a sympathomimetic A sympathomimetic mimics the sympathetic nervous system. Epinephrine is also "adrenaline"; this stimulates the fight-or-flight response. When adrenaline spikes, heart rate increases, blood pressure jumps, etc. It may cause an increase in the blood sugar (hyperglycemia).

You are looking microscopically at a swab of the ear canal from a cat that has been head shaking and scratching at the ears. You identify an 8-legged mite. What is the name the ear mite? Trichodectes canis Otodectes cynotis Ctenocephalides felis Sarcoptes scabiei Demodex gatoi

Otodectes cynotis

What represents depolarization from the SA node through the atria on an electrocardiogram (ECG)? QRS complex T wave ST segment P wave

P wave

Chronic exposure to which hormone leads to a pyometra? Progesterone Luteinizing hormone Testosterone Estrogen

Progesterone Pyometra is often related to increased progesterone levels and their effect on the lining of the uterus. Prolonged exposure to estrogen can cause bone marrow suppression.

A patient with which condition would be most likely to receive weekly subcutaneous fluid administration? Renal failure Cardiomyopathy Diabetes mellitus Hyperadrenocorticism

Renal failure

What is collected when performing an arthrocentesis? Synovial fluid Blood Bone marrow Urine

Synovial fluid Arthrocentesis is inserting a needle into the joint space to collect synovial fluid (joint fluid). This procedure is performed by a veterinarian.

Which of the following is a clinical sign you would see in a dog with chocolate toxicosis? Hyperpigmentation Tachycardia Bradycardia Swollen tongue Paralysis

Tachycardia

You are in the clinic parking lot when you are approached by 2 large dogs who are about to attack you. They are about 25 yards away. You should immediately do which of the following? Run towards them and yell in a loud voice Flail your arms and hit at them when they approach Stand completely still Tuck yourself into a ball on the ground and protect your face and neck Run as fast as you can

Tuck yourself into a ball on the ground and protect your face and neck

Where would you find a Cuterebra parasite? In the nasal cavity In the anterior chamber of the eye In the intestines Protruding from the anus Under the skin

Under the skin

You are training a radiology technician on methods of restraining patients using sandbags and positioning devices that will allow her to step out of the radiology suite during the radiograph exposure. The radiology technician asks you why this is necessary, since she always wears her dosimetry badge and has always been well below the established exposure limits. What important principle of radiation safety dictates that further efforts such as these should still be made to decrease radiation exposure to the radiology technician? Rad ALARA MPD Rem Time, distance, and shielding

ALARA

Which of the following tests can be used to evaluate a patient for immune-mediated hemolytic anemia? Latex agglutination Coomb's test Coggin's test Agarose immunodiffusion

Coomb's test A Coomb's test is used to detect the presence of immunoglobulins against the patient's red blood cells. The Coggin's test is used to diagnose equine infectious anemia. Latex agglutination tests and agarose immunodiffusions are tests that are used to detect the presence of specific patient antibodies.

A deep skin scraping wherein the skin is scraped until capillary blood is seen is generally necessary to recover which of the following parasites? Demodex Cheyletiella Psoroptes Chorioptes

Demodex Surface dwelling mites such as Sarcoptes, Cheyletiella, Psoroptes, and Chorioptes only require a superficial scraping to collect loose scales or crusts. A deeper scraping is necessary to recover Demodex mites which live deeper in the follicles.

Which of the following mites that can be found on dogs is not contagious? Sarcoptes Otodectes Psoroptes Demodex Cheyletiella

Demodex The correct answer is Demodex. Demodex is a normal inhabitant of the skin and causes disease only when there is either a genetic predisposition or systemic disease that allows Demodex to develop into disease. Therefore it is not contagious in dogs. Demodex gatoi in cats is thought to be contagious amongst cats.

You are assisting in radiographing a four-year-old German Shepherd. The dog has been limping on his left hindlimb. The veterinarian elicits a "drawer motion" in the limb during physical exam, and the doctor's suspicion is confirmed by the radiographs. What diagnosis is supported by these findings? Osteosarcoma Hip dysplasia Cranial cruciate ligament rupture Luxating Patella

Cranial cruciate ligament rupture Cranial cruciate ligament injuries are the most common orthopedic problem seen in dogs. Based on the radiographic finding of stifle effusion of the left stifle noted by the displacement of the infrapatellar fat pad, this patient likely has a cranial cruciate ligament rupture. The clue in this question is the veterinarian palpating "drawer motion." This is most commonly associated with a cranial cruciate rupture. (You would not be expected to diagnose this patient, but you should know the general fact that drawer motion is usually caused by a ruptured cranial cruciate ligament.)

A 10-year old Chow Chow needs a whole blood transfusion due to anemia from chronic renal failure. The patient has received multiple blood transfusions over the last 2 years. What blood test is needed prior to initiating the transfusion? Tick-titer Coagulation profile Total protein Blood type Crossmatch

Crossmatch Patients who have had multiple blood transfusions are more likely to have a reaction to subsequent blood product transfusions. In order to reduce the likelihood of a reaction, a major and minor crossmatch should be performed prior to administration. To know what donors would most likely be compatible, the patient's blood type could be determined. Dogs with the same blood type still require a crossmatch, especially if the dog has received prior transfusions.

Ringworm is the common name for which of the following types of organisms? Yeasts Trematodes Ascarids Dermatophytes Staphylococci

Dermatophytes Ringworm is the common name for the cutaneous fungi called dermatophytes. This group of fungi includes Microsporum canis, Microsporum gypseum, and Trichophyton mentagrophytes.

What is new methylene blue stain is used mostly for? Staining yeast Determining if bacteria are gram-positive Determining if bacteria are gram-negative Determining morphology of cells Determining if bacteria are acid-fast

Determining morphology of cells

You have been working in the barn with baby calves that have been having severe diarrhea. You now are having a severe watery diarrhea. Which parasite would be most likely for you to have contracted from these calves? Cryptococcus Toxocara Cryptosporidium Coccidia

Cryptosporidium Cryptosporidium parvum is a protozoan parasite that causes gastroenteritis in animals and humans. People with an adequate immune system have self-limiting disease, but immunocompromised people can develop life-threatening complications from this parasite. Some studies show that greater than 44% of calves with diarrhea can have this parasite isolated. Thus, this is a very important cause of calf scours.

What zoonotic pathogen might a technician contract from treating a 10 day old dairy calf with diarrhea? Francisella tularensis Leptospira pomona Staphylococcus aureus Cryptosporidium parvum E. coli 0157

Cryptosporidium parvum Cryptosporidium is a major cause of calf diarrhea, and just a few oocysts can be infective to humans and cause diarrhea lasting several days.

Which type of cell should not be found in normal joint fluid? Neutrophils Lymphocytes Macrophages Erythrocytes

Erythrocytes Normal synovial fluid should contain no erythrocytes and moderate numbers of nucleated cells (<3000/ul). The cells are normally about 95% mononuclear cells, with approximately equal numbers of lymphocytes and macrophages. There should be <10% neutrophils and very few eosinophils. Blood contamination can make it difficult to interpret synovial cytology.

Which of the following is the intermediate host of Dipylidium caninum and is important in its transmission? Ixodes pacificus Ctenocephalides felis Taenia saginata Otodectes cynotis

Ctenocephalides felis Dipylidium caninum is the common canine tapeworm. The flea (Ctenocephalides felis) is the intermediate host. The parasite is usually acquired by dogs when they groom and ingest fleas carrying the tapeworm cysticercoid. This is important because the diagnosis of tapeworms in a dog may also be a sign that flea control is needed.

Where would you find Anaplasma marginale? White blood cells of horses Urine of sheep Erythrocytes of cattle Platelets of horses

Erythrocytes of cattle Anaplasma marginale is the most prevalent tick-borne pathogen in livestock. This is a rickettsial disease, and the organisms are found in the red blood cells of cattle.

Nerve axons which are covered with a white sheath are known as being which of the following? Dead Slow transmitters Sympathetic Myelinated

Myelinated The myelin sheath is lipid-rich (made of cholesterol and phospholipids) which gives it its white color. The purpose of the myelin sheath is to increase the speed at which impulses can travel.

Which of the following pairs of measurements are equivalent to each other? Nanometer and microliter Cubic centimeter and milliliter Cubic foot and meter Kilogram and kiloliter Kilogram and liter

Cubic centimeter and milliliter A cubic centimeter is the same volume as a milliliter. A liter of water weighs one kilogram, but these terms are not interchangeable because a liter is a volume and a kilogram is a weight. A liter of a heavier fluid would weigh more than a kilogram. A cubic foot is a volume, and a meter is a distance.

How is a bull tested for Trichomonas foetus? Blood culture Wet mount of feces Blood smear Culture of smegma Microscopic exam of sperm

Culture of smegma Trichomonas is a venereal disease of cattle caused by the flagellated protozoan Trichomonas foetus. The prepucial hairs are shaved and the prepuce cleansed and dried. A dry plastic insemination pipette is placed into the prepuce and used to suction and scrape the prepucial membrane and penis to collect approximately 1 mL of smegma for culture. The culture is placed into a vial with 2 mL of saline and then placed on Diamond's medium for culture.

Which syndrome can occur in dogs when long-term corticosteroids such as prednisone are administered? Addison's Hyperthyroidism Cushing's Hypothyroidism

Cushing's Cushing's syndrome, or hyperadrenocorticism, occurs from high levels of cortisol. It can be due to over-production of cortisol by the adrenal glands or can be iatrogenic from long-term or chronic corticosteroid administration. Symptoms of this include a thinning haircoat, polyuria and polydipsia, pot belly, thinning skin, and calcinosis cutis. Long-term corticosteroid administration can also lead to insulin resistance, leading to type II diabetes in some patients.

What is most common anticoagulant used for hematology? Ethylenediaminetetracetic acid Sodium citrate Sodium fluoride Heparin

Ethylenediaminetetracetic acid The correct answer is Ethylenediaminetetracetic acid (EDTA). This is the anti-coagulant found in "purple top" tubes. Gray top tubes contain sodium fluoride and are most often used for accurate assessment of glucose levels. Blue top tubes contain sodium citrate and are used for coagulation testing.

As a technician, it will be your responsibility to administer medications in most situations. Which of the following is not a common medication administered to horses post-operatively? Banamine Dexamethasone Procaine-penicillin G Omeprazole

Dexamethasone Dexamethasone is a steroid and is not a commonly used medication post-operatively. When this medication is noted as a treatment after surgery, the technician should double check with the clinician in charge. Inadvertent administration of steroids can result in immunosuppression and secondary infection at the surgical site. Banamine is a very commonly used non-steroidal anti-inflammatory in horses. Procaine-penicillin G is a commonly used antibiotic which may be used by clinicians post-operatively. Omeprazole is a proton pump inhibitor that decreases the amount of stomach acid production in horses to help keep them from developing gastric ulcers.

Fructosamine levels are used to evaluate the control of which of the following diseases? Cushing's disease Addison's disease Hypothyroidism Hyperthyroidism Diabetes mellitus

Diabetes mellitus The fructosamine level is a measure of the amount of glucose bound to plasma proteins. It goes up when glucose levels are persistently elevated in the blood. It is less subject to the fluctuations seen in blood glucose because plasma proteins have much longer half-lives. Fructosamine levels correspond to the average serum glucose over the previous 1-2 week period. A related test is glycosylated hemoglobin level, which corresponds to average serum glucose over several months.

Chronic administration of prednisone to animals can cause an iatrogenic form of what condition? Cushing's disease Addison's disease Tyzzer's disease Bang's disease

Cushing's disease Cushing's disease (hyperadrenocorticism) naturally occurs when the adrenal glands produce too much cortisol. This can be due to a functional adrenal tumor or due to a functional pituitary tumor that secretes ACTH (adrenocorticotropic hormone) which stimulates cortisol secretion from the adrenal gland. Prednisone is a synthetic glucocorticoid, and chronic administration for conditions such as allergies can lead to iatrogenic Cushing's disease. Addison's disease (hypoadrenocorticism) occurs due to inadequate cortisol levels and can be caused by prompt withdrawal of prednisone after prolonged use. Tyzzer's disease is a bacterial disease caused by Clostridium piliforme. Bang's disease, or Brucellosis, is an infectious disease of cattle caused by the bacteria Brucella abortus. An iatrogenic disease is one caused by a medical or surgical procedure.

Ancylostoma causes which zoonotic disease in humans? Cutaneous larva migrans Crohn's disease Ocular larva migrans Visceral larva migrans

Cutaneous larva migrans Ancylostoma is a hookworm and is the most common cause of Cutaneous larva migrans. This is an intestinal parasite of dogs and cats. In this zoonotic disease, the parasite burrows under the skin in humans. Image shows hookworm eggs. Ocular and Visceral larva migrans are caused by Toxocara canis (roundworm). Crohn's disease is a type of severe inflammatory bowel disease in people with no correlation to our pet patients.

Which of the following would NOT be a potential indicator of intraoperative pain? Increased patient movement Increased respiratory rate Increased heart rate Decreased blood pressure

Decreased blood pressure

A diet high in concentrates or grains may lead to which of the following in cattle? Hyperlipidemia Increased rumen pH Hypocalcemia Decreased rumen pH

Decreased rumen pH Rumenal acidosis (decreased pH) may occur when feeding a high concentrate diet. This is due to microbes fermenting the starches in the grains too quickly such that the pH drops dramatically (less than a pH of 5.5). This increased acidity in the rumen can cause severe damage to the rumenal papillae.

Which of these is the term for the animal that harbors the mature, sexual stage of a parasite? Intermediate host Paratenic host Definitive host Reservoir host

Definitive host The definitive host harbors the adult, sexual stage of a parasite. The intermediate host harbors larval, asexual, or immature forms of a parasite. A paratenic host (also known as a transport host) indicates a type of intermediate host on which a parasite does not undergo development into the next stage. A reservoir host is another vertebrate host for a parasite that serves as a source of infection for people or domestic animals.

What is the most common cause of urate bladder stones in a Sulcata tortoise? Poor thermoregulation Poor lighting Ingestion of sand substrate Dehydration Small habitat enclosure

Dehydration

Which of the following would not be an underlying cause for a urine specific gravity of less than 1.020? Renal failure Diabetes mellitus Dehydration Hypoadrenocorticism

Dehydration

Pre-renal azotemia is most commonly caused by which of the following? Pyelonephritis Urethral obstruction Acute renal failure Chronic renal failure Dehydration

Dehydration Dehydration is the most common cause of pre-renal azotemia. Azotemia is the term for elevated renal values (BUN and creatinine). When a patient is dehydrated, renal values may be elevated. When re-hydration occurs, renal values should return to normal. Renal failure (acute or chronic) causes primary renal azotemia. Post-renal azotemia is most often caused by obstruction either in the bladder or urethra, which prevents urine from being voided.

To perform a bile acids test, what is required? Draw a fasted sample, feed the pet, collect urine 3 hours after feeding Draw a fasted sample 2 consecutive days Feed the pet, draw first sample 1 hour after feeding and a second sample at 2 hours Draw a fasted blood sample, feed the animal, draw another sample 2 hours after feeding

Draw a fasted blood sample, feed the animal, draw another sample 2 hours after feeding

You accidentally write the wrong name for medication in the chart. What should you do? Draw a single line through it and initial White it out Scribble over it until it cannot be read Cover it with an exam sticker

Draw a single line through it and initial

An increase in the total protein, PCV, and BUN with a urine specific gravity of 1.061 may be due to: Renal failure Iron toxicity Increased protein intake Dehydration

Dehydration Hemoconcentration or dehydration may cause an increase in the albumin, PCV or hematocrit, and BUN. The elevated specific gravity helps to determine that the BUN is elevated due to dehydration and not due to renal failure. Animals in kidney failure typically have hyposthenuria because the kidneys have lost the ability to concentrate.

A cigar-shaped mite that is seen on a skin scraping from a dog with severe generalized skin lesions is probably what type of mite? Notoedres Demodex Otodectes Cnemidocoptes Sarcoptes

Demodex Demodex is the cigar-shaped mite. The other mites are much rounder. The generalized skin lesions are not specific for Demodex as they can also be seen with Sarcoptes.

The hip joint is known as which of the following? Femorotibial Glenohumeral Coxofemoral joint Ileocecal

Coxofemoral joint This is the joint where the coxae (pelvis bone) meets the femur (leg bone). The joint is where the head of the femur meets the acetabulum of the pelvis to form the ball and socket joint. Ileocecal does not refer to a joint at all, but refers to the area where the Ileum (distal part of the small intestine) meets up with the colon (the cecum specifically). Remember that the ileum is part of the small intestine, and the ilium is uppermost and largest bone of the pelvis. The glenohumeral joint is the shoulder. Femorotibial refers to the joint at the area of the stifle.

What is the best way to restrain a foal? Cradle one hand under the neck and grasp the base of the tail with the other hand Use a halter Remove the foal from its mother and cradle one hand under the neck and the other under the abdomen It is too dangerous to restrain a foal, it is best just to corner them and not use restraint

Cradle one hand under the neck and grasp the base of the tail with the other hand It is best to keep the foal with the mother. Foals are unpredictable and not trained to use a halter. Best control is achieved when cradling one hand under the neck and grasping the tail base with the other hand.

Which of the following hormones is produced by the adrenal glands? Growth hormone (GH) Thyroid stimulating hormone (TSH) Adrenocorticotropic hormone (ACTH) Antidiuretic hormone (ADH) Cortisol

Cortisol Cortisol is a steroid hormone, or glucocorticoid, produced by the adrenal glands. ACTH, GH, and TSH are all produced in the pituitary gland. ADH is produced in the hypothalamus, then stored and released from the pituitary gland.

You are assisting with an ACTH stimulation test to test for hypoadrenocorticism (Addison's disease). ACTH stimulates the release of which of the following? Insulin Cortisol Amylase Bile

Cortisol The ACTH (cosyntropin) stimulation test measures the ability of the adrenals to respond to the hormone ACTH. This hormone is made in the pituitary and stimulates the adrenals to produce cortisol. Cushing's disease is an overproduction of cortisol due to either a pituitary or adrenal mass. Addison's disease is the lack of production of cortisol from the adrenal glands.

What type of placenta does a ruminant have? Zonary Cotyledonary Diffuse Discoid

Cotyledonary Ruminants (not including camels) have a cotyledonary placenta. This means they have numerous smaller placentae instead of a single large contact area between the mother and fetus. Ruminant placental cotyledons attach to a caruncle to form a placentome. The ruminant placenta is epitheliochorial (they have all three layers: endometrium, connective tissue, and uterine endothelium). The cotyledon is the fetus side of the placenta. The caruncle is the maternal side of the placenta. The placentome is the cotyledon and caruncle together. Dogs and cats have a zonary placenta. The placenta forms a band of tissue surrounding the fetus. They have an endotheliocorial placenta (only has the uterine endothelium). Primates and rodents have a discoid placenta (a single discoid shaped placenta). They have a hemochorial placenta (no maternal layers, but instead direct contact of maternal blood with the chorion). Horses and Pigs have a diffuse placenta. The placenta is formed by the whole allantochorion surface. They also have an epitheliochorial placenta like ruminants.

Which of the following is not typically associated with pruritus? Demodex Microsporum canis Cheyletiella Sarcoptes Otodectes Malassezia

Demodex Demodex mites are typically not itchy. It is important to note however that with severe infections of demodex, secondary bacterial infections are common, and those can be itchy. Sarcoptes (scabies mites), Cheyletiella mites, Otodectes (ear mites), Malassezia (yeast), and Microsporum canis (ringworm) all cause pruritus.

When collecting blood for a blood transfusion, the maximum recommended amount that can be collected from a donor horse (500 kg) is what volume? 6-8 liters 3-6 liters 10-12 liters 12-14 liters

6-8 liters

When conducting a food trial for possible food allergy manifesting as allergic skin disease, how long as a minimum should the trial last? 6 months 48 hours 2 weeks 3 months

3 months

A 12-pound cat needs Clavamox drops at 14 mg/kg BID. The solution is 62.5 mg/mL. How many mLs should this cat receive twice daily? 0.8 mL 3.1 mL 1.2 mL 2.4 mL

1.2 mL 12 pounds/2.2= 5.4 kg 5.4 kg X 14 mg/kg dose = 76 mg 76 mg dose/ 62.5 mg/mL concentration = 1.2 mL

The veterinarian orders 3 mg/kg diphenhydramine IM for a 60-pound dog. The concentration is 50 mg/mL. How much should you give the dog? 1.2 mL 2.5 mL 3.2 mL 1.6 mL 0.8 mL

1.6 mL

The veterinarian asks you use glycopyrrolate to premedicate a patient in the hospital. Using the image, calculate how many milliliters you will administer to a 35 kg dog at a dosage of 0.01 mg/kg? (0.2 mg/ml) 0.18 ml 0.004 ml 1.75 ml 0.08 ml

1.75 ml 35 kg multiplied by 0.01 mg/kg= 0.35 mg divided by 0.2 mg/ml= 1.75 ml

How many chambers does the stomach of a llama have? 4 2 6 1 3

3 The llama has a stomach that functions similar to that of a true ruminant, but it only has 3 compartments (rumen, omasum, and abomasum). True ruminants such as cows, goats, sheep, and deer have 4 compartments including the rumen, omasum, abomasum, and the reticulum.

Frequently, the doctor will ask that a prescription for Cephalexin be filled. How many 500 mg capsules should be sent home for a dog receiving 22 mg/kg PO TID for 10 days? The dog weighs 50 lbs. 60 15 45 30

30 To get the answer, the body weight must first be converted to kilograms. 50 lbs / 2.2 kg/lb = 22.7 kg (There are 2.2 lbs per kilogram) Then, 22.7 kg x (22mg/kg) = 499.4 mg This is rounded up to 500 mg. The patient takes 3 capsules per day for 10 days. 3 x 10 = 30 The patient should be sent home with 30 capsules.

The attending veterinarian asks you to administer 120 mg of enrofloxacin (Baytril) IV slow over 20 minutes to a dog that is hospitalized in your clinic. The strength of injectable enrofloxacin is 2.27%, how many milliliters of drug will you administer? 52.8 mls There is not enough information available to calculate the dosage 5.3 ml 0.53 mls

5.3 ml A 2.27% solution is 22.7 mg/ml. 120 mg divided by 22.7 mg/ml = 5.3 mls

A client calls the clinic and says her 12 pound mixed breed dog was just stung by a bee and his face is starting to swell. You recommend she brings her pet in immediately, and the doctor recommends she give Benadryl 12.5 mg orally now before she leaves her house. The woman has some Benadryl liquid that is 12.5 mg per teaspoon. How many milliliters should she give her pet? 2.5 mLs 5 mLs 0.5 mLs 1.5 mLs

5 mLs

Anesthetic monitoring of pet patients is one of the most important veterinary technician duties. How often should vitals be checked and recorded? 1 minute 5 minutes 20 minutes 10 minutes

5 minutes

The attending veterinarian would like you to administer a 22 mg/kg dosage of Cefazolin to a 67 pound dog that is about to have surgery. You find that the vial that has been reconstituted is empty and you need to make a new bottle. How much sterile water will you add to a 1 gram vial if you want the concentration to be 200 mg/ml? 5 ml 50 mls 200 mls 1 gram of powder can not be reconstituted to 200 mg/ml

5 ml First convert grams to milligrams 1 gram = 1000 mg. 1000 mg divided by 200 mg/ml = 5 mls

When taking a lateral thoracic radiograph of a dog, where should the field be centered? 7th rib and the spine of the scapula 5th rib and caudal border of the scapula 9th rib and the xiphoid process of the sternum 3rd rib and the cranial border of the scapula 2nd rib and the manubrium of the sternum

5th rib and caudal border of the scapula A lateral thoracic radiograph should be taken by drawing the forelimbs forward to prevent them from overlapping the cranial thorax. The film should be centered at the 5th rib and caudal border of the scapula; the film should also be collimated to include the manubrium (as the cranial landmark) and half way between the xiphoid and the last rib (as the caudal landmark). The head and neck should be in a natural position, neither extended nor flexed, and the sternum and dorsal spinous processes should be in a plane parallel to the table, indicating that the animal is not rotated. All lung fields should be included, and the film should be taken on maximum inspiration.

How many 3-rooted permanent maxillary teeth are in the mouth of a dog? 6 8 2 4

6 The correct answer is 6. The 3-rooted permanent teeth in the maxilla of a dog are P4, M1, and M2 (so 3 on each side of the upper jaw). There are no 3-rooted permanent mandibular teeth in the dog.

You are planning to get some goats out on your ranch property and would like to keep them confined to a certain area to keep the weeds down. How tall of a fence (minimum) would likely be required to keep the goats confined? 8 feet 3 feet 4 feet 6 feet

6 feet Fencing is a very important aspect of raising sheep and goats. Most goats can jump or climb over a fence that is shorter than 6 feet, so this would be the minimum recommended height.

What is the average toxic dose of theobromine in dogs? 0.5-2 mg/kg 60-100 mg/kg 10-15 mg/kg 150-200 mg/kg

60-100 mg/kg The LD50 toxic dose of theobromine in dogs which many texts list is 100-150 mg/kg, and may be the correct answer if listed as a choice. HOWEVER, this means that about 50% of pets die at this level; therefore, obviously the true toxic dose would be lower than this. In some sensitive pets, toxic doses could as low as 20 mg/kg. On average, the toxic dose is likely somewhere between 60-100 mg/kg.

A breeder calls and wants to know how long sperm may be viable to fertilize an egg after dogs have mated. What will you tell her? 4 days 14 days 24 hours 7 days

7 days Canine sperm may live or be able to fertilize an ovum for about 7 days.

A patient with a chest tube needs to have lidocaine administered via the tube to help with pain control. The patient weighs 80 lbs and needs 2 mg/kg of lidocaine administered. The doctor asks that the lidocaine be diluted in an equal volume of saline. The lidocaine comes in a 2% solution. How many total milliliters will you administer into the chest? 2.8 ml 7.2 ml 3.0 ml 3.6 ml

7.2 ml First convert the body weight to kilograms. There are 2.2 lbs in 1 kilo. 80 lbs / (2.2 lbs/kg) = 36.4 kg Next, calculate the amount of lidocaine that needs to be given in mg. 36.4 kg x (2 mg/kg) = 72.7 mg Percent is equal to weight (in Grams) / 100 parts volume (in mLs): Example: A 3% solution: 3 Grams/100 mL or 3000 mg/100 mL or 30 mg/mL The easiest way to remember is to just add a zero onto any percentage for the milligrams. (i.e. 2% solution = 20 mg; 10% solution = 100 mg; 25% solution = 250 mg) Therefore, the 2% lidocaine solution is 20 mg/ml. 72.7 mg / (20 mg/ml) = 3.6 ml LIDOCAINE Now, you will add an equal part of saline = 3.6 mL SALINE Add them together: 3.6 mL lidocaine + 3.6 mL saline = 7.2 mL total volume to be injected into the chest. 7.2 ml is the best answer choice.

A dog has recently obtained a fresh superficial linear wound after lacerating its flank on a fence. Within what time period can this wound be optimally repaired by primary closure? 12-24 hours 3-5 days 2 hours 8 hours

8 hours After 8 hours pass, the wound tissue starts to become increasingly compromised, and risk of infection and necrosis increase. Primary closure is used when a wound is classified as clean or clean-contaminated. Closure of these involves immediate lavaging, suturing, and closure of the wound. Delayed primary closure is used in clean-contaminated wounds or contaminated wounds and is performed 2-5 days later. The wound is lavaged and debrided. Closure of the wound takes place before the development of granulation tissue. Secondary closure is wound management consisting of lavage and debridement until granulation tissue develops (3-5 days). Then, the edges of the wound are excised, and the skin edges are sutured over the granulation tissue. Second-intention healing is when a wound is allowed to heal by granulating in, wound contraction, and epithelialization; no surgical closure is performed.

Which of the following best describes a seroma? A fluid pocket at an incision site that is not infected A mass of immune cells that forms due to tissue injury that can occur from trauma or surgery A pocket or distribution of air under the skin A fluid pocket at an incision site containing pus

A fluid pocket at an incision site that is not infected A seroma is a pocket of fluid that sometimes develops at a surgery site post-op. It is typically a clear or pink serous fluid and is not infected. It often builds up in dead space or from a suture reaction. An abscess is a pocket of pus (infection). Air under the skin is referred to as subcutaneous emphysema. A granuloma is a mass of immune cells that forms from tissue injury.

What is the neurotransmitter for skeletal muscle? Dopamine GABA Acetylcholine Acetylcholinesterase Norepinephrine

Acetylcholine Acetylcholine (ACh) is the transmitter for skeletal muscle and is located at the terminal ends of each motor neuron. Acetylcholinesterase destroys ACh and thus turns off muscle activity. GABA and Dopamine are CNS neurotransmitters. Norepinephrine is an inhibitory neurotransmitter in smooth muscle.

If a snake is undergoing "dysecdysis", what does this mean? Retention of the eye cap or "spectacle" after shedding Difficulty breathing Normal physiologic process of skin sloughing Abnormal shedding of the skin External parasitism by mites or ticks

Abnormal shedding of the skin "Ecdysis" means the shedding or sloughing of the skin and occurs in reptiles including snakes. Dysecdysis is the term used when there is abnormal or incomplete sloughing. In pet reptiles, this is most commonly due to a habitat that is not sufficiently humidified.

This 6-year-old Friesian cow has developed a mass on the left jaw region (see image). The farmer reports that it developed over the last several weeks. The mass is firm, immobile, and painful on manipulation. While assisting the veterinarian, you notice a thick granular discharge coming from the mass. The veterinarian suspects "lumpy jaw", which is caused by which of the following agents? Moraxella bovis Corynebacterium pseudotuberculosis Actinomyces bovis Mycobacterium bovis

Actinomyces bovis "Lumpy jaw" is caused by Actinomyces bovis. It is most often diagnosed by palpating a firm mass that is immobile over the mandible. Corynebacterium pseudotuberculosis causes caseous lymphadenitis. It results in an onion appearance in abscesses found most commonly in the neck region and abdomen Mycobacterium bovis causes bovine tuberculosis and is a zoonotic disease. It most often causes coughing and other respiratory symptoms. Moraxella bovis causes pinkeye in cattle.

The licensure we receive before being able to practice as a veterinary technician is an example of which branch of ethics? Official ethics Normal ethics Administrative ethics Descriptive ethics

Administrative ethics Administrative ethics are defined as: Rules set by governmental bodies that regulate veterinary medicine.

Which substance should never be used for prepping the digits for a declaw if a laser is going to be used? Povidone iodine Chlorhexidine 0.9% sodium chloride Alcohol

Alcohol Alcohol is flammable and laser may ignite the area.

Which food is a good source of calcium for large animals? Barley Oats Alfalfa Corn

Alfalfa Legumes (such as clover and alfalfa) and beet pulp are all high in calcium; corn, oats, and barley are low in calcium.

Which of the following drugs is a neuroactive steroid and general anesthetic? Alfaxalone Telazol Propofol Thiopental

Alfaxalone

A dog with a hematocrit of 23% has which of the following? Anemia Dehydration Hypercoagulation Low platelets An elevated PCV

Anemia The hematocrit and packed cell volume (PCV) are red blood cell counts. A hematocrit or a PCV of 23% would be considered low, otherwise known as anemia. Thrombocytopenia is a low platelet count. An animal with dehydration would have hemoconcentration and typically, an elevated hematocrit, unless the animal has both anemia and dehydration concurrently.

A blood smear is evaluated and is noted to have a large variation in cell size as well as a decrease in cell color. What is the appropriate terminology to describe this blood smear? Macrocytosis and hyperchromasia Microcytosis and microchromasia Anisocytosis and hypochromasia Anisokaryosis and macrochromasia

Anisocytosis and hypochromasia Anisocytosis is the descriptive term for cell size variation. Red blood cells that have less color than is normally anticipated are said to by hypochromic. Cells with decreased pallor are likely deficient in hemoglobin. Cells with increased color are hyperchromic. Macrocytes are cells that are larger than typical, while microcytes are the opposite. Varying degrees of color among cells is termed polychromasia and is usually an indication of different aged (young and old) red blood cells present. Younger cells will have ribosomes which give them a different color.

Assuming the daily maintenance fluid requirement is 60 ml/kg/day, how many milliliters per hour should a 4400 lb animal receive per minute to achieve this requirement? Approximately 5000 ml/min Approximately 600 ml/min Approximately 80 ml/min Approximately 120,000 ml/min

Approximately 80 ml/min First, convert the body weight to kilograms. 4400 lbs / (2.2 lb/kg) = 2000 kg Next, calculate daily fluid requirement. 2000 kg x (60 ml/kg/day) = 120,000 ml/day Now, divide by 24 to figure out the hourly requirement. 120,000 ml/day / (24 hr/day) = 5000 ml/hr Lastly, divide by 60 to calculate the amount of fluid necessary per minute. 5000 ml/hr / (60 min/hr) = 83 ml/min

A cat is given dexmedetomidine, and the cat's heart rate has dropped significantly. The veterinarian asks you to give the reversal, which is the following is the appropriate reversal agent? Atipamezole Acepromazine Atropine Naloxone

Atipamezole The reversal of medetomidine or dexmedetomidine (Domitor or Dexdomitor) is atipamezole (Antisedan). Yohimbine may also have some reversal effects but is not as commonly used to reverse medetomidine. Yohimbine is more often is used to reverse xylazine. Naloxone reverses morphine and other opioids. Atropine is an anticholinergic given to increase heart rate. Even though atropine will counteract medetomidine-induced bradycardia, its use results in severe hypertension and is not given with medetomidine. Acepromazine is a tranquilizer and is not a reversal agent.

This ECG demonstrates which arrhythmia? (Couple little's then a big) 3rd degree heart block Atrial fibrillation/atrial flutter Normal sinus rhythm Ventricular fibrillation

Atrial fibrillation/atrial flutter The presence of frequent P waves with no discernible QRS complexes make this an arrhythmia that is atrial in origin.

What is the most common lead system used in a horse for electrocardiography (ECG) tracings? Base-apex lead Lead III Lead II aVL aVR

Base-apex lead The most common is the 3-lead base-apex system in which the electrodes are placed at the left axillary region, right jugular region and a ground on the neck or withers region.

An injury to what region of the brain is most likely to result in respiratory arrest? Cerebellum Diencephalon Brain stem Cerebrum

Brain stem The brain stem consists of the midbrain, pons, and medulla oblongata. The pons contains important respiratory centers. The medulla also influences the respiratory rate as well as heart rate. The cerebellum is responsible for coordination and balance. The diencephalon contains the thalamus and hypothalamus. The thalamus behaves as a relay station for sensory input. The hypothalamus is involved in helping regulate many endocrine functions of the body. Motor control and interpretation of sensory impulses occurs at the cerebrum.

Which part of the brain is associated with regulation and coordination of movement, posture, and balance? Brain stem Cerebellum Hypothalamus Cerebral cortex

Cerebellum The cerebellum is involved in the coordination of voluntary motor, balance and equilibrium. It is located just above the brain stem.

What is the name of the portion of the stomach that attaches to the esophagus? Hypopharynx Cardia Duodenal papilla Pylorus

Cardia The cardia is the anatomical term for the portion of the stomach attached to the esophagus. The cardia overlaps but does not contain the lower esophageal sphincter. The pylorus is where the stomach opens into the duodenum.

A rabbit is brought in for skin problems. Upon exam, you notice large flakes of dead skin over the dorsum. Some of these flakes appear to move. What might you be seeing? Demodex Psoroptes Notoedres Sarcoptes Otodectes Cheyletiella

Cheyletiella Cheyletiella is also known as "walking dandruff". It is a 8 legged mite that can live on the skin of rabbits, dogs, cats and humans. They do not bury in the skin but live in the keratin layer. They have a 21 day life cycle and cannot live more than 10 days off the host.

Endoscopy can be a beneficial diagnostic tool for which of the following conditions? Coughing Pancreatitis Chronic vomiting Portosystemic shunt Nephrolithiasis

Chronic vomiting Endoscopy is a scope that is used in the upper gastrointestinal tract. It is used to take biopsies of the stomach and small intestine. It is very useful to examine for underlying disease grossly (via the endoscope's camera) and microscopically via the biopsy specimens. Colonoscopy is used to examine the large intestine. Bronchoscopy is used in the airways and would be useful in a case of chronic coughing.

You are asked to run an APTT and PT test. What anticoagulant is used for this? Heparin Citrate EDTA No anticoagulant because it is a clotting test Coumadin

Citrate Sodium citrate is the anticoagulant found in the turquoise (light blue) top tube used to run an Activated Partial Thromboplastin Time and a Prothrombin Time. Citrate is a reversible anticoagulant used for coagulation testing.

Which of the following is an appropriate anticoagulant for collection of blood from a donor horse for the purposes of blood transfusion to a recipient horse? Heparin Citrate-phosphate-dextrose adenine (CPDA) Citrate phosphate dexamethasone Sodium fluoride EDTA

Citrate-phosphate-dextrose adenine (CPDA) CPDA is an appropriate anticoagulant to use for blood transfusions; the solution must prevent clotting of blood while providing a "fuel" source, such as dextrose, to the cells.

Sulfasalazine is sometimes used in veterinary medicine to treat which chronic condition? Colitis Pancreatitis Bronchitis Hepatitis

Colitis Sulfasalazine (Azulfadine) is used in treating chronic intestinal conditions, such as ulcerative colitis, because it reduces inflammation in the colon. It is a sulfonamide antibiotic.

Which of the following disorders is caused by a Type IV hypersensitivity, also known as delayed type hypersensitivity, as a result of a T-cell response to an antigen? Atopy Contact allergy to plastic Autoimmune hemolytic anemia Glomerulonephritis

Contact allergy to plastic There are 4 types of hypersensitivity. Type I is known as immediate hypersensitivity and results from inflammatory mediators being released immediately after exposure to an antigen. Common examples are atopy and anaphylaxis. Type II hypersensitivity is known as antibody-mediated hypersensitivity and results from antibodies directed against one's own cells. An example is autoimmune hemolytic anemia. Type III hypersensitivity is known as immune complex disease and occurs when antibodies and antigens form complexes that collect and cause problems. Glomerulonephritis can be caused by type III hypersensitivity. Type IV hypersensitivity is a delayed hypersensitivity that results from T-lymphocytes reacting against antigens. Examples include certain contact allergies (such as poison ivy) or from plastic food dishes (in some dogs).

While the surgeon is focused on the task at hand, it is the anesthetist's job to make sure the patient's vitals are maintained at a normal plane. Which of the following is not suggestive of significant blood loss? Rapid respiratory rate Weak pulses Rapid heart rate Cyanotic mucous membranes

Cyanotic mucous membranes Patients suffering from substantial blood loss can be expected to have tachycardia, tachypnea, and weak pulses. In addition, pale/white mucous membranes as opposed to cyanotic mucous membranes would be noted. Cyanotic mucous membranes are seen when the patient lacks oxygen.

What is the optimal method of sampling if a urine culture is to be performed in a dog? Bladder expression Urinary catheterization Free catch Cystocentesis

Cystocentesis The only way to definitively obtain a sterile urine sample is by performing a cytocentesis. All other methods carry the risk of environmental contamination with bacteria.

You are asked to take a dorsomedial-palmarolateral oblique radiograph of a horse's fetlock with a portable x-ray unit. A cross-section of the limb is diagrammed and labeled below with the red lines representing two possible x-ray cassette placement locations. Pick the letter that corresponds to the correct cassette placement and the correct placement of the marker on the cassette. A B D C

D

Giardiasis may be the cause of which clinical sign? Urticaria Seizures Diarrhea Coughing Panting

Diarrhea Giardia is a protozoan parasite that often causes a watery diarrhea. It may sometimes also cause vomiting. Seizures, urticaria (hives), coughing, and panting are not typical symptoms of giardiasis.

On a CBC on a 5-year old Basenji at a routine wellness exam, you identify the microfilaria shown below. There are two species of microfilariae that can be seen in the peripheral blood in dogs. What are they? Dirofilaria immitis and Uncinaria stenocephala Dirofilaria immitis and Acanthocheilonema (Dipetalonema) reconditum Uncinaria stenocephala and Filaroides osleri Acanthocheilonema (Dipetalonema) reconditum and Filaroides osleri

Dirofilaria immitis and Acanthocheilonema (Dipetalonema) reconditum The correct answer is Dirofilaria immitis and Acanthocheilonema (Dipetalonema) reconditum. Dipetalonema reconditum is a blood parasite that looks similar to the microfilariae of Dirofilaria immitis, the agent of heartworm disease. The two parasites must be differentiated because Dipetalonema reconditum is not pathogenic and is therefore not treated. Uncinaria stenocephala and Filaroides osleri would not be found in the blood. Incidentally, the organism in this blood smear is D. immitis

You are scrubbed in as the surgery technician and are assisting with an intestinal foreign body removal. The surgeon wants to clamp the intestine above and below the foreign body. What instrument could be used for this? Gelpis Doyens Brown-Adsons DeBakeys

Doyens Doyen intestinal forceps are a curved non-crushing occluding clamp with longitudinal grooves. They are used for the purpose described, but over-tightening should be avoided because they can still damage the intestinal tissue. The others listed should not be used on the intestines. DeBakey forceps are vascular forceps that are used to hold vascular tissue with minimal damage to the vessels. Brown-Adson forceps are for holding tissue and have grasping teeth. The Gelpi is a self-retaining small retractor.

Where is the best place to give an intramuscular injection in a dog? Semitendinosus muscle Epaxial muscle Any muscle will do Sternohyoid muscle

Epaxial muscle This is a large group of muscles between the spine and the wings of the ileum. They are easy to find and use. Semitendinosus muscles on the rear legs can be used, but there is a possibility of sciatic nerve damage if not done carefully. The sternohyoid muscle is a muscle of the neck and not appropriate to use for intramuscular injection. Muscles used for intramuscular injection need to be large, vascular, and easy to find and use. Not all muscles are appropriate.

You are assisting with Coggins testing. What does this test for? Equine Herpes Virus-2 Streptococcus equi Equine Infectious Anemia Sarcoidosis

Equine Infectious Anemia The Coggins test checks for Equine Infectious Anemia (EIA) antibodies in a horse's blood. Blood samples must be sent to a state-approved laboratory. This test is often needed before taking a horse to a show and whenever a horse is transported across state lines. EIA is a virus transmitted by the horsefly. Streptococcus equi causes Strangles.

Harderian glands are associated with which part of the anatomy? Eyes Anus Ears Bladder

Eyes The Harderian gland is an accessory lacrimal gland on the inner side of the orbit in reptiles and birds; it is also present but is usually degenerate in mammals. Ears have ceruminous glands. There are perianal glands and apocrine glands associated with the anus of dogs and cats. The bladder is made up of transitional epithelium and is not glandular.

In neonatal calves and foals, what is the most common reason that these animals succumb to disease and death? Failure of passive transfer Iron deficiency Infection acquired by the mother from the placenta Gastrointestinal parasitism

Failure of passive transfer Newborns require essential maternal antibody absorption from the colostrum to help protect against infectious diseases. If they do not get these precious immunoglobulins, they can suffer from increased morbidity and mortality from infections. There are now field test kits that can detect blood levels of IgG in calves and foals to help determine if failure of passive transfer has occurred.

Oviducts are also known as which of the following? Ovaries Uterine horns Fallopian tubes Spermatic cords

Fallopian tubes Fallopian tubes and oviducts are the same structure. It is the structure that transports the egg from the ovary to the uterus.

Which of the following could be used for hemostasis at the site of a liver punch biopsy? Styptic powder Gelfoam Silver nitrate Bone wax

Gelfoam Gelfoam is a sterile compressed sponge that is intended to aid in hemostasis; it allows for absorption of blood and formation of a clot at the bleeding site. It has a very useful application in liver biopsies. Bone wax controls bleeding from bone and functions as a mechanical plug that is pressed into the bleeding bone surface. Silver nitrate applicators are used to stop small superficial bleeders such as a snipped skin tag or a bleeding toenail. These are not sterile. Styptic powder is used to stop bleeding toenails and would never be used on tissues.

Which of the following is true regarding anesthesia of reptiles? Because reptiles are cold-blooded, heat support is not necessary Perioperative antibiotics are not recommended due to reptile sensitivity to these drugs The skin should never be scrubbed but rinsed only with antiseptic Fasting is not usually necessary

Fasting is not usually necessary Regurgitation during anesthesia is not a concern for reptiles, and thus fasting is not usually necessary unless they are undergoing gastrointestinal surgery. The rough nature of reptile skin makes it difficult to aseptically prep for surgery. A stiff brush with antiseptic solution should be used for preparation of the skin. Reptiles are prone to hypothermia. They are ectothermic and hold their breath so heat support is extremely important for better control of anesthesia. Reptiles are susceptible to microbial infections; stress from surgery may potentiate these infections (and the skin is difficult to sterilely prep). Perioperative antibiotics are almost always warranted.

If restrained in a stressful or frightening manner, which species might be prone to seizure? Dog Cat Llama Horse Gerbil

Gerbil

Anticholinergics can be used in small mammals as part of a preanesthetic protocol. Of the following, which anticholinergic is the best choice for use in rabbits? Diazepam Midazolam Glycopyrrolate Atropine

Glycopyrrolate Both atropine and glycopyrrolate can be used in small mammals, but glycopyrrolate is preferred for use in rabbits. This is because the effect of atropine is less effective in this species because many animals have high levels of atropinesterase. Both diazepam and midazolam are benzodiazapines, not anticholinergics.

Which of the following mostly produces endotoxins? Yeast and other fungal organisms Intestinal parasites such as roundworms and hookworms Gram-negative bacteria Gram-positive bacteria

Gram-negative bacteria

Upon examining this 7 year old female spayed Dachshund, you notice her mucous membrane color is abnormal. What term would your write in the medical record to describe your findings? Hemoptysis Cyanosis Hyperemic Icterus

Icterus Icterus, or jaundice, is the yellowing of the mucous membranes, skin, or sclera. It is often seen in patients with liver dysfunction or immune mediated hemolytic anemia. Cyanosis is the blue, purple, or grey tinge seen in the mucous membranes of patients with low oxygen levels. Hemoptysis is the coughing up of blood or blood tinged fluid. Hyperemic mucous membranes are bright or brick red and are usually caused by sepsis, allergic reactions, or shock.

A dog presents with weakness, depression, polyuria, polydipsia, and dilute urine. X-rays reveal a mediastinal mass. Where is the mediastinum located? In the central abdomen Caudal to the kidneys In the cervical spine In the lumbar spine In the chest

In the chest The mediastinum is the central compartment of the chest. It is located between the right and left pleura and extends from the sternum to the vertebral column. It contains all the thoracic organs except the lungs.

What is the effect of administering a diuretic, such as furosemide? Increased blood pressure Increased urine production Sodium retention Water retention

Increased urine production Diuretics work to flush out excess water and sodium from the body (which would decrease blood pressure) and thus increase urine output. They are most commonly used in treating congestive heart failure as the goal is to control pulmonary edema. An example of a medication that increases blood pressure is dopamine.

A 5-year old feline presents after being hit by a car. The veterinarian wants you to administer mannitol intravenously. For what reason was mannitol chosen as a therapy? It increases blood flow to the heart and lungs via peripheral vasoconstriction It reduces intracranial pressure It increases respiratory rate It is a pain medication It helps to prevent GI ischemia

It reduces intracranial pressure Mannitol is an osmotic diuretic and reduces acute increases in intracranial pressure. The veterinarian suspects head trauma has been sustained. Mannitol does not protect the GI tract from damage nor does it provide any pain control. It is not a pressor and therefore does not cause peripheral vasoconstriction.

You find Otodectes cynotis in an ear swab you prepared for a kitten that has been scratching her ears and has dark coffee ground looking debris in them. What could be used to treat this? Albon Clavamox Ivermectin Fluconazole

Ivermectin Ivermectin 0.01% can be applied topically to the ears to treat ear mites. Fluconazole is an anti-fungal that would be effective against Malassezia (yeast). Clavamox would be beneficial for a bacterial ear infection. Albon is used to treat coccidia (an intestinal parasite).

Which of the following are treatments for hyperthyroidism? Methimazole and radioactive iodine Levamisole and I-131 Levothyroxine and I-131 Metronidazole and Levothyroxine

Methimazole and radioactive iodine Hyperthyroidism (elevated thyroid level) in cats is treated with radioactive iodine or methimazole. I-131 and radioactive iodine are the same thing. Levothyroxine is used to treat hypothyroidism (thyroid level is too low).

The small intestine is made up of four layers. Which of the following describes these layers from inside to outside? Mucosa, submucosa, muscularis, serosa Serosa, submucosa, mucosa, muscularis Serosa, mucosa, submucosa, muscularis Muscularis, submucosa, mucosa, serosa

Mucosa, submucosa, muscularis, serosa The inner most layer of the intestine is the mucosa. It is lined with villi which help to absorb nutrients. The next layers working outward are the submucosa, muscularis, and serosa.

What purpose is the administration of the drug Guaifenesin typically used for in horses? Expectorant Intestinal Prokinetic Anti-inflammatory Muscle relaxation Antimicrobial

Muscle relaxation Guaifenesin is administered intravenously to horses to provide relaxation of skeletal muscles during induction of general anesthesia. The exact mechanism of action is unknown but believed to be related to central depression of nerve impulse transmission. Although guaifenesin is administered orally in some species as an expectorant, this is not the typical reason for its administration in horses.

What term would describe inflammation of the kidneys? Nephritis Hepatitis Blepharitis Gastritis Cystitis

Nephritis Inflammatory conditions typically end in "-itis". Nephritis is inflammation of the kidneys. Hepatitis is inflammation of the liver. Cystitis is inflammation of the bladder. Blepharitis is inflammation of the eyelids/follicles. Gastritis is inflammation of the stomach.

At your small animal clinic, the anesthesia cost is included for elective surgeries. An example of this type of surgery is which of the following? Aural hematoma repair Neuter Cystotomy Splenectomy

Neuter Spays, neuters, and routine dentals are examples of elective surgeries. They are surgeries that are scheduled in advance and are not a medical emergency. The other procedures listed are medically necessary and not considered elective.

An owner has fed her diabetic cat and has given the insulin injection. She calls the clinic because there is a wet spot on the fur where she gave the injection and she doesn't think it went in. What will you advise her to do? Not re-dose the insulin, just give the next scheduled dose as directed Give Karo syrup orally and monitor for symptoms of hypoglycemia Repeat the insulin injection since the cat ate Give another 1/2 dose of insulin

Not re-dose the insulin, just give the next scheduled dose as directed The cat should not be re-dosed. It would be more detrimental for the cat to receive additional insulin than for him to miss an injection. Advise the owner to just give the next regularly scheduled dose. Karo syrup is given when hypoglycemia is suspected (if too much insulin is given or if the animal is having symptoms of hypoglycemia)

Which of the following is NOT a contraindication of epidural placement in the dog? Coagulopathy Skin infection in the area of the needle insertion Fractures of the sacrum or pelvis Obesity

Obesity Obesity is not a contraindication, but it may make finding the landmarks difficult while placing an epidural. Occasionally fractures of the sacrum or pelvis can cause abnormal anatomy and an epidural should not be attempted. If a coagulopathy exists, an epidural should not be attempted to avoid potential bleeding. Skin infections at the epidural site can lead to systemic infections if bacteria is introduced to the epidural space by the needle.

Which of the following describes the best method for preparing a urine sediment sample for microscopic evaluation? Once sample is obtained, heat it under a Bunsen burner until an adequate amount of supernatant has evaporated. Then, obtain a small sample with a pipette and transfer it to a microscopic slide for evaluation Once sample is obtained, allow it to sit in a refrigerated environment for 12 hours. Following this, centrifuge the sample, remove the supernatant, and transfer a small amount of urine onto a microscopic slide and evaluate Once sample is obtained, centrifuge the sample and remove the supernatant. Using a pipette, transfer small amount of urine onto a microscopic slide and evaluate Once sample is obtained, transfer a small amount with the aid of a pipette onto a microscopic slide and evaluate

Once sample is obtained, centrifuge the sample and remove the supernatant. Using a pipette, transfer small amount of urine onto a microscopic slide and evaluate

Regarding sterilization in reptiles, which is true? Only the oviducts should be removed and ovaries should be left intact Ovaries should always be removed during sterilization Sterilization of female pet reptiles is recommended so that they do not develop estrogen toxicity Reptiles should not be sterilized due to sensitivity to anesthetics

Ovaries should always be removed during sterilization When sterilizing a reptile, the ovaries should always be removed. Removal of only the oviducts puts the patient at risk for egg binding. With just the oviducts removed, the ovaries are still active and yolks can be released into the coelomic cavity. In ferrets, spaying is recommended due to the risk of bone marrow toxicity from the high levels of circulating estrogen during prolonged heat cycles.

What is of the biggest concern when shipping pigs? Gastric dilatation Dehydration Stress-induced diarrhea Overheating

Overheating

Several Heinz bodies have been identified on a blood smear of a sick dog. What is this an indication of? Oxidative damage Carbon monoxide exposure Cyanide poisoning Viral hemolysis

Oxidative damage

You are taking a lateral radiograph of the left stifle of a dog to assess for a suspected cruciate ligament injury. How should the patient and leg be oriented relative to body and film? Patient should be in left lateral recumbency with the leg against the cassette Patient should be in dorsal recumbency with the leg to the side of the body against the cassette Patient should be in right lateral recumbency with the leg against the cassette Patient should be in right lateral recumbency with the leg away from the cassette Patient should be in ventral recumbency with the leg to the side of the body against the cassette

Patient should be in left lateral recumbency with the leg against the cassette Having the leg against the film instead of away from the film will provide a crisper and less distorted image. Therefore, positioning the dog in left lateral recumbency with the leg against the cassette is the best position.

The lower respiratory tract includes all of the following except for which of these? Trachea Bronchioles Alveoli Pharynx

Pharynx The lower respiratory tract includes portion of the larynx that is past the vocal cords, the trachea, the bronchi, bronchioles, and alveoli. Alveoli are the sites of gas exchange between air and blood and are in the lower-most respiratory tract (lung). The nasal passages, sinuses, pharynx, and the portion of the larynx above the vocal cords are all part of the upper respiratory system.

Which of the following lists the correct order of the phases in the estrous cycle? Metestrus, proestrus, estrus, anestrus, diestrus Estrus, proestrus, diestrus, metestrus, anestrus Anestrus, diestrus, estrus, metestrus, proestrus Proestrus, estrus, metestrus, diestrus, anestrus

Proestrus, estrus, metestrus, diestrus, anestrus Proestrus is the time when ovarian follicles are starting to grow. It immediately precedes estrus. Estrus is the time of actual heat when follicles are mature and ovulation occurs or can be induced depending on species. During Metestrus, estrogen stimulation subsides and the corpus luteum starts to form. The uterine lining begins to secrete small amounts of progesterone. Diestrus is regression of the corpus luteum. Anestrus is resting of the sexual cycle.

What are the segments of a tapeworm that are sometimes seen in canine feces called? Cysticercoids Rostellums Proglottids Scolex

Proglottids The progottids are the individual segments in a long chain that make up most of the tapeworm. When mature, each proglottid segment contains male and female reproductive organs; reproduction takes place, and eggs develop in each segment.

A dog presents with severe seizures, and you are unable to hit a vein for catheter placement and need to stop the seizures immediately. Which of the following is an acceptable means of giving valium to a seizuring dog? Intraosseous Oral Intraperitoneal Rectal Subcutaneous

Rectal Valium can be given rectally to stop seizures. NO oral medications should be given to a seizuring dog. Valium cannot be given via the other routes.

Which of the following may be a cause of post-renal azotemia? Urethral obstruction Severe dehydration Infection in the kidney Primary loss of nephrons

Urethral obstruction Post-renal azotemia is an elevation in BUN and creatinine due to pathology located after the kidney (in the ureters, bladder, or urethra). A urethral obstruction means that the outflow from the bladder is obstructed. This could be from severe inflammation, uroliths, or neoplasia. This puts ascending pressure on the kidneys because the urine has no way of being voided. This may cause an elevation of renal values (BUN and creatinine) and is referred to as azotemia. Treatment of post-renal azotemia depends on relieving the obstruction and subsequent IV fluid diuresis. Pre-renal causes of azotemia include dehydration and lack of perfusion (as could occur in shock from hypotension). Renal azotemia is caused by a primary problem with the kidneys (acute or chronic renal failure).

A 10-year-old male neutered Pug presents in respiratory distress and appears to be having an allergic reaction. He has hives and a swollen face. The doctor asks you to get epinephrine ready at a dose of 0.02 mg/kg. He weighs 9 kg. When you get the bottle of epinephrine from the refrigerator it says it is a 1:1000 solution. How many mLs will you draw up for this patient? 0.18 mL 0.8 mL 0.08 mL 1.8 mL

0.18 mL First you have to understand what a 1:1000 concentration means. A 1:1000 solution is the same as saying 1 gram in 1,000 mL. And we know that 1 gram is equal to 1,000 mg. So it would be 1,000 mg: 1000 mL which is the same as 1 mg/mL (when the thousands cancel out). Next you take the patient's weight in kg X the dose: 9 kg/dog X 0.02 mg/kg (the kg cancel out) = 0.18 mg/dog Now divide the dose by the concentration: 0.18 mg/dog x 1mg/mL (mg cancel out) = 0.18 mL/dog

A dog presents with a history of coughing and rapid breathing. Radiographs confirm congestive heart failure (see image). You are asked to administer 2 mg/kg of furosemide IV. The concentration of furosemide is 50 mg/mL. The dog weighs 25 pounds. How many milliliters should you administer to this patient? 1.2 mL 2.0 mL 0.44 mL 0.12 mL

0.44 mL The dog is 25 pounds. 25 pounds/2.2 pounds per kg= 11.3 kg Now multiply the dog's weight in kg by the dose you need (2mg/kg): 11kg X 2mg = 22 mg The dog needs 22 mg. Now divide this dose by the concentration of furosemide (50 mg/mL): 22mg/ 50 = 0.44 mL

How many liters of 50% dextrose should be added to 5 liters of 0.9% saline to make a 5% dextrose solution? 0.05 L 0.5 L 0.75 L 0.25 L

0.5 L To answer this question, the following formula should be implemented: C(1)V(1) = C(2)V(2) C is the concentration V is the volume C(1) =0.5 V(1) = x C(2) = 0.05 V(2) = 5 L 0.5x = 0.05(5) x = 0.5 L 0.5 L of 50% dextrose should be added to one liter in order to make a 5% solution.

Acepromazine concentration is 10 mg/mL. You are instructed to make a dilute acepromazine solution at 1 mg/mL. Which of the following could be used for how much acepromazine and how much sterile water are mixed to make this concentration? 9 mL acepromazine and 1 mL sterile water 1 mL acepromazine and 9 mL sterile water 10 mL acepromazine and 25 mL sterile water 0.1 mL acepromazine and 2.5 mL sterile water

1 mL acepromazine and 9 mL sterile water 1 mL of acepromazine is equal to 10 mg (since ace is 10 mg/mL). This 10 mg mixed in 9 mL of sterile water results in a concentration of 1 mg/mL. 1 mL (10 mg) of ace + 9 mL of sterile water = 10 mL total volume 10 mg / 10mL = 1 mg/mL

A patient that is currently receiving 30 ml/hr of NaCl needs a medication added to the fluid bag. The clinician would like the patient to receive a total of 3 mg/kg/day of this medication. The medication comes in a 10 mg/ml solution. How many mL should be added to the bag if it is a 250 mL bag and the patient weighs 10kg? 4 ml 1 ml 12.5 ml 6 ml 10 ml

1 ml A 10-kg patient will need 30 mg/day of the medication (10 kg x 3 mg/kg/day) Since the fluid is being administered at an hourly rate we need to figure out how much medication we are administering per hour. 30 mg/day / (24 hr) = 1.25 mg/hr To determine how much volume of the medication is needed the following math is performed: 1.25 mg/hr / (30 ml/hr) x (250 ml) = 10.4 mg (Notice how the units cancel out) 10.4 mg / (10 mg/ml) = 1 ml

What is the normal amount of gastric reflux obtained from a healthy horse via nasogastric tube placement? 5-8 liters 10-12 liters 14-18 liters 1-3 liters

1-3 liters Normally, a healthy horse will have a small amount of gastric reflux, 1-3 liters. If you get back 8-12 or more liters of reflux, the horse likely has an obstructive intestinal disease or ileus of some sort.

"Pablo", a 66 pound Labrador mix, is in oliguric renal failure. The veterinarian has asked you to start a constant rate infusion of dopamine at 5 micrograms per kilogram per minute (ug/kg/min) and a fluid rate of 120 mL per hour. Dopamine comes in a concentration of 40 mg/mL. How much dopamine will you add to a liter of 0.9% sodium chloride solution? 0.78 mL 1.86 mL 5.6 mL 19.5 mL

1.86 mL Constant rate infusions are not something that an entry level technician would typically perform. However, the VTNE may have some questions on CRI and it will benefit you to know how to do these calculations. To solve this problem, break down the basic parts to make it less overwhelming: 1) Convert pounds to kilograms- 66 pounds/2.2 pounds per kg = 30 kg 2) Calculate the number of hours an infusion will last by dividing the volume in the bag (1 Liter, or 1,000 mL) by the rate per hour: 1,000 mL/120mL/hr = 8.3 hours 3) Now take the number of hours (8.3) and multiply by 60 to calculate how many minutes this is: 8.3 X 60 = 498 minutes 4) Now plug into the equation: 5 ug/30kg/498min= 74,700 ug 5) Now convert the ug into mg by dividing by 1,000: 74,700/1,000 = 74.7 mg 6) Calculate the quantity of dopamine to add by now dividing the 74.7 mg by the concentration of the drug (40mg/mL) : 74.7mg/40mg/mL = 1.86 mL To be truly accurate when adding drugs to a bag like this, you should discard 1.86 mL of fluids from the bag prior to adding the 1.86 mL of dopamine.

A cat is having a surgical procedure. How long should food be withheld prior to general anesthesia? 18 hours 10 hours 6 hours 1 hour 3 hours 24 hours

10 hours General anesthesia can decrease swallowing reflexes and esophageal sphincter tone. Patients may experience vomiting during or after anesthetic procedures. This predisposes them to esophagitis, esophageal stricture, or aspiration pneumonia. Domestic animals should be fasted prior to any anesthetic procedure. The amount of time varies by species, but in general food should be withheld for 8-12 hours in dogs, cats, and horses and 24-48 hours for cattle. Neonates should not be fasted prior to anesthesia and small patients (under 2 kg) should be fasted for shorter lengths of time. Water should be withheld for less time to avoid causing dehydration, which can be exacerbated by anesthesia; 2-4 hours is generally sufficient for dogs and cats. Ref. McCurnins Clinical Textbook for Veterinary Technicians, Elsevier, 2010. p. 889

An animal needs to be sedated with Dexmedetomidine. The patient weighs 2200, lbs and the dose that the clinician wishes to administer is 10 micrograms per kilogram. Dexmedetomidine is available as a 1 mg/ml solution. How many milliliters will be administered? 100 ml 1 ml 10 ml 0.5 ml

10 ml To solve this question, it is very important to know that there are 1000 micrograms in 1 milligram. Similarly, there are 1000 milligrams in 1 gram. First, convert body weight to kilograms. 2200 lbs / (2.2 lbs/kg) = 1000kg 1000kg x (10 micrograms/kg) = 10,000 micrograms 10,000 micrograms / (1 milligram/1000 micrograms) = 10 milligrams (Notice the micrograms cancel out.) 10 mg / (1 mg/ml) = 10 ml This question is critical to understand. Not having a good grasp of the difference between a milligram vs. microgram vs. gram can result in misdosing a patient.

Which of the following is an abnormal finding in an intact male dog's urine sediment sample performed from a free catch? Bacteria 10-15 WBC per high-powered field Fat droplets Sperm

10-15 WBC per high-powered field The only abnormal finding in this sample is the excessive number of white blood cells present. Bacteria are not unexpected, as the sample obtained is a free catch, which is not a sterile collection method. Sperm can be seen in intact males. Fat droplets are not an abnormal finding in urine sediment.

What is the maximum time that an ultrasonic scaler can be used on a tooth surface? 30 seconds 10-15 seconds 3-5 seconds 60 seconds

10-15 seconds The scaler must be kept in constant motion on the tooth surface and should not be on a tooth for more than 10 to 15 seconds. This is because heat can buildup and cause thermal damage to the tooth. The water from the scaler also helps to prevent heat buildup. The scaler should be held in a modified pen grasp during the procedure.

The veterinarian prescribes the NSAID carprofen for a 50-pound dog with arthritis at a dose of 4.4 mg/kg and would like this dose divided into two equal daily doses. In your hospital, you stock 25mg, 75mg, and 100 mg chewable tablets. What will you dispense and what are the directions? 100mg tablets- Give 1 tablet in the morning and 1 in the evening 25 mg tablets- Give 1 tablet in the morning and 1 in the evening 75 mg tablets- Give 1/2 tablet in the morning and 1/2 in the evening 75 mg tablets- Give 1 tablet in the morning and 1 in the evening 100 mg tablets- Give 1/2 tablet in the morning and 1/2 in the evening

100 mg tablets- Give 1/2 tablet in the morning and 1/2 in the evening First, calculate the dose needed for this patient. 50 pounds/2.2 pounds/kg (pounds cancel out) = 22.7 kg 22.73 kg X 4.4 mg/kg (kg cancel out) = 100 mg total dose per day (the vet wants this divided into equal doses so now divide the dose in half) 100 mg / 2 = 50 mg twice daily will be given So, the correct answer choice is 100mg tablets- Give 1/2 tablet (50mg) twice a day

An 11-year old intact male Golden Retriever presents with respiratory distress and a temperature of 106.7 F. The attending veterinarian has gone to talk to the client and has requested that the patient be cooled. To what temperature should the dog be cooled? 100.0 104.0 102.0 103.0 99.0

103.0 A patient will continue to cool despite cessation of active cooling, so a patient should not be cooled past the high end of the normal temperature range. Normal temperature range is 101-102.5F in dogs and cats.

The Triadan numbering system is commonly used in human and veterinary dentistry to specify a tooth. What is the correct number of the permanent right maxillary canine tooth of a dog or cat? 301 501 404 104 201

104 In this system, the first numeral denotes the quadrant of the tooth and if it is permanent or deciduous. They are numbered starting at the right maxilla and moving counter-clockwise: 1 = Right maxilla (5 = deciduous) 2 = Left maxilla (6 = deciduous) 3 = Left mandible (7 = deciduous) 4 = Right mandible (8 = deciduous) The next two digits indicate how many teeth over from the midline. For example, in the dog: 701 is the deciduous mandibular left first incisor. 104 is the permanent right maxillary canine. 409 is the permanent mandibular right first molar.

You make a 1:5 dilution of a serum sample and measure the urea nitrogen. The analyzer reads that the concentration of urea nitrogen is 30 mg/dl. What is the urea nitrogen level of the patient's serum? 35 mg/dl 150 mg/dl 6 mg/dl 180 mg/dl 120 mg/dl

150 mg/dl When a dilution is made prior to analyzing a sample, you must multiply that dilution factor with the reported value to obtain the true level in the sample. In this case, the sample was diluted by a factor of 5. Therefore, 30 x 5 = 150

Esmarch bandages and tourniquets are occasionally used in equine surgery. What is the maximum time that a tourniquet may be left in place? 1 hour 2 hours 30 minutes 4 hours

2 hours The maximum recommended time is 2 hours. The purpose of the Esmarch bandage is to help push blood out of a limb by wrapping the bandage from distal to proximal and then applying a tourniquet. In this way, the surgeon can minimize the amount of bleeding at the surgical site. However, prolonged use can result in severe ischemia and deleterious effects.

How many milliliters should be administered to a 100-kg patient that needs 50 mg of a 2.5% solution? 125 ml 2.5 ml 2 ml 25 ml

2 ml The easiest way to remember is to just add a zero onto any percentage for the milligrams (i.e. 2% solution = 20 mg/ml; 10% solution = 100 mg/ml, etc.) In this situation a 2.5% solution is equivalent to 25 mg/ml. Therefore: 50 mg / (25 mg/ml) = 2 ml

What is the typical length of time that an incision should be monitored after an uncomplicated surgery? 2 weeks 5 days 21 days 48 hours

2 weeks Most incisions will heal in approximately 10-14 days. Although the skin has not fully returned to its pre-surgical strength, this is enough time that an incisional complication is extremely unlikely. Incisions should be carefully monitored for excessive pain, swelling, discharge, or bleeding for the first 10-14 days after surgery or until suture/staple removal.

A large dog collapses in the lobby of your veterinary clinic. The dog is taken to the treatment area where an ECG reveals that the patient is in ventricular tachycardia. What drug would be administered intravenously in an attempt to convert the dog to a normal sinus rhythm? 2% lidocaine Ephinephrine Vasopressin Digoxin Atropine

2% lidocaine 2% lidocaine, administered as a bolus dose, can convert a dog into a normal sinus rhythm. If the bolus dose is successful, the veterinarian may prescribe a constant rate infusion to help maintain the rhythm. Epinephrine and atropine are used to in CPCR. Vasopressin is a potent blood pressure medication. Digoxin is an oral medication used for patients with atrial fibrillation.

A 40 kg dog presents to your hospital and is approximately 5% dehydrated. What is the replacement fluid volume needed for this dog? 2,000 mL 1,200 mL 1.5 L 500 mL

2,000 mL To calculate the replacement fluids use the following formula: Body weight (in kg) X percent dehydration= fluid deficit (in Liters) For this dog: 40 X 0.05 = 2 L If you want the fluid measurement in milliliters, just multiply by 1,000 (since there are 1,000 mL in 1 L). For this dog, it would be 2,000 mL (the same as 2 Liters).

You want to give a pre-surgical dose of antibiotics to your patient. You have a 1 gram vial of cefoxitin that is reconstituted with 5 mls of sterile water. You want to give a 20 mg/kg dose to a 25 kg dog. How many mls do you give? 10 1.25 2.5 7.5 5

2.5 The resulting concentration of the cefoxitin solution is 1gm/5ml or 200mg/ml. The desired dose is 20mg/kg x 25 kg = 500 mg 200 mg/ml x ____ ml= 500 mg, Therefore the answer is 2.5 ml

You are recovering an 80 pound patient post-op and the dog is painful. The surgeon would like you to give a Fentanyl injection prior to starting the constant rate infusion. He asks you to give 4 micrograms per kg IV. Fentanyl concentration is 0.05 mg/mL. What quantity do you administer? 2.9 mL 1.2 mL 0.9 mL 0.29 mL

2.9 mL First convert to kg: 80 pounds/2.2 = 36.3 kg 36.3 kg X 4 micrograms= 145 micrograms Now convert the microgram dose into milligrams: 145 micrograms / 1,000 = 0.145 mg Now divide the dose by the concentration of the medication: 0.145 mg/ 0.05 mg/mL = 2.9 mL

A patient is hypoventilating and the end-tidal CO2 is rising. The doctor instructs you to give the patient a breath. The pressure on the manometer to which you ventilate the patient is: 5 cm H2O 40 cm H2O 60 cm H2O 20 cm H2O

20 cm H2O When inflating the lungs, the pressure should reach but not exceed 20 cm H2O.

What quantity of drug is found in 200 mLs of a 10% solution? 20 grams 2,000 grams 2,000 milligrams 20 milligrams

20 grams A 10% solution is equal to 100 mg/mL. The easiest way to remember this conversion is to add a zero to the percentage. (10%= 100 mg/mL, 15% = 150 mg/mL, etc.) If 1 mL is equal to 100 mg, then to find out the quantity of drug in 200 mLs, multiply 100 mg X 200 = 20,000 mg. There are 20,000 mg in 200 mLs of a 10% solution. But none of the choices are 20,000 mg. We need to go one step further and determine how many grams this is. 1 gram is equal to 1,000 mg. We need to divide the milligrams by 1,000. So, 20,000mg / 1,000 = 20 grams.

How many milliliters of 25% dextrose should be added to 1 L of 0.9% saline to make a 5% dextrose solution? 200 ml 20 ml 50 ml 100 ml

200 ml To answer this question, the following formula should be implemented: C(1)V(1) = C(2)V(2) C is the concentration V is the volume C(1) =0.25 V(1) = x C(2) = 0.05 V(2) = 1000 ml 0.25x = 0.05(1000) x = 200 ml 200 ml of 25% dextrose should be added to one liter in order to make a 5% solution.

A 50-kg Golden Retriever has presented with extremely pale mucous membranes and is diagnosed with a hemoabdomen. A blood transfusion has been requested, and the doctor would like a 10ml/kg/hr transfusion rate for 4 hours. How many milliliters of blood will you need? 20 ml 40 ml 2000 ml 1000 ml

2000 ml The patient is receiving 500 ml/hr. 50 kg x (10 ml/kg/hr) = 500 ml/hr In four hours, the patient will receive 2000 ml 4 hr x (500 ml/hr) = 2000 ml.

You are viewing a blood sample in a compound microscope at a 20X objective lens. This microscope (as with most compound microscopes) has an ocular lens that has 10X magnification. What is the total magnification at which you are viewing the sample? .5X 200X 10X 20X 30X

200X

The heart rate for an adult horse should generally be in which of the following ranges? 25-50 beats per minute 60-80 beats per minute 15-25 beats per minute 45-65 beats per minute

25-50 beats per minute

A puppy presents for his first distemper-parvo puppy shot. He is currently 8 weeks of age. When should he receive his next set of vaccines? 1 week from now 4 months from now 8 weeks from now 3 weeks from now

3 weeks from now The puppy series starts at 6 to 8 weeks of age, and the distemper-parvo combo vaccine is given 3 weeks apart for a total of 3 to 4 vaccines, depending on the breed. Black-and-tan breeds such as Rottweilers are thought to be more susceptible to parvo-virus, so this breed may receive a series of 4 to 5 vaccines, depending on the age and status of the pet.

You are instructed to add 40 mEq of potassium chloride (KCl) to a 1 Liter bag of 0.9% sodium chloride for a patient. The patient to receive fluids weighs 50 pounds and will be on a rate of 85 mL/hr. You know that the maximum potassium a patient should receive is 0.5 mEq/kg/hr. What is his potassium intake, and is it too high? 5.6 mEq/hr; Yes 3.4 mEq/hr; No 2.0 mEq/hr; No 12.7 mEq/hr; Yes

3.4 mEq/hr; No The correct answer is 3.4 mEq/hr, which is only about 1/3 of his maximum potassium dose per hour, so it is not too high. Let's do the math on this problem: First, calculate how many mEq/hr he will be receiving with this dose and rate: (There are 1,000 mL in 1 Liter) 40 mEq/1000 mL = 0.04 mEq/mL Next multiply by his fluid rate: 0.04 mEq/mL X 85 mL/hr (mL cancel out) = 3.4 mEq/hr he is receiving currently Now calculate his maximum dose: Convert his body weight to kg: 50 pounds/2.2 pounds/kg = 22.7kg 0.5 mEq X 22.7 kg = 11.3 maximum mEq/hr he can have He is getting 3.4 and his max is 11.3, so he is only getting about 1/3 of his maximum.

You are working in the large animal ward at the hospital. You are receiving a horse that needs immediate treatment for dehydration. The vet asks you to calculate the fluid deficit for this horse. He is a 950-pound horse that is about 7% dehydrated. What is his fluid deficit? 5 L 30 L 63 L 15 L

30 L You can estimate the fluid deficit by the following formula: Body weight in kg X % dehydration = Liters needed to re-establish hydration So for this horse: 950 pounds/2.2 pounds per kilogram (pounds cancel out) = 431.8 kg (will round up to 432kg) Now, multiply by percent dehydrated: 432 kg X 0.07 (which is 7%) = 30 Liters

Perioperative antibiotics are essential in preventing unwanted infections. What is the best time to administer the antibiotic? 30 minutes prior to making an incision At the time of the incision 24 hours prior to surgery 1 hour prior to anesthesia Immediately after surgery

30 minutes prior to making an incision Research has shown that the best time to administer perioperative antibiotics is approximately 30 minutes prior to making an incision. This allows time for the antibiotics to have adequate circulating blood levels, resulting in good protection. By convention, most surgeons will repeat a dose of antibiotics every 90 minutes while in surgery. There is no literature to support the use of antibiotics after surgery in uncomplicated surgeries. If there is substantial contamination, break in technique, or other indication for continued use of antibiotics such as discovery of an abscess, then it is beneficial.

A 38-kg Great Dane is recovering from Gastric Dilatation Volvulus surgery. The doctor would like to send him home on Tramadol. The doctor requested that a prescription be filled out for 5 days at 3 mg/kg orally three times per day. How many pills should you send home? Tramadol is available in 50 mg tablets. Round to the nearest whole number. 50 tablets 30 tablets 25 tablets 40 tablets

30 tablets 38 kg x 3 mg/kg = 114 mg 114 mg / 50 mg = 2.28 (Rounding to the nearest whole number would be 2 tablets) If getting 2 tablets per dose, then 6 tablets per day are needed. Five days' treatment would be 30 tablets.

A 60-lb patient is to go home on Clavamox after his dental cleaning. The dose is 14 mg/kg. What size tablet should the patient be sent home with? 62.5 mg 125 mg 375 mg 250 mg

375 mg After converting the body weight to kilograms the dose needs to be multiplied by the body weight. 60 lb / (2.2 lb/kg) = 27 kg (Notice the pounds cancel out) 27 kg x 14= 378 mg Many medications are available in different concentrations to accommodate different sized patients. This is particularly the case with oral medications so that owners don't have to worry about trying to halve or quarter medications.

A patient has developed a severe run of ventricular premature contractions. The clinician asks you to immediately pull up a 4 mg/kg dose of lidocaine for to administer intravenously. The lidocaine in the hospital comes in a 2% solution. How many milliliters should you draw up for this 20 kg patient? 12 ml 2 ml 8 ml 4 ml

4 ml To determine how many milligrams are need, do the following math: 20 kg x (4 mg/kg) = 80 mg. Now the difficult part is determining how to calculate how many milliliters of a 2% solution of lidocaine is equivalent to 80 mg. Percent is equal to weight (in Grams) / 100 parts volume (in mLs) (This is based on water, since 1 mL weighs 1 Gram): Example: A 2% solution: 2 Grams/100 mL or 2000 mg/100 mL or 20 mg/mL The easiest way to remember is to just add a zero onto any percentage for the milligrams (i.e. 2% solution = 20 mg; 10% solution = 100 mg; 25% solution = 250 mg). Therefore, 80 mg / (20 mg/ml) = 4 ml

A dog with hypothyroidism comes in to have his blood drawn to check his thyroid level. He has been on Levothyroxine since he was first diagnosed. How many hours after the thyroid medication is given should the blood be drawn? 12 hours just before the next dose is due 5-7 hours 8-10 hours 1-2 hours 4-6 hours

4-6 hours It is best to draw the blood 4-6 hours after medication is given because that is when the thyroid level is at its highest peak.

A client has a dog with allergies, and the doctor recommends she give the dog diphenhydramine (Benadryl) twice daily for 1 week. She instructs that the dog have 3.0 mg/kg every 12 hours. Over-the-counter Benadryl contains 12.5 mg per teaspoon, and the owner wishes to give a liquid. What quantity of Benadryl in milliliters should the dog receive every 12 hours if he weighs 8 pounds? 3.1 mL 4.3 mL 1.2 mL 0.85 mL

4.3 mL To solve this question you need to know that there are 5 mL in 1 teaspoon. This means that the concentration of the Benadryl liquid is: 12.5 mg/5 mL = 2.5 mg/mL The dog weighs 8 pounds/ 2.2 pounds/kg = 3.6 kg The dog needs 3.0 mg/kg per dose. So, 3.0 mg/kg X 3.6 kg = 10.8 mg per dose (every 12 hours) 10.8 mg/ 2.5 mg/mL = 4.3 mL

A 120-pound Great Dane has presented with Gastric Dilatation Volvulus. The clinician has asked you to quickly prepare the drug protocol. As a premedication, the dog is to receive 0.08 mg/kg of Hydromorphone. How many milligrams will you administer? 4.4mg 2.2 mg 9.6 mg 6.4 mg

4.4 mg To calculate milligrams administered the dog's body weight should be converted to kilograms. As a general rule, the majority of medications are dosed in a milligram-per-kilogram dosing. Therefore all body weights need to be converted to kilograms so that you are working in the same units. 120 lb/2.2 = 54.5 kg (There are 2.2 lbs per kilogram) 54.5 kg x (0.08 mg/kg) = 4.4 mg (Notice the "kg" will cancel out when you do the math) If your answer was 9.6 mg, you did not convert the body weight into kilograms.

You have a bottle of 50% Dextrose. You are asked to add it to a bag of LRS for a pet in diabetic ketoacidosis to create a 2.5% solution. The pet has 800 mL left in his bag. How much will you add to his bag? 60 mL 80 mL 25 mL 20 mL 40 mL

40 mL 800 mL X 0.025 = 20 mL. If this were 100% dextrose, adding 20 mL to the bag would create the 2.5% concentration. Since it is 50% dextrose, this number must be doubled: So, 20 mL X 2 = 40 mL Draw out 40 mL from the bag (now there are 760 mL left in the bag). Add in 40 mL of 50% Dextrose. Now, 2.5% of the total 800 mL is Dextrose. Double check the math: 40 mL/ 800mL = 0.05 (i.e. 5%) divided by 2 {because it was 50% dextrose that was added) = 0.025 = 2.5%.

You calculate the volume deficit for a patient which is approximately 2 liters. You are instructed to give this volume over the next 5 hours. What should the fluid rate be? 160 mL/hr 310 mL/hr 400 mL/hr 250 mL/hr

400 mL/hr First convert the liters (L) into milliliters (mL): (there are 1000 mL in 1 L) 2 L X 1000mL/L (liters cancel out)= 2000 mL You need to give this over 5 hours: 2000 mL/ 5 hours (mL cancel out)= 400 mL/hr

You perform a dental prophylaxis on a dog that has all of his teeth. How many adult teeth are in the dog's mouth? 24 42 28 32

42

How many days after fertilization will canine fetal skeletons first appear on a radiograph? 50 days 42 days 22 days 14 days 63 days

42 days At 42 days after fertilization, calcium is absorbed into the fetal skeletons and makes them visible on the radiograph.

An owner brings her 4-year old female Labrador Retriever to your clinic because she believes she might be pregnant. She does not remember when the dog's last heat cycle was. The dog's abdomen appears fairly distended and you take a lateral abdominal radiograph which is shown below. What would be the earliest time you would expect to be able to see fetal skeletons on abdominal radiographs in the dog? 23 days gestation 43 days gestation 53 days gestation 33 days gestation

43 days gestation The correct answer is 43 days gestation. The fetal skeleton ossifies at 42-45 days in the dog and 35-39 in the cat. In the dog, a mineralized fetus can usually be seen around 42-46 days. The scapula, humerus, and femur can be made out around 46-51 days. The ribs can be seen at 52-59 days. Teeth and toes can be seen at 58-63 days.

What is the shock dose of fluids for a cat? 45 mL/kg 150 mL total per cat 50 mL total per cat 90 mL/kg

45 mL/kg The full shock dose of fluids for cats is 45 mL/kg of crystalloids. When giving shock doses of fluids, they are often given in 1/4 increments. The shock dose of fluids for dogs is 90 mL/kg.

At what age should pigs be castrated? 5 - 14 days 8 months 4 months 4 weeks

5 - 14 days

A 7-kg Dachshund "Rudy" is being treated for pancreatitis and has been vomiting. You are asked to start him on a constant rate infusion of metoclopramide at 2 mg per kilogram per day (mg/kg/day). He is on a fluid rate of 23 mL/hr. Metoclopramide comes in a concentration of 5mg/mL. How much metoclopramide will you add to a liter bag of LRS? 8 mL 5 mL 3 mL 1.2 mL

5 mL Constant rate infusions are not something that an entry level technician would typically perform. However, the VTNE may have some questions on CRI and it will benefit you to know how to do these calculations. To solve this problem, break down the basic parts to make it less overwhelming: 1) Convert pounds to kilograms- Note that in this problem the units are already in kg so no conversion is necessary. Make sure to pay attention to this. 2) Calculate the number of hours an infusion will last by dividing the volume in the bag (1 Liter, or 1,000 mL) by the rate per hour: 1,000 mL/23mL/hr = 43 hours We want to make sure the units match the equation in the problem which asks for mg/kg/DAY. So divide the hours by 24 (since there are 24 hours in a day): 43 hours/24 hours/day = 1.8 days 4) Now plug into the equation: 2mg/7kg/1.8 days= 25 mg 5) Calculate the quantity of metoclopramide to add by now dividing the 25 mg by the concentration of the drug (5 mg/mL) : 25mg/5mg/mL = 5 mL To be truly accurate when adding drugs to a bag like this, you should discard 5 mL of fluids from the bag prior to adding the 5 mL of metoclopramide.

When scrubbing in for a surgery, the minimum time for a scrub is which of the following? 10 minutes 5 minutes 15 minutes 1 minute

5 minutes

A cat is starting weekly subcutaneous fluid administration for renal failure at the clinic where you work. The client asks how long it will take for him to absorb the fluids after they are given. You tell him most cats will absorb the fluid within: 5-8 hours 1 hour 24 hours 48 hours

5-8 hours Subcutaneous fluids are most commonly absorbed within 5 to 8 hours of administration. However, fluids are absorbed when they are needed, and absorption is dependent on having adequate SQ blood flow, which can be decreased in severely hypothermic or dehydrated patients.

What is the maximum amount of oral fluids that a horse should receive at a time? 20-25 Liters 6-8 Liters 1-2 Liters 10-15 Liters

6-8 Liters The average size horse (800 to 1200 pounds) has a stomach capacity of only about 4 gallons. It is best to give smaller quantities of fluids more frequently, and no more than 6 to 8 liters should be given at one time.

A 20-kg patient is currently receiving 52 ml/hr of Lactated Ringer's Solution. The clinician has requested that 2 mg/kg/day of metoclopramide be added to the bag. Metoclopramide is available as a 5 mg/ml solution. How many milliliters do you need to add to a 1 liter bag? 6.5 ml 3 ml 45 ml 0.8 ml

6.5 ml This is definitely a more complicated calculation question, but when broken down it is not insurmountable. A 20-kg patient will need 40 mg/day of metoclopramide (20 kg x 2 mg/kg/day) Since the fluid is being administered at an hourly rate, we need to figure out how much metoclopramide we are administering per hour. 40 mg/day / (24 hr) = 1.7 mg/hr To determine how much volume of metoclopramide is needed, the following math is performed: 1.7 mg/hr / (52 ml/hr) x (1000 ml)= 32.6 mg (Notice how the units cancel out.) 32.6 mg / (5 mg/ml) = 6.5 ml

A 12 kg Springer Spaniel has been presented to your hospital for pyometra surgery. The veterinarian would like you to give 20 mls of Unasyn over 20 minutes on a syringe pump before the procedure. How many milliliters per hour will you set the pump to run for? 60 20 1 3

60 20 mls multiplied by 60 (minutes in 1 hour)= 1200 divided by 20 minutes= 60 mls/hr.

A 55-pound Pit Bull mix "Rocky" was hit by a car and is hospitalized in the emergency ward. The ECG is showing continuous runs of ventricular tachycardia. The veterinarian asks you to start a constant rate infusion of lidocaine to be given at 80 micrograms per kilogram per minute (ug/kg/min). The dog is currently on a fluid rate of 80 mL per hour. Lidocaine is a 2% solution. How much lidocaine will you add to a liter of LRS? 26 ml 75 ml 19 ml 5 ml

75 ml Constant rate infusions are not something that an entry-level technician would typically perform. However, the VTNE may have some questions on CRI, and it will benefit you to know how to do these calculations. To solve this problem, break down the basic parts to make it less overwhelming. *Note that a 2% solution is the same as 20 mg/mL (add a zero to the % number, so 1% is 10mg/mL, 20% is 200 mg/mL, etc.) 1) Convert pounds to kilograms-55 pounds/2.2 pounds per kg = 25 kg 2) Calculate the number of hours an infusion will last by dividing the volume in the bag (1 liter, or 1,000 ml) by the rate per hour: 1,000 ml/80ml/hr = 12.5 hours 3) Now take the number of hours (12.5) and multiply by 60 to calculate how many minutes this is: 12.5 X 60 = 750 minutes 4) Plug into the equation: 80 ug/25kg/750min= 1,500,000 ug 5) Now convert the ug into mg by dividing by 1,000: 1,500,000/1,000 = 1500 mg 6) Calculate the quantity of lidocaine to add by dividing the 1500 mg by the concentration of the drug (20mg/ml): 1500mg/20mg/ml = 75 ml To be truly accurate when adding drugs to a bag like this, you should discard 75 ml of fluids from the bag prior to adding the 75 ml of lidocaine.

What percentage of dogs and cats, on average, has periodontal disease? 40% 95% 75% 25% 60%

75% It is thought that 80% of dogs and 70% of cats have some degree of periodontal disease over the age of 3 years. This is why it is so important to discuss dental care with pet owners.

A veterinarian hands you a prescriptions that reads: Cephalexin 250 mg, SIG: 3 cap PO BID x 14d What is correct amount of medication to dispense and what directions should be typed on the drug label? 84 capsules (250 mg each) of cephalexin. Give 3 capsules by mouth twice daily for 14 days. 136 capsules (250 mg each) of cephalexin. Give 3 capsules by mouth three times daily for 14 days. 42 capsules (75 mg each) of cephalexin. Give 3 capsules by mouth once daily for 14 days. 84 capsules (75 mg each) of cephalexin. Give 3 capsules by mouth twice daily for 14 days. 42 capsules (250 mg each) of cephalexin. Give 3 capsules by mouth twice daily for 14 days.

84 capsules (250 mg each) of cephalexin. Give 3 capsules by mouth twice daily for 14 days. The signatura (SIG) or transcription is the directions for the medication. PO stands for "Per Os" which means orally. BID means twice daily. So the correct instructions are: Give 3 capsules by mouth twice daily for 14 days. If 3 capsules are given twice daily, then 6 are given total per day. 6 X 14 days= 84 capsules

You are asked to start a dog on maintenance fluids. The dog weighs 80 pounds. What fluid rate do you start on this dog? 45 mL/hr 90 mL/hr 150 mL/hr 25 mL/hr

90 mL/hr There are several ways to calculate maintenance fluids. Some are more accurate than others, with the more accurate ones requiring more complex math. Fortunately, in most patients, being off by a few milliliters does not adversely affect the patient. The most basic calculation to approxiamte maintenance rate fluids in a dog is: 60mL/kg/day 80 pounds/2.2= 36 kg 60mL X 36 kg = 2160 mL per day 2160 mL/24 hours in a day = 90 mL per hour

A 3-year old male neutered Labrador Retriever presents for lethargy and inappetence of a few days in duration. The dog is approximately 7% dehydrated with pale and dry mucous membranes, a capillary refill time > 2 sec, heart rate > 170 bpm and weak femoral pulses. The veterinarian diagnoses the dog with hypovolemic shock and orders isotonic crystalloid fluid therapy. What is the isotonic crystalloid total "shock dose" in dogs? 90 ml/kg 10-20 ml/kg 5 ml/kg 40-60 ml/kg 180 ml/kg

90 ml/kg The isotonic crystalloids shock dose in dogs is 90 ml/kg. 1/4 to 1/3 of this volume should be administered initially and the patient reassessed. In cats, the isotonic crystalloids shock dose is 40-60 ml/kg. Synthetic colloids can be administered at a dose of 10-20 ml/kg in dogs and 5-10 ml/kg in cats; hypertonic saline can be administered at a dose of 5 ml/kg.

Which of the following can occur as an injection of Cerenia (maropitant) is given subcutaneously? Bradycardia A burning or stinging sensation Hyperkalemia Acute collapse

A burning or stinging sensation Cerenia is given as an anti-nausea to prevent vomiting or motion sickness in dogs. Maropitant is a neurokinin (NK1) receptor antagonist that blocks the pharmacological action of substance P in the central nervous sytem (CNS). Some studies have also concluded that this medication provides visceral analgesia to some patients. It may cause a burning or stinging sensation, and sometimes the pet will whine or cry when receiving this injection SQ. Acute collapse and bradycardia can occur secondary to any injection but is not common, nor more prevalent when maropitant is administered. Hyperkalemia is unlikely to occur instantaneously in any situation.

A 6-year old male domestic short hair presents dyspneic with fluid dripping from both nares. The veterinarian suspects heart failure due to hypertrophic cardiomyopathy. What type of injection would most likely immediately be given? Intravenous fluids A corticosteroid A diuretic A sedative An antibiotic

A diuretic Hypertrophic cardiomyopathy is a condition whereby the myocardium becomes thickened and diastolic function becomes impaired, often resulting in congestive heart failure. The diuretic is given to decrease blood volume and therefore reduce the strain on the heart. Intravenous fluids should not be given as they would increase plasma volume. A sedative might be advantageous to calm the patient but should be given after treatment has begun. An antibiotic will not treat HCM. Corticosteroids are contraindicated in cases of heart failure.

Which of the following types of wounds would most benefit from a wet-to-dry bandage? A dog with a clean cut on its side from a fence A dog that has a scrape on its hock with a large surface area A dog hit by a car that has an open wound with gravel debris and macerated tissues A dog that has a lick granuloma on its forelimb that is severely inflamed

A dog hit by a car that has an open wound with gravel debris and macerated tissues Wet-to-dry bandages best serve the purpose for debriding necrotic tissue, especially in the cases of trauma where the wounds are dirty and have the potential of having tissue necrosis around the insult. The goal is to treat these wounds with wet-to-dry bandages until the tissues have healthy granulation tissue and can be adequately debrided and closed. Also, these bandages are beneficial in trauma cases when primary closures are not possible due to more significant internal injuries that need to be addressed first.

What constitutes anemia? A low neutrophil count A low thrombocyte count A low packed cell volume An increased hematocrit

A low packed cell volume Anemia is a condition in which the body doesn't have enough healthy red blood cells (erythrocytes). A low packed cell volume (PCV), or hematocrit (HCT), constitutes anemia. The PCV is the proportion of blood volume that is occupied by the red blood cells. Also important to note is that in mammals, the hematocrit is not related to body size. Normal PCV of a dog is between 37-55% and cats 29-50%. Puppies and kittens normally have lower PCV's. A low thrombocyte count would be thrombocytopenia (low platelets). A low neutrophil count would be neutropenia.

Which animal might receive a perineal urethrostomy? A dog that has a puppy stuck in the birth canal A male cat with a urethral obstruction A female dog with a large bladder stone A dog with a pus-filled uterus

A male cat with a urethral obstruction Perineal urethrostomies are performed in male cats with a urethral blockage. The procedure is performed when a cat has had multiple episodes of obstruction or when the obstruction cannot be relieved manually. The procedure entails making a new opening in the urethra in the perineal area and removing the penis.

What is an osteosarcoma? A malignant bone tumor A benign tumor originating from cartilage A benign bone tumor A benign tumor of the skull sutures A malignant tumor of the spleen

A malignant bone tumor An osteosarcoma is a malignant bone tumor. Osteosarcoma is the most common bone tumor in dogs and has a 90% metastatic rate. An osteoma is a benign bone tumor. Tumors of the skull sutures are rare and do not have a specific name; a benign bone tumor of the skull would be a skull osteoma. There are various types of cancerous splenic tumors with hemangiosarcoma (a tumor of blood vessels) being the most common. A benign tumor of cartilage is a chondroma.

Which ICU patient on fluids needs to be watched closely for jugular distention? A patient with a heart murmur A patient with history of seizures A patient in renal failure An aggressive patient

A patient with a heart murmur A patient with underlying cardiac disease is at the highest risk of pulmonary edema (fluid in the lungs) from fluid overload. An indication of possible fluid overload is jugular distention. A good way to monitor hydration status of patients like this in the ICU is via central venous pressure readings.

A client calls and asks if you could do a necropsy for her pet. What is she asking for? A post-mortem exam to be performed in an effort to find out the cause of death If you offer a taxidermy service If your hospital offers a cremation service on site If your hospital could provide an exploratory laparotomy

A post-mortem exam to be performed in an effort to find out the cause of death Necropsy is examination of an animal after death to try and discern possible causes of death by determining abnormal changes or disease processes which occurred during life. This procedure should only be performed with client consent. Necropsy should be performed immediately if possible. If not, the body should be refrigerated (not frozen) until the procedure can be performed.

In which situation would a post-operative sponge count be necessary following an abdominal exploratory? Sponge counts only need to be performed for mass removals Sponge counts are performed only when there is excessive bleeding A sponge count should always be performed regardless of the nature of the abdominal surgery Sponge counts are not necessary since they are radiopaque

A sponge count should always be performed regardless of the nature of the abdominal surgery Inadvertently leaving a sponge in the abdomen is a completely avoidable complication which unfortunately still occurs. Sponge counts should always be performed at the beginning of surgery and then again at the end of surgery to ensure that no sponge was left behind. If a sponge cannot be located, a radiograph of the patient should be performed. The sponge will only show up on the radiograph if it had a radiopaque marker.

What is colic in a horse? Abdominal pain secondary to intestinal volvulus Abdominal pain that can be secondary to multiple etiologies Abdominal pain secondary to gastrointestinal gas Abdominal pain secondary to intestinal strangulation

Abdominal pain that can be secondary to multiple etiologies The best answer choice is abdominal pain that can be secondary to multiple etiologies. It is a very broad term, and colic can be secondary to any of the other answer choices. In effect, knowing that the patient has colic does not necessarily mean the clinician knows exactly what treatment to provide until the underlying cause of colic is identified.

Which of the 4 stomach compartments in the ruminant is considered the true glandular stomach? Reticulum Omasum Abomasum Rumen

Abomasum The abomasum is the true glandular stomach and is responsible for mixing food with digestive enzymes. The omasum grinds up the food material and absorbs water and bicarbonate. The rumen is the where all the fermentation is occurring thanks to the help of symbiotic organisms that help break down the food. The rumen occupies the majority of the LEFT side of the abdominal cavity. The reticulum acts as a conduit for food and helps pace the contractions of the rumen.

What is a localized accumulation of pus in the body is known as? Abscess Vesicle Cyst Seborrhea Adenoma

Abscess An abscess is a collection of pus that accumulates and is surrounded by inflammatory tissue. A cyst is a closed-off sac that may contain fluid, air, semi-solid material, etc. A vesicle is a small blister on the skin or mucosa, similar to a cyst. Adenoma is a benign glandular tumor. Seborrhea is a skin condition characterized by either greasy skin from excessive sebum (overproductive glands in the skin), or dry skin characterized by flaking scales.

Which of these drugs should be used with great caution in stallions due to the risk of causing paraphimosis or priapism? Acepromazine Atropine Trimethoprim-sulfa Ketamine

Acepromazine The correct answer is acepromazine. Phenothiazine tranquilizers can cause relaxation of smooth muscles and engorgement of the corpus cavernosum with blood, leading to an inability to withdraw the penis into the sheath, or paraphimosis. While this drug side effect is very rare, veterinarians should be cautious in the use of acepromazine in stallions and should consider another sedative such as xylazine or detomidine when sedation is necessary.

A young male cat with symptoms of feline lower urinary tract disease and struvite crystalluria should receive: Diet with low fat Alkalinizing diet Diet with low protein Acidifying diet

Acidifying diet Struvite crystals form when the pH of the urine is high or alkaline. Therefore, an acidifying diet would help to decrease the pH of the urine and prevent formation of the crystals. It can often be of benefit in cases of FLUTD in addition to increasing water intake (canned diets can help with this) and decreasing stress in the home.

Which disease can cause a severe bradycardia that could be life-threatening? Cushing's disease Hyperthyroidism Portosystemic shunt Hypothyroidism Addison's disease

Addison's disease Hypoadrenocorticism (Addison's disease) can cause a high potassium level (hyperkalemia) and can be life-threatening. Addison's occurs when the adrenal glands do not produce enough cortisol, resulting in a crisis. A low heart rate is often associated with high potassium. Sodium chloride is the fluid treatment of choice to help reduce the potassium.

An animal presents with difficulty breathing. You take the animal to the treatment area. The doctor is on the way. What do you do first? Administer supplemental oxygen Don't do anything until the doctor arrives, just restrain the animal Start taking chest radiographs Place an IV catheter and start IV fluids

Administer supplemental oxygen All animals in respiratory distress can benefit from supplemental oxygen. If the patient is stressed, then blow-by oxygen can be used if a mask isn't tolerated. The technician should never attempt to do other treatments such as starting fluids, performing radiographs, etc. without instruction from the veterinarian.

Which of the following is the reason that fluoroquinolones are usually avoided when treating young foals? Ineffectiveness against the most common pathogens of foals Hepatotoxicity in foals Adverse effects on developing dental enamel Adverse effects on the articular cartilage Nephrotoxicity in young animals

Adverse effects on the articular cartilage Fluoroquinolones can be used in mature horses but are believed to have adverse effects on growing immature cartilage.

What is the term that describes nerve fibers that carry an impulse TOWARD the brain? Peripheral Dendritic Efferent Afferent

Afferent Afferent fibers carry impulses toward the brain and/or spinal cord. Efferent fibers carry them away from the brain and/or spinal cord. Peripheral nerves are outside of the brain and spinal cord. Dendritic is an adjective that describes a highly branched structure and refers to the projections from a nerve cell.

You are asked to recover a patient after surgery. When should you pull the endotracheal tube? After the patient's temperature has reached at least 100.0 After the patient is able to move its limbs After the patient has an adequate swallowing reflex After the eyes are alert and the dog is aware of its surroundings

After the patient has an adequate swallowing reflex The endotracheal tube should be pulled when the animal has an adequate swallowing reflex. Brachycephalic breeds should have their tube kept in as long as possible due to their predisposition for respiratory problems.

What adverse side effect can occur in cats given metoclopramide? Dilated and fixed pupils Aggressive behavior Cyanosis Estrus behavior

Aggressive behavior Rare side effects that can occur are nervous, jittery, or aggressive behaviors. The antidote for these types of behaviors is diphenhydramine (Benadryl). Head bobbing can also rarely occur as a side effect of Reglan (metoclopramide).

The veterinarian diagnoses a tension pneumothorax in a dog that was hit by a car. What does this mean? The airway is obstructed Air is building up inside the lungs and causing pressure in the chest A broken rib is protruding from a chest wound Air is building up in the pleural space and can't escape

Air is building up in the pleural space and can't escape Tension pneumothorax is when air accumulates under pressure in the pleural space (around the lungs, not in the lungs). This condition develops when a tear or hold in the chest forms a 1-way valve, allowing air to enter the pleural space but preventing the air from getting out. In this situation, the pressure inside the chest exceeds the pressure outside the chest.

You are assisting with chemotherapy treatments. Which of the following is true regarding blood collection from chemotherapy patients? Do not use the jugular vein if possible The patient should not have blood drawn within 72 hours of their last chemotherapy treatment All blood samples should be drawn from the jugular if possible All chemotherapy patients should be sedated prior to blood draws

All blood samples should be drawn from the jugular if possible Blood should be drawn from the jugular veins when possible. Peripheral veins are spared for intravenous chemotherapy administration or for catheter placement during hospitalization stays. The other answer choices are false.

Diphenhydramine is typically used in treating which of the following conditions? Allergic reactions Urinary incontinence Keratoconjunctivitis sicca Urinary tract infections Auto-immune conditions

Allergic reactions Diphenhydramine is also known as Benadryl. It is a an anti-histamine most commonly used for treating allergies or allergic reactions. A urinary tract infection would require an antibiotic. Keratoconjunctivitis sicca is an immune mediated condition and is usually treated with cyclosporine. Auto-immune conditions also call for immunosuppressive agents. Urinary incontinence may be treated with phenylpropanolamine.

A chihuahua is born with no hair. What is a term for no hair? Hypertrichosis Pruritus Mange Alopecia

Alopecia Alopecia is lack or loss of hair/baldness. This chihuahua has hypotrichosis (lack of hair). It can be familial in this breed and is also more common in poodles. Hypertrichosis is excessive hair. Mange is a skin condition caused by mites that can lead to alopecia and skin pathology. Pruritus is the term for itchy.

Gas exchange occurs between air and blood by diffusion through which structure? Pleura Bronchi Alveoli Trachea

Alveoli The alveoli are the site of gas exchange. They are tiny air sacs in the lungs that exchange carbon dioxide and oxygen and are located at the last branching of the respiratory tract. They are lined with simple squamous epithelium. The oxygen diffuses through the epithelium and the capillary endothelium in the exchange. The upper airway is lined mostly with pseudostratified columnar epithelium.

How can a technician determine if a pack was sterilized appropriately? An indicator strip inside the pack changes color The pack is steaming when removed from the autoclave The pack is double wrapped prior to autoclaving, ensuring sterility The tape on the outside of the pack is not broken

An indicator strip inside the pack changes color The only way to know for sure if a pack has received appropriate sterilization is if the indicator strip changes color inside the pack.

Anemia would likely result from a heavy infection with which parasite? Ancylostoma Cystoisospora Toxocara Dipylidium

Ancylostoma

Which of the following can potentially infect and cause dermatitis in a child walking barefoot through a field? Trichuris vulpis Toxocara spp. Dipylidium caninum Ancylostoma spp. Dirofilaria immitis

Ancylostoma spp. The correct answer is Ancylostoma spp. Ancylostoma caninum, the dog hookworm, can infect a human by penetrating skin. It can cause dermatitis via cutaneous migration of the parasite. Toxocara is a roundworm that causes visceral larval migrans.

Which of the following terms describes variation in cell size? Anisocytosis Agglutination Anisokaryosis Megalocytosis

Anisocytosis Anisocytosis is the term used to describe variation in cell size. Agglutination is a descriptive term for cells red blood cells that are adhering or clumping with each other. Anisokaryosis implies variation between cells in the amount of cytoplasm present. Megalocytosis implies enormous and abnormal cells, usually with multiple nuclei present.

One of the most common incisional complications encountered in veterinary surgery is the formation of a seroma. Which of the following is a poor treatment choice for an incision diagnosed with a seroma? Placement of a drain Exercise restriction Antibiotics Warm compress

Antibiotics Seromas are not infections and do not need to be treated with antibiotics. They consist of serous fluid, which is usually secondary to excessive dissection or "dead-space" that was created during surgery. Excessive activity and motion at the surgical site may predispose to the formation of a seroma. A warm compress will help increase blood flow to the region and thus help the body reabsorb the fluid. Nonetheless, it may take days to weeks for a seroma to completely resolve. Placing a drain at the site of a seroma may help speed up its resolution, as this helps keep the tissues adhered to each other.

Drugs from which of the following categories are inappropriate to inject into the epidural space? Opioids (ie; morphine) Coriticosteroids (ie; methylprednisolone) Local anesthetics (ie; bupivacaine) Antibiotics (i.e; ampicillin) Alpha-2 agonists (ie; xylazine)

Antibiotics (i.e; ampicillin) It is inappropriate to inject antibiotics into the epidural space. All of the other categories listed have drugs that may provide analgesia and are safe when placed in the epidural space.

You are performing an FeLV snap test on a kitten that was found by your neighbor. What type of test is this? Antigen test Immunodiffusion test Antibody test Immunohistochemistry test

Antigen test The FeLV (feline leukemia virus) test is an antigen test, meaning it is testing for the agent itself. The FIV test is an antibody test, meaning it is testing for antibodies made by the host against the agent (immune response). Antigen tests are often referred to as an ELISA test (enzyme-linked immunosorbent assay)

Which of the following terms describes the something that is closer to the root of a tooth relative to another structure? Apical Distal Caudal Buccal Coronal

Apical The following terms are used in veterinary dentistry to describe different positions in the mouth and aspects of the tooth: Rostral - Any structure closer to the front of the head relative to another structure Caudal - Any structure closer to the back of the head relative to another structure Buccal - The tooth surface that faces the cheek Labial- The tooth surface that faces the lips Vestibular- surface of a tooth that is directed outward toward the vestibule of the mouth which includes the buccal (cheek) and labial (lip) surfaces Facial - The surface of the tooth visible from the front (same as the vestibular surface but applies mainly to the incisor teeth) Lingual - The surface of the mandibular teeth that is adjacent to the tongue Palatal - The surface of the maxillary teeth that is adjacent to the palate Mesial - The portion of the tooth that is in line with the dental arcade and closest to the rostral midline of the dental arch Distal - The portion of the tooth that is in line with the dental arcade and closest to the most caudal aspect of the dental arch (the opposite side of mesial) Apical - The portion of the tooth closer to the tip of the root relative to another structure Coronal - The portion of the tooth closer to the crown relative to another structure

The medications Mirtazapine and Cyproheptadine can both be used for which purpose? To stop epistaxis Sedation for brief procedures Appetite stimulation To increase hemoglobin levels

Appetite stimulation

What is the average gestation length of a horse? Approximately 230 days Approximately 340 days Approximately 400 days Approximately 290 days

Approximately 340 days

What is the average gestation length of a cat? Approximately 60 days Approximately 90 days Approximately 290 days Approximately 340 days

Approximately 60 days The average gestation length of dogs and cats is 63 days. Horses have a gestation length of approximately 340 days. Although the average for horses is approximately 11 months, it can vary by as much as 30 days. Cows have an average gestation length of 285 days (about 9 months).

What is the fluid in the anterior chamber of the eye called? Fundus Lens Cornea Vitreous humor Aqueous humor

Aqueous humor

What is the proper location for a hog snare? Around the hind leg, above the hock Around the mandible Around the neck, dorsal to the ears Around the maxilla, over the snout

Around the maxilla, over the snout The hog snare is used to control the movement of the pig. By controlling the maxilla and snout you maintain good control of the whole pig. It is similiar to the twitch in the horse in terms of behavioral control.

Which of the following should be a sterile procedure? Tooth extraction Lymph node aspirate Anal gland abscess culture Fine needle aspirate of a mass Arthrocentesis

Arthrocentesis Arthrocentesis, or joint tap, should be performed with sterile technique. A joint tap is the act of inserting a needle into the joint space to withdraw synovial fluid for culture or analysis. The oral cavity is not sterile, and tooth extraction is not sterile. Lymph node and mass aspirates should be clean procedures but are not sterile. The same applies for culturing an abscess or site of infection.

In which situation would perioperative antibiotics be strongly indicated? Arthroscopy Ovariohysterectomy 3.5 cm lipoma excision Orchidectomy

Arthroscopy Procedures that are considered uncomplicated and clean typically do not require perioperative antibiotic therapy unless there is a break in sterility. Performing arthroscopy carries a very low likelihood of infection; however, because an infection of the joint can be catastrophic, it is recommended that perioperative antibiotics be administered.

What is commonly done to get a mare to cycle out of season? Administration of oxytocin Keep a stallion in the same barn Manual manipulation of the ovaries via rectal palpation Artificial lighting

Artificial lighting Most breeders prefer for a foal to be born early in the year. This is because all horses are considered 1 year old on January 1 no matter when they are born. Many mares are not cycling at the right time for this to occur, so artificial lighting is used to trick the mare that it is the breeding season. While keeping a stallion in the barn may be of benefit, horses are seasonally polyestrous, so they cycle when there are longer days, typically March-September. 16 hours of artificial light mimics this time period. Oxytocin is the hormone responsible for milk letdown. Palpation of the ovaries does not cause cycling.

The skin of a prepped surgical site is considered: Bacteria-free Aseptic Sterile Contaminated

Aseptic Aseptic means the number of bacteria has been reduced below the level that would overwhelm the body's defenses. It is not possible to completely sterilize the skin. Note that "aseptic" and "aseptic technique" have slightly different meanings than the term "asepsis" which is the state of being free from disease-causing contaminants (such as bacteria, viruses, fungi, and parasites).

When is generally considered the optimal time to spay a dog? At 2 years of age Anytime after the third estrus At approximately 6 months of age Anytime after the second estrus, but before the third

At approximately 6 months of age

Padding is extremely important when a horse is under general anesthesia. When the animal is placed in dorsal recumbency, how much padding is required between the surgical table and the horse? At least six inches Two to four inches At least twelve inches Twelve to fourteen inches

At least six inches At least six inches of padding should be used to prevent neuropathies of the head, shoulder, and hip.

You are helping treat a dog with severe papillomas of the muzzle and oral cavity. This new therapy involves weekly intraperitoneal injections. Where is the safest place to do an intraperitoneal injection? On midline 3 inches below the umbilicus With the animal in right lateral recumbency, just caudal to the rib cage At the location of the umbilicus, 2 inches right or left of midline On midline in between the last 2 nipples

At the location of the umbilicus, 2 inches right or left of midline At the area of the umbilicus, 2 to 3 inches on either side is the safest place to give an injection into the abdominal cavity to prevent organ damage.

You need to perform a CBC and take thoracic radiographs on a somewhat fractious cat. You sedate the cat with an intramuscular injection of medetomidine. While on the X-ray table, you become concerned that the cat is not doing well, and the veterinarian asks you to reverse the effects of medetomidine. What should you give the cat? Atropine Naloxone Yohimbine Flumazenil Atipamezole

Atipamezole Atipamezole (trade name: Antisedan), an alpha-2 antagonist, is the reversal agent for medetomidine (trade name: Dormitor). Medetomidine is an alpha-2 agonist. Xylazine would be another example of an alpha-2 agonist. Yohimbine is its reversal agent. Other alpha-2 agonists are clonidine, detomidine, dexmeditomidine and romifidine. Flumazenil is a reversal agent for benzodiazepines. Naloxone is used to reverse opioids. Atropine is a muscarinic antagonist of acetylcholine and may be dangerous to use after administration of alpha-2 agonists. This is because alpha-2 agonists cause marked vasoconstriction and high afterload on the heart. Giving atropine and increasing the heart rate can place further stress on the heart. You do not want to set in motion peripheral vasoconstriction and compensatory bradycardia brought on by the alpha-2, thereby increasing the heart rate against the high afterload.

Which of the following disorders is caused by a Type I hypersensitivity where chemical mediators are released from mast cells after encountering an antigen? Atopy Autoimmune hemolytic anemia Glomerulonephritis Poison ivy allergy

Atopy There are 4 types of hypersensitivity: Type I is known as immediate hypersensitivity and results from inflammatory mediators being released immediately after exposure to an antigen. Common examples are atopy and anaphylaxis. Type II hypersensitivity is known as antibody-mediated hypersensitivity and results from antibodies directed against one's own cells. An example is autoimmune hemolytic anemia. Type III hypersensitivity is known as immune complex disease and occurs when antibodies and antigens form complexes that collect and cause problems. Glomerulonephritis can be caused by type III hypersensitivity. Type IV hypersensitivity is a delayed hypersensitivity that results from T-lymphocytes reacting against antigens. Examples include certain contact allergies (such as poison ivy) or from plastic food dishes (in some dogs).

Quinidine is a medication used to treat which condition in horses? Corneal ulceration Atrial fibrillation Laminitis Colic

Atrial fibrillation Quinidine is a class 1 antiarrhythmic medication used mostly in horses to treat atrial fibrillation.

Which medication is commonly given to patients with organophosphate toxicity? Vitamin K Morphine Atropine IV vodka

Atropine Atropine is an anticholinergic drug and is an important treatment of organophosphate toxicity. This is because organophosphates are acetylcholinesterase inhibitors, and toxicity allows a buildup of acetylcholine. Atropine counteracts these effects.

A canine hospitalized following a TPLO surgery unexpectedly cardiac arrests. Endotracheal intubation and chest compressions have been initiated. What two drugs are usually administered FIRST during CPCR? Propofol and diazepam Dextrose and intravenous fluids Atropine and epinephrine Mannitol and hypertonic saline Dexamethasone sodium phosphate and methylprednisolone

Atropine and epinephrine Atropine is used in CPCR because it blocks signals from the vagus nerve and is used to treat bradycardia. Epinephrine is used because it increases cardiac output by causing peripheral vasoconstriction and causes bronchodilation. Mannitol and hypertonic saline are used to decrease intracranial pressure and are not the first drugs to be administered during CPCR. Dex SP and methylprednisolone are both steroids and do not increase cardiac function or output. Propofol and diazepam are drugs that might be given during status epilepticus, not during CPCR. Dextrose and fluids may be administered during CPCR but are not usually the first drugs of choice.

Pemphigus is a disease affecting the skin and mucous membranes. Pemphigus is which type of disease? Autoimmune Infectious Neoplastic Allergic

Autoimmune Pemphigus is an autoimmune condition that affects the skin and mucous membranes. Antibodies made by an animal's immune system attack the bridges that hold skin cells together. Antibodies are deposited in the intercellular spaces causing the cells to detach from each other (also called acanthosis). Treatment for this condition involves the use of immunosuppressants such as corticosteroids. In some cases it may be caused by a drug allergy or reaction, but is still an autoimmune condition. This dog has Pemphigus vulgaris. Image used with permission, from Clinical Immunology of the Dog and Cat (Day), courtesy of Manson Publishing.

Which of the following foods is considered toxic to parrots? Habanero peppers Chicken Avocado Cheese Blueberries

Avocado

A 5-year-old domestic short hair cat presents with some lesions on his chin. The doctor diagnoses feline acne and prescribes some antibiotics and ointment. Which of the following is a good suggestion a technician could make to the owner? Apply hydrocortisone cream to the chin twice a day Allow the cat to go outside as the air will help the chin to dry out Avoid plastic food and water dishes Gently pop the pimples on the chin twice a day

Avoid plastic food and water dishes In some cats, a hypersensitivity to plastic can cause feline acne. This is a good suggestion to make to the owner of any cat with this condition. You cannot make recommendations for medications other than what the doctor prescribed during the visit. Popping the pimples can further irritate the skin and should not be performed by the owner. Taking the cat outside will not likely help this condition.

What is the term that describes an animal with elevated blood urea nitrogen and creatinine levels measured in the blood? Azotemic Uremic Hyperemic Nephrotic

Azotemic Azotemia is the term meaning an animal has elevated blood urea nitrogen and creatinine. Uremia indicates a clinical syndrome seen in animals with severe kidney disease where they develop nausea, inappetence and potentially other signs such as oral ulcers. Hyperemia refers to red-colored mucous membranes. Nephrotic syndrome refers to a series of changes found in animals with ongoing or severe proteinuria.

Which of the following methods involves collecting nematode larvae by allowing them to pass through a wire net or cheesecloth? McMaster technique Sheather's solution technique Wisconsin technique Baermann technique

Baermann technique The Baermann technique involves placing a fecal sample in a warm water or saline solution over a cheesecloth. This stimulates larvae to travel through the cloth toward the bottom of a funnel. This technique is useful in collecting motile larvae. The Wisconsin technique and McMaster technique are both quantitative fecal count procedures, and Sheather's solution is a sugar solution used for fecal flotation.

What is another name for a non-rebreathing anesthesia circuit? Peep system Circle system CO2 system Bain system

Bain system The Bain system is a non-rebreathing system. All non-rebreathing circuits lack unidirectional valves and soda lime carbon. They are used in patients less than 10 lb. In the non-rebreathing circuit oxygen flows through a flowmeter and into the vaporizer. At this point, gases exiting the vaporizer go directly to a hose for delivery to the patient with no inhalation flutter valve. Exhaled gases pass through another hose and may enter a reservoir bag but do not enter a CO2 absorber. The gas is then released into a scavenger. In a circle or rebreathing circuit (used in patients over 10 lb.), gases exhaled by the patient travel through the expiratory hose and enter the CO2 canister. They are directed into the reservoir bag and back toward the patient through the inhalation flutter valve. Oxygen and anesthetic gas enter the circuit here from the vaporizer and mix with the patient?s exhaled gas.

Which of the following is the most mature neutrophil precursor? Prorubricyte Band Metamyelocyte Promyelocyte Myeloblast

Band

Two black mice present for progressive, partial alopecia that started at the face and head and progressed to the trunk. The underlying skin is normal. The other mouse that the two are housed with appears normal. What is the most likely cause of the alopecia? Pattern alopecia Barbering or chewing of the hair by their cage mate Mite infestation Dermatophytosis (ringworm)

Barbering or chewing of the hair by their cage mate The correct answer is barbering or chewing of the hair by their cage mate. The key to this question is that the underlying skin is normal and the cage mate is normal. Barbering is usually a behavior exhibited by more dominant mice. Dermatophytosis or a mite infestation would cause crusting and scaling of the underlying skin, pruritis, and the cage mate would most likely be affected. Pattern alopecia would not typically start on the face and head and would also not likely affect two of the three mice. The hair would not appear chewed with pattern alopecia, but would show a more uniform or symmetrical loss.

Which of the following appropriately describes a surgical preparation of an incision site? Scrub from cranial to caudal starting at the hairline Alternate chlorhexidine and iodine scrubs for a total of 6 scrubs Begin where the incision will be made and scrub in outward circles, not returning to the central area Scrub and rinse the area a minimum of 6 times

Begin where the incision will be made and scrub in outward circles, not returning to the central area Either chlorhexidine or betadine can be used, but not both together. Scrubbing is started near the center of the incision site in a circular scrubbing motion, moving from the center to the periphery without returning from the periphery to the center. Frequently, the site is scrubbed alternately with alcohol or saline although the benefit of this practice is controversial.

Approximately how much urine should a 40-pound dog produce in a 24-hour period if the dog is drinking and urinating normal amounts and is not dehydrated? About 2.2 Liters of urine Between 150 to 300 mL of urine About 1.5 Liters of urine Between 450 and 850 mL of urine

Between 450 and 850 mL of urine Normal urine output for a dog is 1-2 mL per kg per hour. This dog is 40 pounds, about 18 kg (40 pounds/2.2 = 18.1 kg) 18 kg X 1 = 18 mL/hr X 24 hours = 432 mL 18 kg X 2 = 36 mL/hr X 24 hours = 864 mL Normal urine output for this dog is between 432 mL and 864 mL in a 24-hour period. 1.5 Liters is equal to 1500 mL (there are 1,000 mL per Liter) 2.2 Liters is equal to 2200 mL

Which test would provide the best measure of liver function? Bile acids Serum BUN level Ultrasound Serum phosphorus level

Bile acids When a meal is consumed, the gall bladder contracts and releases bile into the upper small intestine as needed for digestion. The bile acids break down lipids during this process of digestion. The bile acids are then absorbed by the intestine and into the portal bloodstream and returned back to the liver. If the liver is functioning well, the bile acids are removed from the bloodstream and returned to the gall bladder until they are needed again. Comparing the two blood levels (pre- and post-prandial bile acids) allows the veterinarian to see how well the liver is functioning. Bile acids are removed from portal blood by the hepatocytes (liver cells). If the liver cells are not working as they should, the bile acids remain in circulation and enter the systemic circulation where the elevated levels are measured by the bile acids test. Ultrasound is good for looking at architecture of the liver, but does not evaluate function of the hepatocytes. BUN and phosphorus are most often looked at when evaluating kidney disease.

To assess liver function in a patient with a possible portosystemic liver shunt, which of the following tests is most likely to be helpful? Bile acids test Dexamethasone suppression test Water deprivation test Insulin:glucose ratio

Bile acids test A bile acids test is the only choice here that is a test of liver function. Bile acids are one of the most helpful tests in evaluating liver function, particularly in cases of portosystemic shunts. In this test, a fasted blood sample is drawn and a post-feeding blood sample is drawn to compare bile acid levels. Normally, bile acids increase after a meal due to release from the gall bladder, but they do not increase too much because they are reabsorbed by the liver through the portal circulation. In animals with liver dysfunction, including portosystemic shunts, bile acids are not reabsorbed and accumulate in the systemic circulation after a meal. Insulin-to-glucose ratio is a test that may be used to diagnose functional insulin-secreting tumors. Dexamethasone suppression tests are typically used to diagnose Cushing's disease. A water deprivation test may be used to diagnose diabetes insipidus.

The degree to which an administered drug is absorbed intact into the systemic circulation is known as which of the following? Half-life First-pass Metabolism Bioavailability

Bioavailability Bioavailability is a word used to describe how chemicals are absorbed by the body. It is the administered dose of unchanged drug that reaches the systemic circulation. This is the amount of the drug that the body can actually use. IV drugs are 100% bioavailable to the body, because they don't pass through the gastrointestinal system and are directly injected into the circulation. Some oral medications are not fully absorbed so they have a lower bioavailability than IV medications. Drug interactions can also affect the bioavailability of certain drugs.

Uric acid is the major end product of nitrogen metabolism in which of the following animals? Horses Cats Ferrets Birds Goats

Birds Birds metabolize most of their nitrogen to uric acid, whereas mammals metabolize most of their nitrogen to urea.

Which of the following is NOT true regarding platelets? A purple top tube with EDTA should be used when collecting blood for a platelet count Platelet clumping can cause falsely low platelet counts Birds and reptiles have nucleated platelets They are derived from megakaryocytes

Birds and reptiles have nucleated platelets Birds and reptiles have nucleated red blood cells, not nucleated platelets. A purple top is used for platelets counts. Clumping of platelets can cause a low reading, so the periphery of the slide should be checked for clumping. In cases where clumping is present, a platelet estimate may be more valuable. The megakaryocyte is a bone marrow cell responsible for the production of blood thrombocytes (platelets).

Which technique is needed to produce quality intra-oral radiographs? Bisecting angle technique Oblique angle technique Occlusal angle technique Maxillary angle technique

Bisecting angle technique

In order to prevent distortion on a dental radiograph when parallel technique cannot be used (impossible to position the film parallel to the long axis of the tooth), which of the following will prevent image distortion (elongation or foreshortening)? Horizontal technique Bisecting angle technique Volume averaging technique Perpendicular technique Vertical technique

Bisecting angle technique In most radiographs, to avoid distortion (elongation or foreshortening), the x-ray beam should be perpendicular to the long axis of the bone and the x-ray cassette. If the bones are not perpendicular to the beam, they will appear foreshortened. If the cassette is not perpendicular to the beam, the bones will appear elongated. In dental radiography, it is frequently impossible for the long axis of the tooth and the film to be parallel to each other. In this case, the bisecting angle technique may be used, where the beam is angled half-way between the angle of the long axis of the tooth and the angle of the film in order to obtain a true image that is neither elongated nor foreshortened. By angling half-way between those two angles, this technique effectively balances out the two types of distortion to obtain a true image.

Which of the following over-the-counter medications is sometimes given for diarrhea? Famotidine Bismuth subsalicylate Diphenhydramine Simethicone

Bismuth subsalicylate

An owner of a large breed dog calls the clinic and said she just came home from work to find the pantry open and a large portion of the dog food gone. The dog is salivating, dry heaving, and has a distended abdomen. What is a common condition in large breed dogs that can cause these symptoms? Heartworm Disease Pyometra Bloat Urinary Obstruction Allergic reaction

Bloat A dog with a history of the listed symptoms should be seen immediately by a veterinarian. Large breed dogs are at a higher risk for "bloat" which occurs when the stomach becomes distended with gas, fluid, and/or food. The stomach can actually rotate upon itself in some cases and cause a life-threatening emergency called a Gastric Dilatation Volvulus (GDV). Straining to urinate, dribbling, or difficulty urinating could be symptoms of a urinary obstruction. Pyometra occurs in unspayed females and is when the uterus fills with pus. Common symptoms of clinical heartworm disease would include exercise intolerance and coughing. Allergic reactions most typically cause a swollen face or muzzle, hives, and sometimes vomiting.

Blood lactate is a marker of what? Activity of the liver Activity of muscle Amount of tissue carbon dioxide production Blood perfusion and oxygen delivery to tissues

Blood perfusion and oxygen delivery to tissues Blood lactate is a marker of perfusion and oxygen delivery to the tissues. When there is adequate oxygen present in the patient, aerobic metabolism (oxidative phosphorylation) occurs, producing energy. When there is lack of oxygen delivery to the body or a specific tissue/organ, anaerobic metabolism occurs and produces lactate as a by-product. Instances in which lactate may be elevated include sepsis, GDV in dogs, and strangulating intestinal lesions in horses.

All of the following are typically included in a complete blood count EXCEPT for which of the following? Blood urea nitrogen levels Hemoglobin concentration Plasma protein concentration Erythrocyte indices Platelet count and/or estimate Differential white blood cell count

Blood urea nitrogen levels A complete blood count provides a total red blood cell count, white blood cell count, and platelet count and/or estimate in addition to a packed cell volume or hematocrit and plasma protein concentration. It includes a blood smear evaluation to assess the differential white blood cell count as well as blood cell morphology. Hemoglobin concentration and erythrocyte indices of size are included. For anemic patients, a complete blood count should also include a reticulocyte count. Blood urea nitrogen is not part of a complete blood count and is considered a chemistry value.

A Jamshidi needle is used for which of the following procedures? Cerebrospinal fluid tap Joint tap Bone marrow core biopsy Liver core biopsy Bone marrow aspirate

Bone marrow core biopsy A Jamshidi needle is used to obtain a bone marrow core by inserting the needle into the cortical bone, usually of the iliac crest, and then rotating back and forth to cut a piece of bone. A Michele trephine is also used for the same purpose.

For which of the following diseases should an animal be isolated? Diabetes Demodectic mange Ehrlichia canis Bordetella bronchiseptica

Bordetella bronchiseptica Bordetella is one cause of kennel cough and is contagious to other dogs. The other diseases listed are not contagious. Diabetes is not a contagious condition as is a result of an endocrinopathy. Demodectic manage is secondary to a non-contagious mite. Ehrlichia canis is a tick-borne disease and is not directly transmissible.

Ticks can serve as vectors to transmit several diseases in animals including Lyme disease. What is the agent that causes Lyme disease? Rickettsia rickettsii Neorickettsia helminthoeca Borrelia burgdorferi Ixodes pacificus

Borrelia burgdorferi Borrelia burgdorferi is the name of the rickettsial organism that causes Lyme disease. Ixodes is a genus of tick that may transmit the disease. Rickettsia rickettsii is the causative agent of Rocky Mountain spotted fever. Neorickettsia helminthoeca is the causative agent of salmon poisoning.

Increased input from the vagus nerve can result in which of the following responses? Tachycardia Vomiting Seizures Bradycardia

Bradycardia Excessive vagal tone can cause a slower heart rate (bradycardia). Vomiting can sometimes stimulate the vagus nerve but vagal nerve stimulation should not cause vomiting. The vagus nerve carries parasympathetic fibers.

A client calls to schedule an appointment and tells you his male cat is straining to urinate in his litter box but no urine is seen in the box. He wants to know if it would be okay to give him some leftover Clavamox from when he had a skin infection last month. What should you tell him? Orally syringe water to keep him hydrated and schedule an appointment for tomorrow morning since there are no open appointments today Have him pick up some non-absorbable litter from the clinic so that he may collect a urine sample at home when the cat finally urinates Tell him to give the leftover Clavamox he has in his cabinet and come in first thing tomorrow morning so the doctor can collect a urine sample Bring the cat in immediately for an exam Give 1/2 teaspoon of cod liver oil because although it appears he is straining to urinate, he is likely constipated

Bring the cat in immediately for an exam

A 12-year old flat coated retriever presents early for an appointment with a 24-hour history of lethargy. The patient is alert and responsive, has very strong pulses, and a heart rate of 160 beats per minute. The veterinarian has just begun a new pet appointment with a long-term client. This patient should be: Allowed to stay in the lobby until the patient's appointment time Brought into the treatment area and be immediately examined by the veterinarian. Transferred to the local emergency clinic immediately Brought into the treatment area and monitored until the veterinarian is finished with the current appointment Started on IV fluids immediately, prior to the veterinarian's examination

Brought into the treatment area and be immediately examined by the veterinarian.

Myoglobinuria (pigment in the urine) from muscle damage causes the urine to appear what color? Bright yellow Purple Brown Green

Brown Myoglobinuria causes the urine to appear red or brown in color but is negative for red blood cells. This can occur from muscle damage, such as in instances of trauma, electrocution, or exertional myopathy ("tying-up" in horses).

A 3-year old female Doberman presents for a routine spay. The owner states that she has not recently been in heat and has been healthy her entire life. The veterinarian is concerned that Doberman Pinchers are predisposed to having von Willebrand's Disease (vWD). Which of the following diagnostic tests would give the best indication of whether the dog has an increased risk of bleeding due to this disorder? Chest radiographs Total protein level Buccal mucosal bleeding test Abdominal ultrasound

Buccal mucosal bleeding test Doberman Pinschers are predisposed to having von Willebrand's Disease (vWD). Specifically, Dobermans have increased incidence of Type 1 vWD. In this type, there is a reduced presence of the functional von Willebrand factor. This factor is crucial in initial clot formation; it attracts platelets and allows them to bind to exposed subendothelium after injury. A buccal mucosal bleeding test should result in a clot in less than 4 minutes in normal dogs. An abnormal result warrants further investigation to confirm the presence of vWD in order to take the appropriate measures prior to surgery.

Which of the following drugs is NOT useful in the treatment of seizures? Butorphanol Propofol Phenobarbital Potassium bromide Diazepam

Butorphanol Butorphanol is an opiate partial agonist and is mostly used as an antitussive, analgesic, or sedative. The other medications can be used in treatment of seizures. Propofol can be useful when treating status epilepticus.

Parathyroid hormone mostly regulates which blood value? Calcium Potassium Glucose Blood Urea Nitrogen

Calcium Parathyroid hormone is the endocrine regulator of calcium and phosphorus concentration. This hormone is secreted from the parathyroid glands and acts on target cells mostly in bones, kidneys, and small intestine. It acts by mobilizing calcium from bone, helps absorption of calcium in the intestine, and decreases calcium loss in urine. By doing these things, it helps to prevent hypocalcemia (low calcium). Parathyroid hormone also acts on the kidney to stimulate loss of phosphate ions in urine.

A 3-year old female Yorkie presents to your hospital as an emergency for seizures and tetany. In the history, you learn that the bitch had a litter of 3 puppies about 2 weeks ago. What is the most likely treatment this dog will need to correct the cause of its problem? Diazepam IV Phenobarbital Calcium gluconate IV Tetanus anti-toxin

Calcium gluconate IV The correct answer is calcium gluconate IV. Puerperal tetany (eclampsia) is often seen in post-partum toy breed dogs. Emergency treatment for these patients is IV calcium in the form of calcium gluconate. Diazepam (either IV or per rectum) can be given to stop the seizures, but does not treat the problem of hypocalcemia. Tetanus intoxication is less likely in this patient.

A cat may have ingested ethylene glycol. You are examining the urine sediment and are looking for: Calcium oxalate crystals White blood cells Struvite crystals Cystine crystals

Calcium oxalate crystals Antifreeze poisoning causes the formation of calcium oxalate crystals in the urine. White blood cells when seen are usually a sign of a urinary tract infection, but may also be associated with other disease processes occasionally. Struvite crystals are not uncommon to find in urine samples. Cystine crystals, although not commonly seen, in urine samples do not necessarily imply there is a problem.

Mineralized debris on the tooth surface is known as which of the following? Plaque Cementum Dentin Calculus

Calculus Calculus is mineralization of plaque on the teeth. Plaque is a mixture of bacteria, saliva, and food parts that cover the tooth in a thin film. Plaque can turn into calculus in as early as a week. Dentin is the hard, white portion of the tooth. Cementum is calcified connective tissue.

Which of the following diseases should ferrets be vaccinated against? Canine parvovirus Canine distemper virus Feline panleukopenia virus Infectious canine hepatitis virus

Canine distemper virus Ferrets should be vaccinated against canine distemper virus. Ferrets are not susceptible to the other diseases listed.

Which nematode may be found in the bladder or urinary tract of dogs? Uncinaria stenocephala Capillaria aerophila Ancylostoma caninum Capillaria plica

Capillaria plica Capillaria plica is the "bladder worm" in dogs. The eggs pass out of the bladder through the urine. The embryonated egg may be ingested by an earthworm where it transforms to its infective stage, and then may be passed on to a dog if ingested. Capillaria aerophila is a lungworm. Capillaria ova, as seen in the image, look similar to Trichuris eggs but are smaller and have asymmetric terminal plugs. The other two parasites listed are hookworms.

What is the "top" shell of a tortoise called? Carapace Cloaca Plastron Choana Patagium Scute

Carapace The carapace is the top shell and the plastron is the bottom shell. Patagium is the skin or membrane that extends between the body and a limb or wing (such as seen in a flying squirrel or wing of a bat). A scute is a bony external plate or scale. A choana is either of the pair of posterior openings between the nasal cavity and nasopharynx. The cloaca is the posterior opening that serves to pass feces, urine, and reproductive excretions (connects the rectum, vagina, and urethra into a single channel), such as in reptiles and birds.

The main stimulus for ventilation is the partial pressure of which of the following gases? Oxygen Carbon dioxide Carbon monoxide Nitrogen

Carbon dioxide Elevations of carbon dioxide are the main stimulus for respiration. Low levels of oxygen can also stimulate ventilation; however, carbon dioxide is a more potent stimulant.

A 9-year old male castrated Westie presents on emergency after being rescued from a house fire. On presentation, the dog has a respiratory rate of 60 breaths per minute with increased inspiratory effort and moderate distress. The dog's mucous membranes are bright red. What is the likely cause of the bright red color of the dog's mucous membranes? Ash Carbon dioxide Burns Carbon monoxide

Carbon monoxide The bright red mucous membranes in a dog suspected of having smoke inhalation are concerning for carbon monoxide toxicity. Carbon monoxide displaces oxygen on hemoglobin molecules, forming carboxyhemoglobin complexes, which leads to tissue hypoxia. The dog should be supplemented with 100% oxygen which reduces the half-life of the carboxyhemoglobin complexes from 4 hours on room air to 30 minutes. Other treatments for smoke inhalation include humidifying the air to promote mucociliary clearance and to prevent drying injuries to the airways.

Which of the following is the correct term for dental decay, causing demineralization of the hard tissues of the tooth? Odontoclastic resorptive lesions Stomatitis Enamel hypoplasia Caries

Caries Dental caries is the condition where trapped food is fermented by bacteria, forming acids that demineralize the hard tissue of the tooth. Odontoclastic resorptive lesions are a process where the cementum of the tooth is resorbed and replaced with bone-like material. Enamel hypoplasia, is the absence of enamel formation on the tooth crown during tooth development. Stomatitis is inflammation of the oral mucosal surfaces.

What is a main function of a red blood cell? Facilitate phagocytosis of bacteria and viruses Produce histamine Carry oxygen to the tissues Help clot a wound

Carry oxygen to the tissues Red blood cells have hemoglobin in their cytoplasm which binds oxygen and causes the red color of the cells. They are responsible for carrying oxygen to the tissues. Platelets help clot a wound. Leukocytes such as macrophages phagocytose bacteria and viruses. Mast cells produce histamine.

What are the components of a typical X-ray cassette in order from front (where X-rays enter) to back? Cassette front, padding, intensifying screen, film, padding, cassette back Cassette front, intensifying screen, padding, film, padding, intensifying screen, cassette back Cassette front, padding, intensifying screen, film, intensifying screen, padding, cassette back Cassette front, padding, film intensifying screen, film, padding, cassette back

Cassette front, padding, intensifying screen, film, intensifying screen, padding, cassette back

An emasculator tool is used in which procedure? Enucleation Teeth floating Hoof testing Castration

Castration An emasculator is a tool used in the castration of livestock. It functions to simultaneously crush and cut the spermatic cord, preventing hemorrhaging.

Which species regurgitates food for remastication? Cattle Horses Cats Pigs Lagomorphs

Cattle Cattle chew their "cud", which is food that they regurgitate for remastication. When a cow chews her cud, she regurgitates a food bolus into her mouth, which is then re-chewed and re-swallowed. While cattle are chewing their cud, they produce saliva which acts as a natural antacid that helps to buffer and protect the rumen. Cows sometimes spend up to 8 hours a day chewing their cud.

What is the term given to the end of the spinal cord? Cauda equina Spinous terminata Cordus altus Corte finale

Cauda equina The cauda equina is at the lower end of the spine and is made up of nerve roots. The base of this region includes around 3-5 lumbar, 5 sacral, and a coccygeal nerve. The reason it is called the cauda "equina" is that the appearance of this area grossly resembles a horse's tail.

The rear end of an animal is referred to as which of the following regions? Caudal region Dorsal region Ventral region Cranial region

Caudal region

Where is the standard tuburculosis test administered in the bovine species? Ear pinna Proximal axilla Eye lid Caudal tail fold

Caudal tail fold The TB test in cows is given intradermally in the caudal tail fold. This area is used because it is easy to give the injection at this location and a reaction can be easily seen. The eye lid is used in primates.

Where should you palpate a standing animal for the urinary bladder? Caudoventral Rostral Cranioventral Caudodorsal

Caudoventral

Which of the following is an expected side effect of atropine? Cause mild sedation Cause bradycardia Cause inhibition of excessive salivation Cause pupillary constriction

Cause inhibition of excessive salivation Atropine functions as a sympathetic, competitive antagonist of muscarinic cholinergic receptors. It inhibits salivary secretions, causes increased heart rate, and causes dilation of pupils. It is not a sedative.

A hypotonic solution will do what to a red blood cell? Cause the red blood cell to burst Will have an unpredictable effect on red blood cell size Will have no effect on red blood cell size Cause the red blood cell to shrink

Cause the red blood cell to burst

Why is nitrous oxide contraindicated in gastrointestinal surgeries? Longer recovery due to delayed GI emptying Causes nitrous build up in the GI tract Causes flatulence Causes hypersegmentation of the bowel Causes vomiting

Causes nitrous build up in the GI tract Nitrous oxide moves into closed gas spaces such as the intestines. As such, its use is contraindicated in bowel surgeries. You should also be aware that nitrous oxide decreases fractional inspired oxygen levels, although this can be managed and monitored in most instances. Nitrous oxide is also contraindicated when pathology such as pulmonary bullae are present.

You are developing a template for the drug labels at your new hospital. Every prescription drug label should include the following statement: Shake well for liquid medications For veterinary use only Caution: Federal law restricts this drug to use by the order of a licensed veterinarian Give as directed by your veterinarian Stop this medication and contact your veterinarian should any adverse reactions occur

Caution: Federal law restricts this drug to use by the order of a licensed veterinarian Federal regulation requiring labels to bear statements such as "For veterinary use only" represent a sales policy and hold no legal basis. Every label should include the caution label (as indicated in the correct answer), dosage, route of administration, quantity of ingredient, and a lot or control number. While the other choices may be of value, they are not legally required.

In small animals, what is the name of the blind-ended pouch that is part of the large intestine and connects the ileum with the colon? Cecum Guttural pouch Gall bladder Appendix

Cecum The cecum attaches the ileum to the colon to form the ileocecal-colic junction. The appendix is a blind-ended tube connected to the cecum in humans. Most small animals do not have an appendix. The gall bladder is located in the liver. The guttural pouch is an air-filled sac that is dorsal to the pharynx in the horse.

Which of the following is another term for the type of parasite known as a tapeworm? Nematode Trematode Cestode Protozoan Ascarid

Cestode Tapeworms or cestodes are long, segmented, flat worms. Examples include the canine tapeworms, Taenia pisiformis and Dipylidium caninum. Trematodes are flukes, nematodes are roundworms, ascarids are a type of nematode (roundworm), and protozoans are single-celled organisms that may be parasitic.

A patient is hospitalized with a resistant infection and is getting Amikacin injections and IV fluid therapy. Amikacin can cause kidney damage. What could you do to monitor for early renal side effects? Check the urine specific gravity twice daily Check a daily urine protein Monitor urine output Renal ultrasound daily

Check a daily urine protein Proteinuria can be one of the first things seen if there is damage to the kidneys. Urine output and specific gravity would not be reliable indicators since this patient is on IV fluids. Ultrasound would not be helpful picking up early kidney damage since this occurs at a microscopic level in the renal tubules. Kidney changes can be seen with ultrasound only if gross disease is present in the renal architecture. BUN and creatinine levels can be helpful, however, remember that usually these values do not become elevated until more than 70% of the renal function has been compromised.

An owner rushes her dog to your hospital because she noticed his eye looked red. This condition is sometimes referred to as which of the following (red bump)? Exophthalmus Meibomian gland prolapse Cherry eye Raisin eye

Cherry eye

A thoracotomy procedure would involve an incision made into the: Skull Tympanic membrane Forelimb Chest Abdomen

Chest

Which of the following small mammals has the longest life expectancy? Rat Rabbit Chinchilla Guinea pig Hamster

Chinchilla The life expectancy of a chinchilla is 10 to 15 years. This exceeds the life expectancy of most other small mammals including rabbits (5 to 8 years depending on species), rats (3 years), guinea pigs (4 to 7 years) and hamsters (1.5 to 2 years). Other small mammals with relatively long life expectancies are ferrets (8 to 10 years) and sugar gliders (8 to 10 years).

Which of the following medications requires special handling because it can cause aplastic anemia in humans? Chloramphenicol Metronidazole Famotidine Chlorpheniramine Phenylpropanolamine

Chloramphenicol Chloramphenicol can cause a rare reaction in people who are exposed to the drug leading to aplastic anemia. Because of this, chloramphenicol is typically used as a last resort and only in special situations. Direct contact with the drug should be avoided and therefore gloves must be worn when handling. The tablets should never by crushed to avoid inhalation of any powder.

Novalsan is also known as which of the following? Phenol Betadine Chlorhexidine Isopropyl alcohol

Chlorhexidine Novalsan and Hibiclens are both names of chlorhexidine solutions or scrub.

A rupture or tear in the thoracic duct would result in which of the following? Hemoabdomen Pyothorax Chylothorax Pericardial effusion

Chylothorax Chyle (a fluid high in chylomicrons and lymph) effuses from the thoracic duct into the pleural space if the thoracic duct tears or ruptures (usually trauma-related). The effusion typically appears as a milky white fluid and is odorless. This is only one cause of chylothorax. Other causes include lymphangiectasia, lung lobe torsion, heartworm disease, cardiomyopathy, neoplasia, idiopathic, and others. If the triglyceride level of the effusion is greater than that of peripheral blood, this can help to determine the effusion is chylous. The necropsy on this animal reveals a chylous effusion. Note the milky appearance of the fluid.

A 12-year old domestic short hair presents for difficulty defecating. This has become a chronic problem. You perform radiographs and the veterinarian diagnoses megacolon. The cat is de-obstipated and given IV fluids for the day. The cat is currently taking lactulose to help soften the stools. What other medication could be given to help the cat empty his colon? Metoclopramide Cisapride Metronidazole Sucralfate

Cisapride Cisapride is a pro-motility medication that can benefit cats with megacolon. Metoclopramide is also a prokinetic medication, but it acts mostly on the small intestine and not on the colon.

A method of castration in cattle that is performed without incising the skin is known as: Closed castration Inguinal castration Occult castration Cryptic castration

Closed castration Closed castration is a method sometimes used in cattle or small ruminants which involves use of an emasculatome, which crushes the cord within the scrotum without cutting the scrotal skin. It is also referred to as a "bloodless" castration. Incisions are more likely to attract insects and cause disease susceptibility in a field situation. A "closed" castration may also refer to castrations when an incision is made in the skin, but the vaginal tunic is not entered.

A proliferative honeycomb beak (as seen in the image below) along with scaly crusty lesions on the legs and feet of a bird can be a sign of which of the following? Cnemidocoptes pilae Mycobacterium avium Avian pox Chlamydophila psittaci

Cnemidocoptes pilae The correct answer is Cnemidocoptes pilae. Cnemidocoptes pilae, is also known as "scaly face" or "scaly leg." This mite can be treated with topical or oral ivermectin. It is more common in birds that are immunocompromised, and beak deformity may be permanent even after the mite is cleared. Avian pox can cause skin lesions, diptheric membranes, or septicemia. Mycobacterium avium causes granulomatous disease throughout the liver and GI tract. Chlamydophila psittaci is the causative agent of psittacosis, and causes lethargy, respiratory, and GI signs.

Eimeria, Cystoisospora, Cryptosporidium, Sarcocystis, and Toxoplasma are all what type of parasite? Coccidia Trematodes Nematodes Cestodes Ascarids

Coccidia These are all coccidia, a type of protozoan parasite; they are not worms. Tapeworms are cestodes, nematodes are roundworms, trematodes are flukes, and ascarids are a type of nematode (roundworm).

Which of the following could cause a hamster to go into "pseudo-hibernation"? Low Calcium level Poor air quality Low Vitamin C level Cold temperature

Cold temperature Hamsters have the ability to "hibernate" in a sense. If the temperature drops too low (usually below 41 degrees F) they can go into pseudo-hibernation and owners can mistake them for dying. Low Vitamin C can lead to scurvy in hamsters. Scurvy symptoms include loss of fur, lethargy, squealing, weight loss, and weakness. Good air quality is important for overall health which is why hamsters should not be kept in aquarium type housing. Low calcium may lead to brittle bones and broken teeth.

Which of the following is NOT a component of an incisional dehiscence secondary to infection? Swelling Cold to the touch Pain Purulent discharge

Cold to the touch Incisional dehiscences are very serious, and the sooner they are identified the better. When dealing with an abdominal or thoracic incision, it is that much more important to recognize a problem early before there are life-threatening consequences as a result of incision failure. An infection is characterized by all the cardinal signs of inflammation (redness, pain, swelling, heat) as well as purulent discharge.

An animal is hit by a car and has a broken leg in which the bone is sticking out through the skin. This is an example of which fracture type? Spiral Comminuted Compound Greenstick

Compound Fracture types: Oblique - a fracture which goes at an angle to the axis Comminuted - a fracture of many relatively small fragments Spiral - a fracture which runs around the axis of the bone Compound - a fracture which breaks the skin Greenstick - a fracture of soft bone where the bone can bend and be partially broken, seen in young animals

When viewing a slide on a microscope, the objects themselves are in focus but there appear to be haloes or fuzzy rings around the objects you are viewing that you cannot eliminate using the focus knobs. Which of the following is probably not adjusted correctly? Rheostat Course focus Condenser Iris diaphragm Fine focus

Condenser This problem is what is typically seen when the condenser is not appropriately adjusted. The condenser functions to focus light on the object, and when it is not set appropriately, rings of light or haloes appear around objects. The rheostat adjusts light intensity. The iris diaphragm also regulates the amount of light reaching the object. Because the object is in focus and the problem is not resolved by adjusting the focus knobs, this is not a problem with the coarse or fine focus.

Which of the following is NOT a use for an indwelling arterial catheter? Blood pressure monitoring Measurement of arterial blood gasses Constant rate infusions Collection of blood samples

Constant rate infusions Injections should never be given into an arterial catheter (a-line). Arterial catheters are a great way to get real-time blood pressure measurements. These measurements are also helpful when trying to evaluate general cardiac output. They are often used to collect small blood samples, most notably for blood gas measurements.

You are riding along with the veterinarian to a small sheep operation. The operation has had two sheep die in the last 24 hours. Upon arrival, icterus, depression, and weakness are noted in several sheep in the herd. One sheep urinates and the urine is very dark. Toxicity from which mineral can cause icterus and hemoglobinuria (red urine) in sheep? Potassium Selenium Salt Copper

Copper The correct answer is copper. Sheep are highly susceptible to copper toxicity. Because this causes an acute hemolytic crisis, the most common signs are acute death, icterus, depression, hemoglobinuria, increased respiratory rate, and weakness. Pathology often shows dark, hemoglobin filled kidneys ("gun metal blue"). Goats and cattle are less susceptible to copper toxicity. While you will not be responsible for diagnosing animals, you should know that copper toxicity causes icterus in sheep . Salt toxicity in livestock often causes neurologic symptoms. Selenium and potassium toxicities are not common. Selenium deficiency (along with Vitamin E deficiency) may cause poor wool production, decreased fertility, and is the cause of white muscle disease.

When discussing the equine hoof, which structure produces cells that become the hoof wall? Sole Ergot Coronary band Frog

Coronary band The hoof wall grows from the coronary band. The coronary corium produces cells that become the horn of the hoof wall. It is soft skin tissue such as the cuticles on our fingers. If the coronary band is ever damaged, the hoof may grow improperly and could cause lameness. The ergot is a callus-type structure on the underside of the fetlock. The sole is the bottom of the hoof. The frog is the structure that is shaped like a heart and extends forward across two-thirds of the sole. It grows from front to back where it then merges with the heel periople.

A 3-year old female spayed DSH cat presents for evaluation of a fistulous swelling of the ventrum. Physical exam is unremarkable, except for an area of matted hair over an erythematous 1 cm swelling on the ventrum that exudes a small amount of purulent material which you shave and examine. The cat has no history of fighting; what parasite can cause a lesion like this? Ctenocephalides felis Notoedres cati Cuterebra Demodex cati Microsporum canis

Cuterebra

A rancher pulls into the clinic you are working at and rushes in to say his horse is having difficulty breathing. You assist the vet in examining the horse and note that the horse is in respiratory distress. He has dilated pupils and bright red mucous membranes. You are asked to pull blood for testing which you note is also bright red in color. Which of the following may cause this in the horse? Cyanide toxicity Walnut toxicity Choke Chronic Obstructive Pulmonary Disease (COPD)

Cyanide toxicity The correct answer is cyanide toxicity. The bright red-colored blood and mucous membranes with signs of respiratory distress are typical findings for cyanide toxicity. There are many plants that can accumulate cyanide to levels that are toxic to horses. They include Sudan and Johnson grasses, cherries (chokecherries), and Sorghum. Treatment is with sodium nitrate or sodium thiosulfate IV. Use of black walnut shavings in horse bedding can cause laminitis. Choke is an obstruction in the esophagus. It may cause coughing or distress, but not bright red blood. COPD, also called heaves due to heavy expiration, can be caused by dusty or moldy hay, dust and molds in bedding, or pollens, dust and other irritants in the environment. Horses housed with these symptoms should be led out to pasture to remove them from the irritating environment.

When collecting a sample, when should the needle never be re-directed? Thoracocentesis Jugular blood draw Fine needle aspiration Cystocentesis

Cystocentesis

Which of the following is a term for a method of urine collection from a patient in a sterile manner? Cystocentesis Cystotomy Free catch Cystogram

Cystocentesis Cystocentesis involves the sterile introduction of a needle into the bladder for collection of urine. A free catch is the opportunistic collection of urine from a voiding patient. This urine is not sterile because it becomes contaminated as it exits the urethra. A cystotomy is an incision into the bladder. A cystogram is a radiographic contrast study of the bladder.

A fresh biopsy sample from a nasal lesion is pressed against a glass slide several times to leave an impression. The cells that are on the slide are stained with a Romanowsky stain and viewed under a microscope. Which of the following tests is this an example of? Immunocytochemistry Histology Immunohistochemistry Cytology Histopathology

Cytology This is an example of cytology (referring to the examination of cells from body tissues or fluids). Cytology specimens may be collected by impression smear as described in this question or by other techniques including needle aspirate, aspiration of fluid (centesis), or by swab with a cotton-tipped applicator. Histology involves the examination of complete tissue and architecture and is usually performed on tissue collected by biopsy and processed intact. Histopathology is the same except that it involves the assessment of diseased tissue. Immunocytochemistry involves using special stains that are taken up by specific cellular components or markers on a cytologic specimen. Immunohistochemistry involves similar special stains applied to histologic sections.

The current accepted standard image format for digital radiographs is called DICOM (digital imaging and communications in medicine). DICOM files have many advantages over other types of image files. Which of the following is NOT true of DICOM images? DICOM image files can be sent to other veterinarians or to owners and can be viewed in almost any photo viewer DICOM image files contain information about the patient and the type of study performed DICOM image files allow for storage, query/retrieval, and manipulation of images DICOM image files are arranged into images, studies, and series where each study may consist of one or more series and each series may consist of one or more images DICOM images are less easily altered by image editing software than other types of images (i.e. JPG, TIFF) and are therefore more appropriate for a medical record.

DICOM image files can be sent to other veterinarians or to owners and can be viewed in almost any photo viewer DICOM image files require special viewing software (i.e. eFilm or OsiriX) and cannot be opened in most photo viewing or editing software (i.e. Photoshop). This makes it difficult for the images to be altered, which maintains their integrity as a medical document. DICOM files contain information about the patient and study performed, and they can be stored, retrieved, and manipulated. Files are arranged into images, studies, and series where each study may consist of one or more series, and each series may consist of one or more images.

Septic shock may cause the gums to appear: Orange Cyanotic Dark red Pale pink Icteric

Dark red

Which of the following instruments is the least traumatic to use on the intestine? Rat-toothed forceps Curved mosquito forceps Debakey forceps Brown-Adson forceps

Debakey forceps The Debakey forceps are thumb forceps designed to be atraumatic. The other thumb forceps listed are traumatic and should not be used when handling delicate organs. Curved mosquito forceps are great for controlling bleeding vessels, but they are traumatic and should not be used to clamp on intestines.

A dirty wound has edges and tissue that need to be removed. What is the term for removing questionable tissue? Skin scraping Incising Grafting Debridement

Debridement Debriding is removing tissue or material from a wound to aid in healing. A skin scraping is done by taking a dull blade and mineral oil and scraping the skin in an attempt to find mites such as Demodex or Sarcoptes. A graft is a piece of tissue taken from one part of the body to use in another region of the body. Incising is the simple term for cutting or making an incision.

A drug that is an anti-cholinergic agent will have which of the following effects? Decrease heart rate Decrease salivation Suppress the cough reflex Constrict the pupil of the eye

Decrease salivation Anti-cholinergic agents are drugs that antagonize the effects of acetylcholine in the autonomic nervous system. Commonly used examples of anticholinergic agents include atropine and pilocarpine. They are commonly used prior to anesthesia because they increase the heart rate (most anesthetic drugs decrease heart rate) and decrease oral secretions, which can aid intubation.

Cattle, or other livestock, exposed to prolonged rainy conditions are most likely to develop: Pediculosis Photosensitization Habronema infestation Dermatophilosis

Dermatophilosis Dermatophilosis (also known as Rain Rot or Rain Scald) is caused by the bacterium Dermatophilus congolensis. It is a common skin infection seen in livestock during rainy seasons or in damp warm weather. Habronema is a gastrointestinal parasite seen in horses. Pediculosis is a lice infestation. Photosensitization would be more common with prolonged sun exposure.

A refractometer is commonly used in veterinary practice for which of the following? Determining the packed cell volume or hemoglobin concentration Determining the specific gravity of urine or protein concentration in a body fluid Determining the number of red blood cells or white blood cells per microliter of blood Concentrating parasite eggs in a fecal sample

Determining the specific gravity of urine or protein concentration in a body fluid A refractometer is a device that measures the refractive index of a solution. The refractive index is a function of the concentration of solids in a solution. In a veterinary lab, the device is most frequently used to measure the specific gravity of urine. It can also be used to measure the protein concentration of plasma or other fluids such as cerebrospinal fluid.

An 11-year old male German shepherd dog with previously diagnosed dilated cardiomyopathy (DCM) presents to your clinic following a dog fight. The dog is significantly aggressive and resistant to handling. What medication should NOT be used to sedate this patient? Dexmedetomidine Butorphanol Midazolam Atropine Buprenorphine

Dexmedetomidine

Which of the following drugs is considered an alpha-2 agonist? Yohimbine Dexmedetomidine Atropine Atipamezole

Dexmedetomidine Dexmedetomidine (Dexdomitor) is a sedative/analgesic that acts on the alpha-2 receptors of the brain. Xylazine is also an important alpha-2 agonist to know. Atipamezole is the reversal for dexmedetomidine. Therefore it is an alpha-2 adrenergic antagonist (it antagonizes the effects of the dexmedetomidine). Yohimbine is an alpha-2 blocker used to reverse xylazine (which is an alpha-2 agonist). Atropine is an anticholinergic drug (parasympatholytic).

A 10 year old male neutered West Highland White Terrier with a history of 1st degree AV block is admitted to the clinic for a routine dentistry. All of his pre-anesthetic lab work has come back normal. Which of the following anesthetic drugs would be most contraindicated in this case? Butorphanol Acepromazine Dexmedetomidine Diazepam

Dexmedetomidine Dexmedetomidine (Dexdomitor) is an alpha 2 agonist of which the major side effect is bradycardia. Patients with 1st degree AV block are often already bradycardic, and the AV block could be made worse by giving an alpha 2 agonist. While diazepam, butorphanol, and acepromazine all cause different degrees of sedation, they do not cause significant changes in heart rate or contractility. Acepromazine causes vasodilation, which can lead to hypotension, but it is anti-arrhythmic and can actually help stabilize the heart.

Which disease occurs when the body is unable to make or utilize insulin? Cushing's disease Diabetes Hyperandrogenism Addison's disease Pancreatitis

Diabetes Diabetes mellitus occurs when the pancreas does not make insulin or when there is insulin resistance such that the body is unable to utilize insulin.

Which of the following diseases would be most likely to cause the development of cataracts in dogs? Diabetes mellitus Hypothyroidism Hypoadrenocorticism Toxocara canis

Diabetes mellitus Diabetes mellitus very commonly causes the development of cataracts in dogs. Cats do not typically get cataracts from diabetes. Toxocara canis (roundworm intestinal parasite) may cause ocular larval migrans in humans and thus is a zoonotic disease. It does not cause cataracts. Hypoadrenocorticism (Addison's disease) does not cause cataracts. Hypothyroidism could lead to elevated blood lipids and an increased chance of lipid corneal deposition but would not lead to cataract formation.

You are asked to take chest radiographs on a cat that has been hit by a car. You are reviewing the radiographs for technique and see that it appears the cat has intestinal contents in the chest cavity. What is this evidence of? Broken ribs A rotated stomach Ruptured pericardial sac Diaphragmatic hernia

Diaphragmatic hernia A diaphragmatic hernia is a hole in the diaphragm. Through this hole, abdominal contents can protrude into the chest cavity. It can be helpful to hold an animal like this in an upright position to allow gravity to keep the abdominal contents out of the chest. The veterinarian should be notified immediately.

What is the name for the blood pressure measurement when the ventricles of the heart are relaxed? Systolic pressure Mean arterial pressure Mean corpuscular volume Central venous pressure Diastolic pressure

Diastolic pressure Diastolic pressure (DP) is the minimum arterial pressure during relaxation and dilation of the ventricles, which represents the pressure when the heart is resting. The systolic pressure (SP) is the pressure when the heart is contracting. Mean arterial pressure (MAP) is essentially an average blood pressure that represents the perfusion pressure seen by the organs in the body. It is calculated by using the systolic and diastolic pressures as seen by the formula: MAP=DP + 1/3 (SP-DP) Central venous pressure (CVP) is the pressure of blood in the thoracic vena cava and is a good estimation of right atrial pressure. CVP reflects the amount of blood returning to the heart and the ability of the heart to pump blood. Mean corpuscular volume is a laboratory term used to describe average volume of a red blood cell that helps to classify an anemia as microcytic or macrocytic.

An owner calls your vet clinic in a panic because her epileptic dog is currently having a seizure. She says that the vet at your hospital had given her a drug to give rectally to help. She starts to list off the medications in her medicine cabinet. Which of these is the appropriate drug for this instance? Ketamine Clomipramine Potassium bromide Naloxone Diazepam

Diazepam Diazepam is a benzodiazepine that can stop a seizure. It is most commonly given intravenously in the clinic but it can be given rectally by an owner at home and is sometimes dispensed for this purpose. Ketamine is a dissociative anesthetic and an NMDA antagonist that is used in anesthetic drug combinations, but it can be seizurogenic and should not be used in dogs prone to seizures. Potassium bromide is used as an anti-seizure drug; it is usually given orally for seizure prevention but an IV loading dose can be given. It is not effective per rectum and must be given over too long of a period of time to be effective at stopping an ongoing seizure. Clomipramine (Clomicalm) is an antidepressant sometimes used as part of the treatment of behavioral disorders in dogs. Naloxone is an opioid reversal agent.

It is the beginning of the day and you are getting together the anesthetic and pre-anesthetic drugs for all of patients for the day. Which of the following drugs should not be drawn up into a disposable plastic syringe, labeled, and stored in a lockbox unless they are going to be used promptly? Medetomidine and ketamine Acepromazine and etomidate Diazepam and propofol Atropine and thiopental

Diazepam and propofol Propofol should not be stored in a lockbox at room temperature because the vehicle for propofol is a liquid emulsion that serves as an excellent growth medium for bacteria. It should be used promptly or discarded. Diazepam should not be stored in a plastic syringe because the drug will bind to the plastic, and the proper dose will not be dispensed when injected. There is no similar contraindication to the storage of the other drugs listed.

Where is the hypothalamus is located? Brain stem Diencephalon Mid-brain Mediastinum

Diencephalon The diencephalon is composed of the thalamus, hypothalamus, and pituitary gland. The brain stem is the posterior part of the brain adjoining the brain and spinal cord. Most of the cranial nerves originate in the brain stem. The brain stem includes the medulla oblongata, pons, and the midbrain. The thymus is located in the mediastinum. The mesencephalon is the mid-brain (part of the brain stem).

Which parasite would be flat and segmented in appearance? Toxocara Trichuris Cystoisospora Dipylidium

Dipylidium The tapeworm of Dipylidium is usually 6 or more inches in length. It is a flat, segmented worm. It attaches to the intestine via the rostellum which has hooks on it. It has 6 rows of teeth. The segments from the tail end of the worm are dropped off and passed in the feces (look like rice granules). These segments contain the eggs which are then shed into the environment.

Which of the following parasites can a dog acquire by ingesting an infected flea during grooming? Toxoplasma gondii Giardia lamblia Toxocara canis Ancylostoma caninum Dipylidium caninum

Dipylidium caninum The intermediate host of Dipylidium caninum is the flea; dogs are usually infected when they ingest a flea carrying the parasite while grooming.

You are entering an exam room to obtain a history and perform a physical exam on a 2 year old German Shepherd. Which of the following body language signals would indicate that the dog is nervous or anxious? Dog is standing with ears erect, tail straight down, and looking around the room. Dog is standing but crouched down, ears pinned back, tail down, and the head lowered. Standing at attention, making eye contact, hackles raised, and growling. Dog is laying on his back, tail tucked to his abdomen, and ears flattened. Dog is standing with ears erect and forward, tail straight down, and looking forward.

Dog is standing but crouched down, ears pinned back, tail down, and the head lowered. A dog that is crouched down, with the ears pinned back, head lowered, and the tail down is displaying nervous body language. A dog with erect ears, tail down, and looking around the room is calm. A dog holding his ears erect and forward with the tail down is alert. A dog laying on his back with his tail tucked and ears flat is submissive. A dog standing at attention with hackles raised and growling is showing aggression and may be protective of the owner.

Which of the following correctly lists the species in order from shortest to longest gestation length? Pig, Sheep, Horse, Cow, Llama Dog, Pig, Sheep, Horse, Llama Cat, Ferret, Sheep, Cow, Horse Ferret, Dog, Sheep, Pig, Horse

Dog, Pig, Sheep, Horse, Llama

A 6-year old Doberman cardiac arrested and was resuscitated. However, the dog has a consistently low systolic blood pressure of 60 mmHg. What drug may be prescribed to increase the blood pressure and via what route? Dopamine via intravenous infusion Hydromorphone via intravenous administration Dopamine via intramuscular administration Terbutaline via subcutaneous administration

Dopamine via intravenous infusion Dopamine increases blood pressure by increasing systemic vascular resistance and is administered via intravenous constant rate infusion. Dopamine is not labeled for intramuscular use. Hydromorphone is an opioid and may cause a reduction in the patient's blood pressure. Terbutaline is a bronchodilator.

In what position is colic surgery most commonly performed? Dorsal recumbency Standing Lateral recumbency Ventral recumbency

Dorsal recumbency

A tibial fracture needs to be repaired. The surgeon is intending to place a bone plate on the medial aspect of the tibia. How should the patient be positioned and prepped? Lateral recumbency with affected side up Dorsal recumbency with affected leg temporarily hung until the patient is draped Dorsal recumbency with leg tied to the table Lateral recumbency with affected side down

Dorsal recumbency with affected leg temporarily hung until the patient is draped

Which medication has been associated with esophageal strictures in cats, therefore the liquid preparation is preferred? Metoclopramide Famotidine Doxycycline Amoxicillin Sucralfate

Doxycycline Doxycycline and clindamycin are the two medications most associated with esophageal strictures in cats (mostly doxycycline). This occurs when a pill gets lodged in the esophagus. Anytime this medication must be given in tablet form, the tablet should be followed with at least 6 mLs of water to help wash the tablet down. Liquid preparations are preferred.

Which antibiotic choice is NOT bactericidal? Baytril Doxycycline Cephalexin Gentamicin

Doxycycline Tetracyclines (including doxycycline) are bacteriostatic, which means these antibiotics do not kill bacteria. Bacteriostatic antibiotics act to inhibit bacterial growth. Tetracyclines, the macrolides (like erythromycin), chloramphenicol, and trimethoprim sulfa are all bacteriostatic. Bacteriostatic antibiotics can be sufficiently effective if the host's immune system is strong. The other choices listed are bactericidal (they kill bacteria).

Which instrument is specifically designed for atraumatically clamping across the intestine? Doyen forceps Kelly forceps Crile forceps Rochester-Carmalt Halted mosquito forceps

Doyen forceps Doyen forceps are the only forceps used in veterinary medicine for atraumatically occluding a loop of bowel. All the other forceps mentioned will result in trauma to the intestine and are not recommended for use on either the intestine or any other delicate structure that the surgeon is trying to preserve.

A fistulogram involves taking a radiograph after a contrast agent is administered into what location? Intravenous Bladder Carpus Tarsus Draining tract

Draining tract A draining tract, also known as a fistula, can be evaluated by injecting a contrast agent into the tract and taking a radiograph to track where the tract goes.

A pet that is exhibiting the symptom of ptyalism has which clinical sign? A jaw locked shut Drooling A head that is hung downward A penis that is stuck protruding from the prepuce

Drooling Ptyalism is drooling or hypersalivation. The term for a penis that is stuck protruding from the prepuce is paraphimosis.

Which of the following signs likely means that parturition is imminent? Drop in body temperature Vocalization Poor appetite Pruritus of the abdomen

Drop in body temperature A drop in the normal body temperature by 1 or 2 degrees means that labor will likely start within 12-24 hours on average. It is important to evaluate this based on multiple temperatures taken at the same time each day over a period of time (i.e., don't take the temperature after the bitch has been outside using the bathroom in the heat and compare it to temperatures taken in the morning at rest, etc).

Which of the following description of the properties of an inhalant anesthetic is true? Drugs with low minimum alveolar concentrations (MAC) are the most potent Drugs with low solubility (blood/gas solubility coefficients) are the most potent Drugs with high minimum alveolar concentrations (MAC) are the most potent Drugs with high solubility (blood/gas solubility coefficients) are the most potent

Drugs with low minimum alveolar concentrations (MAC) are the most potent MAC is a measure of anesthetic potency or, the concentration of drug required to prevent response to a stimulus in 50% of patients. Listed from lowest to highest MAC, inhalant anesthetics include methoxyflurane, halothane, isoflurane, enflurane, sevoflurane, and desflurane. This means that methoxyflurane is the most potent, or, it requires the lowest percentage of gas to cause its effects. Desflurane is the least potent (requires the highest percentage of gas to cause its effects). Potency is not necessarily related to the speed of induction and recovery with an inhalant anesthetic; this property is determined by solubility.

Which of the following statements regarding inhalant anesthetics is true? Drugs with high minimum alveolar concentrations (MAC) produce rapid induction and recovery rates Drugs with high solubility (blood/gas solubility coefficients) produce rapid induction and recovery rates Drugs with low minimum alveolar concentrations (MAC) produce rapid induction and recovery rates Drugs with low solubility (blood/gas solubility coefficients) produce rapid induction and recovery rates

Drugs with low solubility (blood/gas solubility coefficients) produce rapid induction and recovery rates The solubility of an inhalant anesthetic determines the speed of induction and recovery. Low solubility drugs produce rapid induction and recovery. Listed from least to most soluble, inhalant anesthetics include desflurane, sevoflurane, isoflurane, enflurane, halothane, and methoxyflurane. Therefore, desflurane has the fastest induction and recovery rates, and methoxyflurane has the slowest. MAC is a measure of anesthetic potency or the concentration of drug required to prevent response to a stimulus in 50% of patients.

A patient presents with red eyes and the owner reports he has been rubbing his face on the carpet. He is currently taking the sulfa-based drug Primor for a skin infection. What side effect of this medication could cause these symptoms? Dendritic ulcer formation Uveitis Epiphora Dry Eye Glaucoma

Dry Eye Tears are made up of 3 components, lipid, mucous, and liquid. The liquid and mucous portion of the tear film may decrease from sulfa drugs, thus resulting in dry eye (kerratoconjunctivitis sicca/KCS). Sometimes this problem can be irreversible. This patient should receive a Schirmer Tear Test to test for KCS. The eyes should also be stained, because corneal ulceration can occur secondary to the low tear production. Glaucoma is increased intraocular pressure. Uveitis is inflammation in the uveal tract inside the eye. Epiphora is excessive tearing, which is the opposite of KCS. Dendritic ulcers are most typically seen in cats with herpesvirus infection.

Cyclosporine ophthalmic ointment is most often used in treating which condition? Dry eye Corneal ulcers Herpes keratitis Glaucoma

Dry eye Keratoconjunctivitis sicca (dry eye) is often treated with topical cyclosporine (Optimmune) or with topical tacrolimus. These medications act to stimulate tear production; they may also be beneficial in cases of pigmentary keratitis or Pannus.

Delivering puppies and kittens can be a rewarding experience and a procedure that the technician should always be ready to perform. What are the basic steps that need to be taken once a puppy or kitten is delivered? Dry the newborn, massage gently, dry and suction any secretions from mouth and nose Inject with vitamin B complex subcutaneously, flip patient upside down, dry newborn, massage gently, and remove secretions from mouth and nose Massage gently, vaccinate for rabies virus, dry and suction any secretions from mouth and nose Bathe newborn in dilute chlorhexadine solution, massage gently, remove secretions from mouth and nose

Dry the newborn, massage gently, dry and suction any secretions from mouth and nose

In a fetus, blood is shunted from the pulmonary artery to the aorta via what structure? Foramen magnum Foramen ovale Ductus venosus Ductus arteriosus

Ductus arteriosus The ductus arteriosus is the shunt connecting the pulmonary artery to the aortic arch. In this way, the blood bypasses the lungs, or pulmonary circulation in utero. Patent ductus arteriosus (PDA) is when this opening persists after birth and arterial blood is re-circulated through the lungs, which is abnormal. The foramen ovale allows blood to enter the left atrium from the right atrium. The ductus venosus shunts some blood flow from the umbilical vein to the vena cava. The foramen magnum is the large opening in the occipital bone at the base of the skull through which the spinal cord passes.

Which describes the proper order of intestinal segments? Jejunum, duodenum, ileum, colon Ileum, duodenum, jejunum, colon Duodenum, jejunum, ileum, colon Jejunum, ileum, duodenum, colon

Duodenum, jejunum, ileum, colon The proper order of the GI tract is: esophagus, stomach, duodenum, jejunum, ileum, cecum, colon. The duodenum, jejunum, and ileum make up the small intestine. The cecum is the pouch that connects the small intestine to the colon (the large intestine).

A client calls to schedule an appointment with the doctor because her dog has been shaking his head and now his ear flap feels swollen, warm to the touch, and is squishy. What should you book the appointment as? Pinnal balloon Ear hematoma Ear hemangiosarcoma Ear canal ablation

Ear hematoma An ear hematoma occurs when the vessels inside the pinna rupture (from shaking the head, trauma, or vasculitis). The pinna (ear flap) fills with blood like a balloon. The term for this condition is an ear hematoma. The most common treatment includes placing a drain in the pinna that stays in place for a couple of weeks. Underlying ear disease should be treated as well.

From youngest to oldest, which is the correct order of the tick's life cycle? Larva, egg, nymph, adult Egg, nymph, larva, adult Egg, larva, nymph, adult Nymph, larva, egg, adult Nymph, egg, larva, adult

Egg, larva, nymph, adult The exact time course and types of hosts can vary, but female ticks commonly lay thousands of eggs, usually on the ground. These hatch into larval ticks, which will feed on a host and then molt to the nymph stage. The nymphal tick then feeds off a second host and falls off when fully fed. It then becomes an adult that will feed, and the cycle repeats.

Which of these therapies would prevent weight-bearing of a limb? Mason-Meta splint Ehmer sling Fiberglass cast Schroeder-Thomas splint

Ehmer sling Two common examples of slings which prevent weight-bearing are: 1) Ehmer sling- used for coxofemoral luxations (dislocated hips) to hold hip in place 2) Velpeau Sling- used for shoulder luxation or after shoulder surgery Splints provide rigid support and do not prevent weight bearing on the limb. They do not go around the entire limb. Casts provide support and go all the way around the limb.

Which is a hinge joint connecting the humerus to the radius and ulna? Shoulder Elbow Stifle Tarsus

Elbow The elbow joint is a hinge joint connecting the humerus to the radius and ulna. The shoulder joint is formed by the scapula and humerus. The tarsus and stifle are joints of the hindlimb. The tarsus connects the tibia/fibula to the metatarsus (the ankle); the stifle connects the femur to the tibia/fibula (the knee).

The doctor is going to spay a 6 pound Chihuahua and asks you to make sure the patient stays warm during and after the procedure. Which of the following methods is NOT appropriate to use as a heating source for anesthetized patients? Water Circulating blanket Bair Hugger Warming bean bags Electric heating pad Warmed 1 liter fluid bags

Electric heating pad Severe burns may result from electric heating pads even when using a barrier between the pad and patient. In addition, the electric heating pads are not recommended for conscious animals because of the danger of electrocution from the patient bithing the cord.

A dog presents with excitability, muscle tremors, and hyperthermia. A blood panel shows hypocalcemia, and the patient is prescribed an intravenous dose of calcium gluconate. What monitoring must be performed during calcium gluconate administration? Electrocardiogram Central Venous Pressure Blood pressure End-Tidal CO2 Respiration

Electrocardiogram Electrocardiogram (ECG) monitoring must be performed during calcium gluconate administration, and the administration should be slowed or discontinued if bradycardia or arrhythmias occur. Respiration should be monitored but might not change until the heart rate has already significantly slowed. The same is true of blood pressure. Central venous pressures or CVPs would not provide information on the electrical activity of the heart. ETCO2 would also not provide any information pertinent to the infusion.

What is the purpose of a Potter-Bucky diaphragm on an X-ray machine (sometimes called a "Bucky tray")? Eliminate grid lines on the film Prevent patient movement Increase contrast of the film Decrease exposure time that is necessary to produce a diagnostic image

Eliminate grid lines on the film A Potter-Bucky diaphragm is a moving grid. A grid is a series of short strips of lead that is placed over the cassette, absorbing all radiation that does not go between the strips. The strips are geometrically arranged to allow the primary radiation through but to absorb any radiation that is scattered in other directions. This results in increasing the resolution of structures on the film. However, using a grid results in the appearance of grid lines on the film unless a Potter-Bucky diaphragm is used. Using a Potter-Bucky diaphragm requires increased exposure time and/or kilovoltage.

Which drug is known as an ACE-inhibitor? Diphenhydramine Enalapril Metoclopramide Acepromazine

Enalapril Ace-inhibitors like enalapril are vasodilators which block angiotensin-converting enzyme to prevent angiotensin II (a vacosconstrictor) and aldosterone (causes sodium retention) from forming. Ace-inhibitors help to relax smooth muscle in the arterioles and veins. They are most commonly used in cases of heart and kidney disease. Note that ace-inhibitor drugs usually end in a "pril" (benazapril, captopril, enalapril, etc.)

Which of the following may be seen in a puppy who has had distemper? Retained deciduous teeth Gingival hyperplasia Enamel hyperplasia Enamel hypoplasia

Enamel hypoplasia This dog has enamel hypoplasia, which is an enamel deficiency that causes an abnormal enamel matrix and presents as if the enamel has been eaten away. This may be a finding in adult dogs previously infected with distemper virus as a puppy. Tetracycline antibiotics may also cause this, seen as a permanent brown discoloration of teeth when used in young puppies.

Blood vessels are lined with a type of epithelium known as which of the following? Endothelium Urothelium Keratin Mesothelium

Endothelium Endothelium is the thin layer of cells lining the inside surface of blood vessels. The cells that form the endothelium are called endothelial cells. Mesothelial cells are specialized epithelial cells that line the internal organs and body wall in the peritoneal and pleural cavities and line the pericardium. Keratin is a part of the epidermis, or the skin. Urothelial cells, or transitional cells, are part of the urinary system and line the inside wall of the bladder.

Which of the following medications should be used with caution in cats, as it could cause blindness in some cats? Metacam Enrofloxacin Clindamycin Amoxicillin

Enrofloxacin Enrofloxacin (Baytril) is used in cats at no higher than a 5mg/kg per dose. Higher doses increase the risk for blindness in cats, and in general this drug should be used with caution because of this potential side effect.

Which drug is a fluoroquinolone? Carprofen Cephalexin Enrofloxacin Clindamycin

Enrofloxacin Fluoroquinolones include antibiotics such as enrofloxacin (Baytril), ciprofloxacin (Cipro), and marbofloxacin (Zeniquin). The fluoroquinolones are bactericidal. If possible, they should not be used in young, growing animals due to potential for cartilage disturbances. In some cases, they also have the potential side effect of causing blindness in cats at higher doses and should be used with caution.

Which of the following antimicrobial medications should not be administered to young, growing animals? Enrofloxacin (Fluoroquinolones) Cefazolin (Cephalosporins) Ampicillin (Penicillins) Trimethoprim (Trimethoprim-sulfonamide combinations) Gentamicin (Aminoglycosides)

Enrofloxacin (Fluoroquinolones) Enrofloxacin, and other drugs in fluoroquinolone class, should not be used in young, growing animals as it causes damage to the joint cartilage. The other antimicrobial drugs may have side effects, but these can occur at any age.

A 2-month old male mixed-breed puppy is diagnosed with a urinary tract infection. Culture and sensitivity of the urine show the bacteria to be susceptible to enrofloxacin. Why should a different antibiotic be chosen over enrofloxacin? The half-life of enrofloxacin is greatly reduced in puppies Enrofloxacin causes cartilage abnormalities in growing puppies Enrofloxacin does not reach therapeutic levels in the urinary tract Enrofloxacin causes enamel hypoplasia and teeth staining in young animals

Enrofloxacin causes cartilage abnormalities in growing puppies The correct answer is that enrofloxacin causes cartilage abnormalities in growing puppies. Bubble-like changes to articular cartilage can be seen when given to dogs from 2 to 8 months of age. Enamel hypoplasia and teeth staining is caused by tetracycline antibiotics given to young animals. With enrofloxacin, therapeutic levels are reached throughout most of the body except CSF.

You run a CBC on a dog that is heartworm positive. Which blood cell is commonly elevated in dogs with heartworm disease or other parasitic infections? Bands Platelets Eosinophils Red blood cells Monocytes

Eosinophils Parasitic infections (such as heartworm, intestinal parasites, and fleas) often cause an elevated eosinophil count. Other diseases which may cause an elevation of eosinophils include allergies, chronic dermatitis, and feline asthma.

A 5 year old female spayed dachshund is admitted to the emergency room after blood work shows a PCV of 14%. Based on her physical exam and remaining blood work, she is diagnosed with regenerative anemia. Which of the following would least likely appear on her blood smear? Reticulocytes Metarubricytes Eosinophils Band neutrophils

Eosinophils Eosinophils are more commonly seen in patients with parasitic infections or allergic reactions. Reticulocytes are seen in patients with regenerative anemia. Reticulocytes are immature red cells that the bone marrow has released before they have had time to fully mature. Band neutrophils are immature neutrophils that the bone marrow has released early and is commonly seen in regenerative anemia in response to tissue hypoxia. Metarubricytes are also called nucleated red blood cells and are also immature red blood cells that are released early by the bone marrow in response to tissue hypoxia.

The best locations to administer intramuscular injections, such as penicillin, to a horse include all of the following EXCEPT? Pectorals Semimembranosus Epaxial muscles Muscles of the neck Semitendinosus

Epaxial muscles The epaxial muscles, which are at the dorsum of the horse surrounding the vertebrae, are not an acceptable location for an intramuscular injection due to their relatively small size and location near the spinal cord. All the other injection sites are acceptable.

Where do sperm mature? Vas deferens Prostate Epididymis Oviduct

Epididymis The epididymis is the tube attached to the backside of the testes. It is found within the scrotum and is a coiled segment of the spermatic ducts in which spermatozoa mature. It connects the testicle with the vas deferens. Sperm matures within this tube. Oviducts, also called fallopian tubes, are found in the female reproductive tract. The egg passes from the ovary into the uterus via the oviduct. The prostate stores and secretes a portion of the seminal fluid.

A vet at your clinic is preparing to perform a punch biopsy of a dermal mass in a cat under local anesthesia. He asks you to add epinephrine to his lidocaine before administering the local anesthetic. What is the rationale for adding epinephrine? Epinephrine allows the lidocaine to be more rapidly cleared Epinephrine neutralizes the acidic pH of lidocaine, decreasing the pain on injection Epinephrine prolongs the duration of lidocaine's effects Epinephrine acts as a pharmacologic antagonist to the systemic effects of lidocaine, to which cats can be particularly sensitive Epinephrine, when given by this route, has a sedative effect on cats

Epinephrine prolongs the duration of lidocaine's effects Epinephrine is used with lidocaine to cause local vasoconstriction, preventing rapid systemic absorption. This is typically done to prolong the duration of local anesthetic effects of lidocaine by maintaining it at the site. It can decrease systemic uptake and toxicity, but it is not a lidocaine antagonist. Epinephrine does not neutralize the pH of lidocaine; sodium bicarbonate is sometimes added for this purpose.

Which of the following is the correct medical term for a bloody nose? Hemoptysis Epistaxis Melena Pytalism

Epistaxis Epistaxis is Greek for "to bleed from the nose." Ptyalism is excessive drooling. Melena is the term for digested blood in the stool (black or tarry stools). Hemoptysis is coughing up blood.

How should tubes be placed in a centrifuge? Blood tubes should be placed across from each other and urine tubes should be placed next to each other Equal weight directly next to each other Equal weight across from each other Only 1 tube at a time

Equal weight across from each other

In which species is it common to observe Rouleaux formation in a blood smear? Equine Canine Avian Bovine

Equine It is very common for horses to have blood cells arranged like a stack of coins (Rouleaux formation). This is not typically seen in the other species listed and may be indicative of an underlying inflammatory process in those species. Rouleaux formation may be a normal physiologic finding in horses, cats, and pigs. Of these, it is by far the most common in horses.

Lincosamides (such as lincomycin and clindamycin) are contraindicated for use in which species of animal? Feline Equine Canine Swine

Equine Lincosamides are contraindicated in horses, ruminants, rodents, and rabbits. These drugs can cause severe gastrointestinal effects in these species and can even result in death.

Which of the following vaccines is not available for horses? Equine Protozoal Myeloencephalitis Vaccine Equine Influenza Virus Vaccine Eastern Equine Encephalitis Vaccine Streptococcus equi (strangles) Vaccine Equine Herpes Virus Type 1 Vaccine

Equine Protozoal Myeloencephalitis Vaccine While equine protozoal myeloencephalitis is a common neurologic disease in horses caused by Sarcocystis neurona, there currently is no effective vaccine to prevent this disease. The other vaccines listed are all available.

You are called to a pig operation where some of the adult pigs are experiencing sudden death. The vet points out purple skin lesions that look like a "diamonds." What organism causes "Diamond Skin Disease" in pigs? Haemophilus parasuis Staphylococcus hyicus Streptococcus suis Erysipelas rhusiopathiae

Erysipelas rhusiopathiae Erysipelas may cause diamond shaped purple skin lesions in pigs. This bacterium is commonly found in the tonsils of pigs, but may cause disease and death if the pigs experience stress, poor sanitation or feed, causing the bacteria to invade the bloodstream and cause sepsis. Staphylococcus hyicus causes greasy pig disease (exudative epidermitis). Haemophilus parasuis causes Glasser's disease (polyserositis/arthritis/cyanotic ears and skin). Streptococcus suis is more common in weaning pigs (may cause meningitis, and also has similar symptoms to Glasser's disease).

Administration of which antimicrobial is MOST LIKELY to result in diarrhea in the adult horse? Penicillin Erythromycin Gentacin Enrofloxacin Ceftiofur

Erythromycin Many antimicrobials can result in diarrhea in horses because of disruption of the normal intestinal flora. Of the drugs listed, erythromycin has the highest likelihood of contributing to the development of diarrhea. Erythromycin is administered to foals with Rhodococcus equi pneumonia but is rarely used in adult horses because of its common association with the development of diarrhea. All the other listed drugs can be administered to horses.

Ovulation occurs during which part of the estrous cycle? Proestrus Anestrus Estrus Diestrus

Estrus

What is the most likely period of time during the estrous cycle that a female will be receptive to mating? Diestrus Proestrus Anestrus Estrus

Estrus Estrus is the main period of sexual receptivity. At this point, the uterus and uterine horns are primed for receiving an embryo. Proestrus is the period just prior to estrus and is under the influence of a hormone call follicular stimulating hormone (FSH). During this period FSH stimulates release of estrogen. During diestrus the corpus luteum (ruptured follicle) secretes hormones. If pregnancy does not occur, the corpus luteum will degenerate and stop secreting hormones. Anestrus is the period in which there is sexual inactivity.

Which suture type is non-absorbable and would be ideal for placing skin sutures in a laceration? Ethilon Chromic gut Maxon Monocryl Polysorb

Ethilon Ethilon is nylon suture. Nylon sutures are non-absorbable and are often used for skin sutures that will be removed.

How often should an IV peripheral vein catheter be replaced as a standard? Don't replace as long as it is working fine Every 24 hours Every 3 days Every 5-7 days

Every 3 days

After completing a routine spay, the doctor asks you to administer 0.015 mg/kg of buprenorphine intravenously for post operative pain. How many milliliters will you administer if it comes as a 0.3mg/ml solution and the cat weighs 6.6 lbs? 0.33 ml 1.5 ml 0.5 ml 0.15 ml

0.15 ml Always try to work in kilograms. The body weight must first be converted to kilograms. 6.6 lbs / (2.2 lbs/kg) = 3 kg Now that you have the weight in kilograms, multiply by the dose. 3 kg x (0.015 mg/kg) = 0.045 mg (Notice the kilograms cancel out and you are left with milligrams.) Now, figure out how many milliliters is equivalent to 0.045 mg. 0.045 mg / (0.3 mg/ml)= 0.15 ml (Notice the milligrams cancel out.) When solving these equations, be sure to keep track of the decimal place. Being off by a decimal point can result in a severe overdose.

Which feline disease is most commonly transmitted via saliva sharing, such as through food or water bowls, grooming, etc.? Feline Immunodeficiency Virus Feline Infectious Peritonitis Toxoplasma Feline Leukemia Virus

Feline Leukemia Virus FeLV is shed in saliva and other bodily secretions (from the nose, urine, feces, and milk). Cats can transfer the virus via a bite, through grooming each other, and via sharing food and water dishes/litter boxes, etc. It is also transmitted from an infected mother cat to her kittens either in utero or via nursing. FIV is more commonly transferred via a bite wound and not from casual contact. FIP is caused by a coronavirus which is typically passed in feces, saliva or in utero. The coronavirus mutates in some cats and causes clinical FIP. The Toxoplasma parasite is typically acquired through eating raw meat/prey in cats that hunt.

Which of the following bones is not considered a part of the axial skeleton? Femur Vertebra Skull Rib

Femur The axial skeleton consists of bones found on midline or attached to it. The appendicular skeleton is made up of all the bones associated with the limb.

Which bone articulates with the patella? Fibula Femur Humerus Tibia

Femur The distal femur articulates with the patella at the level of the trochlear groove. The patella attaches to the tibial tuberosity but does not articulate with this bone. The fibula is the bone directly adjacent to the tibia. The humerus is a bone of the fo

Which of the following drugs is commonly delivered via a transdermal patch? Budesonide Butorphanol Fentanyl Diazepam

Fentanyl Fentanyl is often administered via a transdermal patch. It provides analgesia for approximately 4 days, and absorption rates may vary. For post-operative uses, it is best to apply the patch at least 12 hours before the procedure if possible.

A client calls your clinic Monday morning to let you know her dog had an emergency foreign body surgery over the weekend. She said they placed a pain patch on his skin that will need to be removed in 2 more days. This patch likely contains which of the following analgesics? Lidocaine Fentanyl Carprofen Morphine

Fentanyl Fentanyl is the most commonly used transdermal patch in small animals. The patch is usually left on for around 4 days to provide analgesia. Lidocaine also comes in a patch form, but is for more short term and local use (such as after a declaw procedure when they may be applied to the paws overnight).

Which of the following clinical signs is LEAST COMMONLY observed in horses with colic? Pawing with the forelimbs Kicking at the abdomen with the hindlimbs Repeated laying down and getting up Flank watching Fever Frequent rolling

Fever Colic is a non-specific term describing abdominal pain. There are many causes of colic in the horse resulting in various clinical signs, including the ones listed here. Of these listed, fever is the least commonly seen. Occasionally, diseases such as infectious peritonitis or metritis can result in fever and signs of colic but are less common.

Which of the following proteins is typically much higher in plasma than in serum? Trypsin Globulin Amylase Albumin Fibrinogen

Fibrinogen Plasma is the fluid portion of blood. Serum is the same as plasma except that it is obtained after blood has clotted, the process where fibrinogen is converted to a fibrin clot. Other proteins should be at similar levels between plasma and serum.

This tool is most often used for which purpose? (used in spays) Retracting the eyelids during ocular surgery Finding the uterine horns Scooping out bladder stones Probing the intestine

Finding the uterine horns

Which procedure could be performed without using a surgical scrub, and an alcohol wipe would be sufficient? Feline neuter Joint taps Fine needle aspiration IV catheter placement Bone marrow aspirate

Fine needle aspiration

Which of the following correctly describes the order and function of fixer and developer solutions during radiograph processing? First, fixer converts exposed silver crystals into black metallic silver and then developer dissolves unexposed silver crystals First, developer converts exposed silver crystals into black metallic silver and then fixer dissolves unexposed silver crystals First, fixer dissolves unexposed silver crystals and then developer converts exposed silver crystals into black metallic silver First, developer dissolves unexposed silver crystals and then fixer converts exposed silver crystals into black metallic silver

First, developer converts exposed silver crystals into black metallic silver and then fixer dissolves unexposed silver crystals Films go into developer first which reduces exposed silver halide into elemental metallic silver which is black. Crystals that were unexposed do not undergo this process. Fixer then coverts the unexposed crystals into a soluble form so that they dissolve away, leaving a clear image where the film was unexposed.

You are combing through a pet's hair with your fingers and notice a large amount of dark debris which looks like dirt. This may implicate which parasite? Lice Ticks Mites Fleas

Fleas

Lufenuron, Fipronil, and Imidacloprid are all used in treatment of which of the following? Fleas Sarcoptic mites Ascarids Ticks

Fleas Lufenuron (Program) controls flea populations by stopping the life cycle at the egg stage. It is an oral formulation. Fipronil acts to inhibit GABA and kills adult fleas. It is a topical and also works against some ticks. Imidacloprid is a topical flea adulticide and is a neurotransmitter blocker (Ach inhibitor). Selamectin (not listed) would act against fleas and sarcoptic mites.

Which of the following is an indicator that a spinal needle has successfully been placed into the epidural space in a horse? There is moderate resistance when pushing fluid through the spinal needle Spinal fluid fills the hub of the needle Fluid is immediately drawn in when placed on the hub of the needle Fluid bubbles immediately when placed on the hub of the needle

Fluid is immediately drawn in when placed on the hub of the needle

Which of the technical errors in radiography will result in increased film density? Temperature during development too low mAs settings too low Focal-film distance is too short Intensifying screen speed is too slow kV settings too low

Focal-film distance is too short Focal-film distance refers to the distance of the film relative to the X-ray source. As the film is moved closer to the source, density increases exponentially due to the inverse-square law. The other answer choices listed would all decrease film density.

Flunixin meglumine is sometimes used in horses, cattle, and pigs for which purpose? For constipation For tapeworm infestation For analgesia For acid reflux

For analgesia Flunixin meglumine (Banamine) is a non-narcotic, non-steroidal anti-inflammatory drug used to provide analgesia and to reduce inflammation. It may also be used as a fever reducer.

Which of the following is not an acceptable means of euthanasia in reptiles? Freezing Carbon dioxide Injectable pentobarbital Anesthetic followed by decapitation

Freezing The AVMA Panel on euthanasia has published several guidelines for reptilian euthanasia. Many methods remain controversial. It is the consensus that freezing is considered inhumane (see link below). "Immobilization of reptiles by cooling is considered inappropriate and inhumane even if combined with other physical or chemical methods of euthanasia." The other methods listed are considered acceptable. The type of euthanasia method may vary depending on certain factors (if a post-mortem exam needs to be performed, if cultures need to be submitted, etc.).

A patient presents with rodenticide toxicity and is hemorrhaging. The doctor decides the animal needs a transfusion. What blood product is most likely to help stop the patient's bleeding? Fresh frozen plasma Packed red blood cells Any blood product will do Stored plasma

Fresh frozen plasma

When looking at the bottom of a horse's hoof, what is the name of the triangular shaped structure behind the sole of the foot? White line Bars Secondary sole Laminae Frog

Frog The name of the triangular structure is the frog (cuneus ungulae); this structure fills in the gap between the bulbs of the heels. The frog is a soft structure and is a normal part of the hoof anatomy.

A dog presents with lethargy and coughing. The doctor asks you to take a blood sample to test for Coccidioides immitis, also known as Valley Fever. This disease, most commonly seen in the southwestern United States, is caused by which of the following? Mite Rickettsia Bacteria Fungus

Fungus Coccidioides is the fungus that causes Valley Fever which can have many different symptoms. Symptoms most commonly include lameness, coughing, lethargy, and loss of appetite. The treatment of choice is fluconazole. Rickettsia is a genus of Gram-negative bacteria which are transmitted most commonly by a tick bite. Examples of rickettsial diseases include Ehrlichia canis (Tick Fever), Rocky Mountain Spotted Fever (Rickettsia rickettsii), and Salmon Disease (Neorickettsia helminthoeca). These are treated with doxycycline.

Which of the choices below is in the correct order from least-to-most radiodense (darkest-to-whitest on the film)? Gas, fat, soft tissue and fluid, metal, bone Metal, bone, fat, soft tissue and fluid, gas Gas, soft tissue and fluid, fat, bone, metal Gas, soft tissue and fluid, fat, metal, bone Gas, fat, soft tissue and fluid, bone, metal Bone, metal, fat, soft tissue and fluid, gas

Gas, fat, soft tissue and fluid, bone, metal

Emesis should not be induced in dogs that have ingested which toxin? Bromethalin Gasoline Brodifacoum Chocolate

Gasoline

The "boxing glove" shape of air in this dog's abdomen is indicative of what emergency? Splenic torsion Gastric dilatation and volvulus Pneumothorax Bladder rupture

Gastric dilatation and volvulus The presence of gas appears black on films, showing up on this radiograph as a stomach full of trapped air. The "boxing glove" owes it shape to the stomach twisting along its long axis.

An owner rushes in with her Labrador retriever who is having non-productive retching, is lethargic, and has a distended abdomen. She wants to know if he has "bloat". Which condition is she concerned about? Splenic mass Mesenteric volvulus Gastric dilation volvulus Ascites

Gastric dilation volvulus

For which of the following emergency situations would you most likely pass a stomach tube in a dog? Mesenteric torsion Perforated stomach Gastric-dilatation-volvulus Intestinal intussusception

Gastric-dilatation-volvulus Of the answer choices provided, the only condition which results in life-threatening distension of the stomach is gastric-dilatation-volvulus. This condition results in abnormal positioning of the stomach such that blood flow to the stomach is disrupted and gas accumulation within the stomach occurs. The more gas that accumulates, the less blood is able to flow through the stomach walls. In time, this will cause necrosis of the stomach wall. Eventually, there is enough distension that the stomach begins to compress the caudal vena cava, and blood return to the heart is impeded. This results in shock, and if patients are not treated immediately they will die. One of the emergency procedures used to help stabilize the patient is the passing of a stomach tube. The tube is measured to the level of the stomach and then passed through the mouth. Decompression of the stomach will save the patient's life. However, the stomach should be anchored in a normal anatomical position by performing a gastropexy so that a GDV could never recur.

A Great Dane is presented for her ovariohysterectomy. The owner would also like to have a procedure done at the same time to secure her stomach to the body wall to prevent her from a gastric dilatation volvulus in the future. What is this procedure called? Gastropexy Gastric bypass Gastrotomy Gastrectomy

Gastropexy

This retractor is being used to hold open a laryngotomy incision in a horse. Which instrument is this? (curve and then curve out) Gelpi Balfour Senn Deaver

Gelpi The gelpi retractor is a self-retaining spreader suitable for small surgical sites. The blades separate as the handles are closed and are held open by a rachet. The blades turn down at right angles at their tips, and bow towards each other so the edges of the incision are pushed apart. Senn and Deaver retractors are hand-held retractors. Balfours are self-retaining retractors used in abdominal surgery to allow visualization of internal organs.

What is one way to help determine if a nasogastric tube was properly placed in a horse? Fluid will automatically come out of the tube when it is properly placed Infuse water down the tube to see if the horse coughs When the horse breathes, you can feel air coming from the tube Gently blow air into the tube and listen for gurgling

Gently blow air into the tube and listen for gurgling A nasogastric (nose to stomach) tube should go down the esophagus (not the trachea). First measure the length of the tube needed to reach the pharynx and the length to reach the stomach. Lubricate the tube before insertion. Enter the ventral nasal meatus and gently push forward; do not force the tube. Once it reaches where the pharynx would be, flex the neck of the horse which helps to get it into the esophagus and not the trachea. The tube should pass when the horse starts to swallow. If the tube goes into the trachea, the horse does not always cough. You should not feel the horse breathing through the tube; this would indicate that the tube has entered the trachea. You do not want to put water in the tube until correct esophageal placement is confirmed. Often times when the tube is passed into the stomach, there may be a sweet or fetid odor. If you gently blow air into the tube it will cause bubbling in the stomach, and you can hear it by ausculting on the left side between the 8th and 14th ribs. Fluid does not automatically come out of the tube.

Which breed is most at risk for developing Degenerative Myelopathy? German Shepherd Standard Poodle Corgi Dachshund Golden Retriever

German Shepherd This is a progressive spinal cord disease that causes that myelin sheath around the nerves to degenerate. German Shepherds are over-represented for this condition. Dachshunds most commonly get intervertebral disk disease.

A boxer presents with overgrowth of the gums. This condition is known as which of the following? Odontoclastic Resorptive Lesions Gingival hyperplasia Faucitis Stomatitis

Gingival hyperplasia Gingival hyperplasia is overgrowth of the gum tissue characterized by non painful swellings of the gingiva. In some cases, a biopsy may be required to rule out neoplasia. The proliferative tissue can be trimmed away during a dental but dogs with this condition are prone to recurrence. The cause is often unknown, but it may be hereditary or caused by medications such as cyclosporine, phenytoin, or amlodipine.

Buprenorphine is to be sent home with a 17 pound domestic long hair for pain following an abdominal exploratory surgery. The veterinarian would like the prescription to be 0.01 mg/kg PO QID PRN, dispense 12 syringes. If the concentration of buprenorphine is 0.3 mg/ml, what will you put on the label? Give 0.56 mls by mouth every 6 hours as needed for 3 days. Give 0.25 mls three times daily for 12 days. Give 0.25 mls PO QID for 12 days. Give 0.25 mls by mouth every 6 hours as needed for 3 days.

Give 0.25 mls by mouth every 6 hours as needed for 3 days. First you must convert pounds to kilograms dividing 17 by 2.2 = 7.7 kg. 7.7 kg multiplied by 0.01 mg/kg = 0.07 mg. 0.07mg divided by 0.3 mg/ml = 0.25 mls. The patient will recieve 0.25 mls by mouth (PO) every 6 hours (QID), and dispensing 12 syringes will provide 3 days worth of medication.

A dog under anesthesia has an end-tidal CO2 (ETCO2) reading of 60 mm Hg. What should be done? Increase the anesthetic gas because the patient is light Give the patient a breath, or ventilate, to lower the ETCO2 Increase the oxygen flow rate to the patient Give a dose of atropine IV

Give the patient a breath, or ventilate, to lower the ETCO2 If the patient has an ETCO2 greater than 55, the patient is hypoventilating. Giving the patient some breaths, or ventilating the patient, will help to lower this value. Increasing the oxygen flow rate would increase the amount of anesthetic flowing through the circuit. Atropine would increase the heart rate. Typically, a patient in a light plane of anesthesia would be hyperventilating and may have a lower ETCO2 (less than 30).

An owner brings a box of rat poison to your hospital and says her dog ingested it. You help the doctor induce vomiting. Which of the following would be an appropriate treatment for an animal that has ingested an anticoagulant rodenticide? Give IV vodka Give vitamin K Give atropine Give diazepam

Give vitamin K It takes at least a few days for anticoagulant rodenticides to cause prolonged bleeding times and clinical hemorrhage. This could likely be prevented with early intervention with decontamination and Vitamin K1 administration. In patients that have prolonged bleeding times, plasma is also indicated. The bright green vomit confirms the owners suspicion of rodenticide ingestion because many rat poison contains a bright green dye. Dogs that ingest these products may have bright green vomit or stool.

If you take the total protein measurement and subtract the albumin, what measurement is left? Potassium Globulins Calcium Amylase

Globulins Albumin and globulins are combined for the total protein level. Globulins are the protein that plays an important role in the immune system. Calcium binds to albumin so if the albumin is low, the calcium level can appear falsely decreased. This is not measured in the total protein level.

Which food consumed by humans can be highly toxic to dogs and cats? Spinach Carrots Grapes Bananas Tomatoes

Grapes Grapes and raisins can cause severe kidney problems/failure in some cats and dogs. This may be due to the high levels of oxalates they contain. Other toxic foods include macadamia nuts, onions, and chocolate. Coffee, alcohol, yeast dough, nutmeg, and sugarless gum containing xylitol can also cause severe toxicities.

Linear artifacts on a film are usually due to which of the following? Cassette Film Intensifying screen Collimator Processing machine Grid

Grid Linear artifacts can be normal gridlines or may be due to the beam not being appropriately centered on the grid. Alternatively, they may occur if the grid is otherwise out of position, upside-down, or damaged. Processing machine errors may cause streaks on the film which should not be confused with linear artifacts. Errors with the other components listed should not cause linear artifacts.

Many times, horses require additional muscle relaxation during anesthesia. Which of the following is commonly used for its property of being a centrally acting muscle relaxant? Atracurium Ketamine Guaifenesin Succinylcholine

Guaifenesin The correct answer is guaifenesin. Unfortunately, the exact mechanism is unknown; however, we do know that guaifenesin acts centrally by blocking nerve impulse transmission at the internuncial neuron level in the subcortical brain, brain stem, and spinal cord. Early signs of toxicity are in the form of increased rigidity. Ketamine is a dissociative agent and is known for its ability to cause hypertonia. Atracurium is a non-depolarizing neuromuscular blocking agent that acts by competitively binding to cholinergic receptors at the motor endplate. Succinylcholine is a depolarizing neuromuscular blocking agent.

Which anesthetic gas would be most likely to induce a cardiac arrhythmia? Isoflurane Halothane Oxygen Sevoflurane

Halothane While any of the inhaled anesthetics may cause cardiac arrhythmias, they would be more likely to occur with the use of Halothane gas. Because of this, most veterinary practices have moved toward the use of Isoflurane or Sevoflurane for anesthetic maintenance during procedures.

What is most commonly used to restrain a horse for basic procedures? Squeeze chute Twitch Halter Hobbles

Halter The halter is a "head collar" or headgear that fits behind the ears and around the muzzle. A lead rope is attached to the halter. It is a basic piece of equipment that is almost always used when handling an adult horse. Twitches are also used when greater restraint is needed.

Which of the following techniques should NOT be performed when trying to stimulate breathing in a newborn calf? Pour cold water over the calf's head. Place a finger or piece of straw in the calf's nose. Hang the calf off the ground by its back legs and shake or swing it to stimulate breathing. Use of acupuncture points on the muzzle. Use of a respiratory stimulant such as doxapram.

Hang the calf off the ground by its back legs and shake or swing it to stimulate breathing. Recent data suggests that hanging calves to clear the airway is unnecessary. Placing a finger or piece of straw in the nose will elicit a gasping reflex and help aerate the lungs. Pouring cold water over the head will stimulate a hypothermal respiratory response. A pharmacological initiation of respiration can be achieved with medications like doxapram (Dopram). Acupuncture points on the muzzle are also useful for stimulation of respirations.

Traumatic reticuloperitonitis or pericarditis in cattle is most commonly known as which of the following conditions? Metal heart disease Hardware disease Reticulitis White muscle disease

Hardware disease Hardware disease is caused by ingestion of wire, nails, etc; these foreign bodies migrate into the reticulum and may pierce through and cause damage to internal organs (including the heart and diaphragm). It is most easily prevented via a magnet being placed into the reticulum to prevent these ingested metallic foreign bodies from migrating.

You are assisting during surgery, and the surgeon is about to close the incision. She needs a pair of Olsen-Hegar needle holders. What is a fundamental difference between this type of needle holder as compared to others? The jaws are serrated for a better grip Does not have a ratcheting device Has a built-in scissor The tips are magnetized

Has a built-in scissor Olsen-Hegar needle holders are popular among veterinarians who are performing surgery without assistants, because Olsen-Hegar holders have a built-in suture scissor. One major disadvantage is that one may inadvertently cut the suture when tying a knot. All standard needle holders will have a ratcheting device built into them.

What is the meaning of the term multiparous? Ovulates multiple times during a cycle Has had multiple pregnancies producing offspring Born with more than one uterus Has more than one offspring at birth

Has more than one offspring at birth The definition of multiparous is having two or more offspring at birth.

The owner of an 8-year old German Shepherd dog calls to report that the dog may have ingested an unknown quantity of rat bait 12 hours ago. What would be the best course of action to advise over the phone? Have the owner bring the dog to the clinic immediately Have the owner induce vomiting at home The new formulations of rat bait on the market are pet-friendly and no action needs to be taken Take a message and have the veterinarian call the owner back Have the owner monitor the dog's mucus membranes and bring the dog to the clinic if the mucus membranes become pale

Have the owner bring the dog to the clinic immediately Rat bait can cause prolonged clotting times or cerebral edema, and the dog should be seen immediately. It may not be advisable to induce vomiting without a veterinarian examination. Signs such as poor mucus membrane color only develop after a coagulopathy has developed, and by that time, intervention will be much less effective. The veterinarian won't be able to provide any treatment for rat bait ingestion over the phone. Rat bait is never pet-friendly.

The surgeon asks you to please get him some gelatin foam. What will he use this for? He will use it to preserve the biopsy specimen He will use it as an analgesic He will use it to stop bleeding He will use it to test for the presence of bacteria

He will use it to stop bleeding Gelatin foam (gel foam) is used to stop oozing of blood.

Which clinical symptom can be an indicator of vestibular disease? Aggression Head bob Decreased conscious proprioception Head Tilt Hypermetric gait

Head Tilt Vestibular disease usually causes a head tilt (one ear lower than the other) sometimes with leaning and falling to that side. Other signs of vestibular disease include ataxia (in which the body may sway and the pet may have a wide-based stance) and abnormal nystagmus (eyes moving back and forth). Hypermetria is an exaggerated gait seen with cerebellar disease. Head bobbing or intention tremors are more consistent with cerebellar lesions. Decreased conscious proprioception can be seen with central/brain disease or from spinal cord disease of any kind and is not a specific finding. A behavior change such as aggression would not be expected in animals with vestibular disease.

When cattle are placed in lateral recumbency during general anesthesia, how should the head be positioned to decrease the risk of aspiration? Head and oral cavity lower than the neck Head and oral cavity at the same level as the neck Head and oral cavity higher than the neck Head and oral cavity higher than the neck or at the same level as the neck

Head and oral cavity lower than the neck During general anesthesia the esophageal sphincter relaxes, and if regurgitation occurs the fluid can be aspirated. Positioning the head and oral cavity lower than the neck paired with a properly inflated endotracheal cuff can reduce this risk.

Goats exhibiting aggressive behavior may do so most commonly in which manner? Head butting Biting Urinating and defecating Kicking with the hind legs

Head butting Head butting in goats can be play behavior, but is primarily a form of aggression with other goats and possibly humans. Always watch behind you when you are in with a herd of goats, as they will often come up behind you!

Purkinje fibers are found in which of the following organs? Pancreas Stomach Heart Duodenum Bladder

Heart Purkinje fibers are conductive fibers within ventricular walls. They relay cardiac impulses to ventricular cells, which allow contraction.

A cat presents with a history of having difficulty walking and is dragging one of his back legs. The limb is cool to the touch. The veterinarian diagnoses a saddle thrombus. Which of the following underlying diseases is most likely present in this cat? Renal disease Heart disease Thrombocytopenia Liver disease Pancreatic disease

Heart disease Aortic thromboembolism, or saddle thrombus, is a complication associated with heart disease in the cat. A thrombus is a blood clot. An embolism is when a clot lodges inside a blood vessel. In cats, a saddle thrombus develops due to underlying cardiomyopathy. A clot forms in the heart, then breaks free and travels in the blood vessels until it becomes lodged. The most common site for the clot to lodge is in the bifurcation of the aorta, in the area where blood supply branches off to supply the rear legs. The thrombus cuts off the circulation to the legs and is very painful.

A 4 year old male neutered domestic shorthair cat presents to your clinic with a history of straining to urinate. The owner is unsure how long it has been since the cat urinated. What is the first thing you should check while waiting for the doctor? Temperature Blood pressure PCV (Packed cell volume) Pupillary Light Response Heart rate

Heart rate

Oxidative damage to red blood cells commonly causes formation of which of the following? Schistocytes Basophilc stippling Spherocytes Heinz bodies Howell-Jolly bodies

Heinz bodies Heinz bodies are round structures seen in red blood cells that represent denatured hemoglobin caused by oxidative damage. They are seen best when cells are stained with new methylene blue. A spherocyte is a dark-staining red blood cell with no central pallor that usually occurs from partial phagocytosis of the cell, as is seen with immune-mediated destruction. A schistocyte is a red blood cell fragment that is usually formed due to shearing from intravascular trauma; it is seen in cases of DIC or vascular neoplasms such as hemangiosarcoma. Howell-Jolly bodies are basophilic nuclear remnants seen in red blood cells; increased numbers are seen after splenectomy. Basophilic stippling appears as small, dark blue bodies in a red blood cell and represents residual DNA. It may indicate lead poisoning when seen in dogs.

You are looking at a skin scraping from a calf and find the 6-legged parasite shown below. What is this organism? (Big booty) Boophilus, a tick Hematopinus, a louse Sarcoptes, a mite Demodex, a mite

Hematopinus, a louse This is a blood-sucking genus of louse and can cause severe anemia. You can tell this is a louse and not a tick or a mite because lice are insects with 6 legs, and ticks and mites are arachnids with 8 legs.

The urine in this collection set is demonstrating: Isosthenuria Hematuria Pollakiuria Ketonuria

Hematuria The red tinge to the urine demonstrates that there are a high number of erythrocytes in the sample. You would need to check the urine specific gravity to know if there is isosthenuria. You would need to do a dipstick to check for ketones.

Which of the following is an anticoagulant that DOES NOT chelate calcium? Heparin EDTA Oxalate Citrate Fluoride

Heparin EDTA, citrate, fluoride, and oxalate all are anticoagulants that work by chelating calcium, which is needed for clot formation. Heparin works by potentiating antithrombin which prevents conversion of prothrombin to thrombin in the clotting cascade.

Which of the following is a reason why EDTA is a preferred anticoagulant over heparin when collecting a blood sample that will be used for a differential blood film? Heparin interferes with red blood cell staining and causes platelet clumping Heparin shrinks cells Heparin interferes with white blood cell staining and causes white blood cell clumping Heparin will chelate calcium

Heparin interferes with white blood cell staining and causes white blood cell clumping Heparin works by potentiating antithrombin which prevents conversion of prothrombin to thrombin in the clotting cascade. EDTA works by chelating calcium, which is needed for blood clotting. Heparin interferes with white blood cell staining and causes white blood cell clumping, so it should not be used for blood that will be made into a smear for a differential white blood cell count. Heparin is also more expensive than EDTA. Excess EDTA can cause cells to shrink, but EDTA preserves cell morphology better than other commonly used anticoagulants.

What does Eimeria stiedai cause? Cloacal coccidiosis in chickens It is a non-pathogenic coccidia in horses Hepatic coccidiosis in rabbits Intestinal coccidiosis in dogs and cats

Hepatic coccidiosis in rabbits Eimeria stiedai is the species of Eimeria that causes hepatic coccidiosis in rabbits. The rabbits ingest sporulated oocysts, which contain four sporozoites that hatch and travel via the hepatic portal vein to the liver and penetrate the bile duct epithelium. The schizonts in the bile duct cause bile duct hyperplasia; they block the bile duct, causing hepatomegaly, icterus, and liver failure.

An obese cat that is not eating for multiple days is at an increased risk for developing which condition? Megacolon Anemia Inflammatory bowel disease Gastric outflow tract obstruction Hepatic lipidosis

Hepatic lipidosis Hepatic lipidosis is also known as fatty liver syndrome. Excess fats are broken down from the cat's fat stores and transported to the liver. The liver should then process this fat for utilization. In cats with hepatic lipidosis, this is impaired, resulting in accumulation of liver fat. The liver cells fill with fat and swell. This condition must be treated aggressively or can lead to death. The most important treatment for this condition is feeding. Most cats require placement of a feeding tube.

You are enrolled in a technician training session at a laboratory primate center and are observing the handling of macaque monkeys for blood sampling. You are required to wear personal protective equipment, mostly to prevent possible exposure to which disease? Hepatitis B Influenza A Human Immunodeficiency Virus Herpes B Virus

Herpes B Virus Herpes B Virus is a very serious and often deadly zoonotic disease that can be carried by macaque monkeys. It is the disease of most concern in a primate lab animal setting. It causes neurologic signs and can have a mortality rate of around 80% for humans. Bites, scratches, or exposure to mucus membranes including the eyes or mouth from infected saliva can cause transmission. Primates can carry Simian Immunodeficiency Virus (not HIV) and it can be zoonotic.

Some x-ray machines have the option of using 2 different sized filaments at the cathode. Which of the following is an advantage of using a larger filament? Increased penumbra effect Higher exposures can be used Improved image definition Decreased penumbra effect

Higher exposures can be used A larger filament produces a broader electron beam with a larger focal spot. This spreads out the area of the target where heat is generated and allows higher exposures to be used. However, the wider focal spot leads to decreased image definition and increased penumbra effect, both of which are disadvantages. The penumbra effect is the blurring of margins that occurs as a result of the geometry of the x-ray beam.

Which of the following statements about ultrasound probe transducers is true? Higher frequency probes have decreased image resolution and increased penetrating ability relative to lower frequency probes Higher frequency probes have increased image resolution and increased penetrating ability relative to lower frequency probes Higher frequency probes have increased image resolution and decreased penetrating ability relative to lower frequency probes Higher frequency probes have decreased image resolution and decreased penetrating ability relative to lower frequency probes

Higher frequency probes have increased image resolution and decreased penetrating ability relative to lower frequency probes The frequency of an ultrasound probe is usually measured in MHz (megahertz). Higher frequency probes (i.e. 7.5 MHz or 10 MHz) probes have increased image resolution but sacrifice penetration. Lower frequency probes (i.e. 5 MHz) are better for imaging deeper structures or cavities but do not provide as much image resolution.

In order for oxygen to diffuse from the alveoli into the capillaries, the partial pressure of the oxygen in the alveoli in the lungs must be: Higher in the capillaries than in the lungs Oxygen cannot diffuse through the alveoli The same pressure between both the lungs and the capillaries Higher in the lungs than in the capillaries

Higher in the lungs than in the capillaries Partial pressure of oxygen in the alveoli is ~100mmHg, and is ~45mmHg in the capillaries. Gas diffuses from an area of higher concentration to an area of lower concentration.

Which is a "ball and socket" type of joint? Elbow Atlantoaxial Hip Stifle

Hip The hip joint (acetabulofemoral joint) is the ball of the head of the femur sitting in the socket (acetabulum) of the pelvis. The atlantoaxial joint is a pivot joint connecting the first and second cervical vertebrae. The stifle and elbow are hinge joints.

The veterinarian is performing an orthopedic exam and says a dog has positive Ortolani sign. What corresponds with this? Cranial cruciate rupture Cervical instability Luxating patella Hip dysplasia

Hip dysplasia Ortolani is the palpable sensation of gliding the femoral head in and out of the acetabulum and suggests joint laxity, most often seen in hip dysplasia. A "drawer sign" would correspond with a cranial cruciate rupture.

A 10-year old domestic short hair presents with a history of vomiting and inappetance and is severely dehydrated. His CBC shows a hematocrit of 22%. What can be said about this cat? The cat is losing blood His true hematocrit is likely higher than 22% His true hematocrit is likely lower than 22% The cat is likely in renal failure

His true hematocrit is likely lower than 22% A hematocrit of 22% tells us that the cat is anemic. Dehydration causes a falsely elevated hematocrit or PCV. This means that because this cat is dehydrated, the hematocrit is likely actually lower than 22%. The hematocrit or PCV often goes down when hydration is restored. While chronic renal failure can lead to anemia of chronic disease, this assumption cannot be made based on the information given in the question. Even though the hematocrit is low, this doesn't mean the cat is losing blood. To know whether blood loss or chronic disease is causing the anemia, a reticulocyte count would be needed to see if this cat is regenerating (making new red blood cells). In cases of blood loss or lysis, the anemia is usually regenerative (reticulocytes greater than 60,000). In cases of chronic disease, the anemia is usually non-regenarative (reticulocytes less than 60,000).

Which of the following methods is not recommended when picking up a snake? Hold it with a snake hook Hold it by the tail Hold it by the caudal half of the body Hold it by the base of the neck

Hold it by the tail Some snakes can lose their tails as a defense so should not be picked up by the tail. This will prevent skin loss or degloving injury. The same applies to lizards.

After which procedure is it most important that you immediately wash your hands as soon as the procedure is complete? Fluoroscopy Skin scraping on a dog with Demodex Holding a dog that may have Microsporum canis Restraining a cat that has Feline Immunodeficiency Virus (FIV)

Holding a dog that may have Microsporum canis

What purpose does this instrument serve? (blade) Helps to locate the uterine horns Cuts suture material Probes pockets under the gingiva Holds the scalpel blade

Holds the scalpel blade

Which parasite is commonly passed from a mother to her young via the transmammary route? Whipworms Giardia Hookworms Coccidia Tapeworms

Hookworms A common mode of hookworm transmission (Ancylostoma) is transmammary (nursing) through infected milk.

For which species is it most common to clip and clean the surgical site prior to anesthetic induction? Horse Dog Cat Ferret

Horse Out of the species listed, the horse is most amenable to being clipped and cleaned prior to surgery. Clipping and cleaning prior to anesthesia will help reduce the amount of total anesthesia time.

Fermentation is a microbial process that uses enzymes to decompose food so that it may be utilized by the animal. In which species is this process most utilized in the hindgut during digestion? Dogs Sheep Horses Cows

Horses Horses are hindgut (large-intestinal) fermenters. They are herbivores, and the cellulose in the plants they consume is difficult to digest. They have a fermentation system which contains specific microflora, that break sdown the plant material, allowing them to access useful nutrients. Ruminants (sheep and cows) are also herbivores but are foregut fermenters as their fermentation process occurs in the rumen. Dogs are carnivores and monogastrics.

Urine from which of the following species is normally cloudy or milky in appearance? Dogs and ferrets Horses and rabbits Cats and cattle Sheep and goats

Horses and rabbits Urine from horses and rabbits contain high concentrations of calcium carbonate crystals that can make the urine appear milky. Additionally, horses normally have mucous from glands in the renal pelvis that contribute to the urine, which can make it appear cloudy.

Which of the following drugs is a controlled substance? Propofol Hydrocodone Acepromazine Medetomidine

Hydrocodone Hydrocodone (Hycodan, Tussigon) is an opiate agonist and is a controlled narcotic. It is often used for its anti-tussive properties in animals that are coughing. The other drugs listed are not controlled.

What is an expected finding in a patient with Hypervitaminosis D, or Vitamin D toxicity? Hypoglycemia Hyperkalemia Hypocalcemia Hypercalcemia Hypokalemia

Hypercalcemia Vitamin D is the building block of calcitriol, which works with parathyroid hormone. Calcitriol increases absorption of calcium and phosphorus from the intestines and decreases their excretion in the urine. In so doing, calcium levels in the blood rise and PTH levels drop. So, when vitamin D is too high from intake, calcium is too high (there is no negative feedback mechanism with PTH).

What is the most common and life-threatening metabolic abnormality in a cat with a urethral obstruction? Increased BUN and creatinine Hyperglycemia Hypochloremia Hyponatremia Hyperkalemia

Hyperkalemia A high potassium level is life-threatening. The majority of potassium is excreted in the urine. When the cat is unable to urinate, potassium builds up in the bloodstream and can cause serious metabolic consequences. Although the BUN and creatinine are often high, they are usually elevated from pre-renal (dehydration) causes and post-renal (obstruction) causes and will normalize after fluid therapy.

What clinical signs are associated with infection with Clostridium tetani in the horse? Hypersensitivity to sound and touch characterized by signs such as muscle spasms (stiff gait) and protrusion of the third eyelid Increased gastrointestinal motility characterized by increased passage of loose stool Generalized muscle paresis characterized by signs of generalized weakness and difficulty breathing Increased anxiousness and sweating as a result of tachycardia and increased circulating epinephrine

Hypersensitivity to sound and touch characterized by signs such as muscle spasms (stiff gait) and protrusion of the third eyelid Clostridium tetani causes muscle spasms as a result of the organism producing toxins that block normal inhibitory neurons. Classic clinical signs include hypersensitivity to sound and touch resulting in muscle rigidity (lockjaw) and protrusion of the third eyelid.

A bird that is laying eggs with thin soft shells likely has: Giardia Anemia Psittacosis Hypocalcemia

Hypocalcemia Calcium is necessary to construct a rigid egg shell. If the laying bird is hypocalcemic then not enough calcium will be deposited into the shell, resulting in a soft egg. Giardia is a parasitic disease that does not result in soft eggs and usually affects the gastrointestinal tract. Anemia is a decrease in hematocrit below normal levels which may result in pallor and weakness. There are many possible causes for anemia. Pstittacosis is caused by a type of chlamydia bacteria which causes a combination of weakness, ocular discharge, respiratory illness and gastrointestinal disease.

A Yorkshire Terrier who has given birth 2 weeks ago presents with a history of tremoring. She is febrile and not eating well. On her electrolyte panel which of the following would you expect to see? Hypomagnesemia Hypokalemia Hypocalcemia Hyponatremia

Hypocalcemia Eclampsia, also known as "milk fever" is caused by a low calcium level. It most often occurs around 3 weeks after whelping due to losing calcium in the milk and the inability of the body to keep up with demand. It is most common in small-breed dogs with larger litters. It is treated with calcium gluconate IV in an acute crisis and oral supplementation is then required. Milk fever is also seen more commonly in dairy cows that are close to calving. It typically causes a LOW temperature and often causes staggering and recumbency. It can also cause tremors and dilated pupils. It is also treated with injectable calcium. When cows are dry (not lactating), they should be kept on a LOW calcium diet so that calcium can be mobilized more rapidly from the bone when it is needed during calving and lactation.

Bacterial sepsis frequently causes which finding on a blood panel? Eosinophilia Hypoglycemia Anemia Hypercalcemia

Hypoglycemia

You are working at an emergency practice and a woman brings in her ferret "Macy" who is obtunded. The ferret is very weak and appears neurologic. The veterinarian suspects an insulinoma, which is the third most common tumor in the ferret. If this is the case, what laboratory finding would you expect? Hypoglycemia Hypokalemia Ketones in the urine Hyperglycemia

Hypoglycemia An insulinoma is a tumor on the pancreas. The tumor causes the pancreas to secrete excess insulin. Insulin causes the blood sugar to decrease. A low blood sugar is called hypoglycemia. This can lead to neurologic symptoms and extreme weakness seen in this patient. Ketones in the urine would be more common in a chronic diabetic (hyperglycemic) patient. Hypokalemia is a low potassium and can cause extreme weakness and ventroflexion of the neck.

A Boston terrier puppy presents lethargic and minimally responsive at your clinic. The owner says he hasn't been eating very well; his gums are a little pale, temperature is 99.1F. What is the likely cause of his acute symptoms? Hypoglycemia Head trauma Parvovirus Intestinal parasites

Hypoglycemia When a young puppy or kitten presents lethargic and minimally responsive, or has symptoms of weakness or pale gums, hypoglycemia should be at the top of your list of suspected afflictions. You should draw a blood sample to check the glucose level, and be ready to administer dextrose when the veterinarian gives the order. Parvovirus and intestinal parasites can also cause hypoglycemia (what he is acutely presenting for), so additional diagnostics will be necessary to determine the underlying cause of this puppy's problem.

Which electrolyte abnormality is known to cause ventroflexion of the neck and extreme weakness? Hypokalemia Hypermagnesemia Hypercalcemia Hyponatremia

Hypokalemia Hypokalemia (low potassium) can result from various metabolic and endocrine disorders. It causes muscle weakness and can cause cervical ventroflexion. Hypokalemic myopathy has been linked with chronic kidney disease and excessive potassium loss (especially in cats). Hypokalemia can also result from osmotic diuresis, correction of acidosis, or insulin-mediated cell uptake that is seen in diabetic ketoacidosis therapy. Some medications can also contribute to hypokalemia, such as furosemide, which causes excess potassium loss in the urine. Other causes include inadequate potassium intake in the diet, gastrointestinal loss, and hereditary causes; also it is often seen in cases of hepatic lipidosis.

What is the most frequently observed side effect associated with acepromazine use in geldings? Arrhythmias Paraphimosis Seizures Hypotension

Hypotension The correct answer is hypotension. Acepromazine will reliably cause hypotension. Acepromazine may cause paraphimosis in male horses, but this is uncommon. However, due to the risk of paraphimosis, many veterinarians will not use acepromazine on stallions. Acepromazine actually has anti-arrhythmogenic properties. Acepromazine does reduce the seizure threshold, but seizures are not as common as hypotension.

A 5-year-old female spayed Shih Tzu presents to your emergency hospital with a several-day history of vomiting and diarrhea. On presentation, she is dehydrated with pale and dry mucous membranes and a prolonged capillary refill time. Her heart rate is 220 bpm, and she has weak femoral pulses. What type of shock is she in? Cardiogenic shock Hypovolemic shock Anaphylactic shock Distributive shock

Hypovolemic shock Hypovolemic shock is characterized by a critical reduction in intravascular volume (severe dehydration, hemorrhage, third-space loss). The decrease in intravascular volume (preload) results in a decrease in stroke volume and subsequently cardiac output, which will eventually result in decreased tissue perfusion and oxygenation. Cardiogenic shock is the result of a decrease in myocardial contractility with subsequent decrease in oxygen delivery. It is always associated with primary heart disease. Distributive or vasogenic shock is typically secondary to sepsis and anaphylactic reaction, causing vasodilatation.

During calving, the rupture of the umbilical cord before birth causes which of the following? Hyperemia in the calf Fetal sepsis Septic peritonitis in the cow Hypoxia in the calf

Hypoxia in the calf Hypoxia is inadequate tissue oxygenation. The umbilical cord supplies oxygen from the mother's blood to the calf, therefore, if the umbilical cord ruptures, the calf will not receive oxygen and it is a life threatening emergency. Hyperemia is the increase of blood flow to different tissues in the body. Sepsis is caused by infection and does not result from a ruptured umbilical cord.

Which of the following is not considered part of the large intestine? Rectum Colon Ileum Cecum

Ileum The cecum, colon, and rectum are all part of the large intestine. The duodenum, jejunum, and ileum are all part of the small intestine.

When is the optimal time to clean surgical instruments? Immediately after surgery 15 minutes after surgery 24 hours after surgery 1 hour after surgery

Immediately after surgery

Where is the subcutaneous tissue in relation to the dermis? The subcutaneous tissue runs within the dermis Immediately below the dermis Two layers above the dermis Immediately above the dermis

Immediately below the dermis The epidermis is the outermost later of the skin. This is followed by the dermis. Underneath the dermis lies the subcutaneous tissue.

This New Caledonian Giant Gecko (Rhacodactylus leachianus) is suffering from dysecdysis. What is the major cause of this abnormality in captivity? Improper UVB lighting Excessive dietary phosphorus supplementation Improper humidity Excessive vitamin A supplementation

Improper humidity Especially in climates with very low ambient humidity, reptiles kept in an improper humidity range will often have difficulties with normal ecdysis. Owners may need to provide a "humidity box" to give these animals an area with increased humidity to help with shedding. Excessive dietary phosphorus supplementation may sometimes cause excessive bone formation or renal mineralization, but rarely is a direct cause of dysecdysis. Improper UVB lighting may affect normal calcium metabolism but does not directly relate to normal shedding. Signs of hypervitaminosis A include epidermal necrosis and sloughing. Generally, there are other skin lesions present in addition to dysecdysis (if present).

Prior to placement of an endotracheal tube, you measure the tube so you know how far in to place it. Where should your measurement be? Just past the point of the larynx Just in front of the first rib To the caudal point of the mandible In between the larynx and the thoracic inlet

In between the larynx and the thoracic inlet

You have assisted with multiple biopsies on a pet. You are asked to package up the samples for submission. You have taken several samples including GI tract, liver, and skin. What is the best way to package these? In 0.9% saline with a 5:1 ration of saline to tissue In formalin with a 10:1 ratio of formalin to tissue In a sterile container with no additives In isopropyl alcohol enough to cover over the tissue

In formalin with a 10:1 ratio of formalin to tissue Formalin is used to preserve the integrity of the cells for biopsy. A 10:1 ratio of formalin to tissue is recommended.

You are reading a report from an oncologist and read that the popliteal lymph nodes were enlarged. Where are these nodes located? On either side of the neck just under the jaw rostral to the salivary glands At the chest area over the pectoral muscles Inside each thigh at the inguinal area In the caudal portion of the hindlimbs at the level of the stifle and back of the thigh

In the caudal portion of the hindlimbs at the level of the stifle and back of the thigh

In the typical life cycle of the flea (i.e. Ctenocephalides felis), where does the larva pupate and form into the adult flea? In a tapeworm intermediate host In the environment (off of the host) On the surface of the host animal In the gastrointestinal tract of the host animal

In the environment (off of the host) In the flea life cycle, the adult female feeds, mates, and lays eggs on the host, but the eggs are smooth and usually fall off into the environment, often where the host typically lies down. After several days to weeks, the eggs hatch into larvae that feed on skin debris or other organic matter in the environment and then create a cocoon and pupate. The pupa then develops over 10 days or can remain in the environment considerably longer before emerging as an adult flea. This fact is important because this is why treating the host for fleas is not enough to eradicate a flea problem, as the environment is frequently contaminated and can re-infest the host after treatment.

Choose the answer choice with three methods to INCREASE radiographic density (blackness of film). Increase energy of X-rays; decrease the number of photons; increase distance from source to film Decrease energy of X-rays; increase the number of photons; decrease distance from source to film Increase energy of X-rays; increase the number of photons; decrease distance from source to film Decrease energy of X-rays; increase the number of photons, increase distance from source to film Increase energy of X-rays; increase the number of photons; increase distance from source to film

Increase energy of X-rays; increase the number of photons; decrease distance from source to film

Which of the following is expected from a positive inotropic drug? Dramatic decrease blood pressure Prevention of arrhythmias Decrease in adrenal gland stimulation Increase in cardiac contractility

Increase in cardiac contractility

Which of the following can be easily changed by a technician to increase the amount of exposure on a radiograph without affecting the amount of contrast? Increase milliamperage Decrease field size ("cone down") Increase kilovoltage Decrease time of exposure

Increase milliamperage The settings that affect exposure without changing contrast are milliamperage and exposure time. Increasing either will increase the exposure of the film. Changing kilovoltage changes the amount of contrast (higher kVp gives less contrast, more latitude and lower kVp gives greater contrast but less latitude). Decreasing field size will decrease the amount of scatter and may improve image quality but it will not increase exposure to the film.

You draw blood on a 5-year-old female spayed Schnauzer. When you spin down the blood, the serum looks milky. What causes this? Increased blood lipids Elevated bile acids Increased blood glucose Hemolysis of red blood cells

Increased blood lipids Elevated blood lipids or fats cause a milky or white appearance to the serum. It can be seen most commonly in pets that are not fasted or pets with a predisposition for hyperlipidemia (such as Schnauzers).

Which of the following is a classic symptom of pyometra? Seizures Syncope Hives Increased drinking and urinating

Increased drinking and urinating Pyometra is an infection in which the uterus fills with pus. It is most commonly caused by E. coli. It is an emergency in which ovariohysterectomy must be performed. Clinical symptoms include decreased appetite, lethargy, fever, and polyuria/polydipsia.

Which of the following may be a sign of pain in dogs? Increased heart rate Vomiting Hypotension Decreased respiratory rate

Increased heart rate Signs of pain can include increased heart and respiratory rates, increased panting, trembling, whimpering, etc. Pain would likely make the blood pressure go up, not cause hypotension. Pain is not likely to cause vomiting in dogs.

Which of the following statements is most correct about indicator tape? Indicator tape should be placed on the outside of a wrap Indicator tape should be placed on the inside of a wrap Indicator tape is only used if a sterilization strip is not used Indicator tape should never be used on linens

Indicator tape should be placed on the outside of a wrap

Oscillometric or Doppler equipment is a way of measuring what parameter? Oxygen saturation Central Venous Pressure Hematocrit Indirect blood pressure Blood glucose

Indirect blood pressure Oscillometric and Doppler machines measure indirect blood pressure and are often used in the clinical setting. These methods are non-invasive and easy to perform. (The Dinamap is an example of an oscillometric machine.) Direct blood pressure measurements are taken when an animal has an arterial line in place; they are typically used only in patients that are critical and undergoing major surgery.

Which of the following delays wound healing? Jagged edges Bed of granulation tissue Increased blood supply Infection

Infection Bacteria and inflammation from infection cause direct cellular damage. This can cause separation of tissues, which leads to infection and delays healing time. Granulation tissue is a normal part of healing unless it becomes exuberant. Other factors which delay healing include corticosteroid administration or immunosuppression, poor blood supply, and hypoproteinemia.

A "UMIC" would be performed on a urine sample to check for which of the following? Ketonuria Infection and antibiotic susceptibility Diabetic regulation level Crystalluria

Infection and antibiotic susceptibility A UMIC is a urine minimum inhibitory concentration. This is the lowest concentration of an antimicrobial that will inhibit the visible growth of a microorganism after incubation. It basically indicates which bacteria are present and to which antibiotics those organisms are susceptible.

A dog has a high fever, is lethargic, and is diagnosed with peritonitis. What does this dog have? Inflammation of the pleura Fluid in the pericardial sac around the heart Inflammation of the internal abdominal lining A ruptured bladder

Inflammation of the internal abdominal lining The peritoneum is a membrane that lines the abdominal cavity. It also covers most of the abdominal organs. The peritoneum helps support the organs and helps to facilitate blood and lymph flow to these organs. Peritonitis is inflammation ("itis") of the peritoneum. The most common causes of peritonitis include infection (such as a leak in the GI tract or a ruptured gall bladder). In cats, FIP causes a viral peritonitis. Fluid in the pericardial sac is termed pericardial effusion. The pleura is the tissue that protects the lungs. This is a photo of a dog with an intestinal perforation, which resulted in peritonitis.

The clinical condition of "founder" in the horse refers to what? Inflammation of the distal sesamoid bone of the hoof and subsequent crushing of the bone Inflammation of the laminae of the equine hoof and subsequent rotation of the third phalanx Inflammation of the laminar joint of the hoof and subsequent lameness Inflammation of the coronary band of the hoof and subsequent abscess formation

Inflammation of the laminae of the equine hoof and subsequent rotation of the third phalanx

What is the etiology of Clostridium botulinum, the toxin that causes botulism in cattle? Contamination of deep puncture wounds Ingestion Trauma Direct contact

Ingestion C. botulinum is found in decomposing plants and animals, and toxins are produced in the decaying material. The most common source of the toxin for cattle is feed contaminated by carcasses of mice and birds. Botulism is characterized by a progressive muscle weakness (paralysis). Affected animals may be weak, stagger about, or go down. Cattle characteristically display flaccid paralysis and occasionally protrusion of the tongue as seen in this photo.

Which of the following is most appropriate when placing a urinary catheter in a dog or cat? Insert the catheter until urine flows, then advance 1 cm further and secure Insert the catheter no more than 3 inches in any pet, whether urine flows or not Do not use lubrication, as this can irritate the inside of the urethra Insert the catheter until resistance is met

Insert the catheter until urine flows, then advance 1 cm further and secure

What is the primary reason a clinician collects an arterial blood sample (as compared to venous blood sample) when performing a blood gas analysis? Interested in knowing the oxygenation of the patient Interested in knowing if the patient has hypercapnea Interested in knowing the patient's bicarbonate (HCO3) concentration Interested in knowing if the patient has an acidemia or alkalemia Interested in knowing the patient's anion gap

Interested in knowing the oxygenation of the patient There will be mild-to-moderate differences in CO2, HCO3, and other variables between an arterial and venous blood gas sample. However, one of the primary reasons to collect an arterial sample is to evaluate how well the patient is oxygenating (patient's partial pressure of oxygen in blood). Thus, one of the main indications to perform an arterial blood gas is to evaluate the respiratory system of the patient.

Which of the following statements is true regarding radiography of the vertebrae? Intervertebral disc spaces appear wider toward the center of the film and narrower towards the end of the film It is not possible to evaluate intervertebral disc spaces without myelography Normally, the intervertebral discs are more radiodense that the vertebral bones Intervertebral disc spaces appear narrower toward the center of the film and wider towards the end of the film

Intervertebral disc spaces appear wider toward the center of the film and narrower towards the end of the film Due to divergence of the x-ray beam, intervertebral disc spaces will appear wider toward the center of the film and narrower towards the end of the film. This must be taken into account when evaluating the disc space on a vertebral film. A myelogram is useful for assessing the presence of spinal cord compression but does not improve the evaluation of intervertebral disc spaces. Intervertebral discs are much less dense than the bones unless they are abnormally mineralized.

An anthelmintic is used for treating which of the following? Ticks Intestinal parasites Fleas Mange

Intestinal parasites Anthelmintics are drugs used for expulsion of intestinal worms. Ivermectin, praziquantel, fenbendazole, pyrantel, etc. are a few examples of anthelmintics.

A very tiny kitten presents with severe dehydration and is hardly responsive to handling. You are unable to place a peripheral IV catheter. Which of the following routes could be used as a fast or most efficient way of administering fluids and medications to such a small animal? Intraosseus Orogastric tube Subcutaneous Intraperitoneal

Intraosseus Intraosseus fluid administration is delivery of fluid directly into the bone/bone marrow. It can be very beneficial and life saving in a tiny animal with poor peripheral veins when a catheter cannot be placed. Intraperitoneal means inside the abdominal cavity; this route has very slow absorption. Oral/orogastric absorption is very slow, especially in an animal that is obtunded; this may put the patient at an increased risk for aspiration. Subcutaneous fluid administration could be used but may be poorly absorbed in a patient this dehydrated and is not the most efficient.

A 2-year old male Golden Retriever has been brought to your facility in cardiac arrest. Obtaining intravenous access will be extremely difficult due to degloving injuries on all legs and additional trauma to the head and neck. What alternate route would be the first choice for atropine and epinephrine administration? Rectal Subcutaneous Intratracheal Intramuscular Intracardiac

Intratracheal Both atropine and epinephrine can be administered via the endotracheal tube (intratracheal). Three times the intravenous dose must be administered. Intracardiac administration can be used; however, it would not be the first alternate choice as significant cardiac trauma can be caused. Subcutaneous and intramuscular administration both are too slow for resuscitation drugs as it can take up to 30 minutes for drugs to reach the blood stream via these routes. Rectal administration of these medications is not beneficial.

During an emergency you are assisting with CPCR. What is the best way listed to administer the drugs (atropine, epinephrine, and lidocaine)? You are unable to place an IV catheter despite multiple attempts. Subcutaneously Intratracheally Via stomach tube Intraperitoneal

Intratracheally Drugs are rapidly absorbed via the intratracheal route. Establishing an airway is the second step (see below) in CPCR, so a polypropylene urinary catheter or red rubber feeding tube can be placed inside the endotracheal tube; the drug can then be injected through the tube followed with several mLs of sterile saline to help with distribution. The other methods listed are not appropriate for administration of emergency drugs as they would be absorbed at a very slow rate and absorbed very poorly due to poor circulation during cardiac arrest. *Establishing an airway was previously the first step in CPCR. However, the current guidlines put "Circulation" above "Airway", so that chest compressions are to be started immediately even prior to placing the endotracheal tube. (Obviously the endotracheal tube needs to also be placed ASAP)

The doctor tells you that a medication is only to be given parenterally. A parenteral drug is given via which route? Topically Orally Rectally Intravenous

Intravenous Parenteral means administration NOT via the gastrointestinal tract (not orally or rectally). Parenteral drugs are given via injection. Intravenous, subcutaneous, and intramuscular injections are all parenteral routes of administration. In the strictest sense of the word, inhalation or topical applications could be considered parenteral since they are not given through the GI tract, but they are generally not regarded as such.

How is total parenteral nutrition (TPN) administered? Jejunostomy tube Intravenous Esophagostomy tube PEG tube

Intravenous Total parenteral nutrition (TPN) is feeding exclusively intravenously, bypassing the GI tract. It is very useful in cases when animals are unable to eat or will not eat for an extended time period. The other choices listed are all feeding tubes that enter the GI tract. Esophagostomy tubes go into the esophagus, PEG tubes go into the stomach, and jejunostomy tubes go into the jejunum (intestine).

Two technicians are inducing anesthesia on a cat for an ovariohysterectomy. The veterinarian is not present in the room. The cat becomes apneic. In what order should steps be taken to intervene? The cat should be: Intubated, manually ventilated with room air, veterinarian should be called Start flow by oxygen, call veterinarian Call veterinarian and do not start any treatments until the doctor arrives Veterinarian should be called, start on flow by oxygen Intubated, manually ventilated with 100% oxygen, veterinarian should be called

Intubated, manually ventilated with 100% oxygen, veterinarian should be called

A 6-month old puppy has presented to the hospital with a history of severe straining to defecate. A fecal float identifies a severe parasitic infection. An abdominal ultrasound was performed, and the doctor identified that one of the loops of intestine was telescoping into another segment of intestine. What is this called? Strangulation Intussusception Hydrometra Evisceration

Intussusception

What three parts make up the uveal tract in the eye? Sclera, iris, pupil Iris, choroid, ciliary body Ciliary body, iris, lens Iris, lens, conjunctiva

Iris, choroid, ciliary body The uvea is made up of the iris, choroid, and ciliary body. It is the middle layer of the eye. The uvea provides most of the blood supply to the retina. Uveitis is inflammation of this part of the eye.

Which nutrient must piglets be supplemented with? Potassium Calcium Folate Iron

Iron Sow's milk does not contain iron. This must be supplemented to prevent anemia in piglets.

A low packed cell volume could occur due to which of the following? Hyperkalemia Iron deficiency Low phosphorus Iron toxicity

Iron deficiency A low packed cell volume indicates anemia. Iron deficiency is also known as hypochromic microcytic anemia. It is most often suspected in patients with chronic blood loss but could also result from poor nutrition.

Which toxicity is not a common concern in cattle? Salt toxicity Iron toxicity Aspergillus toxicosis Lead toxicity

Iron toxicity Iron toxicity is most common in newborn pigs. Aspergillus toxicosis is caused by moldy feed. Salt toxicity is caused by consumption of excessive salt and lack of adequate amounts of water. Lead toxicity is associated with seeding and harvesting activities when used oil and battery disposal from machinery is handled improperly.

A dog is hit by a car and presents in a Schiff-Sherrington posture. Which is true regarding this condition? The lesion is located at C1-C5 It causes forelimb paralysis It causes paralysis of the hindlimbs It causes hindlimb hypertonicity It is caused by a brain lesion, usually from trauma

It causes paralysis of the hindlimbs The Schiff-Sherrington response is an autonomic reflex that is seen when the animal is in lateral recumbency (lying on its side). The propriospinal tract in the T3 to L3 spinal cord is damaged in this condition, and the forelimbs exhibit extensor hypertonicity (they are stiff). The forelimbs are not paralyzed but, when left alone, will extend. Also, paralysis caudal to the lesion is often seen (hindlimbs are paralyzed). If both forelimb extensor hypertonicity and hindlimb paralysis are seen, it is generally due to a very severe spinal cord injury described as causing Schiff-Sherrington posture.

What does polyvalent mean when talking about a vaccine? It contains multiple antigens It provides immunity for greater than 3 years It may be given to multiple species It has no adjuvant added

It contains multiple antigens A polyvalent vaccine contains multiple antigens. The DHPPC-L vaccine is one example of a polyvalent vaccine and, as such, it contains antigens for distemper, hepatitis, parainfluenza, parvo, corona, and leptospirosis.

Which of the following is true of aspirin? It decreases platelet clumping and thus reduces risk of clot formation It increases thrombus formation to help prevent bleeding at a site of injury It should never be given to canines It should never be given to felines

It decreases platelet clumping and thus reduces risk of clot formation Aspirin decreases platelet aggregation and so reduces risk of clot formation. Because of this, it can also increase bleeding risk. Although cats are very sensitive to the effects of aspirin, it is sometimes used when treating certain types of heart disease. It should be used with caution, since cats cannot metabolize aspirin as quickly as dogs can, and owners should be advised to never give their cat aspirin.

Which of the following is true of tungsten, making it an ideal material for the filament and target of an x-ray machine? It has a very high melting point It is a good conductor of heat It has a low atomic number, making it more efficient at producing x-rays It has a low number of electrons

It has a very high melting point Tungsten has a very high melting point (greater than 3,000 C). It has a high atomic number (74), making it a relatively more efficient target for producing electrons. It also has many electrons, which makes it more efficient at producing an electron cloud at the cathode (filament) via a process called thermionic emission. Despite being relatively efficient at producing x-rays, the process itself is still very inefficient, with more than 99% of the energy generated being converted to heat rather than x-rays. Therefore, the tungsten target is often embedded in copper (a good conductor of heat). Tungsten by itself is a poor heat conductor.

Why should an NSAID and a corticosteroid never be given together? It causes PU/PD It causes profound sedation It increases risk of serious GI ulceration It prevents absorption of nutrients from the intestine

It increases risk of serious GI ulceration Giving an NSAID and steroid together puts an animal at a very high risk for an ulcer which could perforate. This is when a hole is created in the intestine and intestinal contents leak into the abdomen, causing peritonitis. This is a very severe condition which can lead to death.

An 8-year old female spayed Dalmatian presents to your clinic for dribbling urine when sleeping. Bloodwork and urinalysis are performed, and phenylpropanolamine is prescribed. How will this drug help treat the dog's problem? It kills Gram-negative bacteria It increases the tone of the urethral sphincter It provides hormonal replacement to reverse the condition It increases re-absorption of water by the kidneys, resulting in decreased urine volume It acidifies the urine, decreasing urinary crystal formation It alkalinizes the urine, decreasing urinary crystal formation

It increases the tone of the urethral sphincter

Which of the following correctly describes a "surgeon's knot" which may be used to close tissues where tension makes it difficult to apply a regular square knot? When applied correctly, it results in moderate inversion of the tissue It is a continuous pattern It involves passing one strand through the loop twice on the first throw It is an alternative term used to describe a half-hitch knot

It involves passing one strand through the loop twice on the first throw A surgeon's knot is produced by passing one strand through the loop twice on the first throw of a square knot. It is for closure of tissues where tension on the tissues makes it difficult to apply a regular square knot. A surgeons knot can be used with many different suture patterns including interrupted sutures or the first or last knot of a continuous pattern. It can also be used with inverting or everting patterns, but the knot itself does not cause inversion or eversion. A half-hitch is a different type of knot that can be used to slide a knot down a suture line (i.e. toward a pedicle).

The doctor hands you a written prescription for spironolactone and asks you to give it to a client while they are checking out. The client asks you to remind her what this medication is for again, and you reply that: It is a corticosteroid used for treating Addison's disease It is a steroid inhibitor used for treating Cushing's disease It is an anti-inflammatory for her pet's post-operative pain It is a diuretic for her pet's congestive heart failure

It is a diuretic for her pet's congestive heart failure Spironolactone is an aldosterone antagonist and a potassium sparing diuretic used as an adjunctive treatment for heart failure. If furosemide is not enough to control the pulmonary edema, sometimes spironolactone is added on as an additional diuretic to keep fluid out of the lungs.

A Yorkie is presenting for a bile acids test and has been fasted. Which of the following is true regarding a bile acids test? It involves drawing a blood sample, giving a cosyntropin injection, and drawing another blood sample 1 hour after the injection Fasting is not necessary for this test It is a measure of liver function It is a measure of kidney function It is a measure of adrenal function

It is a measure of liver function A bile acids test is a measure of liver function. It requires fasting. A blood sample is drawn. The animal is then fed, and another sample is drawn 2 hours after feeding. The Cortrosyn (cosyntropin) injection protocol describes an ACTH stimulation test which is a measure of adrenal gland function, testing for Cushing's (hyperadrenocorticism) or Addison's (hypoadrenocorticism).

What is the cause of hip dysplasia? Poor diet Environment Breed conformation Genetics It is a multifactorial disease

It is a multifactorial disease Hip dysplasia is multifactorial. Many things such as genetics, environment, nutrition, and breed conformation play a role in the disease. Therefore, the treatment is often multi-modal. This may include an exercise plan, special diet, supplements for cartilage protection, anti-inflammatories, surgery, and/or other treatments.

Which is true of "red bag" during birth in the mare? It is a protrusion of the chorioallantois and is an emergency It indicates a breech position of the foal, and assistance for delivery is needed It is the appearance of a normal placenta as the mare is giving birth It indicates a tear of the uterus, and a c-section needs to be performed immediately

It is a protrusion of the chorioallantois and is an emergency Red bag is a protrusion of the chorioallantois and indicates premature placental separation. It is an emergency situation in which the chorioallantois needs to be broken down manually to assist in delivery of the foal, or death of the foal can occur.

Which of the following is true of the liver? It is the only source for alkaline phosphatase (ALP) It produces amylase for digestion It does not have the ability to regenerate It makes and stores glucose

It makes and stores glucose Glucose comes from liver production and from the GI tract when food is absorbed. The liver helps to break down fats, converts glucose to glycogen, produces urea (the main substance of urine), makes some amino acids, filters harmful substances from the blood, stores vitamins and minerals (vitamins A, D, K and B12) and maintains a proper level of glucose in the blood. The liver also produces cholesterol, albumin, bile, and coagulation factors. Amylase comes from the pancreas and also from saliva. The main sources of ALP are the liver and bone, but smaller amounts can be found in the intestine and kidney. Liver cells can regenerate.

If a region of the film does not come into contact with developer but does run through fixer and the rest of the processing steps, what will this region look like? It will be yellow It will be purple It will be green It will be black It will be clear

It will be clear Films go into the developer first, reducing exposed silver halide into elemental metallic silver, which is black. If a region of the film does not touch the developer, there will be no black on the film even if it was exposed. The remaining crystals would be converted into a soluble form in the fixer solution, dissolving away and leaving a clear image.

What is the name of the tick that is the primary vector for transmitting the bacteria which causes Lyme Disease? Dermacentor andersoni Rhipicephalus sanguineus Amblyomma americanum Dermacentor variabilis Ixodes scapularis

Ixodes scapularis

Of the choices below, which tick is correctly matched with the corresponding disease transmitted? Rhiphacephalus, Lyme disease Ixodes, Lyme disease Ixodes, Ehrlichia Dermacentor, Ehrlichia

Ixodes, Lyme disease The correct tick to rickettsial disease pairs: Rhiphacephalus (brown dog tick), Ehrlichia Ixodes (deer tick), Lyme disease (Borrelia burgdorferi) Dermacentor, Rocky Mountain Spotted Fever (Rickettsia rickettsii)

When performing orthopedic surgery, it is sometimes necessary to advance a pin into the medullary canal of a bone. Which of the following instruments should the surgeon be provided to advance a pin? Osteotome Mallet Jacobs chuck Periosteal Freer elevator

Jacobs chuck A chuck key is used to tighten the Jacobs chuck around the pin. Once the pin is secured, a back-and-forth twisting motion at the wrist is used to drive the pin through the medullary canal of bone. An osteotome is the orthopedic version of a chisel. A mallet is the orthopedic version of a hammer. A periosteal Freer elevator is used to elevate muscle and periosteum off the bone.

In which part of the intestinal tract is nutrient absorption the greatest? Ileum Jejunum Duodenum Colon

Jejunum The jejunum is the longest portion of the small intestine and is the site of the most nutrient absorption.

You are asked to assist the doctor with an intra-articular injection. Where will the injection be given? Bone Joint Under the skin Jugular Middle-ear

Joint Articular means joint. An intra-articular injection is given into the joint space. Jugular would be intra-venous. Bone would be intra-osseous. Under the skin would be subcutaneous.

The dog in this picture is having his blood drawn from which vessel? (Neck) Carotid artery Saphenous vein Cephalic vein Jugular vein

Jugular vein

When a blood sample is needed, the best site for venipuncture in the horse is which of the following? Jugular vein Cephalic vein Facial artery Transverse facial vein

Jugular vein The jugular vein is the best site for venipuncture in horses, especially for large blood samples. The cephalic vein and transverse facial veins can be used in debilitated horses for smaller samples of blood.

What is the most commonly used location for intravenous catheters in horses? Cephalic vein Jugular vein Lateral thoracic vein Facial vein Saphenous vein

Jugular vein The facial vein is relatively small and would be difficult to catheterize. It would also be difficult to maintain in place. The remaining answers are all possible locations to place an IV catheter, with the jugular being the most common. It is a large bore, easily-catheterized vein that allows administration of large volumes of fluid. Veins on the limbs can be catheterized but can also be difficult to maintain because of limb movement. The lateral thoracic vein is difficult to catheterize in some overweight horses but can still be placed.

When assisting with a forelimb declaw, it is helpful to place a tourniquet to help control hemorrhage. Where should the tourniquet be placed? Over the proximal humerus Just distal to the elbow Directly around the digit upon which the surgeon is working Just proximal to the elbow

Just distal to the elbow The optimal placement of the tourniquet is just below or distal to the elbow. Placing the tourniquet above the elbow can result in damage to the radial nerve, which will result in an inability for the patient to use the limb if the damage is permanent. Placing the tourniquet around the digit is unpractical and cumbersome.

Which of the following forceps is most appropriate for use in reducing a fracture? Rochester-Carmalt forceps Rochester-Oschner forceps Kern forceps Kelly forceps

Kern forceps The Kern bone-holding forceps have gripping teeth that allow them to manipulate bone fragments without slipping from the bone. Rochester-Oschner forceps are thin in comparison to Kern forceps. They have a "rat-tooth-like" end and are not suitable for manipulating bone. Similarly, Kelly and Rochester-Carmalt forceps are thin and not designed to manipulate bone.

Which of the following drugs can induce muscle rigidity, or a cataleptic state, during anesthesia? Propofol Ketamine Diazepam Acepromazine

Ketamine Catalepsy can be caused by ketamine and morphine. Ketamine (a dissociative anesthetic) is usually given along with diazepam (Valium) to counteract the muscle rigidity it causes. Ketamine can also result in increased intracranial pressure. Diazepam causes muscle relaxation. Acepromazine is a phenothiazine tranquilizer. Propofol is an anesthetic induction agent that is a hypnotic and does not cause catalepsy.

Which drug choice is a dissociative anesthetic and used for anesthetic induction and analgesia in veterinary medicine? Propofol Ketamine Acepromazine Xylazine

Ketamine Ketamine is a dissociative anesthetic often used for induction of anesthesia. It is an NMDA receptor antagonist and also binds to opioid receptors, which provides analgesic effect. It is often given simultaneously with diazepam for anesthetic inductions.

Which of the following sedative or anesthetic drugs would NOT frequently cause hypotension (low blood pressure)? Propofol Isoflurane Ketamine Acepromazine

Ketamine Ketamine's effects on the cardiovascular system include increased cardiac output, increased heart rate, and increased blood pressure. These cardiovascular effects are secondary to increased sympathetic tone. Isoflurane and other inhalant gases can cause profound hypotension. Acepromazine and propofol also very commonly cause hypotension.

Erythropoetin (EPO) is a glycoprotein hormone produced mainly by the: Spleen Pituitary Kidney Bone marrow

Kidney

Where would you find the Loop of Henle? Heart Kidney Liver Ear

Kidney The Loop of Henle is located in the kidney and is part of the nephron. The nephron is the functional unit of the kidney.

An owner has an elderly cat that has developed significant arthritis. She previously used meloxicam (Metacam) for her dog when he had arthritis. She would like to know if the doctor may prescribe the same medication for her cat. The use of metacam in cats, especially if used beyond a few days, has been known to cause which of the following complications? Gastrointestinal ulceration Kidney failure Urinary incontinence Acute hepatitis

Kidney failure Metacam may cause renal failure in cats. Even though it is still sometimes used post-operatively for a very brief period, risks should be discussed with the owner. For a cat such as this with arthritis, meloxicam is not a good choice for treatment. Any non-steroidal anti-inflammatory medication can cause GI ulceration, but renal failure is the most concerning side effect in cats.

Amphotericin B is often used in cases of resistant fungal infections, such as occurs with diseases like Coccidioides immitis. This medication is known for the potential to cause severe damage to which organ? Skin Liver Bladder Intestine Kidneys

Kidneys

The adrenal glands are closest to what other structure? Pancreas Bladder Kidneys Brain Liver

Kidneys The adrenal glands are located right above the kidneys. The bladder sits in the caudal abdomen ventral to the colon. The liver is just caudal to the diaphragm and is the most cranial organ in the abdomen. The right limb of the pancreas runs parallel with the duodenum while the left limb runs along the body of the stomach.

Which of the following is NOT a measured analyte in a urine dipstick? Ketones Protein Blood Lactate

Lactate

Which of the following is a product of anaerobic metabolism and may indicate poor blood flow to the tissues? Creatine kinase Lactate Blood glucose Alanine aminotransferase

Lactate Lactate is a product of anaerobic metabolism. Elevation of lactate may indicate hypoxia or inadequate perfusion.

Which of the following animals would have an increased energy requirement? A healthy Beagle who was spayed yesterday Geriatric Labrador Retriever Obese Cocker Spaniel Lactating Chihuahua

Lactating Chihuahua Energy requirements are lower in geriatric healthy patients because they are typically less active and sometimes have a slower metabolic rate. Any animal that is pregnant or lactating will have a higher energy requirement.

A toggle-pin fixation for a luxated hip is going to be performed this afternoon. How will the patient be ultimately positioned for surgery after it is has surgically prepped? Lateral recumbency Dorsal recumbency Hanging leg technique Ventral recumbency

Lateral recumbency

Castration is a common procedure performed in horses. How is a horse commonly positioned for the procedure? Dorsal recumbency with rear legs positioned cranially Lateral recumbency with upper rear leg tied around neck Dorsal recumbency with legs spread wide Dorsal recumbency with legs "hog" tied

Lateral recumbency with upper rear leg tied around neck The most common position for a horse castration is lateral recumbency with the upper rear leg tied around the neck. It is extremely important to confirm that both testicles are descended prior to heavily sedating the patient for the procedure.

A dog is fractious, and you need to draw blood with the dog laying on its side. You cannot access the front end of the dog. Which vein would be the best for collecting a sample? Lateral saphenous vein Auricular vein Cephalic vein Jugular vein

Lateral saphenous vein The lateral saphenous vein runs along the lateral side of the hindlimb and is most accessible below the stifle just above the tarsus. The other vessels listed are all located at the cranial end of the dog. Auricular is an ear vein; jugular is under the neck; cephalic is located in the forelimb.

When using the ultrasonic cleaner, it is important to do which of the following? Add surgical milk to the solution Run the clean cycle for at least 27 minutes Scrub the instruments while the ultrasonic cleaner is running Lay the instruments in an open position

Lay the instruments in an open position

Which of the following is a legally acceptable method to identify a radiograph in the event that the radiograph needs to be used in court? An adherent label with patient information is affixed to the film immediately after processing A permanent marker is used to write patient information on the film after the film is developed Lead letters with patient information are placed at the corner of the field during the radiograph exposure The radiograph itself does not need to be labeled, provided that it is kept in a folder that contains all patient information

Lead letters with patient information are placed at the corner of the field during the radiograph exposure Legally, radiographs must be identified during or after exposure and before processing to produce the latent image.

You are at a ranch property and are asked by the vet to tie a horse to a fence. How should the rope be tied? Leave 6 to 8 feet of slack and tie the rope level with the top of the horse's back Leave 3 feet of slack and tie the rope level with the horse's fetlock Leave 3 feet of slack and tie the rope level with the top of the horse's back Leave no slack and tie the rope level with the horse's nose

Leave 3 feet of slack and tie the rope level with the top of the horse's back Tie the rope level with the horse's back or withers (shoulder blade region) and leave approximately 3 feet of slack. Leaving more slack in the rope or tying the rope too low could increase the chances of the horse getting tangled up or injured.

A 3-year old female cow presents with a decreased appetite, decreased manure production, and decreased milk production. She calved three weeks ago and has been less active than normal. You auscultate the GI tract and elicit a pinging noise on the left side of the abdomen. Which of the following emergencies is most likely to be occurring? Mastitis Uterine prolapse Left displaced abomasum Right displaced abomasum

Left displaced abomasum Symptoms of left displaced abomasum include decreased appetite, activity, manure production, and milk production. Eliciting a pinging noise while flicking the abdominal area is an indication of a displaced abomasum. Because the pinging was found on the left, it is most likely a left displaced abomasum. Even though you would not be expected to make a diagnosis, knowledge of this common condition is helpful and may show up on the VTNE.

Which vessel should be avoided when performing venipuncture in pigs? Tail vein Left jugular vein Right cranial vena cava Auricular vein

Left jugular vein The phrenic nerve is located near the left exterior jugular vein, so the left side of the neck in general should be avoided in pigs. The right anterior vena cava is a commonly used site to draw blood from pigs. The auricular vein can be used to draw up to 5 mLs of blood. The tail vein and the orbital sinus near the medial canthus of the eye are other sites that can be used to draw small quantities of blood in the pig.

Which disease may be transmitted to humans through aerosolization or direct handling of urine from an infected patient? West Nile Virus Leptospirosis Rabies Cystoisospora

Leptospirosis Leptospirosis is caused by a spirochete bacterium and is most often transmitted through direct contact with infected urine or infected fetus/vaginal discharge. It most commonly causes kidney and liver disease and is one of the most important zoonotic diseases of concern nationwide. It tends to be diagnosed most frequently in areas of high rainfall or where there are lakes or water sources, because the organism may survive in water. Rodents and wildlife are the most common reservoir hosts.

Which drug treats hypothyroidism? Levofloxacin Levothyroxine Theophylline Methimazole

Levothyroxine Hypothyroidism is the condition of the thyroid being too low. When it is too low, the animal must be supplemented with levothyroxine. Methimazole is the drug used to treat hyperthyroidism (when the thyroid is too high). Radioactive iodine (I-131) also treats hyperthyroidism. Theophylline is a bronchodilator. Levofloxacin (Levaquin) is an antibiotic.

When performing a lameness examination in a horse, perineural anesthesia is used to localize the site of lameness. The most common medication used to perform perineural anesthesia in a horse is what drug? Phenylbutazone Flunixin meglumine Butorphanol Lidocaine Ketamine

Lidocaine

A dog is having continuous runs of ventricular premature contractions following splenectomy and they seem to be happening with greater frequency (see image of his electrocardiogram). Which of the following drugs can be used to treat this condition? Atropine Pimobendan Epinephrine Lidocaine

Lidocaine Ventricular premature complexes (VPCs) are the most common arrhythmia after GDV or splenectomy surgeries. Treatment of VPCs occurs IF: 1. They continue to occur with increasing frequency 2. The continuous heart rate is greater then 180bpm (in an adult animal) 3. R on T phenomenon is observed 4. There is evidence of decreased or impaired cardiac output- the dog is laterally recumbent and minimally responsive, poor arterial pulses, pulses are not synchronous with heart beats. If any of the above are seen beginning treatment with a lidocaine intravenously is strongly considered. If the VPCs persist, a lidocaine CRI may be necessary for up to 24-48 hours post-operatively. None of the other medications listed treat VPCs.

You are assisting with exams on 1-week old piglets. Which of the following may be used as a method of restraint? Lift the piglet by its snout Lift the piglet by its tail Lift the piglet by its rear leg Lift the piglet by its front leg

Lift the piglet by its rear leg A piglet up to 10 kg may be lifted by a rear leg. Be careful not to swing the piglet by the leg; just lift it so that you may then support the chest with your other hand when moving it. When finished with the exam or procedure, lower the piglet back down so that both front legs have contacted the ground and gently lower the back legs to the ground and release your hold.

Ketoconazole can cause toxicity most commonly to which of the organs or tissues listed? Cardiac muscle Retinas Liver Pancreas

Liver Ketoconazole can be associated with hepatotoxicity. Liver values should be closely monitored when using this medication. Fluconazole and itraconazole are more widely used because they are less hepatotoxic, but the cost is higher for these medications.

Which organ(s) most commonly break down or process medications and excrete them from the body? Pancreas and kidneys Liver and kidneys Gastrointestinal tract Gastrointestinal tract and liver Lungs and liver

Liver and kidneys The four main ways medications are excreted from the body are through the kidneys, liver, skin, and lungs. Most drugs are processed by the liver and then the kidneys. Therefore, animals with kidney or liver problems can have problems processing certain medications; this can raise the drug levels higher since excretion may be impaired.

A drug from which of the following categories will cause loss of motor function when administered during an epidural? Opioids (ie; morphine, fentanyl) Corticosteroids (ie; methylprednisolone, dexamethasone) Local anesthetics (ie; bupivacaine, lidocaine) Alpha 2 Agonists (ie; dexmedetomidine, xylazine)

Local anesthetics (ie; bupivacaine, lidocaine)

After parturition, it is normal for the bitch to have a reddish brown vaginal discharge for a few weeks. This discharge is referred to as which of the following? Pyuria Lochia Melena Meconium

Lochia

What is fluoroscein stain used for? Look for bacteria in the urine Check for tear production Look for distichia on an eyelid Look for a leak at intestinal anastamosis site Look for a corneal ulcer

Look for a corneal ulcer

You are working in a new practice that treats exotics. A young rat presents with chronic skin disease and the veterinarian performs a superficial skin scraping and then hands you the slide to examine. You have never seen this before, but you remember from your veterinary technician class that this is which of the following parasites? Flea Cheyletiella Louse Seed tick

Louse

What type of diet should a cat or dog in renal failure should be fed? Low in phosphorus and protein Low in phosphorus and high in protein Low in carbohydrates and high in protein High in phosphorus and low in protein

Low in phosphorus and protein A low-protein diet helps to decrease workload on the kidneys. This is because protein (when metabolized by the liver) leads to production of the waste product urea. The kidneys excrete urea. If the kidneys are not functioning properly, then urea is not excreted efficiently and begins to build up in the peripheral blood and causes the pet to feel sick clinically. Phosphorus is removed from the body via the kidneys. If filtration is impaired, then the phosphorus begins to climb higher in the peripheral blood. Therefore, a diet low in phosphorus is ideal. A low-carbohydrate and high-protein diet would be more ideal for a diabetic patient and is not good for a patient in renal failure.

Which diet is the most appropriate for a cat in chronic renal failure? High carbohydrate Low fat High protein Low protein

Low protein To filter and eliminate the by-products of protein digestion (like creatinine), the kidneys must work harder. A diet lower in protein will decrease the workload on the kidneys as they process and eliminate waste products produced during protein metabolism. Many higher protein diets are also higher in phosphorus, which should be restricted in pets with kidney failure. Protein restriction is more important in pets that have proteinuria as a result of renal failure.

A dog presents with scabies. You are instructed to give the dog a lime-sulfur medicated dip. Which of the following must you do? Deep condition the dog 15 minutes after the dip has been applied Lubricate the dog's eyes with protective ointment Quarantine the dog for 48 hours before the first dip Place a towel or E-collar around the dog's neck to help prevent any solution from contacting the dog's face

Lubricate the dog's eyes with protective ointment Dips can cause serious irritation and even damage eye tissue; protective ointment is mandatory.

Where would you collect cerebrospinal fluid (CSF) from a standing sedated horse? Lumbosacral space Temporohyoid space Atlantooccipital space Cervical-thoracic space

Lumbosacral space There are two sites that are routinely used to collect CSF from horses: the lumbosacral space and the atlantooccipital space. In the standing horse, the lumbosacral space is the only space that can be used. CSF is collected from the atlantooccipital space only in the anesthetized horse.

A dog is having a mass removed from his side. What is the general term for this procedure? Buldgectomy Massectomy Tumorectomy Lumpectomy Mastectomy

Lumpectomy A lumpectomy is the proper term for mass removal. A mastectomy is removal of a mammary gland. The other terms listed are not real terms used in veterinary medicine.

A kitten presents to the emergency service for burns in the mouth after biting into an electrical cord. Aside from the oral burns, what organ system would you most be concerned about? Brain Kidneys Heart Lungs

Lungs In addition to causing painful burns in the mouth, most pets will develop non-cardiogenic pulmonary edema when electrocuted. The fluid in the lungs can be life threatening and most of these pets will need extended hospitalization with oxygen and careful monitoring.

A shock organ is the organ or tissue that exhibits the most marked response to an allergic reaction. What is the shock organ in the cat? Lungs Gastrointestinal tract Liver Heart

Lungs In the cat, the shock organ is the lungs. In the dog, it is the liver. Both species often also have gastrointestinal involvement (vomiting, diarrhea).

Which hormone is the trigger for ovulation and development of the corpus luteum? Follicle stimulating hormone (FSH) Luteinizing hormone (LH) Progesterone Testosterone

Luteinizing hormone (LH) Luteinizing hormone is produced by the anterior pituitary gland. The LH surge is the trigger for ovulation and development of the corpus luteum. Progesterone levels increase after ovulation has already occurred.

Borrelia burgdorferi is the cause of which disease? Ehrlichia canis Leptospirosis Lyme disease Cat scratch fever

Lyme disease Borrelia is a spirochete bacterium that causes Lyme disease. It is transmitted by the Ixodes tick (typically a deer tick). Ehrlichia is a rickettsial disease transmitted by the brown dog tick. Leptospirosis is also a spirochete bacteria transmitted by Leptospira. In veterinary medicine it most commonly transmitted through contact with urine from an infected animal. The causative agent for cat scratch disease is the bacteria Bartonella henselae.

What is the disease Bovine Spongiform Encephalopathy also known as? Polioencephalomalacia Mad Cow Disease Hardware disease Black Leg Disease

Mad Cow Disease BSE is most commonly known as Mad Cow Disease and is a neurodegenerative prion disease in cattle. It causes a spongy degeneration in the brain and spinal cord and is fatal. Polioencephalomalacia is a central nervous system condition that is ultimately a result of thiamine deficiency. One cause may be by an enzyme that destroys thiamine. Black Leg in cattle is caused by Clostridium chauvoei and is an acute febrile disease causing emphysematous swelling in the heavy muscles. Hardware disease is a common term for bovine traumatic reticulopericarditis. It is caused when the cow swallows a sharp, heavy metallic object that falls to the floor of the rumen and is pushed forward into the reticulum, where it can penetrate into the pericardium.

You are using a Bain circuit to anesthetize a ferret for a spay procedure. Which of the following measures is the most important thing to do to prevent re-breathing of carbon dioxide by the ferret? Maintain high flow rates of oxygen in the circuit Make sure the pop-off and one-way valves are opening correctly Make sure the soda-lime canister is fresh Make sure that the endotracheal tube extension is longer than the ferret

Maintain high flow rates of oxygen in the circuit A Bain circuit is a non-rebreathing system (in contrast to circle systems used more commonly in animals over 5kg). Therefore, there is no carbon dioxide absorber such as soda lime. Instead, you must use sufficient oxygen flow rates to constantly flush the system (typically at least 100 ml/kg/min). The system is not dependent on valves like a circle system is. A long endotracheal tube will increase re-breathing and should not be used.

The parathyroid gland is responsible for which of the following? Releasing catecholamines Making insulin Causing milk letdown Maintaining blood calcium level

Maintaining blood calcium level The parathyroid gland is responsible for regulating and maintaining blood calcium levels. The four parathyroid glands are located on or near the thyroid gland. When parathyroid hormone is released into the blood, it circulates to act in a number of places to increase the amount of calcium in the blood (ie. removing calcium from bones). When the calcium level in the blood is too high, the cells of the parathyroids make less parathyroid hormone (or stop making it altogether), thereby allowing calcium levels to decrease. Catecholamines (epinephrine and norepinephrine) are released by the adrenal medulla. Insulin is made by the pancreas. Milk letdown and uterine contractions are stimulated by oxytocin, which is released by the pituitary gland.

What does a lysosome do? Packages proteins and lipids Makes energy (ATP) Makes enzymes to break up cellular debris Provides the cytoskeleton for the cell

Makes enzymes to break up cellular debris A lysosome is a spherical organelle that produces enzymes which break up cellular debris that has been taken up by the cell. Lysosomes digest excess or old organelles, food particles, and viruses or bacteria. Mitochondria are responsible for making energy. The Golgi apparatus packages proteins and lipids after they are synthesized.

Which of the following retractors would you hand a surgeon if he/she needs to be able to retract the liver out of the way? Balfour retractor Malleable retractor Gelpis retractor Senn Rake retractor

Malleable retractor Malleable retractors are bendable in multiple directions and have no sharp edges. This makes them ideal to use in areas such as the abdomen to gently push delicate organs out of the way. Senn rake retractors are small hand-held retractors that are usually used to retract small muscle bellies, tendons, or ligaments and thus would not be appropriate for the retraction of a liver lobe. Gelpis retractors are self-retraining retractors with sharp points. They should never be used to retract abdominal organs. Balfour retractors are only used to retract the body wall and would not be used inside the abdomen to retract a liver lobe.

In normal occlusion, what is the proper position of the incisors and canine teeth when the mouth is closed? Mandibular incisors are buccal to the maxillary incisors, and the mandibular canine is mesial to the maxillary canine Mandibular incisors are palatal to the maxillary incisors, and the mandibular canine is distal to the maxillary canine Mandibular incisors are palatal to the maxillary incisors, and the mandibular canine is mesial to the maxillary canine Mandibular incisors are buccal to the maxillary incisors, and the mandibular canine is distal to the maxillary canine

Mandibular incisors are palatal to the maxillary incisors, and the mandibular canine is mesial to the maxillary canine In normal occlusion, the mandibular incisors are palatal (behind) the maxillary incisors, and the coronal third of the mandibular incisors rests on a smooth convex bulge on the maxillary incisors known as the cingulum. The mandibular canines should be centered between the maxillary third incisor and the maxillary canine (mesial to the maxillary canine), without touching either tooth.

Which medication is typically given through a filter? Dexamethasone Mannitol Metoclopramide Cefazolin

Mannitol Mannitol forms crystals in solution. An in-line 5 micron IV filter or filter needle is recommended. None of the other choices require a filter for administration. Blood products are frequently filtered prior to IV administration.

A client rushes in her puppy which was accidentally dropped and received a trauma to the head. Which medication is an osmotic diuretic and may be used to help decrease intracranial pressure in a situation like this? Dobutamine Diazepam Mannitol Dexdomitor

Mannitol Mannitol is an osmotic diuretic that is often given to help decrease intracranial pressure in cases of trauma or seizures. It may also help to decrease intraocular pressures in acute glaucoma cases by dehydrating the vitreous humor. It also has some renal vasodilation properties and is sometimes given in cases of oliguric renal failure to help stimulate urine production. The other medications listed are not osmotic diuretics and would not be used in this situation. Dexdomitor (dexmedetomidine) is an alpha-2 adrenoreceptor agonist and is often used for sedation. Diazepam is often used to stop seizure activity and causes muscle relaxation. Vasopressors, such as dobutamine, cause vasoconstriction and work to increase blood pressure.

Which medication may help in a case of acute glaucoma? Ketamine Atropine Chloramphenicol Mannitol

Mannitol Mannitol is an osmotic diuretic that will help to decrease intraocular pressure. Atropine raises intraocular pressure and would be contraindicated. Chloramphenicol is an antibiotic, and ketamine is an NMDA receptor antagonist and dissociative anesthetic. Neither of these are indicated in glaucoma.

Which drug could be used to decrease intracranial pressure? Mannitol Phenobarbital Mitotane Ivermectin

Mannitol Mannitol is an osmotic diuretic. It can reduce intraocular and intracerebral pressures. It can also help to reduce edema and enhance urinary excretion of some toxins. This drug can also crystallize, so a filter needle should be used.

A hemocytometer is used or needed for which of the following tasks? Manual blood cell count Serum chemistry analysis Hemoblobin concentration determination Assessment of leukocyte morphology Assessment of erythrocyte morphology

Manual blood cell count

What is the most common method of pregnancy diagnosis in cattle? Progesterone test Ultrasonography Skin fold test Manual rectal palpation

Manual rectal palpation Manually palpating for a pregnancy is the most common method in cows and can be done as early as 30 days. Ultrasound is becoming more popular for cows but is not the most common. Progesterone tests can detect presence of a corpus luteum, but sometimes a CL can be present while the cow is not pregnant. This is not a reliable method. The skin fold test is the test in cattle for tuberculosis.

Which animal is seasonally polyestrous and is not an induced ovulator? Queen Mare Sow Bitch

Mare Mares are seasonally polyestrous, so they repeatedly cycle during the breeding season. This season occurs around the period when the days are longest (which is late March through September). The estrous cycle of the mare is about 21 days in length. During estrus, the mare is receptive to the stallion and lifts her tail and everts the clitoris, which is called "winking".

A canine patient presents for his annual exam. The owner states they will be taking a road trip and "Bailey" gets extremely car sick. What medication has been FDA approved for motion sickness in dogs? Metoclopramide (Reglan) Meclizine (Dramamine) Maropitant (Cerenia) Diphenhydramine (Benadryl)

Maropitant (Cerenia) Cerenia is the only medication that has been approved for motion sickness in dogs. The other medications have been used for similar purposes, but don't work as well for motion sickness specifically in dogs.

Which of the following cells release histamine? Mast cells Lymphocytes Platelets Sebaceous cells Neutrophils

Mast cells

Which of the following is not a typical constituent of a urine sediment sample? White blood cells Mast cells Red blood cells Squamous cells

Mast cells Mast cells would not be expected in a urine sediment sample. Squamous cells, transitional cells, and renal cells may be found in urine sediment. Squamous cells will likely be the largest cell visualized in a urine sample. They are usually derived from the vagina, vulva, or urethra. Red blood cells are a normal finding when in small numbers. Excessive red blood cells may be a sign of disease. White blood cells should also be present in a small number and large numbers of these cells are associated with disease.

If hypochromasia is identified in a blood smear, which value on a complete blood count would one anticipate to be decreased? Mean corpuscular volume Mean corpuscular hemoglobin concentration Red cell distribution width Red blood cell count

Mean corpuscular hemoglobin concentration Color in a red blood cell is attributed to the presence of hemoglobin. Cells that lack a normal amount of hemoglobin are said to lack pallor or color. Mean corpuscular hemoglobin concentration is a measure of the amount of hemoglobin present in the average red blood cell. Mean corpuscular volume is a measure of the average volume of a red blood cell. Red cell distribution width is a measure of the average variation in size present in a sample.

You are asked to place a nasal oxygen tube in a dog. What is the measurement for placement of nasal oxygen? The catheter should be placed 0.5 inches into the nostril Advance the tube until the dog coughs and then pull back slightly Measure to the medial canthus of the eye Measure to the top of the head

Measure to the medial canthus of the eye A topical anesthetic (such as proparacaine) is instilled into a nostril prior to catheter placement. The catheter is lubed and slid into the nostril and introduced into the ventral nasal meatus. It is advanced to the level of the carnassial tooth or the medial canthus of the eye and is sutured to the skin.

Which of the following will cause a false positive fecal occult blood test? Dairy in the diet Steroid administration Diarrhea Meat based diet

Meat based diet Meat but not dairy in the diet will result in a false positive fecal occult blood test. Therefore, the test should be administered after a patient has been on a meat-free diet such as cottage cheese and rice for at least 3 days.

The cat in this picture is having an IV catheter placed into which vein? (inside back leg) Lateral saphenous Medial saphenous Inguinal Cephalic

Medial saphenous

When referring to teeth, which of the following terms describes the portion of the tooth that is in line with the dental arcade and closest to the rostral midline of the dental arch? Lingual Buccal Mesial Distal Apical

Mesial The following terms are used in veterinary dentistry to describe different positions in the mouth and aspects of the tooth: Rostral - Any structure closer to the front of the head relative to another structure Caudal - Any structure closer to the back of the head relative to another structure Buccal - The tooth surface that faces the cheek Labial- The tooth surface that faces the lips Vestibular- surface of a tooth that is directed outward toward the vestibule of the mouth which includes the buccal (cheek) and labial (lip) surfaces Facial - The surface of the tooth visible from the front (same as the vestibular surface but applies mainly to the incisor teeth) Lingual - The surface of the mandibular teeth that is adjacent to the tongue Palatal - The surface of the maxillary teeth that is adjacent to the palate Mesial - The portion of the tooth that is in line with the dental arcade and closest to the rostral midline of the dental arch Distal - The portion of the tooth that is in line with the dental arcade and closest to the most caudal aspect of the dental arch (the opposite side of mesial) Apical - The portion of the tooth closer to the tip of the root relative to another structure Coronal - The portion of the tooth closer to the crown relative to another structure

The fetlock joint of a horse refers to what structure? Carpometacarpal joint Proximal interphalangeal joint Metacarpophalangeal joint Distal interphalangeal joint

Metacarpophalangeal joint The fetlock in a horse is the common or lay term for the metacarpophalangeal joint. In the horse, the proximal phalanx (pastern bone) lies immediately distal to the third metacarpal bone (also known as the cannon bone), with which it articulates to form the condylar metacarpophalangeal joint (fetlock joint). The middle phalanx or second phalanx lies distal to the proximal phalanx, forming the proximal interphalangeal joint known as the pastern joint. The distal phalanx (coffin bone), is the most distal bone of the forelimb, and lies completely within the hoof capsule. The distal phalanx articulates with both the middle phalanx and the distal sesamoid, forming the distal interphalangeal joint known as the coffin joint.

Which type of gas is formed due to rumenal fermentation? Sevoflurane Oxygen Methane Octane

Methane Microbial fermentation of feed leads to production of methane gas. Carbohydrate digestion in the rumen (anaerobic methanogenesis) causes the highest amount of methane gas to be released.

What does MRSA stand for? Moderate Radiopaque Screen Application Modified Recumbent Standing Apposition Methicillin-Resistant Staphylococcus aureus Medical Rating of Staff Accreditation

Methicillin-Resistant Staphylococcus aureus MRSA is a resistant bacterium that is becoming increasingly difficult to treat. Most infections are nosocomial and occur in open wounds, surgical or otherwise. This is why meticulous care and attention to cleanliness of the hospital, hand washing, and sterilization of medical equipment are so important.

Which of the following medications blocks the chemoreceptor trigger zone to help reduce nausea and vomiting in small animals? Sucralfate Famotidine Metronidazole Metoclopramide

Metoclopramide The chemoreceptor trigger zone (CTZ or CRTZ) is part of the medulla that communicates with the vomiting center to initiate vomiting. Metoclopramide (Reglan) is an anti-emetic primarily used to treat nausea and vomiting in small animals. This medication blocks the chemoreceptor trigger zone. Sucralfate is a gastric "band-aid" that helps to protect and heal irritated mucosa. Famotidine (Pepcid AC) is a gastric acid reducer. Metronidazole is an antibiotic often prescribed for diarrhea or colitis.

Which antibiotic often used for diarrhea has potential to cause a neurologic side effect? Bismuth subsalicylate Tylosin Amoxicillin Metronidazole

Metronidazole Metronidazole is used frequently for treating diarrhea. It can cause neurologic symptoms in some sensitive patients or at higher dosages. The treatment for metronidazole toxicity is supportive care, including Diazepam as a main treatment. Owners with pets on higher dosages of this medication should be advised of this potential side effect. The most common use for higher doses is treatment of Giardia. Fenbendazole is often a safer option for this parasite.

A cat presents with watery diarrhea. On fecal examination, a wet mount shows motile protozoal trophozoites; a centrifugal zinc sulfate fecal flotation shows cysts of the parasite. Which of the following is the most acceptable treatment? Amoxicillin Sulfadimethoxine Praziquantel Metronidazole Pyrantel

Metronidazole The diagnostic test findings for this cat are consistent with Giardia infection. While technicians do not diagnose patients, technicians routinely do fecal tests and should be able to identify common parasites under the microscope. Metronidazole has been shown to be effective at clearing the pathogen and reducing cyst shedding of this zoonotic disease. Care should always be taken with metronidazole in kittens due to potential side effects. Fenbendazole, or Panacur, is another treatment option for Giardia, especially for dogs, but efficacy has not been extensively evaluated in cats. Amoxicillin is an antibiotic and is not used in treating Giardia. Pyrantel is used to treat hookworms and roundworms. Praziquantel is used to treat tapeworms. Sulfadimethoxine (Albon) is used for treating coccidia.

This long slender scissor is used for cutting delicate tissue. What is the name of this type of scissor? Mayo Iris Metzenbaum Littauer

Metzenbaum

A kitten presents with hair loss over the top of the head and you take a sample for DTM culture. The culture begins to turn red within two days and grows some white fuzz on the surface. You use a tape preparation to identify the organisms growing and see the following under the microscope (purple footballs). What is this? Pollen, a normal contaminant Malassezia Trichophyton Microsporum

Microsporum This is Microsporum canis, a common cause of dermatophytosis (ringworm). Trichophyton is also a cause of ringworm, though the macroconidia are much longer and more narrow. Malassezia is yeast and often appears budding or looks like bowling pins. Contaminant hyphae are commonly seen on cytology, making it necessary to confirm a positive DTM culture as ringworm by identifying the causative macroconidia microscopically.

You perform a Wood's lamp examination on a cat that has had hair loss over the bridge of his nose (see image). It causes a green fluorescence to appear. Which organism is likely the cause? Trichophyton Cryptococcus Aspergillus Coccidioides Microsporum canis

Microsporum canis In about 50% of ringworm cases caused by Microsporum canis, a Wood's lamp test will be positive. This is the only organism to cause a positive result.

72 hours have passed since performing and exploratory laparotomy. It is important to evaluate the incision on a daily basis to help detect any complications early. Which of the following would you not expect to find on a 3-day-old incision? Moderate swelling Moderate pain Moderate redness Mild serosanguinous discharge

Mild serosanguinous discharge An incision will demonstrate all the classic signs of inflammation (redness, swelling, pain, and heat). Occasionally, there may be mild serosanguinous discharge from an incision, but this usually resolves in 24-48 hours. Persistence of discharge is worrisome as this may perpetuate bacterial colonization and subsequent incisional failure.

Which method is most useful for looking for Otodectes? They are visible with the naked eye Sedi-stain Hematoxylin and Eosin stain Mineral oil swab slide Knott's method

Mineral oil swab slide Otodectes cynotis is the ear mite. It is most easily observed by taking debris from the ear from a swab and making a mineral oil smear on a slide for observation under the microscope. Knott's is used in looking for microfilariae.

Horner's syndrome can cause which of the following clinical symptoms? Exophthalmos Hind limb weakness Miosis Mydriasis

Miosis Horner's syndrome can occur as a result of damage along the sympathetic trunk, especially as the trunk goes through the middle ear. Symptoms include miosis (constricted pupil), ptosis (drooping of the upper eyelid), enophthalmos, and third eyelid protrusion. At least three of these signs must be present to confirm the diagnosis. Weakness of the thoracic limb can also be present on the affected side.

Which of the following medications would be beneficial to a patient who is being treated for an NSAID overdose? Misoprostol Lactulose Potassium Cortisone

Misoprostol Misoprostol is a protective prostaglandin used to treat overdoses of NSAIDs, and helps to provide protection to the gastric mucosa. Overdoses of NSAIDS would cause a high incidence of gastric damage or ulceration. Other concerns would be liver or kidney damage. Potassium would not provide any protection against these effects. Cortisone (or any type of steroid) would be contraindicated in a toxicity such as this as it may exacerbate the signs. Lactulose is a stool softener and would not be beneficial to the GI tract.

"Scabies" in dogs generally refers to skin disease caused by which type of organism? Mite Louse Bacterium Protozoa Flea Fly larva

Mite Scabies refers to infestation with Sarcoptes scabiei var. canis, which is a skin mite that can cause severe pruritus and skin lesions. It is usually seen in dogs and rarely in cats. It may also cause less severe lesions in humans.

What heart valve is responsible for separating blood between the left atrium and left ventricle? Aortic valve Mitral valve Tricuspid valve Pulmonary valve

Mitral valve

Which of the following components of a digital radiography unit has the biggest influence on image quality? Server processor Monitor Workstation processor Workstation graphics card

Monitor A PACS system consists of a server, a workstation, and a viewing monitor. The workstation receives the images and sends them for display on the monitor. The monitor is the most critical to image quality. The other components have a bigger influence on how quickly images may load. A high quality graphics card is more important for viewing movie loops of ultrasound or fluoroscopy images, which require more processing power but are inherently lower resolution.

Which of the following is a phagocytic cell of the immune system? Natural Killer (NK) cell B-Lymphocyte Monocyte Mast cell

Monocyte

Which organism causes pinkeye in cattle? Moraxella bovis Dermatophilus congolensis Bovine herpesvirus Arcanobacterium pyogenes

Moraxella bovis Moraxella bovis is the bacterium that causes bovine keratoconjunctivitis (pinkeye) in cattle. It is transmitted from one bovid to another via flies, fomites, or direct contact. There are many different types of bovine herpesvirus that can result in different types of presentations. BHV-1 also known as infectious bovine rhinotracheitis virus can occur concurrently with moraxella bovis, complicating the disease. Arcanobacterium pyogenes can cause wound infections and mastitis. Dermatophilus congolensis contributes to "rain scald"

A degenerative left shift implies which of the following? Less monocytes than lymphocytes More eosinophils than lymphocytes More neutrophilic bands than normal neutrophils More lymphocytes than neutrophils

More neutrophilic bands than normal neutrophils A degenerative left shift specifically refers to the neutrophil count. In this case, there are more neutrophilic bands present than neutrophils. In a regenerative shift there are more normal neutrophils present than neutrophilic bands. Bands have a smooth nucleus with parallel sides, while mature neutrophils have a twisted nucleus.

What is normal tear production for a dog when performing a Schirmer Tear Test? More than 25 mm/min. Less than 15 mm/min. More than 40 mm/min. More than 15 mm/min.

More than 15 mm/min. Normal tear production is more than 15 mm/min. on a tear test strip. Low tear production is termed "dry eye" or keratoconjunctivitis sicca (KCS). Excessive tearing clinically is termed "epiphora".

You are asked to collect a urine sample for urinalysis on a patient while it is boarding for the week. What would be the best time to collect this sample? Late evening Time of collection does not matter Mid-day Morning

Morning

Which of the following analgesics most often causes vomiting as a side effect in dogs? Fentanyl Meloxicam Butorphanol Morphine

Morphine Morphine is a potent opioid used to provide analgesia and sedation. A common side effect of this medication, especially when given as a pre-medication, is vomiting.

A 4-year old male intact Jack Russell is recovering post-operatively after a lung lobectomy following a coyote attack. The dog is very painful. Which of the following combinations of medications may be given as a constant rate infusion and is commonly used for pain control in the critical care setting? Medetomidine, acepromazine, lidocaine Propofol, morphine, bupivacaine Morphine, lidocaine, ketamine Diazepam, ketamine, morphine

Morphine, lidocaine, ketamine Morphine, lidocaine, and ketamine (MLK), remember the word "milk," is often given as a constant rate infusion for pain in dogs. Propofol is a short acting anesthetic without analgesic properties. Diazepam is given as a CRI to control seizures but not typically for analgesia. Acepromazine also is not an analgesic and is not typically administered as a CRI.

What is the vector for transmission of heartworm disease? Flea Tick Mosquito Fly

Mosquito

What is the usual vector of western equine encephalitis? Soft ticks Mosquito Lice Biting flies

Mosquito The correct answer is mosquitoes. Culex mosquitoes as well as others are the important vectors of western, eastern, and Venezuelan equine encephalitis.

A vessel is bleeding during surgery, and the surgeon uses this tool to clamp the vessel. Which tool is this? Brown-Adson forceps Carmalt clamp Olsen-Hegar needle holders Mosquito hemostat

Mosquito hemostat This small clamp is a mosquito hemostat. It is often used to clamp small bleeding vessels. Olsen-Hegar needle holders are needle holders that also have a scissor function built in to them to allow for cutting suture. Brown-Adson forceps are used for picking up tissues that do not damage easily. Carmalt clamps are a much larger clamp and are used to clamp off structures that are typically going to be excised, such as an ovarian pedicle.

A client calls and plans to breed her prized Yorkie Emmy-Lou. She just finished her heat cycle, and the client would like to breed her the next time she comes into heat. She wants to know when this will likely occur. Which of the following statements is most accurate? Dogs are induced ovulators Most dogs have their heat cycle every 6 months Most dogs have their heat cycle every 3 months Most dogs have their heat cycle every 28 to 30 days

Most dogs have their heat cycle every 6 months Most dogs have their heat cycle every 6 months. Cats are induced ovulators.

You perform a vaginal cytology of a bitch in heat. Which of the following should you normally see? No cornified cells, mostly neutrophils with a few red blood cells Mostly cornified cells No neutrophils, large number of red blood cells A few cornified cells, no red blood cells, and a large number of neutrophils

Mostly cornified cells During heat, cytology will show 90-100% cornified epithelial cells.

Horses with laminitis should be fed which type of diet? Mixed hay and high-quality grains Grains only Mostly grass hay Alfalfa only

Mostly grass hay The cause of laminitis is multifactorial; however, a diet high in grains or grain overload has been identified as a potential cause. Also, alfalfa is higher in energy and protein, and consumption of this hay alone may attribute to laminitis in some horses. Therefore, horses with laminitis should be fed an average or mixed hay only and no grains.

Which of the following is a reason why a tick should not be removed without an effective removal device? Partial removal of the tick may increase the risk of transmission of certain rickettsial diseases Mouthparts may be left embedded and create a focus for infection Improperly squeezing the tick may cause the animal to have an anaphylactic reaction Ticks should never be removed manually and should be allowed to fall off in time or killed with an acaricide

Mouthparts may be left embedded and create a focus for infection Incomplete tick removal can leave the mouthparts of the tick attached. This will maintain an exposed, dirty and open wound that is prone to infection.

In regards to the musculoskeletal system, what is the function of an abductor muscle? Will not allow a limb to move away from the median plane Moves a limb toward median plane Moves a limb away from the median plane Prevents rotational movement of a limb

Moves a limb away from the median plane Abduction is movement away from the median plane. Adduction is movement toward the median plane.

A 14-year old DSH has been prescribed total parenteral nutrition (TPN). What type of catheter would be most appropriate for TPN administration? Single lumen PICC line in the hind leg 18 gauge cephalic catheter Multi-lumen jugular catheter A catheter is not needed; TPN can be delivered subcutaneously 18 gauge saphenous catheter

Multi-lumen jugular catheter TPN must be administered via a central vein due to its hypertonicity. A short saphenous or cephalic catheter would not reach a central vein. TPN is never administered subcutaneously. While a single lumen PICC could be used, a multi-lumen catheter would be preferred as blood samples for blood glucose monitoring could be more easily obtained.

Valium is often given with ketamine to provide which effect? Increased heart rate Increased cardiac contractility Increased muscle tone Muscle relaxation

Muscle relaxation Ketamine can cause muscle rigidity. Valium (diazepam) is often given with ketamine to provide muscle relaxation.

Johne's disease is an intestinal infection that can lead to clinical signs of diarrhea and weight loss. It affects cattle, sheep, goats, and other species. What is the Genus of the causative agent of Johne's disease? Brucella Mycobacterium Fusobacterium Listeria

Mycobacterium The correct answer is Mycobacterium. Mycobacterium avium subsp. paratuberculosis is the cause of Johne's disease and is known to affect cattle, goats, and sheep, as well as many exotic hoof-stock. Infection usually occurs when the animals are young, but clinical signs do not show up until the animal becomes stressed, often around 2 to 3 years of age. Clinical signs are essentially chronic intermittent diarrhea and weight loss. Listeria monocytogenes causes a disease of the central nervous system. Fusobacterium necrophorum is a bacteria often isolated from footrot in cattle. Brucella abortus is a bacterium which causes abortion in cattle (known as "Bang's disease") and also causes a disease in humans known as undulant fever.

Which bacteria will not stain using the Gram staining method? Campylobacter Mycobacterium Salmonella E.coli 0157

Mycobacterium Mycobacteria are acid-fast bacteria that have a waxy coat on their cell walls (a lipoid capsule) and so they will not retain the crystal violet stain. The three types of acid-fast stains are Ziehl-Neelson, Kinyoun, and fluorochrome.

Which of the following terms describe a dilated pupil? Mydriatic Miotic Sympathetic Fixed

Mydriatic Mydriasis is the term for dilated pupils. Miosis is the term for constricted pupils. Fixed pupils are not moving (not dilating or constricting). The sympathetic nervous system is responsible for pupil dilation, but this is not a term given to a dilated pupil.

What procedure requires the use of contrast (dye)? CT scan Ultrasound Myelogram Arthroscopy Magnetic Resonance Imaging (MRI)

Myelogram

You are asked to administer an intramuscular injection to a beef cow. Where should you administer it? Neck muscles Shoulder muscles Epaxial muscles Gluteal muscles

Neck muscles Because this is a beef cow, the injection should be given in the neck muscles to avoid creating a scar or damaging tissues in the muscles that may be used as meat for human consumption. When injections are given in inappropriate locations, the meat could potentially be condemned.

A dog presents with a 1-week old wound on her side. The wound is foul-smelling and has dead tissue. What term best describes this wound? Ulcerated Necrotic Fluctuant Fistulated

Necrotic Necrosis is the death of living tissues and is used to describe lesions such as this. An ulcer is an erosion of the skin or mucous membranes. A fistula is a connection or passageway in between two areas (a hole). Fluctuant means movable or compressible (i.e. a fluctuant mass)

Dirofilaria immitis falls under which category of parasite? Cestode Nematode Trematode Protozoan Coccidian

Nematode Dirofilaria immitis is the heartworm and is a nematode (a type of roundworm). The mosquito is the intermediate host. Trematodes are flukes and require a snail as an intermediate host. Protozoans are single celled organisms (microscopic). They may take on a cyst form or may be in trophozoite form (such as Giardia). Coccidians (such as Cystoisospora) are a type of protozoan. Cestodes are tapeworms. Dipylidium, a type of tapeworm, requires a flea as an intermediate host.

You are riding along on an emergency call to assist the vet with a sick foal who was born two days ago. The foal appeared normal at birth, but is now lethargic and icteric. It has a fever of 102 F. The packed cell volume is down to 15%. Which of the following diseases leads to jaundice and anemia in young foals? Bacterial hepatitis Viral hepatitis Congenital portocaval shunt Neonatal isoerythrolysis

Neonatal isoerythrolysis This foal is hemolyzing its own red blood cells as a result of antibody it acquired in colostrum which is directed at the red cells. This is the best answer choice to explain the low hematocrit and signs of icterus. Only vigorous and effective treatments can save foals which are severely affected.

Feline toxoplasmosis most often presents with which type of symptoms? Respiratory Neurologic Cardiovascular Intestinal

Neurologic Most cats infected with Toxoplasma gondii will not show any symptoms. When disease does occur, it typically occurs when the cat's immune response is suppressed. Fever, loss of appetite, and lethargy can be symptoms. Most commonly, neurologic symptoms occur including seizures or circling. Uveitis can be seen. Toxoplasmosis is treated with Clindamycin.

A breeder of turtles calls to inquire about giving ivermectin he bought at the feed store to one of his turtles. The veterinarian advises NOT to administer ivermectin, and despite this recommendation, the breeder does anyway. What would be expected in this turtle? Regurgitation Neurologic signs Sloughing of the shell Respiratory distress

Neurologic signs The correct answer is neurologic signs. Ivermectin is highly toxic in turtles and causes paresis, paralysis, and death at low doses due to an ability to cross the blood-brain barrier.

Canine parvovirus causes which of the following laboratory changes? Neutrophilia Hemolytic anemia Thrombocytopenia Neutropenia

Neutropenia Parvoviral enteritis causes severe vomiting and diarrhea, which leads to dehydration and often sepsis and fever. The sepsis (overwhelming infection) leads to a low neutrophil count.

What is the predominant cell type seen in this fluid cytology? ( Oval with connected purple blobs) Neutrophil Lymphocyte Epithelial cell Mast cell

Neutrophil

Granulocytes microscopically evaluated on a blood smear (i.e. CBC) include which cells? Neutrophils, Macrophages, Eosinophils Neutrophils, Macrophages, Basophils Neutrophils, Macrophages, Lymphocytes Neutrophils, Basophils, Eosinophils

Neutrophils, Basophils, Eosinophils As the name implies, granulocytes have granules; thus the 3 white cell types with granules (also known as polymorphonuclear cells) are neutrophils, basophils and eosinophils. Macrophages and lymphocytes are also known as mononuclear cells.

In the United States, what type of gas is in a blue gas tank? Medical compressed air Oxygen Nitrous oxide Carbon Dioxide Nitrogen

Nitrous oxide

Which of the following blades is the most commonly used when incising the skin of an animal? No. 20 No. 10 No. 12 No. 11

No. 10 The No. 10 blade is the most commonly used blade, and the technician needs to be familiar with this blade size. The No. 15 blade is also commonly used. This blade is smaller and will be used for more delicate incisions. The No. 11 blade is pointed at the tip and will be used to transect structures such as ligaments. The No. 12 blade is not commonly used but some may use it to lance abscesses.

Viruses with this characteristic are significantly more resistant to environmental degradation; in other words, they survive longer in the environment. Enveloped viruses DNA viruses Non-enveloped viruses RNA viruses

Non-enveloped viruses The correct answer is non-enveloped viruses. Non-enveloped viruses are typically very resistant to environmental degradation and therefore survive longer and are harder to disinfect than enveloped viruses. For example, parvoviruses, which are non-enveloped viruses, are difficult to disinfect and survive longer compared to influenza viruses which are enveloped. Whether a virus is a DNA or RNA virus has no effect on its environmental durability.

Which of the following statements is true regarding non-screen film in comparison to screen x-ray film in radiography? Non-screen film requires less radiation exposure to produce a diagnostic image Non-screen film is more sensitive to x-rays Non-screen film is best for imaging larger animals or body parts Non-screen film produces images that have less detail

Non-screen film is more sensitive to x-rays The main difference between non-screen film and screen film is that non-screen film has the image produced by x-rays themselves, whereas screen film uses light produced in an intensifying screen to generate the image. Therefore, screen film is more sensitive to light, and non-screen film is more sensitive to x-rays. The advantage of using a screen system is that much less radiation is needed to create the image but the trade-off is a loss of detail from the intensifying screen. Non-screen film is not good to use with larger animals or body parts because it requires too much exposure (and therefore a longer exposure which will have greater motion) to obtain a well exposed radiograph.

What type of estrous cycle does a cow have? Induced ovulation when bred Estrus every 6 months Polyestrous in the fall Non-seasonal polyestrous

Non-seasonal polyestrous Cows have non-seasonal polyestrous cycles. This means that they have estrus cycles year round. Average estrus is 21 days and lasts for 18 to 24 hours, but heat stress can shorten this window. Estrus is the time of "standing heat" when the cow will stand to be mounted by the bull. Sheep are polyestrous in the fall. Cats are induced ovulators.

An individual believes that no kill shelters are the correct moral standard for professional behavior. This is an example of which branch of ethics? Official ethics Normal ethics Administrative ethics Descriptive ethics

Normal ethics Normal ethics are defined as: An individual's attempt to discover what he or she believes to be the correct moral standard and norms for professional behavior.

A practitioner's view of convenience euthanasia is an example of which branch of ethics? Normative ethics Official ethics Normal ethics Administrative ethics

Normative ethics Normative ethics is defined as: An individual's attempt to discover what he or she believes to be the correct moral standard and norms for professional behavior.

While evaluating a post-operative patient, a small amount of serosanguinous fluid is noted oozing from the surgical incision. What is the best course of action? Start the patient on antibiotics Apply a bandage over the incision Clean the incision with hydrogen peroxide Notify the veterinarian

Notify the veterinarian

Which of the following is the scabies mite in cats? Cuterebra Sarcoptes gati Cheyletiella blakei Notoedres cati

Notoedres cati Feline scabies is caused by the mite Notoedres cati. It often causes scaly or scabby facial lesions in cats and is intensely pruritic. It is a highly contagious mite and is closely related to the canine scabies mite Sarcoptes scabiei. It is diagnosed via a skin scraping. Demodex gati is the demodex cat mite. Sarcoptes gati is not a real mite. Cuterebra is a fly larva that may infest the skin of domestic animals. Cheyletiella is a surface mite that causes itchy and flaking skin in dogs and cats, also known as "walking dandruff".

You are processing a fluid sample from a cerebrospinal fluid tap. There is some blood contamination of the sample. Which of the following values can be most significantly changed by blood contamination and is frequently corrected for when analyzing the results? Specific gravity Nucleated cell count Glucose level Protein level

Nucleated cell count White blood cells from circulation can artificially raise the nucleated cell count of a CSF sample because there are normally very few (<25/ul) nucleated cells in normal CSF, and even relatively small volumes of blood contamination may introduce white blood cells into the sample. This can also alter the differential cell count. Protein levels and specific gravity should not be dramatically affected. Also, glucose should not be dramatically altered; it is not routinely measured in CSF and is usually only requested in special circumstances.

Choose the proper corresponding acronym for the following in order: Right eye, left ear, both ears, both eyes, left eye. OS, AS, AU, OU, OD AD, OS, OU, AD, AS OD, AS, AU, OU, OS OU, AU, AS, OD, OS

OD, AS, AU, OU, OS

A horse with flared nostrils and a heave line from the tuber coxae towards the elbow with forced expiration likely has which of the following conditions? Congestive heart failure Laminitis Lead poisoning Obstructive airway disease

Obstructive airway disease These symptoms are typical of a horse with chronic increased respiratory effort. Coughing, weight loss, lack of energy, and exercise intolerance are also signs of this condition, termed "chronic obstructive pulmonary disease" (COPD). It is more common in older horses that are stabled during winter months. It is helpful to turn these horses out to pasture to help control the disease. COPD is similar to asthma in humans.

Which of the following is NOT an indication for replacing the bandage on a dog 24 hours after a tibial fracture repair with a bone plate? Distal aspect was slightly urinated on while walking outside Toes are moderately swollen Occasional toe-touching with the limb Strike-through of the bandage

Occasional toe-touching with the limb Occasional toe-touching after a major fracture repair is to be expected. The other answer choices are "red-flags", and the bandage should be replaced and the incision evaluated to ensure there are no impending complications.

Which of the following structures in the body is most sensitive to radiation? Thyroid gland Cerebral neurons Ocular lens Bones of the hand

Ocular lens

Which of the following is NOT a common method of restraint in the horse? Skin twitch Lip chain Nose chain Nose twitch Ocular pressure

Ocular pressure

The National Association of Veterinary Technicians in America adopting a code of ethics is an example of which branch of ethics? Official ethics Descriptive ethics Administrative ethics Normal ethics

Official ethics Official ethics is defined as: Values formally adopted by organizations composed of members of that profession.

In domestic animals, which of the following is true regarding inheritance of a trait that is coded by a recessive gene according to classical rules of inheritance? If one parent displays the recessive trait and one does not, up to 75% of the offspring should display the trait If both parents display the recessive trait, some offspring will show the trait but some will not Offspring may display a recessive trait even if neither parent displays the trait Offspring may display a recessive trait even if only one parent carries a gene for the trait

Offspring may display a recessive trait even if neither parent displays the trait Domestic animals have paired chromosomes and therefore inherit 2 alleles of each gene, one from the mother and one from the father. For classical rules of inheritance, a single gene can be dominant or recessive. If an individual has 2 copies of a recessive gene, it will show the recessive trait. If an individual has 2 copies of a dominant gene, it will show the dominant trait. In the event that an individual has one copy of a dominant gene and one copy of a recessive gene, it will show the dominant trait (the recessive gene is "masked" by the dominant gene). Typically, the dominant gene is indicated by a capital letter and the recessive gene is indicated by a lower-case letter. For this question, let's consider the trait of congenital deafness (h) which is recessive versus normal hearing (H) which is dominant. In order for an individual to show the phenotype of congenital deafness, it must have an hh genotype. An individual that has normal hearing could have either an Hh genotype or an HH genotype. In the correct answer to this question, it states that offspring may display a recessive trait even if neither parent displays the trait. It is possible that both parents could have normal hearing and have a Hh genotype. If both parents contribute the h gene to the offspring, it will display the recessive trait of congenital deafness. In the other choices: Offspring may display a recessive trait even if only one parent carries a gene for the trait - If only one parent carries the h gene, the offspring must receive the H gene from the other parent and will have normal hearing. If both parents display the recessive trait, some offspring will show the trait but some will not - If both parents display the recessive trait, they both must have the hh genotype and will each pass on the h gene to offspring; therefore, all offspring should show the recessive trait. If one parent displays the recessive trait and one does not, up to 75% of the offspring should display the trait - If one parent has the trait (has hh genotype) and one parent does not (has either Hh or HH genotype), the offspring will have a 50% chance of inheriting the trait (if the other parent is Hh) or a 0% chance of inheriting the trait (if the other parent is HH). There is not a scenario where more than 50% of the offspring should inherit the trait.

In domestic animals, which of the following is true regarding inheritance of a trait that is coded by a dominant gene according to classical rules of inheritance? Offspring may inherit the trait if one of the parents does not have the gene for the trait as long as the other one does have the trait Offspring may inherit the trait even if neither of the parents has the trait All offspring will inherit the trait if both parents have the trait No offspring will inherit the trait if only one parent has the trait

Offspring may inherit the trait if one of the parents does not have the gene for the trait as long as the other one does have the trait Domestic animals have paired chromosomes and therefore inherit 2 alleles of each gene, one from the mother and one from the father. For classical rules of inheritance, a single gene can be dominant or recessive. If an individual has 2 copies of a recessive gene, it will show the recessive trait. If an individual has 2 copies of a dominant gene, it will show the dominant trait. In the event that an individual has one copy of a dominant gene and one copy of a recessive gene, it will show the dominant trait (the recessive gene is "masked" by the dominant gene). Typically a dominant gene is indicated by a capital letter and a recessive gene is indicated by a lower-case letter. For this question, let's consider an example of dwarfed limbs (D) which is dominant versus normal limbs (d), which is recessive. In order for an individual to show the phenotype of normal limbs, it must have a dd genotype. An individual that has dwarfed limbs could have either a Dd genotype or a DD genotype. In the correct answer to this question, it states that one of the parents does not have the gene for the dominant trait (has a dd genotype). It is still true that the offspring may inherit the trait because if the other parent has a gene for the trait, it may pass that gene to the offspring and because it is dominant, the trait will be expressed. In the other choices: Offspring may inherit the trait even if neither of the parents has the trait - This choice is not correct because if neither one of the parents has the trait, they both must carry two copies of the recessive allele (dd in the example) and do not have a copy of the dominant gene to pass on. All offspring will inherit the trait if both parents have the trait - If both parents have the trait, it is possible that they both have one dominant and one recessive gene (Dd). Therefore, it is possible that each parent could pass on the recessive allele to the offspring and the trait would not be seen. No offspring will inherit the trait if only one parent has the trait - If one parent has the trait, they can pass on the dominant allele for the trait, which will then be expressed and will be seen in the offspring.

The surgeon uses this instrument to place and cut his sutures. Identify this instrument. Alligator forceps Mayo-Hegar needle holders Olsen-Hegar needle holders Debakey forceps

Olsen-Hegar needle holders The Olsen-Hegar needle holders include the scissors, while the Mayo-Hegar needle holders do not. Debakey forceps are delicate non-traumatic tissue-handling thumb forceps. Alligator forceps are long narrow grasping forceps commonly used to retrieve foreign material out of dog ears.

You are restraining a horse with a halter while the veterinarian is examining a sore on the lateral side of the left pelvic limb. Where should you stand while holding this horse? On the left side, hold lead rope with little slack On the right side, hold lead rope with little slack On the left side, hold lead rope and leave enough slack so that the horse may turn his head around to see what the veterinarian is doing The horse should be tied up and you should stand to the right of the veterinarian at the hind end of the horse to prevent the horse from kicking

On the left side, hold lead rope with little slack When handling a horse, the holder and the person working on the horse should be standing on the same side. If the exam starts on the left, stand on the left; as the examiner moves to the right side, you should also move to the right side. There should be little slack in the lead rope to prevent the horse from moving.

What is the best way to restrain a dog for examination? Both arms under the abdomen and around to the other side with a gentle squeeze One hand under the neck and the other firm grip at the base of the tail One arm under and around the neck and the other under the abdomen and in front of the flank One hand around the dog's muzzle and the other under the abdomen around to the flank

One arm under and around the neck and the other under the abdomen and in front of the flank

If a pulmonary metastatic lesion is visible on a right lateral radiograph but not on a left lateral radiograph, where is the lesion most likely to be? One of the cranial lung lobes One of the left lung lobes One of the ventral lung lobes One of the caudal lung lobes One of the dorsal lung lobes One of the right lung lobes

One of the left lung lobes When a patient is placed into left lateral recumbency for a left lateral radiograph, the left lung fields are compressed and do not inflate fully. This can prevent resolution of small metastatic lung lesions in the left lung lobes due to the lack of contrast with air in the left lung. The same is true of the right lung fields with a right lateral radiograph. Therefore, for evaluation of pulmonary metastasis, both a left and right lateral radiograph should be taken. In this case, a lesion that can be seen on a right lateral but not a left lateral projection is most likely in the left lung lobe because, as explained above, the lesion may be obscured on the left lateral projection due to a lack of contrast in the uninflated lung.

What is the medical term for declawing? Orchiectomy Onychectomy Caudectomy Ovariohysterectomy Enucleation

Onychectomy The procedure consist of surgically removing or amputating the third phalanx of each toe. The majority of practitioners who perform this controversial operation will only do the procedure on the front paws. Caudectomy is tail amputation. Orchiectomy is removal of the testicles. Ovariohysterectomy is surgical removal of the uterus and ovaries. This sterilization procedure prevents pregnancy and is called a spay. Enucleation is surgical removal of the eye.

Contagious ecthyma is a disease that causes painful oral ulcerations, pustules, and scabs. It is most often seen in sheep and goats, and is more commonly known as: Orf Oral Thrush Foot-and-Mouth disease Blue Tongue

Orf Contagious ecthyma, most commonly called Orf is a highly contagious and zoonotic parapox virus. Blue tongue is a virus of ruminants and camelids causing fever, lung disease, and congestion of the lips and oral mucosa. Oral thrush is Candidiasis (yeast). Thrush is caused by an overgrowth of normal yeast flora seen mostly in immunocompromised animals. Foot-and-mouth disease is a contagious vesicular virus that affects cloven-hooved species. It causes vesicles and ulcers in the mouth, lameness of the feet, and is a reportable disease.

Which parasite is the most important cause of parasitic gastritis in cattle and is commonly known as the brown stomach worm? Strongylus vulgaris Haemonchus contortus Ancylostoma braziliense Ostertagia ostertagi

Ostertagia ostertagi Ostertagia is the brown stomach/abomasal worm and is a major cause of parasitic gastritis in cattle. It is the most common roundworm in cattle. Strongylus vulgaris is a large intestinal worm in horses that can migrate to the anterior mesenteric artery. Ancylostoma braziliense is known as the Southern dog hookworm and is a primary cause of cutaneous larval migrans. Haemonchus is known as the barberpole worm or red stomach worm and is especially problematic in sheep.

The doctor is performing a physical exam on a horse and notices the horse has some ticks in the ears. The ticks have large spines on their bodies. He refers to them as "spinous ear ticks". What is the proper name for this parasite? Dermacentor albipictus Oxyuris equi Otobius megnini Otodectes cynotis

Otobius megnini Otobius megnini is the spinous ear tick. These ticks are most commonly seen in the Southwestern states. They may infest many different warm blooded animals (including dogs), but are most commonly seen in horses and livestock. These ticks may crawl deep into the ears and feed on blood. They can spread rickettsial disease and other infectious disease such as Q-fever and tularemia. Otodectes cynotis is the ear mite of dogs and cats. Oxyuris is the equine pinworm. Dermacentor albipictus is the winter tick typically found on horses, cattle, or deer, especially in the mid-Western regions (Nebraska, Colorado, and Wyoming).

The vet thinks there is a fox tail in a dog's ear canal. What instrument will the doctor need? Microscope Otoscope Endoscope Speculum Ophthalmascope

Otoscope An otoscope is used for examining ears. An ophthalmoscope is used to perform eye and fundic exams. An endoscope is used to perform endoscopy: the scope is placed down the esophagus, into the stomach and upper small intestine for visual aid and for collecting biopsies. It can also be used to assist in placement of peg tubes or in retrieving foreign bodies in the esophagus and stomach.

Celiotomies are performed on a routine basis in veterinary medicine. Which of the following procedures is performed through a celiotomy? Lung lobectomy Ovariohysterectomy Femoral fracture repair Tail docking

Ovariohysterectomy A celiotomy or laparotomy is a surgical incision into the abdominal cavity. The term celiotomy is derived from the word coelom, which means body cavity. Knowing this information, the only surgery listed which is performed through a ventral midline incision is an ovariohysterectomy. To review, an ovariohysterectomy is the removal of the ovaries and uterus.

What is the recommended treatment option for a closed pyometra? PGF 2-alpha Estrogen therapy Ovariohysterectomy Antibiotic therapy

Ovariohysterectomy Pyometra occurs secondary to excessive or prolonged progesterone production which causes endometrial hyperplasia. This then sets up an environment conducive to bacterial colonization. The most common culprit is E. coli; however, other organisms have been implicated in pyometra. Antibiotic therapy may effectively treat pyometras which are "open" or draining, however this is only recommended if the patient is stable. PGF 2-alpha, and estrogen therapy is ineffective at treating pyometra.

Orf is the cause of contagious ecthyma primarily in which species? Porcine Canine Bovine Ovine

Ovine Ovine (sheep) and goats are the species which may contract Orf, also known as "soremouth". It is zoonotic and caused by a parapox virus. It is transmitted via direct contact with infected animals or by contact with fomites, so handlers should wear gloves when dealing with infected animals. It is a benign and self-limiting disease in most people but may be serious in immunocompromised people. Symptoms in sheep and goats include papules or pustules on the lips and muzzle that may then crust and bleed.

You are hooking up the anesthesia machine and place the pulse oximeter on the dog's tongue. This is to measure which of the following? Carbon dioxide Oxygen saturation Respiratory rate Electrical impulse of the heart

Oxygen saturation Pulse oximetry measures oxygen saturation of hemoglobin in arterial blood. The pulse-ox reading should generally always be above 94%. If this measurement drops, especially during anesthesia, sometimes rubbing and wetting the tongue can help improve circulation to the tongue and thus help get a more accurate reading.

Which hormone is the main cause of milk letdown? Progesterone Oxytocin Testosterone Estrogen

Oxytocin Oxytocin stimulates the milk letdown reflex. Progesterone and estrogen help with the growth of the mammary tissues that produce milk. Progesterone maintains a pregnancy. Testosterone is the principal male sex hormone.

Anal pruritus in the horse can be associated with an infestation of this parasite. Note the alopecia around the anal region and around the tail caused by rubbing. Oxyuris equi Parascaris equorum Anoplocephala Habronema muscae

Oxyuris equi The correct answer is Oxyuris equi. This is the equine pin worm, which will infest the perineal region and result in perineal irritation after eggs are laid. A frequent diagnostic test performed when visualizing damaged perineal regions is a scotch tape preparation.

All of the following are examples of Regulated Medical Waste (RMW) that require special treatment or handling EXCEPT which of the choices listed? Needle and syringe used to give chemotherapy to a dog Carcass of a cat that was brought to your clinic dead on arrival upon which you performed a necropsy Paint-stripping solvents that may release toxic fumes Scalpel blade that was used to make a spay incision on a healthy cat Gauze sponges used in equine surgery to soak up blood

Paint-stripping solvents that may release toxic fumes RMW, also known as biohazardous waste, applies to waste that is potentially infectious. It must meet both of the following conditions: 1) The waste must be generated as a result of the diagnosis, treatment, or immunization of animals 2) It must be either sharps or biohazardous waste The scalpel and syringe are examples of sharps, and the blood-soaked sponges and carcass are examples of biohazards. While reactive, solvents do require additional consideration when being disposed of, they are not Regulated Medical Waste as they do not meet the 2 criteria above.

Which organ(s) in the body is (are) responsible for producing insulin? Liver Adrenals Pancreas Pituitary gland

Pancreas The two main functions of the pancreas are endocrine or production of hormones (insulin and glucagon) and exocrine or production of digestive enzymes. The pancreatic beta cells are responsible for secreting insulin. The adrenal glands produce several hormones including aldosterone, cortisone, testosterone, and catecholamines. The pituitary gland produces ACTH, growth hormone, thyroid stimulating hormone, prolactin, and others.

A dog presents dragging the hindlimbs. While the doctor is pinching along the dogs back on either side of the spine, the skin twitches, causing contraction of the cutaneous trunci muscle. Which reflex specifically is being tested? Panniculus reflex Patellar reflex Spinal reflex Proprioception

Panniculus reflex The panniculus (cutaneous) reflex tests for superficial pain. A fold of skin is grasped with a small hemostat and pinched just lateral to midline on both sides. A twich of the skin (the panniculus reflex) is observed (contraction of the cutaneous trunci muscle). This helps to determine the caudal margins of normal superficial pain bilaterally. Proprioception is a test to determine if the pet knows where the limbs are in space, or position awareness. The paw is first flipped under so that the dorsal part of the foot is touching the floor, and then the paw is released. The pet should return the paw to the normal positioning. If the remaining paw flipped over, this indicates a proprioceptive deficit. Patellar reflex is performed by a tap on the patellar ligament in the knee. This should extend the stifle. Spinal reflexes are a general term and encompass multiple tests for sensory and motor components.

Which of the following is a causative agent of warts? Dermatophytosis Orbivirus Papilloma virus Dermatophilus

Papilloma virus Papilloma virus is responsible for causing warts. Dermatophilus is a bacterium that invades the skin typically in wet weather and causes crusting and infection of the skin (rain rot). Dermatophytosis is a fungus, most commonly ringworm (Trichophyton) in cattle. Orbivirus causes bluetongue disease in sheep, cattle, and goats.

Which internal parasite is a threat to young (but typically not adult) horses? Large strongyles Tapeworms (Anaplocephala perfoliota) Small strongyles Parascaris equorum

Parascaris equorum Ascarids (Parascaris equorum) can cause significant problems in young (<1 year) horses and can cause poor growth and a pot-bellied appearance. A more severe problem occurs when the small intestine becomes obstructed from a heavy worm burden. Adult horses develop an immunity to ascarids, so they are typically not a problem in adults.

A foal presents for colic caused by the organism shown in the image below (microscopic image from 40X, organism is approximately 100 um). What is this organism? Parascaris equorum Strongyloides westeri Strongylus vulgaris Trichostrongylus axei

Parascaris equorum In foals, a significant ascarid burden with Parascaris equorum can lead to intestinal impaction and associated colic. This is a roundworm of horses.

Which of the following parasites can be associated with colic and respiratory disease in the foal? Echinococcus equinus Parascaris equorum Parelaphostrongylus tenuis Cryptosporidium

Parascaris equorum Parascaris equorum, or the equine ascarid (roundworm), typically only infests foals and can cause ill thrift, coughing, pneumonia, and colic. If a foal has a heavy burden of roundworms and is subsequently given an anthelmintic, impaction colic can result from a mass of dead worms obstructing the intestinal tract. Foals develop immunity to this organism as they age and are typically free of infestation as adults.

A Dachshund presents for back problems and the dog is ambulatory but very unsteady in both hindlimbs. The doctor suspects an intervertebral disk protrusion that is affecting the spinal cord. Regarding the gait, this dog has: Paralysis Paresis Vestibular syndrome Plegia Hypermetria

Paresis

A puppy presents to your clinic for surgery because he has an intussusception. What happened to the puppy? Part of his intestine has telescoped inside of itself His intestine is twisted at the root of the mesentery He swallowed a foreign object He has a tear in his body wall, and his abdominal contents are protruding into the subcutaneous region

Part of his intestine has telescoped inside of itself Intussusception is the telescoping of one part of the bowel into an adjacent segment. This problem is most often seen in younger animals with a history of diarrhea. A tear in the body wall is a hernia. Mesenteric volvulus is twisting of the intestine at the root of the mesentery and is immediately life-threatening.

All of the following pieces of information legally must be present on a radiographic film EXCEPT for which of the following? Patient breed Name of practice Name of patient and owner Date of radiograph

Patient breed A radiograph is part of the legal medical record. As such, it is necessary to identify the patient it was taken from, the date of the exam, and the name of the practice. It is frequently helpful to use a marker to orient the radiograph or to identify which extremity is being examined. For film radiographs, frequently patient information is imprinted by the use of special graphite-impregnated tape where the information can be written and placed on the radiograph cassette or by a light flasher system that exposes printed patient information to the film. For digital radiographs, the patient information must be entered into the computer so that it is saved along with the digital radiograph. Additional patient information such as breed can be helpful but is not required.

What technique should be used to obtain a lateral oblique radiograph of the left tympanic bulla? Patient should be placed in lateral recumbency, right side down; the skull should be rotated 20 degrees towards the VD position Patient should be placed in lateral recumbency, left side down; the skull should be rotated 20 degrees towards the VD position Patient should be placed in dorsal recumbency in a crouching position with the chin resting on a foam block; the skull should be rotated 20 degrees towards the right lateral position Patient should be placed in dorsal recumbency in a crouching position with the chin resting on a foam block; the skull should be rotated 20 degrees towards the left lateral position

Patient should be placed in lateral recumbency, left side down; the skull should be rotated 20 degrees towards the VD position For a lateral oblique film of the tympanic bulla, the side to be imaged should be positioned down (against the table). The Patient should be placed in lateral recumbency, and the skull should be rotated 20 degrees towards the VD position. It is frequently useful to also obtain the same view for the contralateral bulla for comparison and to evaluate if it is also affected.

Where should an injection ideally be given to a pet bird? Wing Neck Leg Pectoral muscles

Pectoral muscles For birds, the best place to give an injection is into the large pectoral muscle mass region. If drugs are injected in the lower half of the body, such as the legs, the medication can be absorbed towards the kidneys and could cause a problem. However, in some large birds, such as ostriches and emus, injections are sometimes given in the legs.

A 7-year-old Lab mix presents to your hospital with a very swollen muzzle. The vet suspects the swelling is from a rattlesnake bite. How could you support this suspicion? Perform a buccal mucosal bleeding time Perform a blood smear and look for echinocytes Perform an aspirate of the swelling for cytology Perform a blood smear and look for spherocytes

Perform a blood smear and look for echinocytes Echinocytes are often seen in rattlesnake envenomation. While rattlesnake bites may cause coagulopathies, an increased bleeding time would not be diagnostic for rattlesnake bite or support the suspicion as strongly the as presence of echinocytes. Spherocytes are most often seen in cases of autoimmune hemolytic anemia. Aspirating the swelling would not be helpful, as the swelling is most often occurring due to edema (swelling within the cells) and not free fluid.

A dog owner is considering having a scheduled cesarean section for her dog and would like some more information regarding the risks associated with the procedure. Which of the following statements is TRUE? Performing a cesarean delivery will result in decreased milk production Performing a cesarean delivery will result in the dog being unable to have another pregnancy Performing a spay at the time of surgery does not affect milk production Performing a spay at the time of surgery is not recommended as there is a risk that the female will not display maternal instincts

Performing a spay at the time of surgery does not affect milk production It is important to be aware that performing a spay at the time of surgery does not have a detrimental effect on the female. There are no adverse effects on milk production or predictable unwanted behaviors. Similarly, there are no long-term negative effects of performing a cesarean delivery; however, there is a possibility of development of scar tissue at the site of a cesarean which may hinder the ability of a future embryo to attach at that specific location.

A purse-string suture would be been placed for which of the following surgeries? Enterotomy Cystotomy Enucleation Perianal mass excision

Perianal mass excision A purse-string suture is a surgical suture passed as a running stitch in and out along the edge of a circular region (as seen with the anus), so that when the ends of the suture are drawn tight the region is closed like a purse. This is often done around the anus prior to initiating surgery to prevent fecal contamination within your surgical site.

Which of the following lymph nodes are not palpated peripherally? Perihilar Prescapular Mandibular Popliteal

Perihilar Above the heart is a triangular depression named the hilum, near which the perihilar lymph nodes are located. They are inside the chest and therefore not palpated peripherally.

Which of the following instruments would be LEAST effective at cutting or removing bone from a patient? Michel trephine Gigli wire Osteotome Periosteal elevator Curette

Periosteal elevator Periosteal elevators are good for elevating muscle attachments from bone or periosteum. They would not be effective at actually cutting or removing bone. Curettes are great for removing small amounts of bone such as when obtaining a bone graft. Gigli wire is still used for cutting through bone. With an osteotome and mallet, precise bone cuts can be made by the experienced surgeon. Alternatively, a battery powered bone saw can be used. A Michel trephine is used to obtain core samples of bone. It is commonly used to help diagnose neoplasia involving the bone.

A 5-year-old female spayed domestic short hair cat presents on emergency for acute vomiting, weakness, hypersalivation, and tremoring. Her heart rate is 240 bpm and her temperature is elevated at 104.2F. The owner applied an over-the-counter flea control product to her skin earlier in the day. What toxicity is suspected? Ethylene glycol Metaldehyde Coumadin Permethrin

Permethrin Permethrin (a synthetic pyrethroid) is very toxic to cats and can be found in some topical flea prevention medications. It can cause neurologic side effects, tremoring, and may even cause seizures. Cats are treated by washing the product off immediately, placing an intravenous catheter, administering IV fluids, and giving muscle relaxants such as valium and/or methocarbamol. Metaldehyde is most commonly found in snail bait and is not part of flea control products. Ethylene glycol is antifreeze and coumadin is Warfarin (an anticoagulant found in some rat poisons); these are not found in flea control products.

Which cat would be most prone to having a hairball problem? Abyssinian Domestic short hair Persian Sphynx

Persian Persians have long hair and are thus most likely to have hairballs. However, all cats can get hairballs. This occurs if the cat swallows hair while it is grooming itself. Laxatone can help to keep the hairballs to a minimum and daily brushing also helps to reduce the amount of shedding.

The pinpoint red marks on this patient's gums can be described as: Erythema Papillomas Ecchymosis Petechia

Petechia Petechia are small (1-2mm)"pinpoint" red bruises under the skin, often associated with thrombocytopenia. Ecchymosis (also often called purpura) are very similar but are much larger areas of bruising, usually over 3mm. Papillomas are warts caused by a virus. Erythema is the general term for redness.

Which of the following methods is the best way to handle a tractable cat? Petting the cat and using minimal restraint Wrapping in a towel to prevent scratching and scruff back of the neck Cat muzzle and wrap in a towel Use sedation and Kevlar gloves for handling

Petting the cat and using minimal restraint

Which of the following drugs is an anticonvulsant? Acepromazine Xylazine Phenobarbital Butorphanol

Phenobarbital

You are about to sedate a dog, and the veterinarian would like to use a drug which is reversible. Which one of these is NOT an option? Hydromorphone Medetomidine Midazolam Phenobarbital Diazepam

Phenobarbital

Which of the following medications is known to cause perivascular tissue sloughing if given out of the vein? Ampicillin Dimethyl Sulfoxide (DMSO) Ascorbic acid Phenylbutazone

Phenylbutazone Phenylbutazone or "Bute" is an NSAID used for analgesia or fever (mostly in horses). It can cause tissue sloughing and necrosis if given out of the vein (it should NEVER be given subcutaneous or intramuscular). Also to note: In humans, phenylbutazone can cause aplastic anemia.

A client has been giving her pet a "nutraceutical" she found that is supposed to help with her dog's arthritis. What are nutraceuticals? Phytochemicals Vitamins Herbs Minerals

Phytochemicals Nutraceuticals are chemicals or foodstuffs derived from plants (and occasionally animals) in the form of a supplement. These are not typically regulated by the government.

A bulldog presents for its fourth foreign body surgery. It eats dirt, toys, and other non-food substances on a regular basis. This condition of eating non-food items is known as: Pica Aerophagia Dysphagia Coprophagy

Pica Pica is a word used to describe the eating of foreign materials. Coprophagy is eating of feces. Dysphagia is difficulty swallowing. Aerophagia is swallowing of air.

Which of the following species have teeth that only grow for a limited time during development and do not continually erupt or grow throughout the life of the animal? Pigs Horses Rabbits Rats

Pigs

An owner presents her diabetic cat and new glucometer with lancets. Where is the best place for her to collect a sample? Medial hindlimb Pinna Dorsal frontlimb Pad

Pinna Lancets are best to use on superficial smaller veins such as veins on the ear. Pads are very thick and therefore difficult to use for sample collection.

An 11-year old female spayed dog is diagnosed with a bladder tumor. She is still able to pass urine, but she strains towards the end of her urination and needs to go out frequently. Her bloodwork is unremarkable. What medication is sometimes prescriped specifically to alleviate symptoms from bladder tumors in dogs? Tramadol Cranberry extract Piroxicam Amoxicillin-clavulanic acid

Piroxicam Piroxicam is a non-steroidal anti-inflammatory drug that has some anti-cancer properties against transitional cell tumors and nasal adenocarcinoma in dogs. Piroxicam can cause renal papillary necrosis as a rare side-effect, so is usually reserved for cases that have no underlying renal issues.

You are assisting with a bone marrow aspirate on a dog with suspected neoplasia. You are asked to make slides to send to the laboratory with the samples the veterinarian is collecting. Which describes proper technique for this? Place a drop on one end of the slide, tilt slightly and make a pull smear Place a drop on the slide and place 1 drop of 0.9% saline onto the drop to dilute and let dry Place a drop on the slide but do not make a pull smear or press the cells, just let air dry as-is Place a drop on the slide and place another slide on top in an up-and-down motion to press the cells firmly onto the slide

Place a drop on one end of the slide, tilt slightly and make a pull smear Pull smears should be made of bone marrow samples. The pull smear should be in a light fashion as to spread the fluid but not to crush the samples. The sample should be dried immediately to preserve integrity of the cells. A blow dryer can assist by quickening drying time. A stain of one of the slides should be performed to make sure the sample appears adequate before submission.

What is the biggest risk when giving an animal fluids or medications via an orogastric or a nasogastric tube? Placing tube into the lungs Regurgitation through the nares Perforation of the esophagus GI foreign body from animal biting off the tube

Placing tube into the lungs Sometimes animals require medications through a tube. Horses that are having colic symptoms often receive fluids and mineral oil through a nasogastric tube. Sometimes small animals need to receive activated charcoal or barium through an orogastric tube. The biggest risk when tubing a patient is accidentally placing the tube into the lungs, so the tube goes down the trachea instead of the esophagus. This leads to compromise of the lungs and can lead to subsequent pneumonia and can have fatal consequences.

Yersinia pestis is the causative agent for which zoonotic disease? Q fever Parrot fever Plague Cat scratch fever

Plague Yersinia pestis is the causative agent of plague. Plague is usually transmitted by the bites of infected fleas. Yersinia pestis is a gram-negative bacterium and has a bipolar safety-pin appearance. Definitive diagnosis is based on culture. However, before collecting any samples, the state vet or CDC should be contacted. Plague is highly zoonotic, and bubonic, septicemic, pneumonic, and meningeal forms can occur. Early treatment is critical for survival. Antibiotic treatment early in the course of disease can greatly improve prognosis. Cat scratch fever is caused by the bacterium Bartonella henselae. Parrot fever is caused by the bacterium Chlamydophila psittaci. Q fever is caused by the bacterium Coxiella burnetii.

You are working in the emergency clinic and are assisting with treating a dog that has been hit by a car. You are asked to set up for a blood transfusion, and the doctor would like you to give whole blood. A whole blood transfusion would include which of the following components? Plasma and red blood cells only Plasma, red blood cells, white blood cells, and platelets White blood cells and red blood cells only Platelets, plasma, and red blood cells

Plasma, red blood cells, white blood cells, and platelets

A blood smear is performed and a large variation is seen in erythrocyte shape. What is this called? Anisokaryosis Poikilocytosis Anisocytosis Polychromasia

Poikilocytosis Anisocytosis is a variation in cell size (not shape). Anisokaryosis is a variation in the amount of cytoplasm present in a cell and not necessarily shape or size. Polychromasia describes a variation red blood cell color. Poikilocytosis describes a variation in cell shape. Some shapes occur with enough frequency that they have their own name, such as acanthocytes and echinocytes.

A surgeon is asking for any non-absorbable suture to close a skin defect. Which of the following sutures would NOT be appropriate for you to give the surgeon? Polydioxanone Polypropylene Nylon Silk

Polydioxanone Polydioxanone is the real name for PDS. This suture is a monofilament absorbable suture which lasts approximately 180 days before being completely broken down. Silk, polypropylene, and nylon are all considered non-absorbable. Polypropylene and nylon are synthetic. Most people know polypropylene as Prolene and nylon as Ethilon; these are their trade names.

What is the term used for increased urine output? Pollakiuria Anuria Polyuria Oliguria

Polyuria Polyuria is the term that describes increased amount of urination. Pollakiuria refers to an increase in frequency in urination. Oliguria is a decrease in urination. Anuria is the complete absence of urine formation.

Which breed does not typically shed its haircoat? Golden retriever Pug Poodle Labrador

Poodle

You are on a farm call with a veterinarian to investigate the cause of agalactia in the farmer's cows. These cows have: Blood in their milk Poor milk production Infertility Low calcium

Poor milk production Agalactia (agalactorrhea) is either an absence of milk production or abnormally low milk production following parturition. There are many common causes including infection (especially in sheep, goats, and pigs), poor nutrition or water deprivation, hormonal imbalances, anatomical causes(inverted teats or absence of mammary tissue, scarring of tissue), or failure of the neonate to suckle properly so that the milk let-down reflex is not stimulated.

Certain drugs may cause a rapid rise in temperature during anesthesia. This is known as malignant hyperthermia. Which species is at greatest risk for this condition? Canine Caprine Equine Porcine

Porcine Pigs are at greatest risk for malignant hyperthermia. It is thought that this condition may be hereditary. During anesthesia in pigs, this must be monitored very closely as early detection is essential to prevent fatality. Cooling measures, Dantrolene, and oxygen are all implicated in treatment.

A 3-year old male neutered Maltese presents to your clinic because the owner saw "some white rice-looking things" around his anus. They were small and flat and seemed to be moving. Some of them were dried up. Which of the following drugs is effective against this organism? Fenbendazole (Panacur) Fipronil (Frontline) Pyrantel (Strongid) Selamectin (Revolution) Praziquantel (Droncit)

Praziquantel (Droncit) This dog likely has a tapeworm infection. Tapeworm segments are typically flat and white and small, resembling a grain of rice. The most common tapeworms in cats are Taenia taeniaeformis and Dipylidium caninum. The only medication that will treat both types is praziquantel. Praziquantel is in the products Drontal Plus and Profender, approved for the use in cats. Drontal Plus also contains pyrantel. Profender also contains emodepside. Both are also effective against roundworm and hookworm. Pyrantel is not effective against tapeworms and treats hookworm and roundworm infection. Fenbendazole (or Panacur), treats Taenia but not Dipylidium, and also treats hookworm, roundworm, and whipworm infection. Revolution treats and prevents hookworm, roundworm, heartworm, fleas, and ear mites in cats. Frontline treats and prevents fleas and ticks. A flea control should be recommended, since Dipylidium is transmitted by ingestion of an infected flea. Taenia is transmitted through eating an infected prey.

Which of the following scenarios describes the proper care of surgical instruments? Place instruments in surgical milk. Remove from surgical milk and rinse to remove and debris and residue. Place instruments in ultrasonic cleaner for 10 minutes. Let instruments dry. Rinse the instruments to remove the debris and residue. Place in an ultrasonic cleaner for 30 minutes. Then place the instruments in surgical milk. After removing instruments from the surgical milk rinse them again. Place instruments in ultrasonic cleaner for approximately 10 minutes. Rinse instruments with distilled water and scrub as necessary. Place instruments in surgical milk. Remove from surgical milk and let instruments dry. Pre-rinse the instruments immediately after surgery to remove residues. Place in ultrasonic cleaner for approximately 10 minutes. Place in surgical milk. Remove from surgical milk and let instruments dry.

Pre-rinse the instruments immediately after surgery to remove residues. Place in ultrasonic cleaner for approximately 10 minutes. Place in surgical milk. Remove from surgical milk and let instruments dry.

In a pregnant mare, "red bag" delivery implies what situation? Rupture of the chorioallantois Premature placental separation Delivery of a premature foal Rupture of the amniotic sac

Premature placental separation Red bag is the layman's term for premature separation of the chorioallantois from the placenta and is a medical emergency. The chorioallantois interfaces with the mare's endometrium and is the interface that transfers oxygen to the fetus. Under normal circumstances, the fetus breaks through the chorioallantois at the cervical star, and the fetus and amniotic sac are delivered first. In a red bag delivery, the entire fetal-placental unit is being delivered all at once and thus can cut off the oxygen supply to the fetus prior to birth; this can result in death of the fetus if parturition is not facilitated immediately.

Which of the following species require Vitamin C in their diets because they cannot synthesize it? Hamster Chinchilla Primate Cat

Primate Primates, including humans, are unable to synthesize adequate amounts of Vitamin C and must have it included in the daily diet. Guinea Pigs too have the same requirement for additional daily Vitamin C.

What is the function of a goblet cell? Produce platelets Phagocytize bacteria and viruses Secrete insulin Produce mucus

Produce mucus

Myositis in a horse can occur as a result of which of the following? A blood lactate level of 1.0 mg/dl and higher will result in myositis Prolonged recumbency during anesthesia Intraoperative hypertension will result in myositis Administration of nonsteroidal anti-inflammatories prior to anesthesia will result in myositis

Prolonged recumbency during anesthesia Prolonged recumbency puts a horse at a high risk for developing myositis. It is extremely important for the anesthesia team to provide appropriate padding by means of rubber pads, inflatable tubes, or even an inflatable water bed to help decrease the pressure exerted on muscles as result of recumbency. Intraoperative hypotension can contribute to myositis due to lack of blood flow and subsequent ischemia at the level of the muscle. Myositis can result in extreme muscle pain and soreness. In severe cases, horses can develop myoglobinuria which can cause renal damage. The blood lactate level of 1.0 mg/dl is actually in the normal range; however, an increasing blood lactate level is a sign of anaerobic metabolism, which may be secondary to poor muscle oxygenation; the cause should be investigated and addressed.

A 5-year old female spayed Yorkie presents with a history of decreased vision. The doctor asks you to perform tonometry on this dog. Which of the following may be used as a topical anesthetic to the eyes so that you may check the intraocular pressures with your Tonopen? Benzocaine Proparacaine Bupivacaine Lidocaine

Proparacaine Proparacaine is a topical anesthetic for the eyes. The other choices should not be used in the eyes. The other options are local anesthetic agents but are likely to cause irritation and/or corneal ulceration.

You are working on assembling drugs for a canine anesthetic/analgesic protocol for an upcoming surgery. You will be giving hydromorphone or morphine as a premedication, diazepam just before induction with propofol, and you will place a fentanyl patch after surgery in addition to carprofen for pain control. Which of these mentioned drugs does not need to be logged as a controlled substance? Morphine Fentanyl patch Diazepam Hydromorphone Propofol

Propofol Propofol is not a controlled substance and is not required to be recorded in a drug log by DEA regulations. Any controlled substances such as morphine, hydromorphone, butorphanol, ketamine, diazepam, phenobarbital, fentanyl, buprenorphine, hydrocodone, etc. must be logged in a drug log. Drug logs should include the name of the pet owner, date, prescribing doctor, name and amount of drug used.

A dog presents to the emergency hospital in status epilepticus. Which of the following drugs may be given for this condition? Ketamine Propofol Diphenhydramine Atropine

Propofol Status epilepticus is the state of continuous seizuring. Propofol and valium are the most common drugs used for this condition. Rectal valium may be given initially until a vein can be accessed. Ketamine would be contraindicated, as this drug causes muscle rigidity.

Which of these sedative/anesthetic drugs does not have an antagonist? Propofol Diazepam Hydromorphone Xylazine Medetomidine

Propofol The correct answer is propofol. The reversal agents are: For diazepam: flumazenil For hydromorphone: naloxone or naltrexone For medetomidine: atipamezole For xylazine: yohimbine Acepromazine is another sedative that does not have an antagonist.

A 10-year old male intact Rottweiler is currently undergoing surgery for removal of a prostatic mass. Which term best describes the procedure? Orchidectomy Prostatitis Onychectomy Prostatectomy Cholecystectomy

Prostatectomy There are a few basic rules that will allow you to comprehend a majority of the medical terms. Any term that ends with "-ectomy" infers to removal of that organ. Onychectomy is another word declawing or removal of the distal phalanx. Orchidectomy is the removal of a testicle. A cholecystectomy is removal of the gall bladder. The ending "-itis" means inflammation. In this case, prostatitis would be inflammation of the prostate.

The veterinarian asks you to retrieve a psittacine bird out of its cage for examination. Which of the following restraint tools would be least recommended for this type of bird? Towel Net Wash cloth Protective falconry gloves

Protective falconry gloves

Which of the following in an example of a Gram-negative bacterium? Staphylococcus Leptospira Clostridium Pseudomonas Lactobacillus

Pseudomonas Gram stain is used in classifying two distinct types of bacteria based on the structural differences of their bacterial cell walls. Gram negative bacteria lose the primary complex, take up the secondary dye safranin, and appear red or pink in color. Gram-positive bacteria will retain the crystal violet iodine and appear dark blue or purple.

Which of the following is considered a reportable swine disease? Sarcoptic mange Erysipelas Pseudorabies Pasturella multocida

Pseudorabies Pseudorabies is a virus caused by porcine herpesvirus 1. Wild mammals, cattle, sheep, dogs, and raccoons are also susceptible, and the disease can be fatal to these species. It is shed in oral and nasal secretions of swine and spread through oral/nasal contact or by fomites. Symptoms in pigs include abortion, nasal discharge/sneezing, seizures, depression, circling, and increased salivation. In cattle, it causes intense itching, also known as "mad itch", then seizures and death. The other diseases listed are more common in pigs and are not considered as reportable.

Which of the following breeds would be considered brachycephalic? Greyhound Pug Golden Retriever Chihuahua Dachshund

Pug A brachycephalic skull is relatively broad and short. Because of their conformation, dogs of this type more susceptible to respiratory problems. "Brachycephalic Airway Syndrome" includes elongated soft palate, everted laryngeal saccules, and stenotic nares (narrow nostrils). These animals are also more prone to ocular problems. The most common (and visually obvious) dogs in this class include the Pug, Shih Tzu, Bull dog, Pekingese, Boston terrier, and Cavalier King Charles spaniel.

A parrot presents because it is bleeding profusely from one of its feathers. What would likely be the best course of action to take after examination by the veterinarian? Cauterize the feather to stop bleeding Place a tight wrap around the feather shaft Pull the feather out Do not manipulate the feather and allow it to clot on its own

Pull the feather out Blood feathers are immature feathers that still have a blood supply to the shaft. The shaft usually appears dark as opposed to white or clear as would be seen in a mature feather. If one is cut or breaks from trauma, it can bleed profusely and not clot. The bird can actually die of blood loss in some cases. Typically, the best course of action is to pull the blood feather out. This is typically done using a hemostat and giving a firm pull.

A dog with bradycardia and a low albumin on a high rate of IV fluids is at risk for developing which of the following? Thromboembolism Syncope Pulmonary edema Seizures

Pulmonary edema Albumin provides oncotic pressure which helps maintain fluid in the vasculature. If the oncotic pressure is decreased from low albumin and movement of blood through the body is slow due to a low heart rate (bradycardia), fluid can build up in the lungs (pulmonary edema).

Oxygenated blood returning from the lungs travels in which blood vessel? Pulmonary vein Aorta Pulmonary artery Vena cava

Pulmonary vein Vessels that contain blood moving toward the heart are known as veins despite their oxygenation status. The pulmonary vein is responsible for returning oxygenated blood to the heart (via the left atrium) for distribution the rest of the body. The pulmonary artery contains deoxygenated blood from the right ventricle that is moving toward the lungs for oxygenation.

What is the ideal diet to feed a healthy lactating (nursing) dog? High carbohydrate food It is best to keep her on her normal diet Geriatric food Low protein food Puppy formula food

Puppy formula food Puppy food is higher in protein and is usually enriched with higher amounts of fats, vitamins, and minerals. A nursing mother needs about twice as much food than when she is not pregnant or nursing.

Newborn mice are often called which of the following? Colts Kits Pups Squabs

Pups Newborn mice are also referred to as pups or sometimes pinkies. They are pink and hairless and blind and deaf when born. Newborn rabbits are called kits and are also blind, deaf, and hairless when born. Ferrets are also called kits. Guinea pigs are also referred to as pups and are "precocious" when born. This means their eyes are open and they are aware and can see and hear and walk from birth.

When performing CPCR (cerebro-cardio pulmonary resuscitation) in lateral recumbency, how far should you press down on the chest during compressions? Push down about 1/3 of the chest width Push very lightly almost as if tapping on the chest Push down about 1/2 of the chest width Push all the way down so both sides of the ribs meet

Push down about 1/3 of the chest width

Ingesta must travel through which opening as it passes through the stomach into the duodenum? Anal sphincter Bile duct Pyloric sphincter Lower esophageal sphincter Duodenal papilla

Pyloric sphincter The pylorus connects the stomach to the upper small intestine. The lower esophageal sphincter is between the esophagus and stomach. The anal sphincter is muscle around the anus that helps with anal tone. The duodenal papilla is a raised area that marks the opening of the common bile duct and pancreatic duct into the duodenum. The bile duct helps connect the liver to the duodenum and carries bile from the liver to aid in digestion.

A pus-filled uterus is known as which of the following? Pyogenes Pyometra Pythiosis Pyoderma

Pyometra A pyometra is a uterus filled with pus and requires removal of the uterus surgically. The prefix "pyo" means pus.

You are assisting with thoracocentesis and the fluid removed from the chest has a very foul odor. Which of the following types of pleural effusions is most likely? Pyothorax Chylothorax Pneumothorax Hemothorax

Pyothorax Pyothorax is purulent fluid (pus) in the pleural space and most often has a very foul odor. Chylothorax is chyle (lymphatic fluid) in the pleural space. Hemothorax is blood in the pleural space. Pneumothorax is air in the pleural space.

What is the term used for an elevated number of white blood cells in a urine sample? Pyuria Stranguria Anuria Hematuria

Pyuria The correct answer is pyuria. Hematuria is the excessive presence of red blood cells in urine. Stranguria is difficulty or straining to urinate. Anuria is the lack of urine production.

Coxiella burnetti is the cause of what zoonotic disease in humans? Undulant fever Plague Cat scratch fever Q fever

Q fever

What is the name for the muscle group that includes the rectus femoris, vastus lateralis, vastus medialis, and vastus intermedius? Hamstrings Triceps Gluteals Quadriceps

Quadriceps "Quad" means four, and the Quadriceps femoris is a large group of muscles that cover most of the middle thigh. This muscle group extends the knee. The rectus femoris originates on the ilium and also flexes the hip. The other three muscles originate from the femur. All four parts attach to the patella through the quadriceps tendon. The hamstrings are the biceps femoris, semitendinosus, and semimembranosus muscles.

A urine protein:creatinine ratio may be requested for which purpose? Test for bilirubin in the urine Quantify urine protein Look for urinary tract infection Measure the glomerular filtration rate

Quantify urine protein Urine protein:creatinine ratio is helpful when quantifying the severity of proteinuria. Urine reagent test strips are not sufficiently quantitative, and the sulfosalicylic acid turbidity test merely provides a measure of the amount of protein in a single urine sample.

The sacculus rotundus is part of the intestinal anatomy of which species? Camel Rabbit Psittacine bird Horse

Rabbit Just proximal to the cecum is a gastrointestinal structure containing a large amount of lymphoid tissue called the sacculus rotundus which has some immune function. It is present only in lagomorphs.

Which species does not have canine teeth? Rabbits Horses Llamas Cows

Rabbits

Tularemia would most likely be carried by which animal? Goats Deer Guinea pigs Cows Rabbits

Rabbits Tularemia is caused by the bacteria Francisella tularensis and is known as rabbit fever. It is zoonotic, and the bacteria can gain entrance through a cut or scratch when handling an infected animal. It leads to a skin ulcer and then swollen glands, fever, headache, and rash. It can also be transmitted to humans via tick bites or deerflies.

A dog presents deceased to your clinic with foam coming from the mouth and his neck appears swollen. The owner says the dog had been sick for a few days and then had a seizure and died. They live on the outskirts of town on some acreage, and the dogs roam free on the property. What disease would you be concerned about? (Hint: You need to wear protective clothing before handling this pet) Brucellosis Rabies Distemper Leptospirosis

Rabies While you are not making a diagnosis, you should know that swollen neck area or enlargement of lymph nodes, foam from the mouth, and history of seizure and death may be symptoms of Rabies virus. While it is not very common, cases of Rabies are diagnosed every year in domestic pets. It is best to take precautions, get a history from the owner, and discuss with the veterinarian proper handling of the animal and if any testing will be required. Exposure of the owner or other people to the dog should be documented if Rabies is suspected; specimens will need to be sent to lab for testing.

Name one of the bones that is fractured in this radiograph. (small bone on front) Humerus Radius Metacarpal Tibia

Radius This is a radiograph of a forelimb. Notice the elbow and the carpus to help you get oriented. The humerus is the bone proximal (on the left side of the radiograph) which is connected to the scapula. The radius and ulna are the two bones fractured in this radiograph. The ulna is the smaller in diameter of the two bones.

For a centrifuge, the relationship between revolutions per minute (RPM) and relative centrifugal force (g or RCF) can be calculated based on which of the following? Time setting on the centrifuge Radius of the centrifuge head Amperes of the centrifuge Size of the tube in the centrifuge Watts of the centrifuge

Radius of the centrifuge head Many spin protocols indicate the RCF at which a tube needs to be spun. While some centrifuges may have an RCF setting, many can only be set in RPM. For a given centrifuge, the conversion factor is based on the radius of the centrifuge head from the center to the axis of rotation. The actual full formula for the conversion is RCF = 1.18 x .00001 x radius x rpm x rpm. This is probably not a formula that you need to memorize but you should be aware that it is the radius of the centrifuge that determines how much force is generated for a given RPM speed. A centrifuge with a larger radius will generate more force at a given RPM than a centrifuge with a smaller radius at the same RPM. None of the other factors listed influence the force generated.

The glomerular filtration rate (GFR) is directly related to which of the following? Albumin level in the blood Size of the kidneys Rate of blood flow to the kidneys Tubule secretion rate

Rate of blood flow to the kidneys Glomerular filtration rate (GFR) is a test used to check how well the kidneys are working by estimating how much blood passes through the glomeruli each minute.

When performing a major cross-match when a blood transfusion is necessary, which of the following samples are mixed? Recipient serum with donor neutrophils Recipient serum with donor erythrocytes Recipient serum and donor platelets Recipient platelets with donor serum Recipient erythrocytes with donor serum

Recipient serum with donor erythrocytes In a major cross-match, the recipient's serum is combined with the donor erythrocytes, and the sample is examined for agglutination or hemolysis, indicating incompatibility. A minor cross-match combines recipient erythrocytes with donor serum.

Which of the following is NOT considered a leukocyte? Neutrophils Red blood cells (RBCs) Macrophages Lymphocytes Eosinophils

Red blood cells (RBCs)

Which of the following is true about red blood cells? Red blood cells are exclusively produced in the spleen Red blood cells have a lifespan of over 2 years Red blood cells are normally nucleated in birds Nucleated red blood cells are never seen in dogs

Red blood cells are normally nucleated in birds Interestingly, red blood cells are normally nucleated in birds and reptiles. Nucleated red blood cells can be seen in other species whenever there is a demand on the body to rapidly release red blood cells into the bloodstream. If demand exceeds the bone marrow's production capabilities, the body will begin to release juvenile red blood cells, which still have a nucleus.

What do spherocytes look like under the microscope? Red blood cells that are spiculated White blood cells with blue round inclusion bodies White blood cells with solid round nuclei Red blood cells that are small, very dark, and perfectly round Red blood cells with blue round inclusion bodies

Red blood cells that are small, very dark, and perfectly round

Erythematous skin might also be described as: Red skin Thickened skin Itchy skin Scaly skin

Red skin Erythema is redness of the skin. It is usually caused by hyperemia (increased blood flow) to the affected area. Itchy skin is described as being pruritic. Lichenification is a descriptor for thickened skin.

A urinalysis should ideally be performed within 30 minutes of collection. How should urine be stored prior to evaluation if it cannot be evaluated immediately? In a 98 degree F water bath Refrigerate At room temperature Freeze

Refrigerate

Microbiology materials or media that are not in use should be stored where? Freezer In the dark room at room temperature Refrigerator In a lighted cabinet at room temperature

Refrigerator

In radiation safety, what is the difference between the "rad" and the "rem"? Rad applies a quality factor to rem Rem only applies to the patient, and rad only applies to personnel Rad only applies to the patient, and rem only applies to personnel Rem applies a quality factor to rad Rad is the old unit and rem is the new unit of absorbed dose Rem is the old unit and rad is the new unit of absorbed dose

Rem applies a quality factor to rad Rad and rem are both old units of types of absorbed dose that can be applied to either the patient or to personnel. More commonly, rem is used in radiation safety for personnel, and rad is used in radiation therapy in dose calculations. The difference between the two is that rad is the basic unit of absorbed dose, and rem applies a quality factor depending on the type of radiation. For x-rays, rad equals rem; but for other types of radiation such as neutrons and protons, rem may be greater than rad. Rem exposure is of greater relevance in radiation protection than rad because the biologic effects of 10 rad of neutron exposure is far greater than the biologic effects of 10 rad of x-ray exposure. The units that should be used currently are the Gray (Gy) and the Sievert (Sv) for absorbed dose and effective absorbed dose respectively, but many people still use rad and rem.

An ovariohysterectomy is one of the most common procedures performed in veterinary medicine. This procedure entails which of the following? Removal of the ovaries and uterus Removal of the ovaries only Removal of the uterine body only Tying off of both fallopian tubes

Removal of the ovaries and uterus

A Golden retriever ingests several tablespoons of antifreeze. This will cause the most damage to what organ system? Gastrointestinal Cardiac Renal Nervous Hepatic

Renal Ethylene glycol (EG), the primary ingredient in antifreeze, causes damage to the renal system (kidneys). EG causes damage to the renal tubular epithelium, causing build up of oxalate crystals in the renal tubules and metabolic acidosis.

Which of the following findings in a urine sample is associated with renal tubular damage or disease? Red blood cells Bilirubin White blood cells Renal casts

Renal casts Renal casts are suggestive of renal disease. There are several different types of casts including hyaline, cellular, and granular casts. The presence of a few hyaline casts may be a normal finding, but a large number of these should make the interpreter suspicious. Cellular casts can contain epithelium from the renal epithelium and are seen with acute tubular degeneration. Granular casts in large numbers may be indicative of acute renal disease. These contain fragments of cells or plasma proteins.

Of the following options, oral ulceration is most common in patients with which condition? Renal failure Pancreatitis Liver failure Diabetes

Renal failure

A 2-year old domestic short hair cat presents for ingestion of the flowers seen in the picture below. The ingestion occurred about 2 hours ago and no clinical signs have been noted yet. What does the ingestion of this plant cause in cats? Seizures Hypoglycemia Renal failure Muscle paralysis

Renal failure The flowers are Tiger Lilies. Lilies of all different varieties as well as their leaves cause acute renal failure in cats when ingested. Treatment is similar to any other cause for renal failure and includes high rates of IV fluids, supportive medications, and occasionally diuretics such as furosemide or mannitol if the patient is anuric or oliguric. Image used with permission, from Feline Medicine (Sparkes), courtesy of Manson Publishing.

A veterinarian has diagnosed a horse with a fracture of the distal metatarsal. In regards to emergency management, which of the following is NOT part of triage? Developing a treatment plan Stabilization of the fracture Repair of the fracture Assessment and treatment of pain

Repair of the fracture

Which of the following is the "minute volume" of a patient under anesthesia? Oxygen consumption multiplied by tidal volume Respiratory rate multiplied by lung capacity Tidal volume divided by lung capacity Respiratory rate multiplied by tidal volume

Respiratory rate multiplied by tidal volume Minute volume is the volume of air that is inspired by a patient in one minute. It is calculated by multiplying tidal volume by the number of breaths per minute. The tidal volume is the volume of air that is inspired on each breath. The respiratory rate is the number of breaths per minute. Tidal volume is usually considerably smaller than the total lung capacity.

Where is the sinoatrial node located? Right atrium Left ventricle Left atrium Ventricular septum

Right atrium The sinoatrial node provides automaticity to the heart and is located on the right atrium. The impulse then travels to the atrioventricular node which then conducts the impulse down the Bundle of His to the Purkinje fibers. This pattern of depolarization results in a very coordinated contraction of the heart, allowing for smooth blood flow.

Which is the correct direction through the heart in which blood flows? Right atrium, mitral valave, left atrium, tricuspid valve Left atrium, mitral valve, right atrium, aortic valve Right atrium, tricuspid valve, left atrium, mitral valve Right atrium, aortic valve, left atrium, pulmonic valve

Right atrium, tricuspid valve, left atrium, mitral valve The tricuspid and mitral valves are both atrioventricular valves that divide the right sided chambers and left sided chambers, respectively. To follow blood flow through the entire body: Blood flows from the right atrium, to the tricuspid valve, right ventricle, pulmonic valve, pulmonary artery, lungs, pulmonary vein, left atrium, mitral valve, left ventricle, aortic valve, aorta, then to the rest of the body.

Phenylbutazone (bute) is often used for pain relief in horses. What side effect has been associated with this drug? Right dorsal colitis Colonic impaction Esophageal stricture Laminitis

Right dorsal colitis Phenylbutazone is an injectable non-steroidal anti-inflammatory drug commonly used for the relief of inflammatory conditions associated with the musculoskeletal system in horses. This drug may cause right dorsal colitis associated with ulcerations in this specific region of the bowel.

A dog presents in critical condition and the veterinarian detects an abnormal heart rhythm. She asks you to get an ECG (electrocardiogram) on this patient. Which position should he be placed in for this reading? Left lateral recumbency Dorsal recumbency Sternal recumbency Right lateral recumbency

Right lateral recumbency An ECG should ideally be recorded on a patient in right lateral recumbency. An easy way to remember where leads are placed: You read your newspaper in the morning (black/white) front limbs Christmas comes at the end of the year (green/red) back limbs white/right rhyme grass grown on the ground (in right lateral, the limb on the "ground" or table in the right hindlimb) So: white: right forelimb black: left forelimb green: right hindlimb red: left hindlimb Not all ECG units will have all leads (the green one is sometimes omitted) but the positioning remains the same.

A 3-year old castrated male Doberman Pinscher comes in to the emergency clinic with signs consistent with gastric dilation and volvulus (GDV). Following standard initial emergency treatment for suspected gastric dilation and volvulus, what imaging procedure would be most helpful in confirming the diagnosis? Ventral dorsal abdominal radiograph Right lateral recumbency abdominal radiograph Abdominal ultrasound exam Dorsal recumbency abdominal radiograph Barium contrast abdominal radiograph Ventral recumbency abdominal radiograph

Right lateral recumbency abdominal radiograph A typical GDV occurs with repositioning of the pylorus to the left dorsal abdomen. A right lateral radiographic image is the best perspective position for revealing a-gas filled right dorsally-displaced pylorus with a gas-filled ventral fundus separated by a soft tissue band (compartmentalized stomach). Although the features of a malpositioned stomach may be observed on the other radiographic views, they are often more difficult to interpret than that represented by the right lateral recumbency view. The severe amount of gas within the GI tract that is associated with GDV would make ultrasound exam interpretation extremely difficult at best.

You are taking thoracic radiographs to evaluate for lung metastasis in a dog with splenic hemangiosarcoma. What radiographic views should be taken? A left lateral and a dorsoventral projection A right lateral and a ventrodorsal projection Dorsoventral, ventrodorsal, and either a right lateral or left lateral projection Right lateral, left lateral, and either a dorsoventral or ventrodorsal projection Either a dorsoventral or a ventrodorsal projection AND either a right lateral or left lateral

Right lateral, left lateral, and either a dorsoventral or ventrodorsal projection When a patient is placed into left lateral recumbency for a left lateral radiograph, the left lung fields are compressed and do not inflate fully. This can prevent resolution of small metastatic lung lesions due to the lack of contrast with air in the left lung. The same is true of the right lung fields with a right lateral radiograph. Therefore, for evaluation of pulmonary metastasis, both a left and right lateral radiograph should be taken. Either a ventrodorsal (VD) or dorsoventral (DV) view is acceptable for visualizing the lung fields in this case.

You are reading a cardiologist report discussing tricuspid valve insufficiency and grade 3 murmur. The mitral valve is said to be normal. If you listened to this patient, where would you expect to hear the murmur the loudest? Sternal Left side cranial portion of the chest Right side Left side caudal portion of the chest

Right side The tricuspid valve is located between the right atrium and right ventricle. Mitral murmurs are usually left-sided holosystolic murmurs.

Name the heart chamber responsible for pumping blood toward the lungs. Right atrium Right ventricle Left atrium Left ventricle

Right ventricle The right atrium receives deoxygenated blood from the vena cava. This blood is then pumped into the right ventricle (passing through the tricuspid valve) which then gets pumped into the lungs (passing through the pulmonary valve) via the pulmonary artery for oxygenation. The newly oxygenated blood returns to the left atrium via the pulmonary vein. Once in the left atrium, the blood is pumped into the left ventricle (passing through the atrioventricular valve) from which it is then pumped into the circulation via the aorta (passing through the aortic valve).

Which of the following conditions in rats results from living in an area with the humidity too low? Mycoplasma Ringtail Chronic Respiratory Disease Red tears

Ringtail

Which of the following instruments is best suited for removing bone, such as may be done when removing the dorsal spinous process during spinal surgery? Rongeur Rochester-Carmalt forceps Kern forceps Mallet

Rongeur The rongeur can have a single-action or double-action mechanism that allows for relatively easy removal of dense material such as bone. It comes with a sharp, scoop-shaped tip. The kern forceps is used to help hold bone fragments in reduction during a fracture repair. Rochester-Carmalt forceps are used to grasp masses of tissue or vessels. A mallet is a device similar to a hammer used in surgery.

What is the drug of choice for treating Tritrichomonas foetus in cats? Sulfasalazine Ronidazole Metronidazole Fenbendazole Sulfadimethoxine

Ronidazole The only proven effective treatment for treating T. foetus is ronidazole. It is a nitroimidazole antimicrobial that has mutagenic properties and should be handled as would a chemotherapeutic agent. Ronidazole is not licensed for use in cats and is thus used off-label for this parasite; it should only be used with caution and with informed, signed, owner consent.

Which of the following is an example of an endodontic treatment? Dental scaling Soft palate resection surgery Root canal Tumor removal from the tongue

Root canal Endodontics deals with the tooth pulp and the tissues surrounding the root of a tooth. The root canal is the most common endodontic procedure.

A cat has a skin lesion that is not improving with antibiotic therapy, and you assist the veterinarian in performing a biopsy and culture. The result shows Sporothrix schenckii. This is a zoonotic disease also known as what in humans? Rose gardener's disease Cat scratch fever Tularemia Tuberculosis

Rose gardener's disease Sporothrix is a cigar-shaped yeast and is typically found in soil and vegetation in coastal regions or river valleys. Infection may occur from direct inoculation of the organism into a skin wound during contact with plants or soil, or secondary to a penetrating foreign body. It is known as rose gardener's disease because the organism may gain entry to the body during gardening or digging in the soil. Cat scratch fever is caused by Bartonella henselae. Tularemia is caused by the bacterium Francisella tularensis and is most common in rabbits (rabbit fever). Tuberculosis in humans is caused by a mycobacterium, causing lung disease. Various mycobacteria also cause disease in animals, such as M. bovis, and M. avium.

A wound in front of the eye in a dog (toward the front of the nose) may be described as being which of the following? Caudal to the eye Ventral to the eye Dorsal to the eye Rostral to the eye

Rostral to the eye

On a bone marrow smear, which of the following is the most immature red blood cell precursor? Metarubricyte Rubriblast Reticulocyte Metamyelocyte

Rubriblast The erythorocyte lineage, from most immature to mature, proceeds as follows: Rubriblast, prorubricyte, basophilic rubricyte, polychromatophilic rubricyte, metarubricyte, reticulocyte. The metamyelocyte is part of the lineage of the segmented leukocytes.

The correct order of the ruminant digestive system in regards to the chambers of their stomach is: Abomasum, omasum, rumen, reticulum Reticulum, rumen, omasum, abomasum Rumen, reticulum, abomasum, omasum Rumen, reticulum, omasum, abomasum.

Rumen, reticulum, omasum, abomasum. This is the order of the "stomachs" in ruminants such as cattle. The abomasum is the true glandular stomach. The rumen is the largest chamber. The reticulum, which has a honeycomb type lining, is positioned up against the diaphragm and is the chamber where a magnet can be placed to try and prevent Hardware Disease. If the cow swallows a metal object, it falls into the reticulum where the magnet can attach to it and prevent the object (such as a nail or wire) from piercing through the diaphragm into the heart.

When a horse is feeling threatened or is resisting restraint, what is its first instinct to do? Raise up on its back legs Run away Kick with the back legs Bite

Run away Horses typically will try to run away first if possible, but if they are cornered and feeling threatened with no escape, they will also kick, bite, or rear up. When working with a horse that is not used to handling, it is important to have a plan of attack if the horse is not cooperating to avoid injury.

In what order does an electrical impulse travel through the heart? SA node, Purkinje fibers, AV node, Bundle of His AV node, SA node, Bundle of His, Purkinje fibers Bundle of His, SA node, AV node, Purkinje fibers SA node, AV node, Bundle of His, Purkinje fibers SA node, Bundle of His, AV node, Purkinje fibers

SA node, AV node, Bundle of His, Purkinje fibers

The parotid gland is responsible for making and secreting which of the following? Tears Saliva Epinephrine Oxytocin Sebum

Saliva The parotid is the largest of the salivary glands. Sebum is made by the sebaceous glands. Epinephrine is made by the adrenal glands. Oxytocin is made by the hypothalamus and released by the pituitary gland. Tears are produced by the lacrimal gland.

You volunteered at the zoo and spent the day handling the tortoises. A day later you have gastrointestinal symptoms. Reptiles are a major source for which of the following? E. coli 0157 Streptococcus Giardia Salmonella

Salmonella Salmonella is a Gram-negative rod of the family Enterobacteriaceae. It is transmitted via the fecal-oral route. Reptiles and sometimes birds can be chronic carriers of Salmonella. Turtles, iguanas, and snakes are especially known to harbor the bacteria.

Which organism causes equine protozoal myeloencephalitis (EPM) in the horse? Sarcocystis neurona Gasterophilus intestinalis Micronema deletrix Haemonchus contortus

Sarcocystis neurona EPM is caused by the aberrant migration of Sarcocystis neurona through the central nervous system of the horse. This causes variable clinical signs such as ataxia and muscle atrophy in the horse.

A pot-bellied piglet presents with a history of hair loss and severe itching. You help the doctor perform a skin scraping around one of the lesions on his side and you see the following under the microscope. What parasite is this? (Giardia) Hematopinus Demodex Otodectes Sarcoptes

Sarcoptes This is Sarcoptes scabiei which causes scabies- typically a very itchy disease. This piglet was treated successfully with 2 doses of Ivermectin. Otodectes is the ear mite, and Hematopinus is a louse. Demodex is a cigar-shaped follicular mite that is not typically itchy unless there are secondary infections present.

The pulse oximeter measures what variable? Saturation of hemoglobin with carbon dioxide Saturation of plasma with oxygen Partial pressure of oxygen in arterial blood Partial pressure of carbon dioxide in arterial blood Saturation of hemoglobin with oxygen

Saturation of hemoglobin with oxygen Pulse oximetry detects the amount of oxygen present on hemoglobin and is expressed as a percentage (i.e. 97% saturation). This monitor measures saturation by using the different wave lengths of saturated and desaturated hemoglobin. This is different than the amount of pressure that oxygen exerts on a vessel (i.e. the partial pressure of oxygen).

The shoulder joint links which two bones? Humerus and radius Scapula and humerus Humerus and ulna Radius and ulna

Scapula and humerus The shoulder joint connects the scapula and the humerus. The head of the humerus sits at glenoid cavity of the scapula to form the shoulder joint. The elbow would connect the humerus to the radius and ulna.

Which of the following represents the source of the most radiation exposure to a veterinary technician who is restraining a cat for a radiograph, taking the exposure, and processing the film? Scatter from the patient area Radiation from the radiograph processing equipment Scatter radiation from the collimator housing Radiation from standing too close to the cathode

Scatter from the patient area

Which test checks for keratoconjunctivitis sicca? Tonometry Schirmer Tear Test Nasolacrimal flush Fluoroscein stain

Schirmer Tear Test The Schirmer Tear Test is the test for keratoconjunctivitis sicca (KCS), or most commonly known as "dry eye". Tonometry is the measurement of intraocular pressures and tests for glaucoma, which is increased pressure in the eye. Fluoroscein stain checks for corneal ulcers or abrasions; it is a yellow dye that these defects will uptake on the cornea and can be seen under a blue light. Nasolacrimal flush is used to flush/clear out the nasolacrimal duct (tear duct) if it becomes clogged.

A red blood cell fragment, as might be seen from shearing of the cell by trauma in an animal with disseminated intravascular coagulopathy, is known as what? Keratocyte Echinocyte Spherocyte Acanthocyte Schistocyte

Schistocyte A schistocyte is a red blood cell fragment that is usually formed due to shearing from intravascular trauma; they are seen in cases of DIC or vascular neoplasms such as hemangiosarcoma. An acanthocyte is a red blood cell with surface projections that are seen mainly in animals with altered lipid metabolism or liver disease. An echinocyte is a red blood cell with uniform blunt projections that are usually artifacts from blood drying but may also be seen with some diseases. A keratocyte is a red blood cell that appears to contain a vacuole; they are sometimes called blister cells and may occur from intravascular trauma. A spherocyte is a dark-staining red blood cell with no central pallor that usually occurs from partial phagocytosis of the cell seen with immune-mediated destruction.

Which structure are you most concerned about when giving an injection in a hindlimb? Sciatic nerve Sacrotuberous ligament Femoral nerve Femoral artery

Sciatic nerve The sciatic nerve starts where the lumbar spine ends and the sacral spine begins. It is located on the underside of the L6-S1 vertebrae. It travels through the ileum and continues along the back and upper thigh area. Care must be taken when giving injections (especially intramuscular injections) in this area as damage to the sciatic nerve can occur.

Which structure is the white part of the eye? Sclera Lens Cornea Optic nerve Vitreous

Sclera The sclera is the white fibrous tissue that extends from the cornea (the clear surface of the eye) to the optic nerve in the back of the eye.

If a veterinarian says an animal has pruritus, which of the following clinical signs is the animal likely displaying? Coughing Diarrhea and vomiting Polyuria and polydipsia Scratching Lethargy

Scratching Pruritus is the term indicating that an animal is itchy. Scratching is the most common sign of pruritus. Others might include rubbing or head shaking, depending on the location of the pruritus.

A fracture has just been repaired with a 3.5 mm screws. What is the 3.5 mm in reference to? Pitch Thread type Screw diameter Screw length

Screw diameter The number is in reference to the screw diameter. The pitch of a screw describes the number or distance between each thread. The two main types of thread used are cortical and cancellous. Cortical screws are designed for dense bone; cancellous screws are made for softer bone. The screw length is also measured in millimeters. The most common sizes used in small animal surgery are 2.0, 2.7, and 3.5 mm screws. In large animal surgery 4.5, 5.5, and 6.5 mm diameter screws are used.

The sac containing the testicles is known as which of the following? Epididymis Scrotum Cremaster Glans

Scrotum

Which of the following arrhythmias is common in horses and can be alleviated by exercise? Third degree AV block Ventricular Fibrillation Second degree AV block Atrial Fibrillation

Second degree AV block Second degree AV block is a very common arrhythmia in adult horses as a result of high vagal tone. This arrhythmia is "regularly irregular" and often alleviates with exercise. It is more common in athletic horses. Atrial fibrillation is an irregular arrhythmia and a more serious condition that occurs when the SA node isn't firing properly. Electrocardiogram is the best way to determine atrial fibrillation by identifying a lack of p waves. It is treated with Quinidine, an antiarrhythmic. Ventricular fibrillation is a severely abnormal heart rhythm that is most often identified in patients that have undergone cardiac arrest. Third degree AV block, or complete heart block, is rare in horses and usually associated with degenerative or inflammatory changes in the heart. Horses with this condition usually have exercise intolerance or syncope.

A 4-month-old Angus calf in a beef herd in Northern California has died. Symptoms before death included lethargy and coughing. You assist with a necropsy and find pale areas in the cardiac muscles (see image). The veterinarian diagnoses white muscle disease. He asks you to assist with collecting blood samples from the herd to test for what deficiency? Copper Selenium Vitamin A Potassium

Selenium Nutritional myodegeneration (white muscle disease) is associated with low dietary selenium levels and compounded by low dietary vitamin E. Unsupplemented cattle grazing soils of volcanic origin are at high risk, and calves born in these herds may show skeletal or cardiac muscle syndromes. Measurement of whole blood selenium in some of the cows from the herd will indicate if the herd needs supplementation with selenium.

Clomipramine is a veterinary approved drug used most commonly to treat which condition? Separation anxiety Epilepsy Allergies Urinary incontinence

Separation anxiety Clomipramine (Clomicalm) is most often used to treat generalized anxiety or separation anxiety in small animals. It is also approved for obsessive compulsive behaviors in dogs. Chlorpheniramine sounds similar, and is an anti-histamine used in treating allergies. The most commonly prescribed medication for urinary incontinence is phenylpropanolamine. Potassium-bromide and phenobarbitol are commonly used medications for epilepsy.

A foal has petechial hemorrhages on its pinna, injected sclera, and brick red mucous membranes. These are signs of which of the following? Cryptosporidiosis Sepsis Failure of passive transfer Milk allergy

Sepsis Depression, cold limbs, aural petechiations, injected tongue and mucous membranes, hypoglycemia, increased respiration, decreased suckling of the mare, hypothermia, and recumbency are all symptoms of sepsis in the foal. Failure of passive transfer predisposes the foal to sepsis.

After performing intestinal surgery, there is a small possibility of complications. What would be the consequence of an intestinal dehiscence? Inflammatory bowel disease Persistent diarrhea and intermittent vomiting Sterile peritonitis Septic abdomen

Septic abdomen The intestines are lined with bacteria, and any contamination of intestinal contents within the abdomen may result in bacterial colonization and, subsequently, septic peritonitis (septic abdomen). The peritoneum is the lining of the abdomen, and the "itis" implies inflammation. The process is secondary to bacterial contamination and is therefore not a sterile peritonitis. Inflammatory bowel disease is an autoimmune small intestinal disease that both dogs and cats may develop. Vomiting may be a clinical sign of a septic abdomen, and a cause should be identified if vomiting occurs after surgery.

Which of the following types of shock may cause brick-red mucous membrane color? Cardiogenic Hypovolemic Obstructive Septic shock

Septic shock Septic shock, sometimes called distributive shock, may cause brick-red or injected mucous membrane color and bounding pulses. This is because in cases of sepsis, the bacteria in the bloodstream cause severe vasodilation, which leads to hyperemic gums and low blood pressure. Septic shock may be caused by infectious diseases or by secondary infections. Cardiogenic shock is caused by decreased cardiac output. It may occur from primary heart diseases like cardiomyopathy, arrhythmia, or valvular disease. Obstructive shock is a type of cardiogenic shock, caused by an obstruction of blood flow. Examples of this include heartworm, pulmonary thromboemolism (PTE), or pericardial disease. Hypovolemic shock is caused by decreased perfusion, especially from blood loss or dehydration. Hypovolemic shock is the most common cause of shock in small animals.

Which of the following is FALSE regarding the use of Humulin insulin? Shake well before use It can be given intramuscularly It causes blood sugar to decrease when administered Keep refrigerated

Shake well before use Most Insulin types should never be shaken. Insulin should only be gently rolled prior to use. The exception to this is Vetsulin, which should be shaken prior to use. Insulin should be refrigerated, and although typically given subcutaneously, it can be given intramuscularly and is often given this way in cases of diabetic ketoacidosis. Insulin causes a drop in blood glucose.

What species, if overworked, is most at risk for hyperthermia? Horses Cats Cows Rats Sheep

Sheep Sheep are at risk for hyperthermia due to their thick coat. A sheep's normal body temperature is around 102-103F. Pigs are also at risk for hyperthermia due their increased amount of body fat.

Which breed should not receive ivermectin for treating Demodex? Boxer Chihuahua Pit Bull Sheltie

Sheltie Ivermectin should not be given to collie-type breeds due to a possible MDR1 gene mutation that could cause the drug to be toxic. Breeds included for possible ivermectin toxicity include Collies, Shelties, and Australian Shepherds. The saying is "white feet, don't treat".

Patients that have received prednisone chronically need to be tapered off slowly to avoid which of the following? Signs of exocrine pancreatic insufficiency Signs of hypoadrenocorticism Signs of hyperinsulinism Signs of hyperadrenocorticism Signs of diabetes insipidus

Signs of hypoadrenocorticism Chronic prednisone administration can lead to atrophy of the adrenal gland due to the absence of signaling to produce cortisol. If prednisone is suddenly stopped, the adrenal gland will not be capable of producing enough cortisol, and the animal may show signs of hypoadrenocorticism (Addison's disease).

What is the metallic element present on radiograph film that turns black when exposed and run through the radiograph development process? Zinc Lead Magnesium Tungsten Silver Gold

Silver Films go into developer first which reduces exposed silver halide into elemental metallic silver which is black. Crystals that were unexposed do not undergo this process. Fixer then coverts the unexposed crystals into a soluble form so that they dissolve away, leaving a clear image where the film was unexposed.

Electrical activity that serves as the pacemaker in the heart originates from what specific area? Sinoatrial node in the left atrium Bundle of His in the right ventricle Bundle of His in the left ventricle Atrioventricular node in the right atrium Atrioventricular node in the left atrium Sinoatrial node in the right atrium

Sinoatrial node in the right atrium In health, the pacemaker of the heart is the sinoatrial node within the right atrium. This impulse is transmitted to the atrioventricular node located at the top of the interventricular septum, followed by the bundle of His (within the walls of the interventricular septum) and through the Purkinje system.

You are asked to restrain a dog for a cephalic draw. How are you going to hold the dog? Dorsal recumbency and stabilize the hindlimbs Sitting position, holding the head steady and pointing the muzzle upward Sitting position, holding off and stabilizing a forelimb Lateral recumbency, holding off and stabilizing the lateral side of the pelvic limb

Sitting position, holding off and stabilizing a forelimb

A dorsoproximal-dorsodistal oblique radiograph of the carpus is frequently referred to as what type of radiograph? Bisecting angle Horizontal beam Flexed lateral view Cross-sectional view Skyline view

Skyline view With a standing patient, a skyline view is taken with the carpus flexed and the x-ray cassette placed along the anterior aspect of the limb. The x-ray beam is aimed from proximally to distally and an oblique angle. This technique is used in equine radiography most frequently to view the distal radius, the proximal row of carpal bones, and the distal row of carpal bones where chip or slab fractures commonly occur, especially in racehorses. Sclerosis of the carpal bone can also be visualized here. This technique decreases the overlap of surrounding bones and structures in order to improve the ability to see lesions specifically in this area. A flexed lateral radiograph also involves flexing the carpus, but the beam is aimed from laterally and the cassette is placed medially. This technique allows the carpal bones to separate and makes the margins of the bone more clearly visible. A horizontal beam radiograph generally is used to confirm the presence of air or fluid in a body cavity (thorax, abdomen). A bisecting angle radiograph is a technique used to prevent distortion when it is not possible to place the film parallel to the structure of interest and perpendicular to the x-ray beam, commonly in dental radiography.

A drug that is a cholinergic agent will have which of the following effects? Cause central nervous system depression Dilate the pupil Slow the heart rate Decrease salivation Decrease intestinal motility

Slow the heart rate Cholinergic agents are drugs that mimic the effects of acetylcholine within the autonomic nervous system. These drugs stimulate the parasympathetic nervous system. They promote salivation, lacrimation, urination, and defecation (sometimes abbreviated SLUD or referred to as "rest and digest" functions). They can cause pupil constriction or decreased heart rate. A commonly used cholinergic drug is bethanechol which is structurally similar to acetylcholine but resists degradation. It is used to treat gastric atony or urinary retention in the bladder.

Stimulation of the Vagus nerve results in which of the following? Diarrhea Dilated pupils Hypersalivation Tachycardia Slowed heart rate

Slowed heart rate Vagal stimulation slows the heart rate.

You are working at the specialty center and are observing an echocardiogram for the first time. The cardiologist prescribes the drug propanolol to a canine patient. What effect will this medication have on the heart? Stops ventricular premature contractions Increases the heart rate Increases contractility Slows the heart rate

Slows the heart rate Propanolol is a Beta-1 and Beta-2 blocker. It blocks the sympathetic nervous system (sympatholytic) which allows for the parasympathetic effects on the heart to dominate. The parasympathetic nervous system slows the heart. Remember- parasympathetic- rest and digest (slower heart rate), and sympathetic- fight or flight (increased heart rate).

You are asked to do a direct smear on a fecal sample of a patient with diarrhea. What is the negative aspect of a direct smear to identify parasites? Small sample size Creates too much mess Parasite eggs too fragile Takes too long

Small sample size

A 13-year old diabetic cat arrests during hospitalization during a blood glucose curve. During resuscitation efforts, an emergency blood panel is run which reveals a blood pH of 6.4. What drug may be requested by the veterinarian to help correct this abnormality but would not be used as a long-term therapy? Insulin Methylprednisolone Potassium gluconate Sodium bicarbonate Darbopoetin

Sodium bicarbonate A blood pH of 6.4 is low, or acidic. Sodium bicarbonate can increase the cat's blood pH but ultimately the underlying problem (the diabetes) must be treated. Insulin may be used in both resuscitation efforts and long-term therapy. Darbopoetin causes an increase in red blood cell production and has no effect on blood pH. Potassium gluconate is administered orally to correct hypokalemia. Methylprednisolone can cause an increase in blood glucose levels and is not a drug used to correct pH. Corticosteroids such as this should be avoided in diabetic patients when possible.

A double-contrast cystogram involves which of the following? Positive-contrast media injected directly into the bladder and air injected intravenously Soluble positive-contrast media and air injected directly into the bladder Air only injected directly into the bladder Soluble positive-contrast media only injected directly into the bladder Positive-contrast media injected intravenously that will be excreted via the kidneys into the bladder and air injected directly into the bladder

Soluble positive-contrast media and air injected directly into the bladder Double-contrast cystograms are typically performed by retrograde injection via catheter of a soluble positive-contrast agent and air.

The California Mastitis Test (CMT) is an assay that gives a score that corresponds to which of the following? Presence of contagious bacteria in milk Presence of environmental bacteria in milk Fat content of milk Somatic cell count in milk

Somatic cell count in milk A CMT involves adding a test reagent that reacts with the DNA in somatic cells to cause a gelling reaction. The score of this test corresponds to the somatic cell count in the milk. A higher score indicates the presence of more somatic cells and is indicative of mastitis.

Which species is particularly aggressive when with her young, and extreme caution must be used? Ewes Mares Sows Cows

Sows In general, pigs are a more aggressive species. The lactating sow is particularly dangerous; and when handling piglets, it is best to remove the sow to a separate area where she cannot hear her piglets.

A red blood cell that stains darkly with no central pallor is called what? Spherocyte Schistocyte Echinocyte Keratocyte Acanthocyte

Spherocyte A spherocyte is a dark staining red blood cell with no central pallor that usually occurs from partial phagocytosis of the cell as is seen with immune-mediated destruction. A schistocyte is a red blood cell fragment that is usually formed due to shearing from intravascular trauma and they are seen in cases of DIC or vascular neoplasms such as hemangiosarcoma. An acanthocyte is a red blood cell with surface projections that are seen mainly in animals with altered lipid metabolism or liver disease. An echinocyte is a red blood cell that is speculated with uniform blunt projections that are usual artifacts from blood drying but may be seen in some diseases. A keratocyte is a red blood cell that appears to contain a vacuole; they are sometimes called blister cells and they may occur from intravascular trauma.

The in-house blood chemistry machine broke down this morning and won't be repaired until late afternoon. What should you do with the laboratory samples? Spin down and freeze Spin down serum tubes and refrigerate the samples Freeze the samples The samples are stable at room temperature for up to 12 hours

Spin down serum tubes and refrigerate the samples Samples can be stable for around 4 to 7 hours at room temperature, but if there is a delay in processing they should be refrigerated. All serum tubes should be spun down as soon as the clot has formed in the tube for most accurate results. Samples can be refrigerated for 5 to 7 days. Frozen samples can last longer but freezing may change some test results and should be avoided if possible. In all cases, it is best to run the bloodwork as soon as possible.

What is the biggest risk when restraining a rabbit? The rabbit can easily escape Inguinal hernia Spinal cord trauma Broken ear

Spinal cord trauma

What is the best type of feeding/drinking bowls to give to patients staying in your hospital? Glass Bowls Plastic Bowls Ceramic Bowls Stainless Steel Bowls Paper Bowls

Stainless Steel Bowls

Mannitol Salt Agar, or MSA, selects for growth of which species of organism? Clostridium Campylobacter Streptococcus Staphylococcus Escherichia coli

Staphylococcus This media contains a high concentration of salt, making it mostly selective for Staphylococci, since this level of NaCl inhibits most other bacteria. This media contains the indicator phenol red.

It is a very busy day at the clinic. Multiple emergencies walk in all at the same time. Choose the order in which the following cases should be seen. Status epilepticus, dyspnea, fracture, stranguria, cervical pain, abscess Dyspnea, status epilepticus, stranguria, cervical pain, fracture, abscess Dyspnea, fracture, status epilepticus, cervical pain, abscess, stranguria Stranguria, status epilepticus, dyspnea, fracture, abscess, cervical pain

Status epilepticus, dyspnea, fracture, stranguria, cervical pain, abscess In a perfect situation both the seizuring pet and the pet with difficulty breathing would be treated immediately. Sometimes this may not be possible and triage is necessary. Status epilepticus is a state of seizuring that is continuous. This is life-threatening, and the seizures must be stopped immediately or permanent brain and organ damage can occur. Dyspnea or difficulty breathing is always a top priority emergency. The dyspneic animal should be assessed and placed on oxygen while the seizuring dog is being treated. Fractures, while painful, are not immediately life-threatening as long as there is no bleeding vessel that needs to be controlled, which is sometimes the case with an open fracture. Straining to urinate is not usually life-threatening unless it is a "blocked cat", or urethral obstruction. If this were a blocked cat, it should be seen before the fracture. Neck pain and abscesses are not life-threatening and should be seen last if the patients are stable.

Sometimes drugs, such as phenobarbital, are given in a higher dose as a loading dose and then tapered back to a maintenance dose. This helps the drug to accumulate to a therapeutic level in the body until the concentration plateaus. When this occurs, the drug has reached which of the following? Nadir Half-life Bioavailability Steady state

Steady state The steady state is the accumulation of a drug concentration in the body until the concentration plateaus. A nadir is a low point, often used to describe the lowest blood sugar during a glucose curve. Half-life describes the time it takes for the blood concentration of a drug to drop to half of its steady state. The bioavailability is the degree to which an administered drug is absorbed. The drug may be absorbed even if peak concentrations or steady state has not yet been reached.

You are assisting with an exploratory surgery, and the dog has a septic peritonitis from a perforated bowel. Before closure, the veterinarian lavages the abdomen with what? Sterile Lactated Ringer's solution Sterile physiologic saline with betadine Sterile physiologic saline Sterile physiologic saline infused with antibiotics

Sterile physiologic saline Copious lavage of the abdomen with sterile physiologic saline solution is performed prior to closure of the abdomen.

After finishing a routine enterotomy, the surgeon prepares to perform a leak check of the site. What instruments should you hand the surgeon? Sterile red rubber catheter and saline Sterile syringe, needle, and saline Sterile saline and the appropriate sized suture Sterile suction tip and gauze

Sterile syringe, needle, and saline

A dog has torn his cranial cruciate ligament and is having surgery. What area should you clip and aseptically prep? Shoulder Stifle Elbow Abdomen Hip

Stifle The correct answer is stifle. An extracapsular repair involves placing a synthetic suture material like heavy gauge fishing line spanning in a similar orientation as the cranial cruciate ligament, except the material is outside the joint. This is a very common surgery performed in small breed dogs with ruptured cranial cruciate ligaments.

The main purpose of stimulating the perineal area of puppies and kittens is to: Help comfort them when orphaned Stimulate the suckle reflex Stimulate respiration at birth Stimulate urination and defecation

Stimulate urination and defecation Orphaned puppies and kittens need manual stimulation of the perineal area to help them to urinate and defecate. This mimics the mother dog licking the region to stimulate evacuation.

What type of muscle is the heart? Smooth muscle, involuntary Striated muscle, involuntary Smooth muscle, voluntary Striated muscle, voluntary

Striated muscle, involuntary The three muscle types are cardiac, smooth, and skeletal. Cardiac muscle cells are located in the walls of the heart. This is striated muscle that is under involuntary control. Smooth muscle fibers are located in walls of hollow visceral organs, except the heart and are spindle-shaped. This type is also under involuntary control. Skeletal muscle fibers are striated and are in muscles attached to the skeleton. They are under voluntary control.

Which of the following conditions would cause the bowel to appear plicated (accordion)? String foreign body Parvovirus Inflammatory bowel disease GI stasis

String foreign body

Infection of foals with the parasite whose egg is shown in the image below can be associated with diarrhea in young horses. What is the parasite? The microscopic image was taken at 40X; the egg is approximately 50 um. Oxyuris equi Parascaris equorum Strongyloides westeri Strongylus vulgaris

Strongyloides westeri This image shows a Strongyloides westeri egg. Strongyloides pass from the host in the larvated form and are recognizable as an oval-shaped, thin-shelled embryonated egg. Larvae of Strongyloides westeri are transmitted to foals in the mare's milk. Adult horses rarely have patent infections except when larvae harbored in their tissues migrate into a mare's milk after parturition. The worms are found in the small intestine and may cause diarrhea in young horses. Ivermectin or oxibendazole are effective in treatment of S. westeri.

Which of the following is NOT considered a clinically important large strongyle in the horse? Strongylus edentatus Strongylus equorum Strongylus equinus Strongylus vulgaris

Strongylus equorum Strongylus vulgaris, Strongylus edentatus, and Strongylus equinus are the large strongyles of horses. Strongylus equorum does not exist. Although Parascaris equorum is a relevant parasite (round worm) in foals,

A 4-year old male Thoroughbred horse presents for colic due to verminous arteritis caused by an equine parasite. An egg of this parasite is shown below. What is the parasite that causes verminous arteritis of the cranial mesenteric artery in horses? Strongylus vulgaris Strongyloides westeri Taenia coli Trichostrongylus axei

Strongylus vulgaris Colic with an associated painful mass at the root of the mesenery is seen with verminous arteritis, caused by damage to the cranial mesenteric artery and its branches by Strongylus vulgaris. The strongyle egg shown in the picture is an extra hint. A number of anthelmintics are effective including benzimidazoles, pyrantel and ivermectin.

What organism is verminous arteritis associated with in the horse? Salmonella Parascaris equorum Cryptosporidium Strongylus vulgaris

Strongylus vulgaris S. vulgaris is a large strongyle that causes arteritis and thrombosis of the vessels of the GI tract in the horse and eventually can result in segmental ischemia or infarction of the bowel wall. This, in turn, results in signs of colic in the horse. Fortunately, with the advent of modern anthelmintics, this form of colic is not common.

What type of crystal tends to form in urine that is alkaline and typically appears as a 6-8 sided prism with tapering sides? Calcium oxalate Ammonium biurate Calcium carbonate Struvite

Struvite Struvite crystals are sometimes described as resembling a coffin lid, although they may sometimes have a slightly different appearance. They are composed of magnesium ammonium phosphate. Calcium carbonate crystals are round with radiating lines from the center; they may also have a dumbbell shape. Calcium oxalate crystals appear as small squares with an "x" across them. Ammonium biurate crystals are round with long, irregular spicules.

On a routine urinalysis an abundant amount of colorless prism-like crystals that have a coffin lid appearance are observed. What type of crystals are these? Calcium oxalate Ammonium biurate Struvite Cysteine

Struvite Struvite crystals are clear/colorless and have a rectangular appearance. They are commonly described as having a "coffin lid" appearance. Ammonium biurate crystals are round and brownish in color. They have spicules coming off of them. Calcium oxalate crystals have a clear-colored square appearance with an X across the surface.

The veterinarian asks you to please administer an FVRCP vaccination to a cat that is boarding. Where should you administer the vaccination? Subcutaneous at the intrascapular region Subcutaneous on the lateral aspect of a forelimb as distal as possible Into the epaxial muscles on either side Subcutaneous at the nape of the neck

Subcutaneous on the lateral aspect of a forelimb as distal as possible Because of the risk associated with feline vaccine-associated sarcomas, it is recommended to give feline vaccines on the lateral aspect of a limb as distal as possible. The intrascapular region and nape of the neck area should be avoided.

The interns at the hospital where you work are required to "SOAP" their patients daily. What does this stand for? Subjective, Outline, Action, Plan Subjective, Objective, Assessment, Plan Symptoms, Outline, Assessment, Prognosis Symptoms, Outline, Assessment, Plan

Subjective, Objective, Assessment, Plan

How are bladder stone types identified? Look for crystals on the urinalysis Gross examination Check the urine pH Submit to lab for analysis

Submit to lab for analysis The only way to know what a bladder stone is composed of is through lab analysis of its composition. Sometimes we can make an educated guess based on the appearance of the stones grossly, on a radiograph, crystals in urine, or based on urine pH, but none of these things are definitive and can be unreliable.

What is the best way to administer long-term medications to the eye of a horse with a severe corneal ulcer? Sub-conjunctival injection Subpalpebral lavage system Sedation Manual direct application of the medication

Subpalpebral lavage system The horse has very strong palpebrae (eyelids) that tend to spasm and close when they are manually manipulated, especially if the eye is painful from a corneal ulcer. The subpalpebral lavage system is placed under the eyelid (not in contact with the cornea) and a long tube extends to the withers. This allows the frequent administration of medications without touching the eye. This is the best method if long term eye medications are necessary.

Which medication is considered a gastric "band-aid"? Metoclopramide Sucralfate Metronidazole Pepcid-AC

Sucralfate Sucralfate (Carafate) acts as a sticky paste that binds with proteins found in ulcers and adheres there like a band-aid to aid in protection and healing. Pepcid (famotidine) is an antacid that helps to decrease the gastric acid that can cause an ulcer to occur. Metoclopramide is a pro-motility agent that helps to reduce vomiting. Metronidazole is an antibiotic often used in treating diarrhea.

Which stain is used to look for lipids or fats and is helpful in looking for intestinal malabsorption? Sedi-stain Sudan stain Hematoxylin and eosin stain Silver stain Sulphur dye

Sudan stain

In the horse, the condition known as Sweeney is caused by damage to which structure? Radial nerve First cervical vertebrae Brachial plexus Suprascapular nerve

Suprascapular nerve Sweeney is the term for atrophy of the shoulder muscle in the horse. It is caused by damage to the suprascapular nerve. This nerve is responsible for innervation to the infraspinatus and supraspinatus muscles, which are found in the scapula (shoulder blade).

Which of the following surgical instruments should never be autoclaved? Surgical towels Gauze sponges Mosquito forceps Scalpel blades Surgical drills

Surgical drills Autoclaves can be used to sterilize drapes, gowns, gauze, glassware, and most surgical instruments such as forceps, clamps, scissors, and scalpels. Heat-sensitive items that must be sterilized by other means include fiber-optic equipment, lenses, plastic, electric and pneumatic devices.

The veterinarian you work for is referring a Cocker Spaniel for an ear canal ablation procedure. What will this procedure entail? Opening of the tympanic membrane and flushing of the middle ear Surgical excision and closure of the external ear canal A deep flush and culture of the ear canal Ligation of the 8th cranial nerve

Surgical excision and closure of the external ear canal

A feline patient presents for onychectomy. What does this entail? Bladder stone removal Surgical nail removal Nail trim Anal gland expression Neuter

Surgical nail removal Onychectomy, mostly known as declawing, is an operation to surgically remove an animal's nails by means of amputating the distal phalanx.

What are the "Joints" in the skull are referred to as? Planes Ball and socket Hinge Sutures

Sutures Sutures are fibrous bands of tissue that connect the bones of the skull. In neonates, the sutures are open and allow for growth of the brain and skull.

Bandages are commonly placed on the legs of animals after surgical procedures to help prevent swelling and protect the incision. They can also be used to help stabilize fractures or protect wounds. Which of the following is NOT a sign of a potential bandage complication when monitoring the bandage of a forelimb bandage? Soiling of the bandage Slipping of the bandage Swelling of the toes Swelling of the neck

Swelling of the neck Swelling of the neck would definitely be alarming but would not be associated with a bandage complication. Bandage complications are usually seen as a result of slippage, wetting, or soiling, which result in bandage sores. Additionally, slippage or improper placement can result in constriction and subsequent problems with circulation. If circulation is interrupted, a tourniquet effect may occur; this can be a very serious complication resulting in loss of the limb if gone unchecked.

Blood pressure increases as blood vessels constrict. Which system causes blood pressure to increase when stimulated, and which corresponding stimulus could induce this response? Sympathetic nervous system, fear Parasympathetic nervous system, digestion Central nervous system, seizure Parasympathetic nervous system, exercise Sympathetic nervous system, digestion

Sympathetic nervous system, fear The sympathetic nervous system comprises the "fight or flight" response of the nervous system; the parasympathetic nervous system comprises the "rest and digest" portion. When our blood pressure rises (think of fear-induced increases in heart rate and blood pressure), we are stimulating our sympathetic nervous system. Exercise also causes these increases during activity and is a stimulant of the sympathetic nervous system.

The mucin clot test can be used when evaluating which fluid type? Peritoneal fluid Synovial fluid Cerebrospinal fluid Serum

Synovial fluid

A cat presents in respiratory distress and displays very rapid and shallow breathing. Which of the following is an accurate description of this breathing pattern? Agonal Tachypneic Obstructive Tachycardic

Tachypneic Tachypnea is rapid breathing. Tachycardia is a rapid heart rate. Agonal breathing is characterized by shallow, slow, irregular inspirations followed by irregular pauses. Obstructive breathing is characterized by a slow inspiratory phase followed by a rapid expiratory phase and can sometimes have respiratory noise (i.e. stertor) associated with breathing.

A puppy presents for his caudectomy procedure. What is being performed? Tail removal Soft palate repair Dewclaw removal Testicle removal

Tail removal

A patient under general anesthesia has become hypoxic. You suspect that the endotracheal tube might be in too far. How can you be sure? Move the endotracheal tube cranially an inch and check to see if the hypoxemia resolves Take an endotracheal tube with a similar outside diameter, line up the connector on the proximal end, and then lay it against the outside of the patient approximately following the route of the trachea If the endotracheal tube connector is past the incisors, it is in too far Take an endotracheal tube with a similar outside diameter, line up the measurement numbers on the proximal end of the tube, and then lay it against the outside of the patient approximately following the route of the trachea

Take an endotracheal tube with a similar outside diameter, line up the measurement numbers on the proximal end of the tube, and then lay it against the outside of the patient approximately following the route of the trachea The endotracheal tube's distal end should not extend past the point of the shoulder, and its proximal end should not extend past the incisor teeth. Proper placement insures that the animal is not endobronchially intubated which can cause hypoxemia and one-sided lung collapse. Lining up the connectors on the proximal end of the endotracheal tube may give you a false measurement because endotracheal tubes can be cut shorter to lessen the dead space on smaller patients. Pulling the endotracheal tube cranially without measuring could cause the patient to become extubated, making matters worse. If the endotracheal tube connector is past the incisors, it doesn't necessarily mean that it in too far. The tube might have been cut shorter for a previous patient to lessen dead space and may not be long enough for another patient. The distal end might be perfectly placed.

What is the name of the reflective layer behind the retina that allows animals to have improved night vision? Cornea Tapetum Lens Optic nerve

Tapetum

A healthy 1-year old Labradoodle presents for a wellness exam. You are asked to evaluate the dog's fecal float; you find the structures seen in the image below (shown from 40X magnification, each is about 40 um in diameter). What are they? (oval shaped) Giardia cysts Giardia trophozoites Roundworm eggs Hookworm eggs Tapeworm eggs

Tapeworm eggs

The "hock joint" of a horse is more appropriately identified as what structure? Metatarsophalangeal joint Tarsocrural joint Distal interphalangeal joint Tarsometatarsal joint

Tarsocrural joint The hock joint, more specifically called the tarsocrurual joint in the horse, is a high-motion joint composed of numerous bones.

Which essential amino acid is a requirement in the feline diet? Arginine Leucine Taurine Lysine

Taurine

Telazol is commonly used in veterinary anesthetic immobilizations. Telazol is a combination of which two drugs? Teletamine and zolazepam Ketamine and xylazine Ketamine and diazepam Acepromazine and butorphanol

Teletamine and zolazepam Telazol is an injectable anesthetic/tranquilizer used for restraint or anesthesia. It is a controlled substance made of teletamine and zolazepam. Teletamine is a classified as a dissociative while zolazepam is a benzodiazepine.

Anesthetic monitoring is extremely important in the operating room. If you are in surgery with a cat, which of the following parameters would you consider abnormal? Systolic blood pressure of 132 mmHg PCV of 39 End-tidal CO2 of 45 Temperature of 93.7F

Temperature of 93.7F Of the parameters listed, the low temperature is most alarming. Heat support is extremely important in the anesthetized patient. Draping of the patient, using a warm air system or heat blanket, placing a towel between the patient and the operating table are all measures to help maintain a normal temperature. It is not abnormal for that recovering patient to be hypothermic, but every effort should be made to minimize that. A systolic blood pressure above 90 and below 160 mmHg is ideal for a cat or dog. An end-tidal CO2 level in the range of 35-45 is considered normal. Any acute drop in PCV should be relayed to the surgeon. The PCV measurement in this question falls within the normal range.

You are examining a 4-year old Paint gelding and notice the pictured abnormality on physical examination. Which of the following diagnostic tests would help determine if platelets are associated with this finding? Antithrombin Activity (AT) Template Bleeding Time (TBT) Prothrombin Time (PT) Activated Partial Thromboplastin Time (APTT)

Template Bleeding Time (TBT) The image demonstrates petechial and ecchymotic hemorrhages. The TBT is a test that determines the functional ability of platelets to plug a minute wound. A buccal mucosal bleeding test is similar to template bleeding time. The APTT and PT determine if there are deficiencies in the coagulation factors and do not assess platelet function. Antithrombin activity is sometimes measured when disseminated intravascular coagulation (DIC) is a concern but is not used to evaluate platelet function.

What are hoof testers used for in horses? Test if wide enough to support weight Test hardness Test for pain Test thickness

Test for pain The hoof tester is use to apply pressure over parts of the hoof, to see if a pain response results.

Which of the following conditions causes muscle rigidity and a "saw horse"-like stance? Myasthenia gravis Immune-mediated thrombocytopenia (ITP) Cushing's disease Tetanus

Tetanus Tetanus, also known as "lock-jaw", causes muscle rigidity. Animals with tetanus are light-sensitive and have a "saw horse" stance. Tetanus is caused by the bacteria Clostridium tetani and can be present in the soil, gaining access to the body through an open or penetrating wound. Horses should be vaccinated against tetanus.

If a horse gets a cut on loose barbed wire or steps on a rusty nail, which disease would be the biggest risk? Rhodococcus Leptospirosis Tetanus Blackleg

Tetanus Places where horses are kept (pastures, barns, etc.) should be free of loose nails, barbed wire, or other things that may penetrate the skin and cause infection with Clostridium tetani. Horses should be vaccinated against tetanus. Rhodococcus is a gram-positive bacterium and is often found in dry and dusty soil and is most commonly transmitted via inhalation. Clostridium chauvoei is the cause of blackleg in cattle and is a soil-borne bacterium spreading through food, water, and wounds. Horses are not usually susceptible to blackleg bacteria and are protected by natural resistance. Leptospirosis may be transmitted to horses through ingestion of contaminated water or soil. The environment becomes contaminated through infected urine and is found most often in damp ground where the bacteria can most easily survive.

If a horse receives a cut on the leg from barbed wire and is not vaccinated, what is the protocol for preventing tetanus? Antibiotics Tetanus antitoxin within 24 hours Tetanus toxoid 2 weeks after injury No vaccine can be given after exposure, wash the wound with 2% chlorhexidine solution daily

Tetanus antitoxin within 24 hours A horse that is potentially exposed and not vaccinated should receive the tetanus antitoxin within 24 hours of injury. This will provide some temporary immunity for about 2 weeks. If the wound is still present at that time, the vaccination is given again. A horse that actually has tetanus is given both tetanus toxoid and antitoxin.

Which of the following drugs can cause discoloration of puppy teeth? Metronidazole Enrofloxacin Amoxicillin Tetracycline

Tetracycline

A dog under anesthesia is hypoventilating. This means which of the following is true? The oxygen level is high The oxygen and the CO2 levels are high The CO2 level is low The CO2 level is high

The CO2 level is high Hypoventilation (inadequate respirations) causes an increase in carbon dioxide levels (hypercapnia) and respiratory acidosis. If the end tidal CO2 is climbing, the patient should be given a breath or may need to be placed on a ventilator (especially if the ETCO2 climbs over 50).

You take a series of chest radiographs and notice that all of the films appear darker (more exposed) towards one side of the film. Which of the following is the most likely explanation? The processor is running low on developer The X-ray beam is usually more intense on one side of the field The radiograph cassette is not closing fully The processor is running low on fixative

The X-ray beam is usually more intense on one side of the field This phenomenon is known as the "Heel effect". X-ray distribution in diagnostic X-ray units is uneven, with the beam being most intense at the cathode side of the machine and weakest at the anode side. This effect is frequently minor but can be noticeable. It is advantageous to radiograph patients of uneven thickness (i.e. the body of a deep-chested dog) by orienting the thickest part toward the cathode side of the x-ray tube. The other choices listed could cause artifacts but should not cause such a gradient on the film.

Clipping an abdomen for a ventral midline laparotomy is one of the more common procedures done as a surgical nurse working with dogs and cats. Which of the following statements is most accurate? The hair should be clipped from the xiphoid to the base of the pubis in all males Only enough hair should be clipped to perform the procedure As long as 0.5 cm margins of clipped areas are present around the incision, the surgeon should have no trouble The abdomen should be clipped from the manubrium to the pubis with 2 cm margins cranial to the manubrium and 2 cm caudal to the pubis The abdomen should be clipped widely with at least 2 cm margins cranial to the xiphoid and 2 cm caudal to the pubis

The abdomen should be clipped widely with at least 2 cm margins cranial to the xiphoid and 2 cm caudal to the pubis

Capnography is a means of measuring what? The amount of oxygen in the blood The amount of carbon dioxide in the blood The amount of oxygen in exhaled gas The amount of carbon dioxide in respiratory gases Continuous blood pressure monitoring

The amount of carbon dioxide in respiratory gases Capnography is a means by which carbon dioxide is measured in gas such as inhaled or exhaled respiratory gases. Capnography measures CO2 tension by passing an infrared light beam across a measuring chamber in which a stream of exhaled gas passes. A photodetector on the other side of the chamber measures the intensity of light that is transmitted. The intensity of light transmission is inversely related to the concentration of CO2 in the exhaled gas sample.

Many x-ray machines used in veterinary medicine have a 2-part exposure process. The first step involves depressing a button or pushing a pedal half-way to prepare the machine for the exposure and then a second button or fully depressing the pedal to take the exposure. What happens in the x-ray machine when the first step is started? The anode rotates and the filament is cooled The anode rotates and the filament is heated The cathode rotates and the anode is charged The anode rotates and the cathode is charged The cathode rotates and the filament is cooled

The anode rotates and the filament is heated To generate x-rays, the filament must be heated for a process called thermionic emission to occur, wherein a cloud of electrons forms around the filament and can be accelerated from cathode to anode for the exposure. Because a great deal of heat build-up occurs during x-ray production, most x-ray machines have a rotating anode to distribute the heat. This is done in preparation for the exposure. When the exposure button is pressed, charge in the x-ray tube accelerates the electrons, which hit the target at the anode, which emits x-rays.

Meningoencephalomyelitis is an inflammatory neurologic condition that affects the central nervous system. What comprises the central nervous system? The vagus nerve The peripheral nerves The lower motor neurons The cerebellum alone The brain and spinal cord

The brain and spinal cord

A woman adopts a pet from the humane society and you are reading through the record from the shelter. The record says the cat presented with a history of pyrexia. What does this mean? The cat had a dropped jaw The cat was extremely painful The cat had a fever The cat was severely dehydrated

The cat had a fever

A 2-year old cat presents 15 minutes after being attacked by a dog. The cat is in cardiac arrest. The owner wants CPR to be undertaken, and the veterinarian is not currently on the premises. What course of action should be taken? The cat should receive no medical care without the supervision of a veterinarian. The cat should be immediately intubated and CPR should commence. The cat may be intubated but chest compressions should not occur. The owner should be told the cat is beyond saving. The cat should be transferred to the closest emergency facility.

The cat should be immediately intubated and CPR should commence.

Which of the following is true of a dog with polycythemia? The dog has a low red blood cell count The dog has an increased hematocrit The dog has an increased white blood cell count The dog has a low platelet count

The dog has an increased hematocrit Polycythemia is an increase in the absolute number of circulating red blood cells (which results in an increased hematocrit or PCV). This is not the same as an increased hematocrit due to hemoconcentration from dehydration. A low red blood cell count is anemia. A low platelet count is thrombocytopenia. An increased white blood cell count is leukocytosis.

When performing an epidural using the hanging drop technique, which of the following is the indication you have correct placement of the needle in the epidural space? The patient's tail drops between its legs when they are hanging over the edge of the table. The drop of saline placed in the hub of the needle is sucked in. When a drop of saline is placed in the hub of the needle and does not suck in, instead it is "hanging out" or forms a meniscus. The hanging drop is not a technique used during the placement of an epidural.

The drop of saline placed in the hub of the needle is sucked in. When using the hanging drop technique while placing an epidural, the patient is placed in sternal recumbency; the spinal needle is placed through the skin, and a drop of saline is put in the hub of the needle. When the needle is in the correct place, the drop of saline should be sucked in through the needle.

Which of the following errors in taking and processing a radiograph would result in a completely clear film? The film was exposed to light after x-ray exposure due to a leak in the darkroom The cassette was not closed completely and the film was exposed to light prior to x-ray exposure The collimator was left all the way open The exposed film was put in fixer before developer

The exposed film was put in fixer before developer In normal radiograph processing, silver halide crystals in the film that are sensitized by exposure during the x-ray are converted to black metallic silver in the developer. The remaining crystals are removed by the fixer. If a film is accidently run through the fixer first, all of the silver halide crystals will be removed, leaving a clear image. The other choices will all increase exposure of the film, making it darker.

Which of the following statements is LEAST correct regarding dystocias and Cesarean sections? The gravid female may have difficulty breathing when anesthetized Not all dystocias can be treated medically The fetuses are not affected by the anesthetics delivered, as they do not cross the placenta Metabolic alterations such as hypocalcemia should be corrected

The fetuses are not affected by the anesthetics delivered, as they do not cross the placenta

A patient has sustained a burn to the skin corresponding to the area where the electrocautery ground plate had been placed. What is the most likely reason this occurred? The ground plate had poor contact with the patient's skin The power level was too high The electrocautery unit was inadvertently switched to bipolar mode The surgeon used the cut mode for over 30 seconds

The ground plate had poor contact with the patient's skin

Which of the following is true about the appearance of an image relative to the orientation of the slide when viewing through a compound microscope? The image is upside-down and reversed The image is upside-down and oriented normally left to right The image is right-side up and oriented normally left to right The image is right-side up and reversed

The image is upside-down and reversed Images are seen upside-down and reversed when viewed on a compound microscope. The left side of the image is seen on the right side of the visual field. Accordingly, movement of the stage is reversed such that as the stage is moved to the left, the image appears to move to the right.

A woman presents with her 10-week-old kitten who has been having diarrhea. She brought a fresh fecal sample and you perform a fecal flotation while the doctor is performing his exam. You see the following under the microscope (round). What do you tell the doctor when he asks about the fecal? The kitten has hookworms The kitten has tapeworms The kitten has roundworms The kitten has coccidia

The kitten has roundworms This is a photo of a Toxocara egg, which is a roundworm. Roundworms are very common in kittens and puppies.

A woman presents with her 10-week-old kitten that has been experiencing diarrhea. She brought a fresh fecal sample and you are checking it while the doctor is performing his exam. You see the following under the microscope (circle). What do you tell the doctor when he asks about the fecal sample? The kitten has hookworms. The kitten has coccidia. The kitten has tapeworms. The kitten has roundworms.

The kitten has roundworms.

What is the carnassial tooth? This is another term for a supernumerary tooth This is another term for a missing tooth The tooth with the deepest root in each dental arcade The largest shearing tooth of each dental arcade This is another name for the canine tooth

The largest shearing tooth of each dental arcade "Carnassial" literally means tearing of flesh but is used in veterinary dentistry to describe the largest shearing tooth of the mandible and maxilla in carnivores. This is the tooth that does the most work during mastication. In dogs and cats, the carnassial teeth are the maxillary fourth premolar and the mandibular first molar.

When placing craniocaudal radiographs of the stifle joint on a viewer for a veterinarian to evaluate, how should the film be oriented? The medial side of the right limb should be to the viewer's left; the medial side of the left limb should be to the viewer's right. The lateral side should be to the viewer's right The lateral side should be to the viewer's left The lateral side of the right limb should be to the viewer's left; the lateral side of the left limb should be to the viewer's right.

The lateral side of the right limb should be to the viewer's left; the lateral side of the left limb should be to the viewer's right.

Which region of the blood smear should be used for the differential cell count? The thickest region of the smear The monolayer region The feathered edge The center of the slide

The monolayer region When a blood smear is made, the feathered edge is where cells may be distorted or erratically distributed. It is appropriate to assess this area and the rest of the slide for platelet clumps, large irregular cells, and parasites. However, it is best to look in the monolayer region for the differential cell count because cells here should be randomly distributed with minimal distortion. Thicker regions or the center of the slide will frequently have so many red blood cells that they obscure the view of leukocytes.

Which of the following is true regarding Dirofilaria immitis, the parasite that causes heartworm? The mosquito is the paratenic host The mosquito is the definitive host The dog is the intermediate host The dog is the paratenic host The dog is the reservoir host The mosquito is the intermediate host

The mosquito is the intermediate host The dog is the definitive host of D. immitis, and the mosquito is the intermediate host. The definitive host harbors the adult, sexual stage of a parasite. The intermediate host harbors larval, asexual, or immature forms of a parasite. A paratenic host (also known as a transport host) indicates a type of intermediate host where a parasite does not undergo development into the next stage. A reservoir host is another vertebrate host for a parasite that serves as a source of infection for people or domestic animals. For this parasite, the mosquito is the intermediate host but not a paratenic host because the first, second, and third larval stages are found within the mosquito.

Regarding blood draws, which of the following is true? A small amount of air should be placed in the syringe before inserting the needle into the vein The bevel may face in any direction when drawing blood The needle should be held parallel to the vein at a slight angle with the bevel facing up The needle is directed straight into the vein in a perpendicular fashion with the bevel facing down

The needle should be held parallel to the vein at a slight angle with the bevel facing up There should be no air in the syringe before drawing blood. The bevel is held facing upward and the syringe held parallel to the vein at a slight angle, depending on how superficial the vein is.

Ancylostoma caninum and Cystoisospora are parasites that have a direct life cycle. What does this mean? The parasite does not require an intermediate host The parasite does not undergo migration outside of the intestinal tract The parasite does not have a larval or oocyst form The adult of the parasite is the infective form of the parasite

The parasite does not require an intermediate host A direct life cycle indicates that the parasite does not require an intermediate host and can undergo its entire life cycle within a single species. This is true for both Cystoisospora and Ancylostoma caninum, the canine hookworm.

What does it mean if an animal is a paratenic host of a parasite? The animal is where the parasite develops from nymph to adult form The parasite does not undergo development on the animal The animal is another vertebrate that serves as a source of infection for humans or domestic animals The animal is where the parasite develops from larval to nymph form The parasite's adult or sexual stage lives on the animal

The parasite does not undergo development on the animal A paratenic host is also known as a transport host. This indicates a type of intermediate host on which a parasite does not undergo development into the next stage. The definitive host harbors the adult, sexual stage of a parasite. The intermediate host harbors larval, asexual, or immature forms of a parasite. A reservoir host is another vertebrate host for a parasite that serves as a source of infection for people or domestic animals.

How can you differentiate a male tortoise from a female? The females are smaller in size The hemipenes are evident in males The eyes are darker in males The plastron is concave in males The female's carapace is concave The plastron is convex in males

The plastron is concave in males In order for the males to be able to mount the females, their plastron is concave. This allow for a good fit over the convex shape of the female's shell.

Ectopic pregnancy means which of the following? There is more than one fetus The fetus is getting resorbed by the body There is no fetus, just an empty amniotic sac The pregnancy is occurring outside of the uterus

The pregnancy is occurring outside of the uterus Ectopic pregnancy occurs when a fertilized egg has implanted outside the uterus, most often in the fallopian tubes. Mostly these pregnancies are not viable and pose a bleeding risk which can lead to death.

Where would you give a sub Q injection in a snake? Distal 1/3 of the body, along the lateral edge of the ventral scales Near the vent, proximal to the opening The proximal 1/3 of the snake along the lateral edge of the ventral scales. Visually divide the snake in half, inject into the distal 1/2

The proximal 1/3 of the snake along the lateral edge of the ventral scales. Due to the portal system of reptiles, it is important to give injections towards the head. The drug will distribute much more slowly and evenly. The dorsal and ventral scales meet along the lateral sides. This is where the drug can be injected.

A client has called your hospital to schedule a castration to be performed on her rabbit. The owner says she remembers that she had to fast her dog when he got neutered and asks if she should do the same for the rabbit. What is the correct response? The rabbit should be fasted for 12 hours prior to being dropped off at the hospital. The rabbit should be fasted for 6 hours prior to being dropped off at the hospital. The rabbit should not be fasted. The rabbit should fasted for 24 hours prior to being dropped off at the hospital.

The rabbit should not be fasted. Small rodents and rabbits do not vomit, therefore, there is generally no reason to withhold food or water prior to anesthesia. Withholding food from rabbits can trigger digestive disturbances that can be fatal.

When taking a radiograph of the radius and ulna, which of the following will occur if the x-ray beam is perpendicular to the cassette but not to the long axis of the radius and ulna? The radius will appear elongated, and the ulna will appear foreshortened The radius and ulna will appear foreshortened The radius and ulna will appear elongated The radius will appear foreshortened, and the ulna will appear elongated

The radius and ulna will appear foreshortened To avoid distortion (elongation or foreshortening), the x-ray beam should be perpendicular to the long axis of the bone and the x-ray cassette. If the bones are not perpendicular to the beam, they will appear foreshortened. If the cassette is not perpendicular to the beam, the bones will appear elongated. In dental radiography, it is frequently not possible for the long axis of the tooth and the film to be parallel to each other. In this case, the bisecting angle technique may be used, wherein the beam is angled half-way between the angle of the long axis of the tooth and the angle of the film in order to obtain a true image that is neither elongated nor foreshortened.

Which of the following best describes the therapeutic index of a drug? The number of milligrams of a drug that is needed to be effective divided by 100 The ratio between the effective dose and the toxic dose The amount of variability in the response of a population to a drug The range of microbial species that a drug is effective against The ratio between the maximum effective dose and the minimum effective dose

The ratio between the effective dose and the toxic dose The therapeutic index is a measure of a drug's safety margin. It is defined as the ratio between the effective dose (ED50) and the lethal dose (LD50) to a population.

What is the "chief complaint" of a client? The reason why they brought the pet The medical bill Another term for signalment The most severe problem the animal has

The reason why they brought the pet

What maintains the membrane potential in excitable cells? Neurons Calcium channel pump Chloride anions The sodium-potassium pump

The sodium-potassium pump The principal ions involved in an action potential are sodium and potassium cations. During depolarization ion channels allow sodium to enter the cell, and potassium to leave the cell. The sodium-potassium pump then helps to restore the normal resting membrane potential. The sodium-potassium pump maintains the normal ratio of ion concentrations across the membrane. Calcium cations and chloride anions are involved in a few types of action potentials, such as the cardiac action potential.

The autonomic nervous system refers to what? The limbic nervous system The central nervous system (CNS) The sympathetic and parasympathetic nervous systems The cranial nerves The peripheral nervous system (PNS)

The sympathetic and parasympathetic nervous systems

When taking a radiograph of the tibia and fibula, which of the following will occur if the x-ray beam is perpendicular to the tibia and fibula but not to the x-ray cassette and film? The tibia will appear elongated, and the fibula will appear foreshortened The tibia and fibula will appear foreshortened The tibia and fibula will appear elongated The tibia will appear foreshortened, and the fibula will appear elongated

The tibia and fibula will appear elongated To avoid distortion (elongation or foreshortening), the x-ray beam should be perpendicular to the long axis of the bone and the x-ray cassette. If the bones are not perpendicular to the beam, they will appear foreshortened. If the cassette is not perpendicular to the beam, the bones will appear elongated. In dental radiography, it is frequently not possible for the long axis of the tooth and the film to be parallel to each other. In this case, the bisecting angle technique may be used, wherein the beam is angled half-way between the angle of the long axis of the tooth and the angle of the film in order to obtain a true image that is neither elongated nor foreshortened.

Approximately 5ccs of an inhalant anesthetic is spilled while the vaporizer on an anesthetic machine is being filled. What course of action should be taken? The fire department should be called Nothing, continue filling the vaporizer The spill should be cleaned up using paper towels and bleach The hospital should be evacuated Dilute the spill 1:1 with water and then wipe up with a towel The vaporizer should be closed and the room well ventilated

The vaporizer should be closed and the room well ventilated The fire department should only be summoned if a significant amount of inhalant (gas) anesthetic is spilled, such as an entire bottle being dropped or broken. The hospital does not need to be evacauated unless there is no way to seal off the area where the spill occurred. Bleach is very reactive with a great number of chemicals and should never be used to clean up any chemical spill. Continuing to fill the vaporizer will expose staff to inhalant as the anesthetic evaporates. The best course of action would be to close the vaporizer, leave the room/area for 5-10 minutes and allow the air in the room to circulate either by opening an external door or increasing the air flow into the room using a fan or air conditioning unit.

A dog is being referred to the neurologist for a cervical disk compression that is causing forelimb paresis. Where will the surgical incision likely be made? Between the shoulder blades The dorsal neck region The ventral side of the neck Inside the mouth

The ventral side of the neck A cervical disk surgery typically requires an incision made on the underside of the neck. This surgery is referred to as a "ventral slot".

A pet-sitter is holding a 4-year old male intact Bichon while the veterinarian examines a rash on the dog's abdomen; you are assisting with a skin scraping. The dog reaches back and bites the pet-sitter on the arm. She had insisted on holding the dog for examination despite the posted sign (see image). This dog has been known to be aggressive in the past. If the pet-sitter seeks compensation for being bit, who is most likely to be found liable for her injury? You, the technician This would be considered an accident and fault of both parties, therefore no one is liable. The pet-sitter is liable and therefore no damages can be claimed The dog's owner The veterinarian

The veterinarian Even though the owner insisted on holding the dog for the exam, the veterinarian could still be liable for the owner's injury. While most clients would not pursue damages for an incident such as this, courts have ruled that veterinarians are liable if a pet hurts its owner while the two are located on the premises of a veterinary hospital. The best solution to prevent this is to have a technician or assistant trained to restrain animals hold the patients for examination. It is a good idea for a veterinary practice to post a sign in each exam room stating that owners please not ask to restrain pets during exam and this rule should be enforced. In animals that are known to be aggressive, a muzzle should be placed prior to the exam for the safety of all involved.

Which of the following statements is NOT included in NAVTA's Code of Ethics for the Profession of Veterinary Technology? Aid society and animals through providing excellent care and services for animals. The veterinarian must assume accountability for individual professional actions and judgements of the veterinary technician. Safeguard the public and the profession against individuals deficient in professional competence or ethics. Assist with efforts to ensure conditions of employment consistent with the excellent care for animals. Veterinary technicians shall represent their credentials or identify themselves with specialty organizations only if the designation has been awarded or earned.

The veterinarian must assume accountability for individual professional actions and judgements of the veterinary technician.

You are working in a laboratory where the rats are getting "ringtail." Which of the following may be contributing to this condition? The wire-bottom cages The humidity in the cage is too high The temperature is too low in the housing unit Too much nesting material in the cages

The wire-bottom cages The correct answer is the wire-bottom cages in which the rats are housed. Ringtail is an annular constriction of the tail found in weanling rats and rats kept in wire-bottom cages. The condition causes the tail to constrict, undergo dry gangrene, and fall off. Factors contributing to ringtail include increased environmental temperatures, low humidity, impaired blood supply to the tail, and possibly drafts. The tail stumps usually heal with no complications. Increased humidity, low temperature and too much nesting material would not typically contribute to the development of ringtail.

You are asked to take a DLPMO radiograph of a horse's fetlock with a portable x-ray unit. Where will the x-ray unit be and where should the film cassette be? The x-ray beam should come from the more dorsal side of the limb, and the film will be on the more medial aspect of the limb. The x-ray beam should come from the more dorsal side of the limb, and the film will be on the more lateral aspect of the limb. The x-ray beam should come from the more plantar side of the limb, and the film will be on the more medial aspect of the limb. The x-ray beam should come from the more plantar side of the limb, and the film will be on the more lateral aspect of the limb.

The x-ray beam should come from the more dorsal side of the limb, and the film will be on the more medial aspect of the limb. With a DLPMO or dorsolateral-palmaro/plantaromedial oblique, the beam comes from 45 degrees between dorsal and lateral sides of the limb, and the film is placed 45 degrees between the medial aspect and the palmar (or plantar) aspect.

Why are guinea pig sows prone to C-section if they are bred for the first time after 6 months of age? Their pelvic bones fuse between 4 and 6 months. Guinea pigs are known to have low blood calcium. The babies are too big to be born naturally. The brachycephalic skull of the babies will get stuck. The older the sow is, the greater number of young they produce in one litter. You can't do a c-section on a guinea pig.

Their pelvic bones fuse between 4 and 6 months. Guinea pigs must be bred for the first time prior to 6 months of age because their pelvis must be flexible enough to allow the the fetus to pass through the birth canal. Their pelvic bones fuse at 4 - 6 months. If this occurs and the sow is bred, dystocia will occur.

Asepsis implies which of the following? There are only symbiotic bacteria present There are less than 1000 colonies of bacteria present There are less than 100 colonies of bacteria present There is no living organism present

There is no living organism present Asepsis means absolutely no living organisms present. The concept of aseptic technique is very important, and technicians should be very aware of sterility in the operating room. This includes walking within the room, passing instruments, prepping the patient, and working within the operating room. Adhering to strict aseptic principles decreases the likelihood a patient will develop a post-operative infection.

A Bearded Collie is to receive an injection of maropitant (Cerenia). If the dose prescribed is 1 mg/kg and the concentration of maropitant is 10 mg/ml, how many milliliters should the patient receive? 2.1 1.8 0.5 6 3.5 There is not enough information to complete this problem

There is not enough information to complete this problem

Why are polyurethane catheters more appropriate for long-term (> 7 days) use in horses? They do not require maintenance They are the largest They are the least thrombogenic They are rigid

They are the least thrombogenic Polyurethane catheters (such as those made by Mila) are the least reactive and thrombogenic and, if placed correctly, can be left in place for more than a week. Polyurethane is a flexible material. All IV catheters require maintenance and monitoring.

Phenothiazine tranquilizers such as acepromazine are frequently used as preanesthetic agents. All of the following are true statements regarding these drugs EXCEPT for which? They frequently cause vomiting They are synergistic with opioids They frequently cause hypotension They are not analgesics

They frequently cause vomiting Phenothiazines are frequently used with opioids because they act synergistically to improve their sedative effects. They have anti-emetic properties which can reduce the vomiting that can be caused by opioids. They cause relaxation of vascular smooth muscle which leads to hypotension. Phenothiazines do not have analgesic properties.

Regarding controlled substances, which is true? When they expire, they should be thrown in the trash Should never be prescribed for home use and should only be used in-hospital They have a high abuse potential, and a written log must be kept on these drugs They are currently controlled by the Environmental Protection Agency

They have a high abuse potential, and a written log must be kept on these drugs

Which of the following is true of non-elective procedures? They should be performed at the veterinarian's convenience They have to be performed immediately An example of an non-elective procedure is ear cropping An example of a non-elective procedure is an ovariohysterectomy

They have to be performed immediately

Bacteria differ from animal cells in that they lack which of the following? They lack a true membrane-bound nucleus They lack cell walls They lack ribonucleic acid They are unable to metabolize glucose

They lack a true membrane-bound nucleus

Which is true regarding umbilical hernias? They usually consist of a small piece of fat protruding through the body wall and are not life-threatening They should not be repaired at the same time as a spay or neuter procedure They usually progress until a loop of bowel is strangulated through the defect and should be repaired as an emergency procedure The best treatment for all umbilical hernias is benign neglect

They usually consist of a small piece of fat protruding through the body wall and are not life-threatening MOST umbilical hernias are stable and consist of a small piece of fat protruding through a body wall defect. Many small dogs with stable hernias do not require repair since they are not life-threatening. Larger dogs with hernias require repair more often since the weight of their abdominal contents on the defect can enlarge it and increase the risk that abdominal contents (such as intestine) could protrude through. The best time to repair these is when they are already under anesthesia for their spay or neuter. The course of action for repair or monitoring depends upon the risk factors (size of hernia, size of dog, etc.).

Which of these drugs should not be used in Greyhounds? Ivermectin Propofol Morphine Thiopental

Thiopental The correct answer is thiopental. Thiopental is an ultra-short acting barbiturate. Recovery depends on redistribution to tissues, including fat. Because sighthounds have very little fat, they have prolonged recoveries and greater complications with these drugs.

Which of the following is NOT a drug used for the treatment of internal parasites? Praziquantel Pyrantel Thiopental Fenbendazole Sulfadimethoxine

Thiopental Thiopental (Pentothal) is an ultra-short acting barbiturate anesthetic induction agent. Pyrantel pamoate (Strongid, Nemex) is an anti-nematode agent. Fenbendazole (Panacur) is a broad spectrum anthelmintic. Praziquantel (Droncit) is an anti-cestodal (tapeworm), and Sulfadimethoxine (Albon) is a sulfonamide used to treat coccidia.

A multi-lumen jugular catheter must be placed in a canine with renal disease for CVP monitoring. To what anatomical landmark should the catheter be inserted to? Thoracic inlet Third thoracic vertebrae Aorta Caudal vena cava The heart

Third thoracic vertebrae

You are new on the ICU floor and are taking time to look through the crash cart so you will be prepared when emergency arises. You find a 60 cc syringe attached to a 3-way stopcock and extension set with a 22-gauge needle attached. What might this setup be used for? Tracheal drug administration Thoracocentesis Jugular catheter placement Decreasing intracranial pressure

Thoracocentesis This is commonly used for removing fluid from the pleural space (pleural effusion).

An animal with pleural effusion may require which procedure for stabilization? Thoracocentesis Radiographs Abdominocentesis Orogastric tube

Thoracocentesis A large amount of fluid in the pleural space (around the outside of the lungs) will prevent the lungs from fully expanding. A chest tap (thoracocentesis) will allow the fluid to be drained from the pleural space and allow the patient to expand the lungs and breath easier. While radiographs may be needed, an unstable patient in respiratory distress should not have radiographs. The chest tap should occur first to stabilize the patient. Image used with permission, from Veterinary Cytology (Freeman), courtesy of Manson Publishing.

Generally, how much time do you have before cardiac arrest causes irreversible brain damage? Ten minutes Thirteen minutes Seven minutes Three minutes

Three minutes That is why it is imperative that cardiac arrest is identified quickly, CPR status is clear for each patient, and that high-risk cases should be discussed at the time of admission.

How is tularemia transmitted (what is the vector)? Ticks Mosquitoes Leeches Lice

Ticks The correct answer is ticks. Ticks are the usual vector, although flies and fleas may be able to transmit Francisella tularensis as well.

What three things help to minimize your exposure to radiation while taking radiographs, according to the U.S. Nuclear Regulatory Commission? Time, Distance, Shielding Time, Equipment, Restraint Distance, Time, Restraint Equipment, Restraint, Shielding

Time, Distance, Shielding The correct answer is Time, Distance, and Shielding (try to remember TDS). Decreasing exposure time to the x-rays, distancing yourself from the source as much as possible, and using proper shielding that includes lead (in personal protective equipment) or leaving the room during x-ray if you are not needed are all ways to minimize radiation exposure. While proper restraint and equipment are also important in minimizing exposure, they are not sufficient without the time, distance, and shielding; this question is asking specifically about the U.S. Nuclear Regulatory Commission.

What is the best method for handling tissues during abdominal surgery when they must be placed outside of the peritoneal cavity? Tissues and organs outside the abdomen should be covered in an antibiotic-impregnated solution and sponges Tissues and organs outside the abdomen should be covered with saline-moistened sponges Tissues and organs should never be exteriorized from the abdomen Tissues and organs outside the abdomen should be wrapped in a dry gauze or sponge

Tissues and organs outside the abdomen should be covered with saline-moistened sponges Tissues and organs should always be handled with exceptional care. To avoid desiccation, all tissues/organs should be intermittently moistened with sterile saline or wrapped in saline-impregnated gauze or lap sponges. Antibiotic-impregnated solutions are not necessary, can be irritating, and may select for resistant organisms. Allowing the tissues/organs to dry will cause damage to their surfaces and result in subsequent inflammation.

Why are oblique radiographic views of certain joints typically obtained? To change the position of air or fluid in or around the joint To decrease superimposition of structures To prevent foreshortening or elongation artifacts To decrease the amount of scatter that is causing distortion on the film

To decrease superimposition of structures Oblique radiographic views are typically requested in areas where there are multiple overlapping structures. The purpose of these views is to decrease superimposition of certain bones or structures so that a specific region is better isolated on the film. Typically, there is no change in the air or fluid when oblique views of a joint are obtained. Because oblique views are still usually taken across the entire joint, there is no change in the amount of scatter. Properly taken conventional radiographs of a joint should not have foreshortening or elongation artifacts.

Pre-anesthetic drugs are used for all of the following reasons except which of the following? Smoother anesthetic induction and recovery period Reduce anxiety To decrease the blood pressure Less induction drugs are needed Provide analgesia

To decrease the blood pressure

What is the function of the screen of an X-ray cassette? Protect the film from light Absorb scatter radiation to increase image clarity Allow use of higher kVp settings To decrease the exposure necessary to create an image on the film

To decrease the exposure necessary to create an image on the film The screen of an X-ray cassette, sometimes called intensifying screens, are the shiny white inner surfaces of the cassette. They are made of crystals that fluoresce, emitting light, when exposed to x-rays, producing the diagnostic image.

What is the purpose of the condenser of a microscope? To focus light on the object being viewed To focus the lens on the object being viewed To adjust the intensity of light reaching the object being viewed To filter out wavelengths of light before they reach the object being viewed

To focus light on the object being viewed The condenser of a microscope consists of two lenses that focus light on the object. It is adjusted by being raised or lowered below the stage. It may have a diaphragm that can control the amount of light but this is not the main purpose of the condenser. The rheostat of the light source is the best way to adjust light intensity.

In regards to rabbit anesthesia, which of the following best explains why the use of sedative premedications should be given prior to inducing anesthesia using a mask or induction chamber to deliver gas agents such as Isoflurane? To help prevent breath holding Allows intubation prior to induction Anesthesia in rabbits can not be achieved by gas agents alone To cause an increase in heart rate

To help prevent breath holding Because rabbits often hold their breath and struggle when exposed to gas anesthetics, premedications such as dexmeditomidine, diazepam, or midazolam will facilitate smoother inductions.

A 7-year old Saint Bernard dog has a lesion of the left distal radius that is highly suspicious for a bone tumor (osteosarcoma). Prior to surgery, a bone scan is ordered and performed. What is the main reason for this study? To provide greater anatomic detail regarding the lesion in the left distal radius To identify possible pulmonary metastatic lesions To confirm the diagnosis of osteosarcoma of the left distal radius To identify possible additional bone lesions (other than the left radius lesion) that may represent bony metastases

To identify possible additional bone lesions (other than the left radius lesion) that may represent bony metastases While the lungs are the most common site of metastasis of bone tumors in dogs, these tumors can also metastasize to other bony sites. Thoracic radiographs are generally sufficient to assess gross pulmonary metastasis but radiography is not typically practical for assessing all of the bones in the body. A bone scan is a nuclear medicine test that involves administration of a bone-seeking radioisotope (Technetium phosphate) that is taken up in areas of bone turnover. The test does not provide significant anatomic detail and is not confirmatory of osteosarcoma because it is not specific for cancer (osteoarthritis or osteomyelitis also are positive on bone scans). The test is used to identify additional bony sites that should be evaluated radiographically to assess for metastasis. Because this type of test is not routinely available at all practices and the overall incidence of bony metastasis is relatively low, it is usually reserved for cases where there is greater suspicion for bone metastasis. It would also be used to rule-out the presence of bone metastasis prior to performing complex or expensive limb-sparing surgeries.

Why is a small amount of heparin frequently added to saline syringes that will be used for flushing catheters? To prevent degradation of the sodium and chloride in saline To prevent blood clots from forming To prevent growth of bacteria in the saline To prevent irritation caused by injection of saline

To prevent blood clots from forming

What is the purpose of a grid on an x-ray table? To decrease the exposure necessary to create an image on the film To provide a reliable scale for measuring structures that appear magnified on the film To protect the film from light To prevent scatter radiation from reaching the film To prevent primary radiation from reaching the film

To prevent scatter radiation from reaching the film A grid is a series of short strips of lead that is placed over the cassette, absorbing all radiation that does not go between the strips. The strips are geometrically arranged to allow the primary radiation through but to absorb any radiation that is scattered in other directions. This results in increasing the resolution of structures on the film.

A tablet may have enteric coating to serve what purpose? It increases transit time so the drug can make it to the colon more quickly for absorption It prevents gastric ulceration To protect the medication from being destroyed by the acids of the stomach It slows transit time so that the drug will be absorbed mostly through the stomach wall

To protect the medication from being destroyed by the acids of the stomach Enteric coatings are used to protect the medication from the acidic stomach environment so that more of the active drug can make it and be absorbed in the small intestine. The enteric coating does not increase or decrease transit time. Drugs are mostly absorbed in the small intestine, not the colon.

You are running some blood tests, and the veterinarian asked that the pet be fasted. Why is this necessary? To reduce lipemia To prevent icterus To prevent clotting To prevent hemolysis

To reduce lipemia Some test results require fasting for accuracy. Fasting an animal helps to reduce the lipemia (fats) in the blood that can interfere with testing. This is especially important when checking values such as triglycerides and cholesterol.

A hematocrit tube provides the means for evaluating a patients packed cell volume and what other parameter? Total plasma protein Lactate Hemolytic index Arterial oxygen

Total plasma protein Hematocrit tubes are used for two main purposes. One of these is to determine the packed cell volume (hematocrit) and the other is to measure the total plasma protein. Another observation that is made with the hematocrit tubes is to note the color of the plasma. This can give an indication of lipemia, icterus, hemolysis, etc.

It is recommended that all puppies be treated for which of the following parasites? Dirofilaria immitis Toxocara canis Ctenocephalides felis Toxascaris leonina

Toxocara canis

Ocular larval migrans is a zoonotic disease caused by which parasite? Ancylostoma caninum Taenia taeniaformis Uncinaria stenocephala Toxocara canis Dipylidium caninum

Toxocara canis Toxocara canis (a roundworm) is the cause of Ocular and Visceral larva migrans. It is transmitted fecal-oral. Backyards, sandboxes, parks, and beaches accessible to dogs are often contaminated with Toxocara ova, which can remain infective for years. These areas can be exposure sites for people (especially children) who accidentally ingest the infective eggs. Ancylostoma and Uncinaria are hookworms and can cause Cutaneous larva migrans. Taenia taeniaformis and Dipylidium caninum are both tapeworms.

Which parasite has the feline as its only definitive host? Ancylostoma caninum Dirofilaria immitis Giardia lamblia Toxoplasma gondii Trichuris vulpis

Toxoplasma gondii The definitive host of T. gondii is the cat, but the parasite can be carried by many warm-blooded animals. D. immitis, A. caninum, and Giardia lamblia can have multiple hosts. T. vulpis is canine specific.

Which small animal disease has been implicated as a specific danger to pregnant women? Clostridium Toxoplasmosis Camylobacter Coccidiosis

Toxoplasmosis

A 9-year old male neutered German Shepherd is diagnosed with an osteosarcoma at the distal radius. The owner has chosen palliative care and wants to keep him comfortable as long as possible. He is already on carprofen (Rimadyl) for pain. What other medication could he take for additional analgesia for his bone pain? Meloxicam Tramadol Glucosamine Prednisone

Tramadol Tramadol is a synthetic drug that acts on the opiate receptor and is often effective against bone pain in dogs. It is safe to give with anti-inflammatory medications. Since this dog is already on Rimadyl, he cannot receive any other NSAIDS or any steroids. Anti-inflammatories (steroids or NSAIDS) cannot be mixed or given concurrently due to the high risk of gastrointestinal ulceration and other complications such as renal damage. Glucosamine, while it would not cause harm, would not provide any analgesia for this dog.

The clinician has been unsuccessfully treating a chronic cough. A culture of the lower airways is needed. What is the best method for this? Nebulize the dog and culture the mist in the mask after 10 minutes Nasal swab culture Transtracheal wash Culture of the back of the throat with a sterile swab after coughing episode

Transtracheal wash Transtracheal wash is used when a culture of the lower respiratory tract is needed. It is typically done under general anesthesia for small dogs or cats or under a local in larger dogs. It is a method of collecting bronchial exudate for culture and cytology. The procedure entails inserting a catheter into the trachea in the ventral neck region between two tracheal rings, and infusing and aspirating sterile saline in order to get a sample.

When collecting an arterial blood sample, the best location to obtain arterial blood in the standing adult horse is where? Jugular Artery Carotid Artery Dorsal Metatarsal Artery Femoral Artery Transverse facial artery

Transverse facial artery The transverse facial artery is the easiest and most commonly used artery to collect arterial blood in a standing adult horse. It is typically located caudal and ventral to the eye and is easily detected. The dorsal metatarsal and carotid arteries are possible sites to collect arterial blood, but an adult horse typically will not stand still to collect blood from the extremity (hind limb). The carotid is also possible to collect arterial blood but is a deeper structure and is obviously a very large vessel; damage to this vessel during collection can be problematic.

Which of the following is another term for the type of parasite known as a fluke? Cestode Nematode Trematode Ascarid Protozoan

Trematode Trematodes, or flukes, are leaf-shaped flatworms with unsegmented bodies. Adults are hermaphrodites. They primarily are found in the intestinal tract, liver, and lungs. Examples include the liver fluke of cattle (Fasciola hepatica) and the lungworm of cats (Paragonimus kellicotti). Nematodes are roundworms, ascarids are a type of nematode (roundworm), cestodes are tapeworms, and protozoans are single-celled organisms that may be parasitic.

Ivermectin would not be used when treating which parasite? Trematodes Ticks Mites Nematodes

Trematodes The correct answer is trematodes (flukes). Ivermectin is ineffective against flukes. Ivermectin has activity against ticks, mites, nematodes, and even lice. Ivermectin works by stimulating GABA which is an inhibitory neurotransmitter.

Rabbits are prone to hairballs. What is the medical term for a hairball? Trichobezoar Piloerection Phytobezoar Dermatophyte

Trichobezoar

Regarding lice parasites, the biting louse found in canines is known as which of the following? Trichodectes Hematopinus Damalinia Felicola

Trichodectes Trichodectes canis is the chewing or biting louse in dogs. Hematopinus asini is the sucking louse in cattle. Damalinia bovis is the chewing louse in cattle (also called Bovicola). Felicola subrostratus is the chewing or biting louse in cats. Lice are ectoparasites; infestation with lice is known as pediculosis. Trichodectes canis can also serve as an intermediate host for Dipylidium caninum (although more commonly, the intermediate host is a flea).

You are performing a fecal examination on feces from a sick dog and find the following under the microscope (football). You inform the veterinarian of your finding, which is what parasite? Trichuris Ancylostoma Taenia Cystoisospora

Trichuris The eggs shown are Whipworm eggs (Trichuris vulpis). Note the bipolar plugs (the plugs on each end of the eggs). Here are some photos of the other parasites listed (also included Toxocara).

Schnauzers are notorious for having hyperlipidemia, and when their blood is spun down, the serum often appears milky. This is due to a high level of what in the serum? Chyme Leukocytes Lymph Triglycerides

Triglycerides Hyperlipidemia is the term for elevated triglycerides or cholesterol.

What is the name for the triangular region of the bladder where the ureters and urethra are connected to the bladder? Body Medulla Cortex Fundus Trigone

Trigone The trigone is a smooth triangular part of the internal urinary bladder formed by the two ureteral openings and the internal urethral opening. The fundus is an anatomical term referring to the portion of an organ opposite from its opening (i.e. fundus of the stomach). Medulla generally refers to the middle of an organ (i.e. renal medulla). Cortex generally refers to the outer layer of an organ (i.e. adrenal cortex).

A chihuahua is diagnosed with pituitary-dependent hyperadrenocorticism. Which of the following medications may be used for treating Cushing's disease in this patient? Trilostane DOCP Pimobendan Prednisone

Trilostane Trilostane (Vetoryl) is a steroid inhibitor used in the treatment of Cushing's disease. This medication works by stopping the production of cortisol in the adrenal glands. In Cushing's disease (hyperadrenocorticism), the adrenal glands are over-producing cortisol due to either a tumor on the pituitary gland stimulating the adrenal glands, or due to a tumor on an adrenal glad itself. This medication is reversible (it doesn't actually destroy the adrenal gland as does mitotane (Lysodren). Trilostane is also sometimes used to treat Alopecia X. Prednisone is a steroid, and you would not want to give a steroid to a dog who is already over-producing steroids. DOCP (desoxycorticosterone pivalate) or Percorten-V is also a steroid injectable used in treating hypoadrenocorticism (Addison's disease). Pimobendan is an inodilator heart medication, most often used in treating congestive heart faliure.

A dog presents with Gastric Dilatation and Volvulus (GDV), and you have tried multiple times to pass an orogastric tube to no avail. If this is not working what else can be done? Prep the dog for surgery and not decompress the stomach Trocharize the stomach ventrally Flip the patient over Trocharize the stomach on the right side

Trocharize the stomach on the right side Decompression should be performed; the best place for the trocharization is on the right side. Trocharization involves passing a large sterile needle or catheter through the abdominal wall into the stomach to relieve the gas within the stomach. This is necessary in order to make the patient more stable prior to surgery.

You are monitoring a 2 kg kitten being spayed at the humane society; she is intubated and is on isoflurane gas for maintenance. She is on IV fluids at 10 mL/hr. Her blood pressure starts to drop during the procedure. You have already given her a bolus of fluids. Which of the following should you do first to try to increase her blood pressure? Turn off the heat support to prevent peripheral vasodilation, which decreases blood pressure Give a 90 ml/kg bolus of a crystalloid fluid intravenously Turn down the anesthetic gas Give a dose of atropine

Turn down the anesthetic gas Anesthetic gas is a potent vasodilator and causes profound hypotension, especially in small patients. Decreasing the vaporizer setting in addition to crystalloids and colloids can help to facilitate raising the blood pressure. If the pressure cannot be managed by these simple measures, drugs such as dobutamine may be needed to help with blood pressure during the procedure. A dose of atropine will not increase blood pressure directly; it will only increase the heart rate. Good heat support actually helps to keep blood pressure more stable and is especially important in small patients to prevent hypothermia. A 90 ml/kg bolus would be a "shock dose" of fluids for a dog but is too much for a cat, and this cat is hypotensive for other reasons.

When a tortoise is kept in an indoor enclosure, what is the most important light in that enclosure? UVA Incandescent lightbulb Heat Lamp Basking bulb UVB

UVB A UVB light is needed to properly process and absorb calcium from the diet. Sunlight is the natural way to process calcium. Other lighting is also needed for proper digestion. A basking bulb and heat lamp help regulate temperature, and UVA light helps regulate activity.

The olecranon is a structure of which bone? Fibula Ischium Ulna Tibia Humerus

Ulna

The placenta supplies the fetus with oxygen via which structure? Foramen ovale Ductus venosus Umbilical vein Left portal vein Umbilical arteries

Umbilical vein The umbilical vein carries oxygenated blood from the placenta to the growing fetus. The umbilical arteries carry deoxygenated blood from the fetus to the placenta. The foramen ovale is an opening in the septum between the atria in the heart of the fetus with allows blood to bypass the lungs. The ductus venosus shunts a large portion of blood flow in the fetus from the umbilical vein to the inferior vena cava such that oxygenated blood from the placenta can bypass the liver. The left portal vein connects umbilical circulation with portal (liver) circulation.

Which of these forms of chocolate contains the highest concentration of theobromine? Semi-sweet chocolate Milk chocolate Unsweetened baking chocolate White chocolate

Unsweetened baking chocolate The correct answer is unsweetened baking chocolate. This contains about 7 times more theobromine than milk chocolate. White chocolate has very little methylxanthines. Semi-sweet chocolate rates in between. Theobromine per oz. of milk chocolate= 44-60 mg/oz Baker's chocolate has 390-450 mg/oz, Semi-sweet chocolate has about half as much as Baker's or dark chocolate.

What is the dental formula for the adult horse (i.e. incisor-canine-premolar-molar)? Upper: 3-1-4-3 Lower: 3-1-3-3 Upper: 3-1-2-3 Lower: 3-1-2-3 Upper: 4-1-3-3 Lower: 3-1-3-3 Upper: 4-1-2-3 Lower: 4-1-2-3

Upper: 3-1-4-3 Lower: 3-1-3-3 The upper quadrant of the horse's dental arcade has 3 incisors, 1 canine (primarily male horses), 4 premolars (including "wolf" tooth) and 3 molars; the lower arcade has 3 incisions, 1 canine, 3 premolars and 3 molars. Not all horses have the 1st upper premolar, the so-called wolf tooth. Generally, females do not have canine teeth.

Which type of urinary stone is lucent and does not typically show up on a radiograph? Struvite Urate Calcium oxalate Magnesium ammonium phosphate

Urate Urate and Cystine are the two types of stones that do not usually show up on radiographs and require ultrasound or contrast studies to diagnose. Calcium oxalate and struvite stones are radiodense and usually visible on a radiograph. Magnesium ammonium phosphate is another name for a struvite stone.

What structure connects the kidneys to the bladder? Urethra Loop of Henle Ureter Trigone

Ureter The kidneys connect to the bladder via the ureters. The bladder excretes urine to the outside via the urethra. The trigone is the smooth triagular region inside the bladder that is formed by the area where the ureters and urethra attach to the bladder. The Loop of Henle is part of the nephron in the kidney.

For which of the following disorders is an intravenous pyelogram most useful in detecting? Lymphangiectasia Ureteral stones Gall bladder stones Urethral stones Bile duct obstruction

Ureteral stones An intravenous pyelogram (IVP) is an x-ray examination of the kidneys, ureters and urinary bladder using iodinated contrast material injected intravenously and excreted via the kidneys. It is used to detect problems in the urinary tract (particularly the upper urinary tract) including kidney and ureteral stones or tumors involving the kidney, ureters, or urinary bladder. An IVP is not as useful for urethral stones because they are further down in the urinary tract; in this case, retrograde cystography would be more helpful.

A cystogram is a radiographic study performed to identify the position, shape, and lesions involving what structure? Urinary bladder Kidney Liver Esophagus Skin and subcutis

Urinary bladder

Which of the following must be sterile before use? Urinary catheter Otoscope Thermometer Cotton-tipped swab use to collect sample for ear cytology

Urinary catheter Urinary catheters must be sterile so that bacteria are not introduced into the bladder. The other things listed should be clean but don't have to be sterile. If you were planning on doing a culture of the ears, a sterile swab would be warranted but not needed for cytology.

Micturition is a medical term for which of the following? Vomiting Coughing Urinating Straining

Urinating Micturition is the act of voiding urine, or urinating. Emesis is another term for vomiting. Stranguria is the term for straining to urinate.

A 7-year old female spayed domestic short hair previously diagnosed with diabetes mellitus presents to your clinic dull and unresponsive. In addition to obtaining a blood glucose, what other laboratory test would be important for determining the cat's diabetic status? Total protein Blood lactate Hematocrit BUN Urine ketones

Urine ketones A patient with ketones in the urine is likely to be in a diabetic ketoacidotic state. An elevated BUN would be an indication of renal disease. A low hematocrit level would cause a patient to be dull, but this value does not relate directly to diabetes. Blood lactate levels directly correspond to shock. A total protein level would help in determining the patient's hydration and protein status.

What does isosthenuria indicate? Urine that has a lesser specific gravity than the plasma Urine that has a greater specific gravity than the plasma Urine that has significantly elevated protein concentration Urine that has the same specific gravity as the plasma

Urine that has the same specific gravity as the plasma Isosthenuria describes urine that has the same concentration (specific gravity) as the plasma. Urine-specific gravity describes the concentration of a fluid as compared to water (specific gravity of water = 1.000). Isosthenuria may be observed during renal failure, as the kidney cannot concentrate the urine.

During ultrasound of the bladder, a hyperechoic (white) linear structure is seen against the bladder wall that creates a black shadow obscuring all structures deep to it. What is this structure most likely? Round ligament of the bladder Bladder wall tumor Free fluid Urolith

Urolith Sound is strongly reflected by mineral and gas resulting in a hyperechoic appearance. The structure most likely to be mineralized inside the bladder is a urolith. The bladder wall should be of intermediate echogenicity as are most bladder wall tumors. The round ligament of the bladder is usually not visible sonographically but also would not create such a dramatic acoustic shadow as described in the question. Free fluid appears anechoic (black). Air inside the bladder as might be seen in emphysematous cystitis could also be echogenic and cast a shadow but this was not an answer choice.

Which condition occurs least commonly in the horse? Intestinal incarceration Urolithiasis Enterolithiasis Laminitis

Urolithiasis As compared to small animals, urolithiasis is uncommon in horses. If they do develop stones in their bladder, it is usually secondary to formation of calcium oxalate crystals. Enterolithiasis and intestinal incarcerations are common reasons for colic in horses and usually require surgical intervention. Laminitis is a very serious and often life-threatening condition in horses. The pathophysiology of laminitis is poorly understood but involves inflammation of the lamina of the feet. It can involve all feet or even just one foot. There are multiple conditions that can lead to laminitis, such as endotoxemia, grain overload, and pneumonia

For taking a diagnostic radiograph, which of the following techniques will reduce the amount of scatter radiation that reaches a film? Use of a grid with a Potter-Bucky diaphragm Moving the cassette closer to the surface of the patient Decreasing exposure time Use of an intensifying screen Use of high-speed film

Use of a grid with a Potter-Bucky diaphragm A Potter-Bucky diaphragm is a moving grid. A grid is a series of short strips of lead that is placed over the cassette, absorbing all radiation that does not go between the strips. The strips are geometrically arranged to allow the primary radiation through but to absorb any radiation that is scattered in other directions. This results in increasing the resolution of structures on the film. However, using a grid results in the appearance of grid lines on the film unless a Potter-Bucky diaphragm is used. Scatter can also be reduced by utilizing the "air-gap" technique which involves moving the cassette and film away from the patient. This will magnify and may decrease sharpness of the film but will decrease the amount of scatter radiation reaching the film. Exposure time and the use of high-speed film or an intensifying screen do not affect the amount of scatter radiation reaching the film.

All of the following radiation protection measures are intended to decrease the dose to the technician EXCEPT which of the following? Collimating the beam to the smallest aperture necessary Lead glasses Use of lead gloves with at least 0.5 mm lead equivalent Use of filtration at the portal of the X-ray tube

Use of filtration at the portal of the X-ray tube Filtration of the X-ray beam is performed to eliminate low energy (soft) radiation that is part of the emitted spectrum. The soft radiation will be primarily absorbed by the patient with minimal scatter. This radiation would not contribute to the radiographic image and unnecessarily increases patient dose. Collimating the beam decreases radiation dose to the patient and the technician. Protective equipment such as lead gloves, aprons, thyroid shields, and lead glasses all decrease the radiation dose to the technician.

Which of the following principles will always improve radiographic image quality? Use the lowest milliampere (mA) setting and the highest time (s) setting possible to achieve the desired mAs Use the highest kilovoltage (kVp) setting and the lowest milliampere (mA) that adequately exposes the film as possible Use the highest milliampere (mA) setting and lowest time (s) setting possible to achieve the desired mAs Use the lowest kilovoltage (kVp) setting and the highest milliampere (mA) that adequately exposes the film as possible

Use the highest milliampere (mA) setting and lowest time (s) setting possible to achieve the desired mAs The product of multiplying the milliamperage (mA) and time (s) is the mAs, which determines the degree of film exposure for a given kVp. Minimizing the time of exposure decreases motion artifact that occurs from the patient moving. It is always preferable to use the highest milliamperage and lowest time setting for this reason. Depending on the type of radiographic study and the size of the animal, a certain kVp and mAs will produce a radiograph with a corresponding amount of contrast and latitude. Maximizing contrast at the expense of latitude or vice-versa may not yield the best radiograph.

Which of the following is NOT considered good surgical instrument care? Using a brush to scrub the instruments Using tap water Using an ultrasonic cleaner Using cleaning agents at a pH of approximately 10

Using tap water The use of tap water for cleaning surgical instruments is not recommended since it carries a high likelihood of leaving mineral deposits on the instruments; distilled water is preferred. Cleaning agents that have a higher pH (between approximately 9 and 11) are used because these do not promote corrosion. An ultrasonic cleaner is highly recommended as it is much more effective than manual cleaning alone. The use of a brush to scrub the instruments is common practice and facilitates the removal of dry blood or residue.

What is the proper way to restrain a rat for routine procedures? Scruff the rat at the nape of the neck. Hold the rat from the tip of tail and grasp the thorax. Place the rat in a squeeze cage. Using two hands, one around the thorax and under the front legs, and one hand grasping the base of the tail. Scruff the rat and hold the back legs. Cup the rat in the palm of your hand.

Using two hands, one around the thorax and under the front legs, and one hand grasping the base of the tail.

Which of the following is an example of an analytical variable that may adversely affect the accuracy of a laboratory result? Genetic variation within a breed or population A mislabeled tube Obtaining a blood sample from a non-fasted patient Variation in temperature at which an assay is performed

Variation in temperature at which an assay is performed

In a horse, what is the most common surgical approach to the abdominal cavity? Ventral midline incision Parasaggital incision Flank incision Transverse incision

Ventral midline incision Similar to small animals, the most common approach performed is a ventral midline incision. This provides excellent exposure of abdominal organs. A flank incision may be preferred in special circumstances but is not performed as routinely. A parasaggital incision is not ideal as it results in trauma to the abdominal musculature. A transverse incision would not provide good visualization as compared to a ventral midline incision.

The dog's abdominal region may also be referred to as which of the following? Proximal region Ventral region Dorsal region Anterior region

Ventral region Cranial is toward the head; caudal is toward the tail. Ventral is toward the abdomen; dorsal is toward the backbone. Lateral is away from midline and medial is toward midline. Proximal is close to the spine or body while distal is away; these are typically used when describing limbs (ex. fracture of the proximal femur vs. fracture of the distal femur). Palmar is the bottom of the paw of the forelimb; plantar is the bottom of the paw of the hind limb. Anterior is toward the head and posterior is away from the head. In veterinary medicine, anterior and posterior are usually only used to describe distal extremities or areas of the head.

During which cardiac status would a defibrillator most likely be used and effective? Ventricular tachycardia with a pulse Ventricular fibrillation Normal sinus arrhythmia Asystole

Ventricular fibrillation Ventricular fibrillation usually occurs just prior to asystole, when there is uncoordinated contraction of the ventricular muscles (uncontrolled quivering of the muscle fibers). It is during the ventricular fibrillation that a defibrillator is most often used to try to induce a normal rhythm to the heart. Defibrillation delivers a dose of electrical energy to the heart. This depolarizes the heart muscle, stops the arrhythmia, and allows normal sinus rhythm to be reestablished in the sinoatrial node (the "pacemaker" of the heart). The heart will generally not respond to defibrillation in asystole because it is already depolarized. Defibrillation only works if electrical activity is occurring. If a pulse is present, defibrillation for ventricular tachycardia is not indicated. Normal sinus arrhythmia is a normal rhythm in which there is mild acceleration and slowing of the heart rate that occurs during inhalation and exhalation; many normal pets have this rhythm.

A dark, amber-colored sample of urine is likely to have what specific gravity value? Low specific gravity Average specific gravity Very high specific gravity Cannot be determined

Very high specific gravity In urine samples generally, the darker the color the higher the specific gravity. Conversely, a very clear urine sample will have a very low specific gravity value. In most species, urine color ranges from light-yellow to dark-yellow. The exception to this is if the urine contains blood or bilirubin; these can make the urine appear darker, even if the specific gravity is low.

A dog presenting with a head tilt and loss of balance most likely has disease associated with which nerve? Optic Trigeminal Vestibulocochlear Vagus

Vestibulocochlear Head tilt and loss of balance are symptoms of vestibular syndrome, which is most often assoiated with damage to the vestibulocochlear nerve (CN VIII). Other symptoms may include nausea and nystagmus.

The designation of VTS in someone's credentials stands for which of the following? Veterinary Technician Scientist Veterinary Technical Surgeon Veterinary Technician Specialist Veterinarian of Technical Surgeries

Veterinary Technician Specialist VTS is a designation given to a credentialed technician in their field of expertise after spending 3 years full time in that specific field and passing a rigorous board examination. There are several different specialities that can be achieved by credentialed veterinary technicians looking to go beyond their state credentials.

The Veterinary Technician Code of Ethics contains which of the following statements? Veterinary technicians must never put a client's needs above an animal's needs. Veterinary technicians shall protect confidential information provided by clients. Veterinary technicians shall keep the health of the animals they treat their priority. Veterinary technicians should care for all animals equally.

Veterinary technicians shall protect confidential information provided by clients.

All drugs need to be stored and handled so that they are safe for the patient, client, and veterinary staff. When handling drugs that are considered cytotoxic or hazardous, personal protective apparel (PPA) should be worn. With which of the following drugs is it imperative to pay special attention to safe handling? Vincristine Metoclopramide Metronidazole Ascorbic Acid Sucralfate

Vincristine

You are on a house call with the veterinarian to evaluate some goats that are having "night blindness" per the owner. Upon arrival you can see that the goats have a dull haircoat, are very thin, and have nasal discharge. What deficiency can cause these symptoms? Vitamin E Iodine Vitamin A Zinc

Vitamin A Vitamin A deficiency causes a rough dry haircoat, thick nasal discharge, diarrhea, and can also lead to night blindness. Vitamin E can cause white muscle disease (a nutritional muscular dystrophy). Zinc deficiency can cause hypersalivation, chronic skin problems, testicular hypoplasia, and deformed hooves. Iodine deficiency most often causes a goiter.

How is toxicity from the rodenticide, brodifacoum, treated? Vitamin K1 Vitamin D3 Potassium bromide Atropine Methylene blue

Vitamin K1 The correct answer is vitamin K1. Brodifacoum is a vitamin K antagonist (anticoagulant) commonly used in rodenticides. Ingestion of this compound causes hemorrhaging. Treatment for this condition requires vitamin K1 administration for 4-6 weeks.

Which of the following is not a nutrient that provides energy? Fats Vitamins Proteins Carbohydrates

Vitamins

The somatic nervous system is responsible for which of the following functions? Secretion of saliva Voluntary motor movement Regulation of heart rate Contraction of the intestines

Voluntary motor movement Secretion of saliva, contraction of the intestines, and regulation of the heart rate falls under the responsibility of the autonomic nervous system. More specifically, the heart rate is influenced by the parasympathetic and sympathetic nervous systems, both of which are branches of the autonomic nervous system.

What is a symptom of myasthenia gravis? Hyperesthesia to touch Muscle rigidity Papules on the dermis Weakness and muscle fatigue

Weakness and muscle fatigue In Myasthenia gravis, there are few acetylcholine receptor sites (AChR) on the muscles, and acetylcholine is broken down before it can fully cause muscle stimulus. This results in muscle weakness, and with this condition, animals experience severe muscle fatigue.

What should be done when administering nitroglycerin paste to a patient? Have the patient hooked up to IV fluids to prevent hypotension Have the patient on oxygen Wear gloves Apply it directly to the mucous membranes

Wear gloves Nitroglycerine is a potent vasodilator used in cases of heart failure. It is critical to always wear gloves when handling this medication as it is absorbed through the skin. Mostly, it is applied to the skin inside the pinna of the ear. The ear should be marked with tape so that others know not to touch the area after the medication has been applied.

A castrated male goat is known as a: Buck Gelding Wether Barrow

Wether Wether is the term for a neutered male goat. Buck is an intact male goat. Barrow is a castrated male pig. Gelding is a castrated male horse.

What is a pulse deficit? When a pulse is synchronous with a heartbeat When some heart beats do not result in a palpable pulse When an extra pulse occurs sporadically When no pulses are palpable in a patient

When some heart beats do not result in a palpable pulse A pulse deficit is the condition in which a peripheral pulse rate is less than the ventricular contraction rate because some heart beats do not create a palpable pulse. This can be indicative of a lack of peripheral perfusion.

Performing an abdominocentesis involves which of the following? Removing an organ from the abdomen Making an incision into the abdomen Obtaining a urine sample from the abdomen Withdrawing fluid from the abdomen

Withdrawing fluid from the abdomen "Centesis" is the act of puncturing a body cavity or hollow organ in order to remove fluid; therefore, abdominocentesis implies the removal of fluid from the abdominal cavity. Similarly thoracocentesis would be removal of fluid from the chest cavity. A cystocentesis describes the removal of fluid from the urinary bladder. Terms that involve the removal of an organ usually end with "-ectomy". Examples of terms would include splenectomy, nephrectomy, and lung lobectomy.

A horse is presented for a surgical procedure. What is the best time to administer perioperative antibiotics to the patient? Within 1 hour of cut time When the incision is being closed 1 hour post-operatively When the surgeon makes the first cut

Within 1 hour of cut time The best time to administer antibiotics for a surgical patient is within 1 hour of cut time. Antibiotics are most effective and can provide therapeutic levels to the patient when they are given within 60 minutes of cut time (when the incision is made). Subsequent doses may be needed if the surgical procedure is more than 1-2 times the half-life of the chosen antibiotic.

When are needle teeth in piglets usually clipped? At 3 days of age Within 24 hours of birth At 3 weeks of age At 3 months of age

Within 24 hours of birth Piglets are born with 8 sharp needle or milk teeth. The teeth can lacerate the sow during nursing, and playing or fighting behavior with other piglets may cause trauma. The teeth are most often clipped within the first 24 hours after birth.

A pet Guinea Pig is presented to your hospital for emergency treatment after sustaining a crushing injury. Which of the following is not considered part of the initial emergency assessment of the patient? Control of hemorrhage Heat support Assessment of airway, respiratory, and cardiovascular systems Wound stabilization Ruptured organs Fluid or air accumulations

Wound stabilization Just as in other species, assessment of 'ABC's, control of hemorrhage, treatment of hypothermia, diagnosis of ruptured organs, and fluid or air accumulations are part of initial triage. Stabilization of the most critical conditions are performed first and then wound and supportive care is addressed.

Which of the following choices most accurately describes the "heel effect" in diagnostic radiology? There is increased radiation dose to bone relative to soft tissues with lower energy x-rays There is increased radiation dose to bone relative to soft tissues with higher energy x-rays At the edge of the collimated field, there is a region where the x-ray beam gradually declines known as the penumbra X-ray beam intensity declines toward the anode side of the x-ray machine

X-ray beam intensity declines toward the anode side of the x-ray machine

You take and develop an abdominal radiograph of an animal, but the radiograph appears much too light (underexposed) throughout the area of the patient. You increase the exposure (mAs) and the radiograph is still much too light. The portion of the radiograph outside the body of the patient is completely black (exposed). Which of the following is the most likely problem? X-ray energy (kVp) is too low The developer in the processor is low X-ray energy (kVp) is too high The tungsten target in the X-ray machine needs to be replaced The fixative in the processor is low

X-ray energy (kVp) is too low Since part of the film is exposed correctly, you can conclude that X-rays were produced by the machine; there does not seem to be a problem with the target. Also, there are processing artifacts mentioned. The problem here appears to be that the energy of the X-rays produced is insufficient to penetrate the patient. Therefore, the patient is essentially shielding the film from exposure in that area. To fix the problem, try using a higher kVp.

Which of the following is true regarding the properties of x-rays compared to visible light? X-rays and visible light have the same wavelength X-rays have higher frequency and higher wavelength X-rays have shorter frequency and shorter wavelength X-rays have higher frequency and shorter wavelength

X-rays have higher frequency and shorter wavelength The energy of a photon is directly proportional frequency and inversely proportional to wavelength. X-rays are higher energy rays than photons in the visible light spectrum, which gives them greater penetrating ability.

Which is responsible for the transmission of Yersinia pestis? Vampirolepis nana Dermacentor variabilis Xenopsylla cheops Dipylidium caninum

Xenopsylla cheops Xenopsylla is known as the plague or rat flea. Yersinia pestis are gram-negative rod shaped bacteria that are the cause of plague. Plague transmission from infected animals generally occurs via bites from infected rodent fleas (Xenopsylla), direct contact with infected tissues or fluids, or breathing infected droplets. Dipylidium and Vampirolepis are both tapeworms. Dermacentor is the American dog tick.

Which of the following drugs is NOT approved for the use in cattle? Diazepam Albendazole Butorphanol Xylazine

Xylazine Ruminants are highly sensitive to xylazine when compared with horses, dogs, and cats. They generally require only one-tenth of the dosage required for horses to exhibit the same effect. It is not approved for the use in ruminants but is sometimes used off-label. It is important to know how sensitive ruminants are to this drug. Albendazole is a broad spectrum de-wormer commonly used in cattle. Diazepam or Butorphanol can be used as a sedative or tranquilizer in cattle.

Which of the following drugs should be used with extreme caution in ruminants? Acepromazine Lidocaine Atropine Xylazine

Xylazine Xylazine is an alpha-2 adrenergic agonist and is classified as a sedative/analgesic with muscle relaxant properties. It is not approved for any species to be consumed for food purposes. Ruminants are very sensitive to xylazine and it is not used much in these animals. In fact, ruminants generally require only 1/10 of the dosage required for horses to exhibit the same effect. Due to the unique GI system of the ruminant, much care must be taken when administering drugs so as not to interfere with proper digestion, and xylazine inreases risk of ileus.

Which drug is a commonly used sedative for horses? Diazepam Propofol Xylazine Ketamine

Xylazine Xylazine is a very common sedative for horses. Detomidine is also commonly used for sedation in horses. Both of these drugs are alpha 2-adrenergic agonists used for their sedative and analgesic effects.

A Cavalier King Charles Spaniel presents to your clinic with a history of chronic otitis externa. You perform an ear cytology showing large numbers of Malassezia. Which of the following types of organisms is this? Ear mite Yeast Gram-negative bacteria Acid-fast bacteria Gram-positive bacteria

Yeast Malassezia pachydermitis is a yeast/fungus very common in skin and ear infections. It resembles a bowling pin under the microscope. The ear mite is Otodectes. This is a cytology showing Malassezia. Note the bowling pin appearance of these yeast.

Icterus is a term that means which of the following? Small hemorrhages on the mucus membranes Yellow discoloration of the skin, eyes, and mucous membranes Straining to urinate Blood in the stool

Yellow discoloration of the skin, eyes, and mucous membranes Icterus, or jaundice, means yellow discoloration of the skin and sclera. It is most often associated with liver disease from a poor flow of bile or biliary obstruction in the liver. It can also be caused by hepatitis or from red blood cell destruction. Small hemorrhages on the mucus membranes are petechiae. Straining to urinate is termed stranguria. Blood in the stool is hematochezia (bright red blood suggests large intestine bleeding) or melena (black/digested blood suggests small intestine bleeding) This is a photo of a foal with neonatal isoerythrolysis, which causes icterus. Note the yellow appearance of the mucous membranes and sclera.

An owner presents her puppy for severe itching. The veterinarian diagnoses Sarcoptes mites with a superficial skin scraping. The owner wants to know if this mite is contagious to her and her children. What should you tell her? Yes, these are highly contagious and can be easily passed between all species Yes, there can be a temporary transmission from her dog to those with direct contact, but the mites are host-specific and the infection is temporary No, these mites are only contagious between dogs and cats Yes, but the infections are transient, and her dog does not need treatment

Yes, there can be a temporary transmission from her dog to those with direct contact, but the mites are host-specific and the infection is temporary Sarcoptic mites (scabies) from the dog can transfer to humans and cause skin irritation. However, since humans cannot host the canine sarcoptic mite, it dies off without treatment being required for the human, just the dog. Humans have their own sarcoptic mite that is contagious between humans. Notoedres cati causes feline scabies and is host-specific to the cat. It can also temporarily infect humans.

You are attempting to give an intra-jugular injection to a horse and insert the needle to the hub. Blood is forcefully pulsating and ejecting from the needle. What is the most likely explanation? This is normal when injecting medication into the jugular of a horse You are in the carotid artery The vein is spasming, and this is normal; it is okay to give the medication The horse has hypertension

You are in the carotid artery This is typical of insertion into an artery. If this occurs, the needle should be removed immediately and pressure placed on the site for several minutes to assure hemostasis.

A dog presents to your hospital to have surgery for a cranial cruciate ligament rupture (see x-ray). The veterinarian wants you to place an epidural after the patient is under anesthesia and before the procedure is performed. While performing the epidural you get blood in the hub of the spinal needle. What does this mean and what should you do? You have created a hematoma going through the skin, it is ok to administer the full dose of anesthetic once the needle is in its proper place. You have created a hematoma going through the skin, you should withdraw the needle and start over. You have entered the sinus, you should withdraw the needle and start over. You have entered the sinus and you should give only 1/2 the dose of anesthetic.

You have entered the sinus, you should withdraw the needle and start over. If you see blood flow into the spinal needle it means that you have entered a venous sinus, the needle should be removed and the procedure can be started again.

Which of the following solutions used for fecal flotation floats protozoal organisms with the least distortion? Sodium nitrate Zinc sulfate Saturated sodium chloride Formalin

Zinc sulfate

What is the most commonly used method for fecal parasite testing for dogs and cats? Zinc sulfate centrifugation Fecal cytology using H&E staining method Direct smear Baermann funnel technique

Zinc sulfate centrifugation Zinc sulfate centrifugation is the most common and best of these choices. Technique: Zinc sulfate (ZNSO4) solution is added to 2-3 grams of feces. It is mixed and then centrifuged for a few minutes on high speed. A loop can then skim the surface and place the sample on a slide for examination, or the tube can be filled to the top with more zinc solution and a coverslip set on top for a few minutes. Then the coverslip can be placed onto a slide for examination. Sometimes a drop of iodine is added to the sample to help identify the eggs more easily. Baermann is useful for detecting lungworm and some other nematodes. Direct smears can help detect trophozoites, but they are not useful for finding tapeworm or nematode eggs, or coccidia. Cytology of feces would mostly just demonstrate the bacterial flora.

What toxicity is caused by ingestion of these items? (pennies) No toxicity is caused by penny ingestion Zinc toxicity Nickel toxicity Lead toxicity

Zinc toxicity Any penny minted after 1982 contains a large amount of zinc, leading to zinc toxicity in animals even from just a single penny.

Four patients present to your emergency clinic at the same time: a) 2-year old male domestic short hair with a urethral obstruction b) 5-year old female Labrador with a tracheal foreign body and cyanosis c) 10-year old male standard poodle with a laceration on its forelimb d) 4-year old male hamster with elongated teeth In what order should these patients be triaged? c, d, a, b a, b, c, d a, d, b, c a, c, b, d b, a, c, d

b, a, c, d A patient with an airway obstruction is in immediate danger of respiratory arrest and therefore takes priority over all other patients. A cat with urethral obstruction would be the next priority due to the possibility of a bladder rupture. A laceration needs care within a few hours. Elongated or overgrown teeth in a hamster, while needing treatment, would have the lowest priority in this group of emergency patients.


Ensembles d'études connexes

how to refer a friend on quizlet so you can get quizlet+

View Set

Week 2 of 5: Start the UX Design Process: Empathize, Define, and Ideate

View Set

CB - ch. 13 Situational Influences

View Set

Squares, Cubes, Square and Cube Roots, Simplifying Square and Cube Roots, Simplifying Radicals (Numbers with Variables), Negative Exponents, Exponent Rules, Product Rule and Power to a Power Rule for exponents, Order of Operations with Integers

View Set

Evolve Adaptive Quiz - Leadership and Management

View Set

Phases and Phase Changes-Chemistry Chapter 5

View Set